Skip to main content

Full text of "High school algebra"

See other formats


MiiniTljlMl^  ■ " "  ■ "'  IMMHI 


<liwww»ii»lijj}lii«ii<iiiMWjm<MWg:<ri)i>^'Fy.f>"'»rT><TO^''"i"M''<'«^'^^^^^ 


\' 


n 


aI 


^ 


.«;'■ 


.0% 


.ev^^AV^^ 


.^A 


f^t;;--'"' 


S>" 


HIGH   SCHOOL  ALGEBEA 


Digitized  by  the  Internet  Archive 

in  2010  with  funding  from 

Ontario  Council  of  University  Libraries 


http://www.archive.org/details/highschalgebracOOcraw 


HIGH  SCHOOL  ALGEBRA 


^ 


J.  T.  CRAWFORD,  B.A. 

CHIEF    INSTRUCTOR    IX    MATHEMATICS,     UNIVERSITY    SCHOOLS, 

LECTURER   IN    MATHEMATICS,    FACULTY    OF   EDUCATION, 

UNIVERSITY   OF  TORONTO 


AUTHOEIZED   BY 
THE  MINISTER  OF  EDUCATION  FOR  ONTARIO 


TORONTO 
THE  MACMILLAN  COMPANY  OF  CANADA,  LIMITED 

1916 


Copyright,  Canada,  1915 

By  the  MACMILLAN  COMPANY  OF  CANADA,  Limited 

Reprinteh  1916 


PREFACE 

This  text  covers  the  work  prescribed  for  entrance  to  the 
Universities  and  Normal  Schools. 

The  book  is  written  from  the  standpoint  of  the  pupil,  and 
in  such  a  form  that  he  will  be  able  to  understand  it  with  a 
minimum  of  assistance  from  the  teacher.  The  question 
method  is  frequently  used  in  developing  the  theory.  The 
purpose  of  this  is  to  lead  the  pupil  to  think  for  himself. 

The  close  connection  between  algebra  and  arithmetic  is 
constantly  kept  in  view,  and  in  many  cases  the  arithmetical 
and  algebraic  processes  are  shown  in  parallel  columns. 

There  are  numerous  diagrams  for  the  purpose  of  illus- 
trating the  theory,  and  algebraic  methods  are  applied  to  many 
of  the  theorems  which  the  pupil  meets  in  elementary  geometry. 

Special  emphasis  is  placed  upon  the  verification  of  results. 
In  the  past,  sufficient  attention  has  not  been  given  to  this 
important  part  of  mathematical  work. 

Provision  is  made  for  oral  work,  many  of  the  exercises 
being  introduced  by  a  number  of  oral  examples  for  use  in 
class. 

The  equation  and  the  solution  of  simple  problems  are 
introduced  in  the  second  Chapter.  It  is  ho})ed  that  the 
pupil  will  thus  become  interested  much  earlier  in  the  work. 

Long  multiplications  and  divisions  are  not  included  in  the 
work  of  the  first  year.  They  are  difficult  for  the  beginner 
and  of  little  interest,  as  there  is  not  much  to  offer  in  the 
way  of  practical  illustrations. 


vi  PREFACE 

Chapter  X.,  with  which  the  pupil  would  begin  the  second 
year's  work,  contains  a  thorough  review  of  the  simple  rules. 
Here  the  more  complicated  processes  are  dealt  with. 

The  graphical  work  is  introduced  naturally  in  illustrating 
the  negative  quantity  and  in  the  solution  of  equations. 
Only  graphs  which  can  be  drawn  with  the  ruler  and 
compasses  are  included  in  the  book. 

More  attention  is  given  to  methods  of  inspection  in  the 
extraction  of  roots.  The  long  process  for  cube  root  is 
eliminated,  as  cube  root  is  not  now  required  in  arithmetic. 

The  work  on  ratio  and  proportion  is  presented  in  as  simple 
a  form  as  possible,  and  is  intended  only  as  an  introduction  to 
the  senior  work  in  this  subject.  The  geometrical  illustrations 
which  are  given  should  make  it  more  interesting. 

The  division  method  of  finding  highest  common  factor  has 
been  discarded,  as  it  is  usually  performed  mechanically  and 
not  understood  by  pupils.  The  elimination  method  which 
is  used  will  be  found  easj'  to  apply  with  expressions  which 
are  not  too  complicated.  Finding  the  highest  common 
factor  of  expressions  of  the  fovirth  or  higher  degrees  is  of 
little  algebraic  value,  and  few  examples  of  such  problems 
will  be  found  in  the  book. 

The  review  exercises  at  the  end  of  each  Chapter  will  be 
found  useful,  particularly  for  the  purpose  of  reviewing  the 
work  of  a  previous  term. 

On  the  recommendation  of  experienced  teachers  the  answers 
are  not  given  to  simple  examples,  or  to  such  examples  as  the 
pupil  can  verify  without  difficulty. 


CONTENTS 
CHAPTER  I 

PAGE 

Algebraic  Notation 1 

Arithmetical  and  Algebraic  Signs  and  Sj'mbols  (1). 
Fmidamental  Laws  (3).  Factor  and  Pro- 
duct (6).  Power  and  Index  (7).  Terms  (8). 
Coefficient  (9).  Addition  and  Subtraction  of 
like  Terms  (9).  Use  of  Brackets  (11).  Review 
Exercise  (13). 

CHAPTER  II 
Simple  Equations ■        .        .        .        16 

Meaning  of  Equation,  Solving  an  Equation,  Root 
of  an  Equation  (16).     Axioms  used  (18).     Veri- 
fjnng     Results     (19).     Problems     solved     by 
Equations  (21).     Review  Exercise  (25). 

CHAPTER  III 

Positive  and  Negative  Numbers 28 

Graphical  Representation  of  Positive  and  Negative 
Numbers  (28).  Concrete  Examples  of  Negative 
Numbers  (30).  Signs  of  Operation  and  Signs 
of  Quality  (32).  Absolute  Value  (32).  Review 
Exercise  (34). 

CHAPTER  IV 

Addition  and  Subtraction 36 

Addition  of  Quantities  with  hke  Signs  (36).  Com- 
pound Addition  (37).  Addition  with  unhke 
Signs  (39).  Indicated  Additions  (41).  Sub- 
traction the  Inverse  of  Addition  (42).  Rule  for 
Subtraction  (43).  Removal  of  Brackets  (45). 
Review  Exercise  (47). 


viii  CONTENTS 

CHAPTER  V 

PAOK 

Multiplication  and  Division 49 

Multiplication  of  Simple  Positive  Quantities  (49). 
Index  Law  for  Multiplication  (49).  Rule  of 
Signs  (50).  Compound  Multiplication  (53). 
Verifications  (55).  Division  by  a  Simple 
Quantity  (57).  Index  Law  for  Division  (58). 
Review  Exercise  (60). 

CHAPTER  VI 

Simple  Equations  (continued) •    .        62 

Equation  and  Identity  (62).  Transposing  Terms 
(63).  Simple  Fractional  Equations  (67).  Steps 
in  the  Solution  of  an  Equation  (67).  Problems 
(69).  Algebraic  statements  of  Arithmetical 
Theorems  (74).     Review  Exercise  (75). 


CHAPTER  VII 

Simultaneous  Equations 78 

Equations  with  two  Unknowns  (78).  Method  of 
Solution  (79).  Elimination  (80).  Fractional 
Simultaneous  Equations  (82).  Problems  (83). 
Review  Exercise  (85). 


CHAPTER  VIII 

Type  Products  and  Simple  Factoring 88 

Monomial  Factors  (88).  Product  of  two  Binomials 
(89).  Factors  of  Trinomials  (90).  Radical 
Sign  (92).  Square  of  a  Binomial  (94).  Square 
Root  of  a  Trinomial  (95).  Difference  of  two 
Squares  (97).  Numerical  Applications  (99). 
Review  Exercise  (101). 


CONTENTS 


CHAPTER  IX 


Simple  Applications  of  Factoring   .  .  .103 

Highest  Common  Factor  (103).  Algebraic  Fractions 
(104).  Lowest  Terms  (105).  Multiplication  and 
Division  of  Fractions  (106).  Lowest  Common 
Multiple  (107).  Addition  and  Subtraction  of 
Fractions  (108).  Mixed  Expressions  (HO). 
Review  Exercise  (111). 

CHAPTER  X 

Review  of  the  Simple  Rules 114 

Brackets    (114).  Collecting    Coefficients    (117). 

Multiplication  with  Detached  Coefficients  (118). 
Division  by  a  Compound  Quantity  (121).  Veri- 
fjdng  Division  (122).  Inexact  Division  (124). 
Review  Exercise  (125). 

CHAPTER  XI 

Factoring  (continued) 128 

Factors  by  Grouping  (128).  Complete  Squares  (130). 
Difference    of    Squares    (132).  Incomplete 

Squares  (135).  Trinomials  (137).  Sum  and 
Difference  of  Cubes  ( 140) .  The  Factor  Theorem 
(141).  Easy    Quadratic    Equations    (144). 

Review  Exercise  (146). 

CHAPTER  XII 

Simultaneous  Equations  {continued) 149 

Elimination  by  Substitution  and  by  Comparison 
(149).  Equations  with  three  Unknowns  (152). 
Special  Forms  of  Equations  (154).  Solution  of 
Problems  (156).     Review  Exercise  (161). 


X  CONTENTS 

CHAPTER  XTII 

PAOK 

Geometrical  Representation  of  Number         ....      165 

Function  of  x  (165).  Variables  and  Constants  (165). 
Arithmetical  Graphs  (166).  The  Axes  (169). 
Graph  of  an  Equation  (170).  Coordinates  (170). 
Plotting  Points  (171).  Linear  Equation  (173). 
Graphical  Solution  of  Simultaneous  Equations 
(175).  Special  Forms  of  Equations  (176). 
Review  Exercise  (177). 


CHAPTER  XIV 

Highest  Common  Factor  and  Lowest  Common  Multiple  180 

By  Factoring  (180).  By  Ehmination  (183).  Product 
of  the  H.C.F.  and  L.C.M.  (185).  Review 
Exercise  (187). 


CHAPTER  XV 

Fractions 188 

Changes  in  the  Form  of  a  Fraction  (188).  Lowest 
Terms  (190).  Addition  and  Subtraction  (191). 
Special    Types    (193).  Cyclic    Order    (194). 

Multiplication  and  Division   (197).       Complex 
Fractions  (199).     Review  Exercise  (201). 


CHAPTER  XVI 

Fractional  Equations       .......      204 

Cross    Multiplication    (204).  Special    Forms    of 

Fractional  Equations  (206).  Literal  Equations 
with  one  Unknown  (209),  with  two  Unknowns 
(212).     Review  Exercise  (214). 


CONTENTS  XI 


CHAPTER  XVII 


Extraction  of  Roots  •        .        .  216 

Square  Root  by  Inspection  (216),  by  the  Formal 
Method  (217).  Verifying  Results  (218).  Cube 
of  a  Binomial  (222).  Cube  Root  (223).  Higher 
Roots  (224).     Review  Exercise  (226). 


CHAPTER  XVIII 

Quadratic  Surds 228 

Surd  and  Rational  Quantities  (228).  Mixed  and 
Entire  Surds  (229).  Like  Surds  (231).  Addition 
and  Subtraction  (231 ).  Square  Root  Table  (232) 
Conjugate  Surds  (233).  Rationalizing  a  De- 
nominator (235).  Surd  Equations  (237). 
Review  Exercise  (238). 


CHAPTER  XIX 

Quadratic  Equations 240 

Standard  Form  (241).  Solution  by  Factoring  (242), 
by  Completing  the  Square  (244).  Irrational 
Roots  (247).  Inadmissible  Solutions  (248). 
Review  Exercise  (250). 


CHAPTER  XX 

Ratio  and  Proportion      .  .  ....      253 

Methods  of  Comparing  Magnitudes  (253).  Com- 
parison of  Ratios  (254).  Proportion  (256). 
Finding  a  Ratio  by  Solving  an  Equation  (257). 
Mean  Proportional  (261).  Ratio  Theorems  (263). 
Review    Exercise  (266). 


xii  CONTENTS 

CHAPTER  XXI 

PAGE 

The  General  Quadratic  Equation 268 

Solution  of  Literal  Quadratics  (268).  Solution  by- 
Formula  (270).  Imaginary  Roots  (271).  Equa- 
tions Solved  like  Quadratics  (273).  Review 
Exercise  (276). 

CHAPTER  XXII 

Simultaneous  Quadratics 279 

Three  Types  of  Simultaneous  Quadratics  (279). 
Special  Methods  (284).  Graphical  Solutions 
(288).     Review  Exercise  (290). 

CHAPTER  XXIII 

Indices 293 

The  Index  Laws  (294).  Fractional,  Zero  and 
Negative  Indices  (297).  Operations  with 
Fractional  and  Negative  Indices  (301).  Con- 
tracted Methods  (303).     Review  Exercise  (305). 

CHAPTER  XXIV 

Surds  and  Surd  Equations 308 

Surds  of  different  Orders  (308).  Varying  Forms  of 
Surds  (309).  Surd  Equations  (312).  Extraneous 
Roots  (313).  Square  Root  of  a  Binomial  Surd 
(317).  Imaginary  Surds  (320).  Impossible 
Problems  (323).     Review  Exercise  (324). 

CHAPTER  XXV 

Theory  of  Quadratic  Equations  327 

Sum  and  Product  of  the  Roots  (327).  Reciprocal 
Roots  (328).  Functions  of  the  Roots  (331). 
Character  of  the  Roots  (335).  The  Discriminant 
(337).  Factors  of  the  Quadratic  Expression 
(338).     Review  Exercise  (341). 


CONTENTS  xiii- 

CHAPTER  XXVI 

Supplementary  Chapter .  344 

Factors  of  the  Product  of  two  Trinomials  (344). 
Sum  and  Difference  of  Cubes  (346).  Factors 
by  Grouping  (349).  The  Factor  Theorem 
(351).  Symmetry  (354).  Factoring  by  Sym- 
metry (355).  Identities  (359).  Review  Exercise 
(362). 

Answers 365 


HIGH  SCHOOL  ALGEBRA 


HIGH   SCHOOL   ALGEBRA 

CHAPTER  I 
ALGEBRAIC  NOTATION 

I.  Use  of  Arithmetical  Signs.  In  arithmetic,  signs  are 
used  to  abbreviate  the  work.  In  algebra  the  same  signs  are 
used,  with  the  same  meanings  and  for  the  same  purpose. 

EXERCISE   1 

Write  the  following  statements  in  the  shortest  way  you  can,  using 
the  signs  and  symbols  with  which  you  are  famihar  in  arithmetic. 

1.  Two  and  two  make  iour. 

2.  The  sum  of  five,  ten  and  twenty  is  thirt^y  five. 

3.  Six  and  four  is  the  same  as  four  and  six. 

4.  Seven  times  eight  is  the  same  as  eight  times  seven. 

5.  The  diiference  between  twelve  and  five  is  seven. 

6.  Ten  exceeds  six  by  four. 

7.  The  excess  of  twenty  over  fifteen  is  five. 

8.  The  defect  of  thirty  from  a  hundred  is  seventy. 

9.  Thirty-six  divided  by  four  is  nine. 
10.  Three  score  and  ten  is  seventy. 

II.  One  half  of  the  sum  of  seven  and  five  is  six. 

12.  The  sum  or  the  product  of  three,  five  and  seven  is  the  same  in 
whatever  order  they  are  written. 

13.  Three  multipUed  by  four  is  twelve,  therefore  twelve  divided 
by  three  is  four. 

14.  The  square  of  four  is  sixteen,  therefore  the  square  root  of 
sixteen  is  four. 

B 


2  ALGEBRA 

2.  Algebraic  Symbols.  In  the  preceding  exercise  you  have 
used  symbols  to  represent  the  numbers  stated  and  signs  to 
show  the  operations  performed  on  those  numbers. 

In  algebra,  symbols  are  used  more  extensively  than  in 
arithmetic. 

A                                                           B 
I 1 

If  the  length  of  this  line  be  measured  it  will  be  found  to 
be  two  inches.  But  without  measuring  it,  we  may  say  that 
the  measure  of  its  length  is  some  definite  number  which 
might  be  represented  by  the  letter  a. 

The  measure  of  the  length  of  another  line  might  be  repre- 
sented by  6.  The  cost  of  an  article  might  be  c  cents,  or  the 
cost  of  a  farm  might  be  x  dollars,  or  the  weight  of  a  stone 
might  be  m  pounds. 

Here  a,  b,  c,  x,  m  are  algebraic  number-symbols,  or  briefly 
algebraic  numbers. 

The  symbols  1,  2,  3,  etc.,  used  to  represent  numbers  in 
arithmetic  are  called  arithmetical  number-symbols  or  arith- 
metical numbers. 

In  algebra  the  number  symbols  of  arithmetic  are  also  used. 
For  the  present,  when  letters  are  used  to  represent  numbers, 
it  will  be  understood  that  each  letter  represents  some  integral 
or  fractional  number. 

3.  Signs  of  Multiplication.     In  this  square  the  measure  of 
D  C      the  length  of  the  side   AB  is   a.     What  is  the 

measure    of    the    length    of    BC ;     of    CD  ;   of 
AB+BC  ;  of  AB+BC^CD  ? 

The  measure  of  the  perimeter  (sum  of  all  the 
sides)  is     a-f-a+a+a  or  4  times  a  or  4xa. 
In  algebra,  4xa  or  ax4  is  usually  written  4a,  the  sign  of 
multiplication  being  understood.     It  is  also  written  4. a,  the 
dot  representing  multiplication. 

Thus,  4  X  a  =  4 .  a  =  4a,  and  as  in  arithmetic,  is  a  short  way  of  writing 
a-\-a-\-a-\-n. 

Thvjs,  if  o=6,  the  measure  of  the  perimeter  of  the  square  is 
6+6+6+6  =  4x6  =  24. 


ALGEBRAIC    NOTATION  3 

It  will  be  observed  that  in  algebra  the  multiplication  of  a 
and  4  is  only  indicated  in  the  form  4a,  while  in  arithmetic 
it  may  be  actnallj^  performed  as  in  the  result  24. 

The  pupil  must  recognize  the  difference  between  24 
{twenty -four)  and  the  product  of  2  and  4  or  2  x  4  or  2  . 4.  When 
two  numerical  quantities  are  to  be  multiplied,  the  sign  of 
multiplication  must  be  used,  so  that  as  stated,  24  may  be 
distinguished  from  2x4.  When  both  factors  are  not  numerical 
as  4xa  or  axh,  the  sign  is  omitted  and  these  are  written  in 
the  form  4a,  ah. 

4.  Signs  of  Division.  As  in  arithmetic,  the  quotient  ob- 
tained by  dividing  one  number  by  another  ma}^  be  written 
in  the  fractional  form. 

In  arithmetic  the  division  may  be  actually  performed,  as 
in  6-i-3,  which  may  be  written  f  or  2,  but  it  is  frequently 
only  indicated  as  in  6-^7,  which  is  written  ^. 

So  in  algebra,  the  quotient  obtained  on  dividing  a  by  h, 

or  a^b,  is  written  j,  and  here,  as  in  multiplication,  the  division 

can  only  be  indicated  unless  the  numerical  values  of  a  and 
b  are  known. 

5.  Some  Fundamental  Laws.  Since  the  letters  used  in 
algebra  represent  arithmetical  numbers,  all  the  laws  of 
arithmetic  must  be  true  also  in  algebra. 

In  arithmetic.  I  In  algebra, 

(1)  7  +  3  =  3  +  7.  i  (1)  a+b  =  b+a, 
6+2+5  =  6+5+2  =  2+5+6.                  a+6+c  =  o+c+6  =  6+c+a. 

(2)  3x5  =  5x3.  (2)  ab  =  ba. 
2x4x3  =  2x3x4  =  3x4x2.                          abc  =  acb  =  cba. 

(3)  10+5-2=10-2  +  5.  (3)         a  +  b-c=a-c+b. 
10-5-2  =  10-2-5.  a-b-c  =  a-c-b. 

(4)  3x10-:- 5  =  3  +  5x10=10-1-5x3    (4)  axb^c  =  a-^cxb  =  b^cxa. 

From  (1)  and  (2)  it  follows  that  the  sum  or  the  product  of 
several  numbers  is  independent  of  the  order  in  ivhich  they  are 
written. 

B  2 


4  ALGEBRA 

From  (3)  and  (4)  it  follows  that  a  series  of  additions  and 
std)tractions,  or  of  multiplications  and  divisions,  may  be  mxide 
in  any  order. 

In  finding  the  numerical  value  of  an  expression  hke 
3a+4&— 2c  for  given  values  of  a,  h  and  c,  the  operations  are 
performed  in  the  same  order  as  in  arithmetic,  the  multi- 
plications being  performed  first  and  then  the  additions  and 
subtractions  in  any  order. 

Thus,  when  a  =  2,  6  =  3,  c  =  l, 

3a+46-2c  =  3x  2+4x3-2x1  =  6+12-2=16. 
Similarly,  for  the  same  values  of  a,  b,  c, 

ab  +  bc      2x3+3x1  _  6+3_9_ 
a  +  b    ~         2  +  3        ~     5     ~  5" 

Note. — Many  of  the  examples  in  the  following  exercise  may  be 
taken  orally.  The  pupil,  however,  is  advised  to  write  the  algebraic 
forms  so  that  he  may  thereby  become  familiar  with  them. 

EXERCISE  2 

1.  When  a =6,  what  are  the  numerical  values  of : 

„      1      a    5      12    2    5a « 
Sa,  -a,  -,  -a,  — ,  -,  —  ? 
2      3    6       a     a     S 

2.  When  a:=5  and  2/=3,  what  are  the  vahies  of; 

x~\-y,  x—y,  xy,  3x-\-2y,  2x—3y,  \xy  ? 

3.  When  m=4,  w=6,  r=2,  find  the  values  of : 

m^n-\-r,  m-^-r—n,  mn-\-mr,  mr—n,  4w— 3m— 6r. 

4.  Express  algebraically  the  sum,  'the  difference  and  the  product 
of  a  and  b.     What  are  their  values  when  a =8  and  6==3  ? 

5.  The  quantities  a,  b  and  c  are  to  be  added  together.  Express 
the  sum  algebraically.     What  is  its  value  when  a=6,  6=4,  c=12  ? 

6.  When  a  is  divided  by  6  the  quotient  is  expressed  in  the  form  --. 

When  c  is  added  to  the  quotient  of  x  by  y,  how  is  the  result  expressed  ? 
What  is  its  value  when  a;=12,  y=4t,  c=10  ? 

7 .  A  boy  has  p  marbles ;  he  wins  q  marbles  and  then  loses  r  marbles. 
How  many  has  he  now  ?     How  many  if  ^=5,  5'=11,  r=4? 


ALGEBRAIC   NOTATION  6 

8.  When  a=4  and  6=5,  find  the  numerical  value  of 

12a— 56+6«— 76+10. 

9,  The  sides  of  a  triangle  are  a,  6  and  c ;  express  algebraically  the 
perimeter  and  the  semi-perimeter.  What  do  they  become  if  a=13, 
6=14,  c=15? 

10.  Find  the  cost  of  8  articles  at  5  cents  each  ;  of  7  articles  at  x 
cents  each  ;  of  x  yards  of  cloth  at  6  cents  a  yard  ;  of  m  tons  of  coal  at 
n  dollars  a  ton. 

11.  How  many  cents  are  there  in  4  dollars  ;  in  x  dollars  ;  in  x 
dollars  and  y  cents  ;  in  a  quarters  and  6  ten-cent  pieces  ? 

12.  Find  the  number  of  inches  in  2  yards  ;  in  3  feet  and  7  mches  ; 
in  a  yards  ;  in  6  feet ;  in  x  feet  and  y  inches  ;  in  m  yards  n  feet  and 
p  inches. 

13.  What  operations  are  to  be  performed  to  find  the  numerical 
value  of  vm-\-nb,  when  a=2,  6=5,  m='i,  ?i=6  ?     What  is  the  value  ? 

14.  What  operations  are  to  be  performed  to  find  the  value  of  ^Zl^, 

a+6 
when  a=5,  6=6,  x=15,  y— 7  ?     What  is  the  value  ? 

15.  By  varying  the  order  of  the  letters,  in  liow  many  ways  can 
you  write  a-\-h-\-c  ? 

16.  In  how  many  different  ways  can  you  ^vrite  xyz  ?  \_ 

17.  In  the  figure,  BC  is  twice  as  long  as  AB.  li  A  Q  C 
AB  \s  I  units  in  length,  what  is  the  length  of  BC  ?  '  ' 
of  ^C? 

18.  In  the  figure,  BC  is  three  times  as  long  as  AB  and  CD  is  twice 
as  long  as  AB.     If  AB  is  x  units  in    ^       B  C  D 

length,  what  are  the  lengths  of  BC  1     ' 1 1  ' 

CD?    BDt    ADt 

19.  In  the  following  statements  c  represents  the  cost  of  an  article, 
s  the  selling  price,  and  g  the  gain  : 

(1)  s-c=g,  (2)  c^g=s,  (3)  s-g=c. 

Read  them  and  explain  their  meanings. 

20.  What  is  the  next  integer  above  27  ?  The  next  below  27  ? 
What  is  the  next  integer  above  n  ?     The  next  below  n  ? 

21.  If  w  is  an  even  integer,  what  is  the  next  even  integer  above  it 
and  the  next  even  integer  below  it  ? 


6  ALGEBRA 

22.  If  a;  is  any  number,  what  is  the  number  which  is  5  greater  than 
X  1     5  less  than  x  ? 

23.  A  boy  is  10  years  old.  How  old  will  he  be  in  6  years  ?  In 
m  years  ?     How  old  was  he  4  years  ago  ?     n  years  ago  ? 

24.  A  man  is  x  years  old.  How  old  wUl  he  be  in  n  years  ?  How 
old  was  he  m  years  ago  ?  In  how  many  years  wUl  he  be  three  times 
as  old  as  he  is  now  ? 

25.  A  boy  was  p  years  old  3  years  ago.  How  old  wQl  he  be  15  years 
from  now  ? 

26.  Explain  the  difference  between  ,  a  +  -  and  a  .  -.       What 

c  c  c 

are  their  values  when  a =6,  b=9,  c=3  ? 

27.  The  side  of  one  square  is  a  and  of  a  smaller  one  is  b.  Indicate 
the  difference  in  their  perimeters.  What  is  the  difference  if  a=10  and 
6=6? 

28.  The  sides  of  one  rectangle  are  a  and  b,  and  of  another  are  c 
and  d.  Indicate  the  difference  in  their  areas,  (1)  when  the  first  rect- 
angle is  the  larger,  (2)  when  the  second  is  the  larger. 

29.  What  arithmetical  number  does  10x-\-y  represent  when  x=5, 
y=3  1     When  x^l,  y=9  ? 

30.  When  a=3,  6=4,  c=5,  d=0,  find  the  values  of  : 

(1)  I0a+4b-5c  +  3d.        (2)  5ab+2cd—3ac. 

6a—2b+3c—d 


(3)  ^ac^lbc-lad.  (4) 


2a+b—c+d 


6.  Factor  and  Product.  When  numbers  are  multiplied 
together  the  result  is  called  the  product,  and  the  numbers 
which  were  multiphed  are  called  the  factors  of  the  product. 

Thus,  3x5=15,  therefore  the  factors  of  15  are  3  and  5,  so  a  xb  =  ab, 
therefore  the  factors  of  ab  are  a  and  b. 

The  factors  of  ,3.t  are  3  and  x.  The  3  is  called  a  numerical 
factor  and  the  x,  a  literal  factor. 

Just  as  12  may  have  different  sets  of  factors  as  3x4,  2x6, 
2  X  2  X  3,  so  3x1/  has  the  factors  3  X  xy,  3x  x  //,  a:  x  3y  or  3  X  a;  x  ?/. 

The  prime  factors  of  12  are  2,  2  and  3>  and  the  simplest 
factors  of  3xy  are  3,  x  and  y. 


ALGEBRAIC    NOTATION  7 

In  whatever  order  the  factors  are  written  the  product  is 
the  same,  but  it  is  usual  to  write  the  numerical  factor  first 
and  the  hteral  factors  in  alphabetical  order. 

7.  Power  and  Index.  What  is  the  area  of  a  square  whose 
side  is  7  inches  in  length  ?  The  measure  of  the  area  of  the 
square  in  art.  3  is  axa,  which  is  written  a-,  and  is  read  "  a 
square,"  or  "  a  to  the  second  power." 

The  product  when  2  a's  are  multiphed  together  is  called 
the  power,  and  the  2  is  called  the  index  or  exponent  of  the 
power. 

If  the  edge  of  a  cube  is  6  inches,  what  is  the  sum  of  all  the 
edges  ?  What  is  the  area  of  each  face  of  the  cube  ?  What 
is  the  area  of  all  the  faces  ?  What  is  the  volume  of  the 
cube  ? 

If  the  edge  of  a  cube  is  a,  the  sum  of  all  the  edges  is  12a, 
The  area  of  each  face  is  a^,  and  of  all  the  faces  is  Qa^. 

The  volume  is  ax  ax  a  or  a^,  which  is  read  "a  cube,"  or 
"  a  to  the  third  power." 

The  pupil  must  distinguish  between  3a  and  a^.  The  former 
means  3  X  a,  and  the  latter  ax  ax  a. 

Thus,  if  a=5,  3a=3x5=15, 

but  0^  =  5x5x5=125. 

EXERCISE  3  (1-14,  Oral) 

1.  Whai  are  the  prime  faotors  of  35,  of  42,  of  75  ? 

2.  What  are  the  simplest  factors  of  5xy,  of  6mn  ? 

3.  Express  3abc  as  the  product  of  two  factors  in  four  different  ways. 

4.  Give  two  common  factors  of  15ab  and  256c. 

5.  Find  the  values  of  3~  X  2",  10^  x  5^,  2*  x  5,  3^  x  2^  x  5. 

6.  Using  an  index,  express  100  as  a  power  of  10,  16  as  a  power  of 
2,  27  as  a  power  of  3,  625  as  a  power  of  5. 

7.  What  is  a  short  way  of  writing 

a+a  ?     a+a+a  ?     axal     axaXat     aaxtat 

8.  What  is  the  area  of  a  square  whose  side  is  6  inches  ?  whose 
side  is  x  inches  ? 


8  ALGEBRA 

9.  What  is  the  volume  of  a  cube  whose  edge  is  3  inches  ?  whose 
edge  is  m  inches  ? 

10.  When  a=4,  what  is  the  vaUie  of  a-  ?  of  2a  ?  What  is  their 
difference  ? 

11.  When  x=2,  what  is  the  difference  between  x^  and  3,r  ? 

12.  What  is  the  difference  between  "  x  square  "  and  "  twice  x  " 
when  a;=ll  ? 

13.  If  TO=10,  what  is  the  difference  between  the  square  of  3m  and 
three  times  the  square  of  m  ? 

14.  The  side  of  a  square  is  x  inches  and  of  a  smaller  one  is  y  inches. 
What  is  the  sum  of  their  areas  ?  What  is  the  difference  ?  What  do 
these  results  become  when  a;=10  and  y=G  ? 

15.*  If  x=6  and  y=2,  find  the  numerical  values  of 

Sx^,  x^-\-y,  x-\-y'^,  x"—y'^,  2x"^  — 3?/^. 

16.  Find  the  values  of  x^-\-x'^^x  for  the  following  values  of  x : 
x=\,  2,  3,0. 

17.  If  7/=4.r--7,  find  the  value  of  y  if  x=2,  if  x^3,  if  .r=2|. 

18.  The  unit  of  work  is  "  a  day's  work,"  that  is,  the  work  which 
one  man  can  do  in  one  day.  How  many  units  of  work  can  3  men  do 
in  5  days  ?     6  men  in  x  days  ?     m  men  in  n  days  ?     a  men  in  a  days  ? 

19.  If  a=3,  6=2,  c~  I,  find  the  quotient  when  a^-irb^+c^  is  divided 
by  2a-\-b—c. 

20.  Show  that  a;^4-26.r  has  the  same  value  as  9x^+24  when  x=2 
or  3  or  4. 

21.  If  a;=10  and  y=5,  how  much  greater  is  x"-\-y^  than  2xy  ? 

22.  li  d  represents  the  diameter  of  a  circle  and  c  the  circumference, 
we  know  that  c=3}d.     Find  c  when  (^=14.     Find  d  when  c=22. 

23.  If  ^  represents  the  area  of  a  circle  and  r  the  radius,  A^==B]r^. 
Find  A  when  r—7  ;   when  r=14. 

24.  By  arranging  the  factors  m  the  most  suitable  order,  find  the 
values  of  2* .  5^,  25^ .  4^,  125 .  2^. 

8.  Terms  of  an  Expression.  The  parts  of  an  algebraic 
expression  which  are  connected  by  the  signs  of  addition  or 
subtraction  are  called  the  terms  of  the  expression. 


ALGEBRAIC   NOTATION  9 

Thus,  the  expression  2a-\-3b  has  two  terms,  and  the  expression 
ix'^  —  Sxy  —  y^  has  three  terms. 

Quantities  which  are  connected  b}^  the  signs  of  multiplication 
or  division  are  not  different  terms. 

Thus,  iax  is  only  one  term,  so  is  -7-  • 

9.  Coefficient.  In  the  product  4.r,  4  is  called  the  coefficient, 
or  co-factor,  of  .r.  In  ab,  a  is  the  coefficient  of  6  and  b  is 
the  coefficient  of  a. 

The  4  is  a  numerical  coefficient,  and  the  a  or  h  is  a  literal 
coefficient. 

In  any  product,  any  factor  is  called  the  coefficient  of  the 
rest  of  the  product. 

Thus,  in  5abx,  5  is  the  coefficient  of  abx,  5a  is  the  coefficient  of  bx, 
and  5ax  is  the  coefficient  of  b. 

In  any  term  where  the  numerical  coefficient  is  not  stated, 
the  coefficient  1  is  understood. 

Thus,  in  xy  the  numerical  coefficient  is  1. 

10.  Addition  and  Subtraction  of  Like  Terms.  When  terms  do 
not  differ  or  differ  only  in  their  numerical  coefficients,  they 
are  called  like  terms. 

Thus,  2ab,  5ab,  ^ah  are  like  terms,  but  3a,  46,  6ab  are  unlike  terms. 

In  arithmetic,  quantities  which  have  the  same  denominations 
may  be  added  or  subtracted. 

Thus,  3ft.  +  4ft.- 2ft.  =  5ft. 

.S12-.$10  +  $8-$3  =  $7. 

We  cannot  add  or  subtract  quantities  of  different  denomina- 
tions, unless  we  can  first  reduce  them  to  the  same  denomination. 
Similarly,  in  algebra,  like  terms  may  be  added  or  subtracted. 

Thus,  5a+2a=la,5a—2a  =  3a, 

6afe4-5a&-  3a6=  1  \ab-3ab  =  8ab, 
8x2  -  6a;2+ a-c*  -  2a;2  =  1 7a;2  -  8.r2  =  9a;2. 

In  the  last  example  we  may,  of  course,  perform  the  operations  in  the 
order  in  which  they  occur  and  obtain  the  same  result. 


10  ALGEBRA 

Unlike  terms  can  not  be  added  or  subtracted. 

Thus,  the  stun  of  3a  and  56  can  be  indicated  in  the  form  3a-\-5b, 
but  they  can  not  be  combined  into  a  single  term  unless  the  numerical 
values  of  a  and  b  are  given. 


EXERCISE  4  (1-8,  Oral) 

1.  What  is  the  numerical  coefficient  of  each  term  in  the  expression 

2.  What  is  the  sum  of  the  numerical  coefficients  in 

2x^+3xy+x+y  ? 

3.  Which  are  like  terms  in  the  expression 

5a2+26— 3a+76— 4a2  ? 

4.  In  6bcy,  what  is  the  coefficient  of  bci/  1   oi  cy  1   oi  by  1  of  6  ? 

5.  What  is  the  sum  of  : 

(1)  2a,  3a,  4a.  (2)  5m,  |m,  fm. 

(3)  4a2,  1a%  5a\  (4)  3xy,\xy,  2xy. 

6.  If  a; =2,  find  the  numerical  value  of  the  sum  of  3.T-  and  4a;2 
in  two  different  waj'S  and  comj^are  the  results. 

7.  Simplify  the  expression  3a+86-l-2a+6+a+36   by   combining 
like  terms. 

8.  Express  in  as  simple  a  form  as  possible  : 

(1)  5m+7m— 3mi— 2to.  (2)  6a6— 3rt6-f  2a6— a6. 

(3)  3a;+a+2a;+a.  (4)  15a+106— 7a+46. 

9.*  Combine  the  like  terms  in  the  expression  : 

2x-^ly+bz—X'\-2y~~3z-\-Zx—4:y—z 
and  find  its  value  when  .t=3,  ^=5,  z=10. 

10.  If  a  =  6,  find  the  value  of  / 

15a2—  10a2_3a2+8rt— 5a— 20. 

11.  What  arithmetical  number  does  lOOa+lOft+c  represent  when 
a=2,  6=3.  c=4  ?     When  a^Q,  b=5,  c=7  ? 

12.  Simplify    2a:2+3.T+7— a;2+lla:— 2— a-^— 4a:-|-5. 

13.  A  man  walks  4.f  feet  East,  then  x  feet  West,  then  3.i:  feet  East 
then  5a;  feet  West.  How  far  is  he  now  from  the  starting  point  and  in 
what  direction  from  it  ? 


ALGEBRAIC    NOTATION  11 

14.  A  man  began  to  work  for  a  firm  on  a  salary  of  x  dollars  a  year. 
If  his  salary  for  each  year  was  double  the  salary  for  the  preceding  year, 
how  much  did  he  earn  in  four  years  ? 

15.  If  x-i-3x-\-5x  is  equal  to  72,  what  is  the  value  of  x  ?  How  do 
you  know  that  j^our  answer  is  correct  ? 

16.  Write  in  the  shortest  form  you  can 

17.  Find  the  average  of  (1)  10,  8,  15,  (2)  3a,-,  7.r,  5x. 

11.  Use  of  Brackets.  In  algebra  brackets  are  used  for  the 
same  purpose  and  with  the  same  meanings  as  m  arithmetic. 

In  finding  the  value  of  10+8+5,  we  may  perform  the 
additions  in  any  order,  but  if  we  write  it  10+ (8+5),  it  is 
understood  that  the  8  and  5  are  first  to  be  added  and  the 
sum  of  10  and  the  result  is  to  be  taken. 

Similarly,  a+(6+c)  means  that  the  sum  of  the  numbers 
represented  by  b  and  c  is  to  be  added  to  the  number  represented 
by  a. 

In  the  expression  7+5x2,  the  multiplication  is  to  be 
performed  first,  and  then  the  addition.  If,  however,  we 
wish  the  value  of  (7+5)  X  2,  we  must  add  the  7  and  5  before 
multiplying  by  2. 

Although  10+(8+5)  is  equal  to  10+8+5,  it  is  clear  that 
(7+5)  X  2  is  not  equal  to  7+5x2,  the  former  being  equal  to 
24  and  the  latter  17. 

When  a  is  to  be  multiplied  b}^  b,  the  sign  of  multiphcation 
is  omitted  in  the  indicated  product ;  so  when  (7+5)  is  to  be 
multiplied  by  2  we  may  write  2(7+5)  or  (7+5)2,  the  sign 
of  multiphcation  being  understood. 

It  is  thus  seen  that  one  of  the  Kses  of  brackets  is  to  indicate 
the  order  in  which  operations  are  to  be  'performed. 

Thus,  10— (7  — 3)  means  that  3  is  to  be  subtracted  from  7  and  the 
result  is  to  be  subtracted  from  10. 

If  the  values  of  the  letters  were  given,  what  operations  would  you 
perform  to  find  the  values  of  : 

a+(6+c),    a-(ft+c),    a-{b-c),    (a-b) -{c~d)'i 

The  pupil  should  recognize  that  3a^  is  not  the  same   as 


12  ALGEBRA 

(3a)  2.     The  latter  means  that  a  is  first  to  be  multipHed  by 
3  and  the  product  is  to  be  squared. 

Thus,  ifa  =  2,  3a2  =  3x4=12, 

and  (3a)2  =  3«x3a=6x6=36. 

Brackets  also  indicate  that  the  numbers  within  the  brackets 
are  to  be  considered  as  a  single  quantity,  thai  is,  they  are  used 
for  the  purpose  of  grouping. 

The  dividing  Una  between  the  numerator  and  denominator 
of  a  fraction  has  the  same  value  as  a  pair  of  brackets. 

Thus,  in j,  a-\-h  is  a  single  quantity  and  so  is  c  +  d.     The  fractional 

form  is  another  way  of  writing  {a-\-b)-^{c-\-d). 


EXERCISE  5  (1-18,  Oral) 
Perform  the  operations  indicated  : 


1. 

10-(6+3). 

2. 

8-(4-2). 

3. 

15-6^3. 

4. 

(15-6)^3. 

5. 

3(4  +  7-5). 

6. 

(10+2)(5-l). 

7. 

10+2x5-1. 

8. 

(16+12)^(6-2). 

9. 

7.r-(8.c— 4x). 

10. 

(6a— 2a)— (7a— 4a). 

1. 

(3.r+4x)-^7. 

12. 

(10a— 6a)^2a. 

3. 

(3a;+9a;)^(6a:-3a;). 

14. 

3(7-5)-2(8-6). 

15. 

43a;— (7.T— 4x)+2a;. 

16. 

x-{4:y^Zy-ly). 

-7. 

(56-46)(3z-22). 

18. 

6a— {la— 3a) 

8a— (6a+a) 
Indicate,  using  brackets  : 

19.  That  X  is  to  be  added  to  the  sum  of  p  and  q. 

20.  That  the  sum  of  x  and  y  is  to  be  added  to  w. 

21.  That  the  sum  of  a  and  b  is  to  be  multipUed  by  2. 

22.  That  the  difference  of  m  and  n,  where  m  is  greater  than  n,  is 
to  be  subtracted  from  a. 

23.  If  p  is  greater  than  q,  that  the  difference  of  p  and  q  is  to  be 
divided  by  the  sum  of  7n  and  n. 

If  a=10,  6=3,  c=2,  find  the  value  of  : 

24.*  8a-(26+c)-5(a-6).  25.     7(a-6-c)-3(a-26+c). 


ALGEBRAIC   NOTATION  13 

26.     (3a+2b-c){a-3b).  27.     a^--{-f^+c^-2{ab+bc+ca). 

a+36— c        2a— 36— 3  ^_      a—b       b—c  _  c~\-a 

2a—5b+2c  ~  a+6— 2c  '        "  *      be     "^  c+a      a+26  " 

30.     When  a— 6  and  6=3,  show  that 

5{a-6)+3(a+6)=2(4a-6). 


EXERCISE  6  (Review  of  Chapter  I) 

1.  If  a;  represents  a  certain   number,   what  does    4x   represent  ? 
Jx  ?    a;2  ?  3a;2  ? 

2.  If  a   represents   a   nvimber,    what    wiU    represent   5  times  the 
number  ?  y  times  the  number  ? 

3.  How  do  you  indicate  that  y  is  to  be  added  to  a;  ?      That  x  is 
to  be  subtracted  from  y  ? 

4.  Indicate  the  sum  of  x  and  y  diminished  by  a. 

5.  If  one  yard  of  cloth  costs  x  cents,  how  many  cents  will  10  yards 
cost  ?     How  many  dollars  ? 

6.  If  a  yard  of  ribbon  is  worth  y  cents,  how  much  is  a  foot  worth  ? 

7.  A  man  bought  an  article  for  x  dollars  and  sold  it  at  a  loss  of 
y  dollars.     What  did  he  sell  it  for  ? 

8.  If  I  paid  a  dollars  for  b  articles,  how  much  did  I  pay  for  each  ? 
What  would  c  articles  cost  at  the  same  price  ? 

9.  A  boy  has  a  dollars.     He  earns  b  cents  and  then  spends  c  cents. 
How  many  cents  has  he  left  ? 

10.  I  have  x  dollars.     If  I  pay  two  debts  of  a  dollars  and  b  dollars, 
how  much  shall  I  have  left  ? 

11.  If  one  number  is  x  and  another  is  5  times  as  large,  what  is  the 
sum  of  the  numbers  ? 

12.  If  one  part  of  10  is  x,  what  is  the  other  part  ? 

13.  A  man  worked  m  hours  a  day  for  6  days.      If  he  was  paid  $2 
per  Ifour,  how  much  did  he  earn  ? 

14.  How  far  can  a  man  walk  in  5  hours  at  4  miles  per  hour  ?     In  a 
hours  at  6  miles  per  hour  ? 

15.  A  man  bought  x  acres  of  land  at  a  dollars  per  acre  and  sold  it 
at  a  loss  of  b  dollars  per  acre.     What  did  he  sell  it  for  ? 

16.  What  number  is  15  greater  than  a;  ?      15  less  than  x  ? 

17.  By  how  much  does  a^  exceed  6'  when  a  —  I,  6  =  3  ? 


14  ALGEBRA 

18.  When  x=\,  y=2,  z  =  3,  what  are  the  values  of  x-{-y  —  z, 
2x+5y-3z,  lx-3y+z  ? 

19.  If  a  =3,  b  =  4,  c  =  0,  find  the  values  of  2ab,  4ac,  a^+b^+c^, 
5a2-262+4c2. 

20.  If  x=5  and  y=\,  what  are  the  values  of  Zx—2y,  6xy,  2x^—2y^, 
8x3-272/3? 

21.  If  a=  10,  6  =  5,  c  =  3,  find  the  values  of  a{b+c),  a{b-c),  a{b^~c^)' 
c{a^-b^). 

22.  What  is  the  sum  of  2x,  5x,  Ix  and  3a;  ? 

23.  Simplify  5a— 3a+lla+«— 10a. 

24.  What  is  the  average  of  20,  15,  0,  8,  12  ?  Of  2a,  3a,  la  ?  Of 
a,  b,  c,  d  ? 

25.  In  8  years  a  man  will  be  x  years  old.  How  old  was  he  8  years 
ago  ? 

26.  B  has  $20  more  than  A,  C  has  $20  more  than  B.  If  A  has 
$x,  how  much  has  C  ? 

27.  What  is  the  sum  of  the  numerical  coefficients  in  the  expression 
3a+^ab+ac+^ad  1 

28.  Express  1000  as  a  power  of  10  ;  32  as  a  power  of  2  ;  81  as  a 
power  of  3  ;    64  as  a  power  of  4. 

29.  Express  15,  105,  3a6,  35x^y,  as  the  products  of  simple  factors. 

30.*  How  long  will  it  take  me  to  walk  a  miles  at  3  miles  per  hour 
and  ride  b  miles  at  1 2  miles  per  hour  ? 

31.  A  farmer  buys  5  lb.  of  tea  at  x  cents  per  lb.  and  20  lb.  of  sugar 
at  y  cents  per  lb.  He  gives  in  exchange  7  lb.  of  butter  at  z  cents 
per  lb.     If  he  still  owes  something,  how  much  is  it  ? 

32.  If  I  buy  100  lb.  of  nails  at  a  cents  per  lb.  and  200  lb.  at  6  cents 
per  lb.,  what  is  the  average  cost  per  lb.  ? 

33.  What  is  the  total  number  of  cents  in  x  five-cent  pieces,  y.  ten- 
cent  pieces  and  z  half-dollars  ? 

34.  What  number  is  represented  by  lOOOa+lOOfe-f  lOc-f  d,  when 
a=l,  6  =  2,  c  =  3,  d  =  4  ?     When  a  =  4,  6  =  0,  c=l,  d  =  9  ? 

35.  When  a  =  -2    and   6  =  -l,  what  are  the  values  of  a-\-b,   o6,  — 

"+i',  a^+b^  a^-b^l 
ab 


ALGEBRAIC  NOTATION  15 

36.  If  A  can  do  a  piece  of  work  in  10  days  and  B  in  15  days,  what 
fraction  of  the  work  can  they  together  do  in  1  day  ?  Wliat  fraction 
if  A  could  do  it  in  x  days  and  B  in  y  days  ? 

37.  If  a  =  2p,  6  =  15,  c=10,  d  =  5,  find  the  difference  between 
(a+6)  — (c+d)  and  (a  —  b)  —  (c  —  d),  also  between  3(a+6)  — 5(c  — rf)  and 
5(a-rf)-3(6-c). 

38.  Wlien  x  —  7  and  y=l,  the  product  of  x+y,  x-\-2y,  x  —  5y  is  how 
much  greater  than  the  product  oi  x—y,  x—2y,  x—Sy"! 

1 


39.     If  a=  3,  find  tlie  value  of  1  + 


^     a+1 


CHAPTER  II 


SIMPLE  EQUATIONS 

12.  Idea  of  Equality.  In  weighing  an  article,  when  you 
see  that  the  scales  are  balanced,  what  conclusion  do  you 
draw  ?  If  a  5  lb.  bag  of  salt  is  placed  in  one  scale  pan,  what 
weight  (w)  must  be  placed  in  the  other  pan  to  restore  the 
balance  ?  What  must  w  be  to  balance  a  3  lb.  bag  a-nd  a 
4  lb.  bag  ? 

If  the  scales  are  balanced  in  each  of  the  following  figures, 
what  must  w  be  equal  to  ? 


Fio.  1. 


Pio.  2. 


Fio.  3. 


If  w-^4:  =  9,  as  in  fig.  1,  what  is  w  equal  to  ? 

If  w-\-w=  12,  as  in  fig.  2,  what  is  w  equal  to  ? 

If  w-\-3  =  5-\-2,  as  in  fig.  3,  what  is  iv  equal  to  ? 

If  the  scales  are  balanced  and  I  add  2  lb.  to  one  side,  what  else  must 
I  do  to  preserve  the  balance  ?  What,  if  I  take  away  3  lb.  from  one  side  ? 
If  I  double  the  weights  on  one  side  ?     If  I  halve  the  weights  on  one  side  ? 

13.  The  Equation,  When  a  certain  number  is  added  to 
10  the  result  is  27.     What  is  the  number  ? 

The  condition  expressed  in  this  problem  might  be  more 
briefly  shown  in  the  form  : 

10+ a  certain  number =27, 
or  in  the  form  10+? =27,  where  the  question  mark  stands  for 
the  required  number. 

Any  other  symbol  would  answer  the  same  purpose  as  the 


SIMPLE   EQUATIONS  17 

question  mark.     Thus,  if  x  represents  the  required  number, 
then  the  in-oblem  states  that 

10+a:=27. 

This  statement  is  called  an  equation  and  is  merely  a  short 
way  of  stating  what  is  given  in  the  arithmetical  problem 
preceding.  In  order  that  the  statement  may  be  true,  it  is 
easil}^  seen  that  the  symbol  x  must  stand  for  the  number  17. 

Ex. — When  a  number  is  multiplied  by  3,  and  5  is  sub- 
tracted from  the  product,  the  result  is  19  What  is  the 
number  ? 

Here,  if  x  standi^  for  the  number,  the  problem  states  that 

3a;-5=19. 
Before  the  5  was  subtracted  the  product  was  evidently  5  more  than 
19  or  19+5  or  24. 

If  3  times  the  number  is  24,  then  the  number  must  be  J-  of  24  or  8. 
The  solution  may  be  written  more  briefly  thus  : 

If  3a:- 5=  19, 

.-.      3a;=  19+5  =  24, 

a;=iof24  =  8. 

That  8  is  the  correct  value  for  the  number  is  shown  by  the 
fact,  that  when  it  is  multiplied  by  3  and  5  is  subtracted 
from  the  product,  the  result  is  19. 

14.  Solving  an  Equation.  The  process  of  finding  the  value 
of  X,  such  that  3.r~5=19,  is  called  "solving  the  equation," 
and  the  value  found  for  x  is  called  the  root  of  the  equation. 

EXERCISE  7  (Oral) 

1.  State  the  number  for  which  the  question  mark  stands  in  each 
of  the  following : 

(1)  5  +  ?=]2.  (2)  ?  +  12=20.  (3)  10-?=2. 

(4)  15=^8  +  ?.  (5)  40=62-?.  (6)  ?-8=42. 

2.  What  is  the  number  for  which  x  stands  in  each  of  the  following  : 
(1)  x+6=20.  (2)  8  +  a:=32.  (3)  25=a;+6. 

(4)  a;-15=7.  (5)  10-a;=8.  (6)  12=17-a;. 

C 


18  ALGEBRA 

3.  The  first  equation  in  Ex.  2  states  that  when  a  number  is  in- 
creased by  6  the  result  is  20.  What  does  each  of  the  other  equations 
say? 

4.  If  3  times  a  number  is  45,  what  is  the  number  ?  If  one-half 
of  a  number  is  16,  what  is  the  number  ?  If  n  stands  for  a  given  number, 
what  would  represent  ^  of  the  number  ?     4  of  the  number  ? 

5.  For  what  number  does  n  stand  in  each  of  the  following  equations : 

(1)  4n=24.     (2)  4n=10.     (3)  fw=36.     (4)  |w=14. 

6.  If  2x+5=ll,  what  is  the  value  of  2x  ?    of  .r  ? 

7.  If  3to — 2=13,  what  is  the  value  of  3»i  ?    of  m  ? 

8.  If  125+3=10,  what  is  the  value  of  i?)  ?    of  ;p  ? 

9.  If  fa:— 11  =  7,  what  is  the  value  of  'ix  ?    of  a;  ? 

10.  If  2(a;+4)  =  14,  what  is  the  value  of  a;+4  ?    oi  x  1 
Solve  the  equations  : 

11.  a;+10=30.  12.     3,t— 2=16.  13.     5y+2=n. 

14.  4<— 5=27.  15.  2h=11.  1«.  In— 4=24.. 
17,  3w+2=38.  18.  ix— 1=4.  19,  2«+l=4. 
20.  3n-|=5|.  21.  Jw+2=5.  22.  |a;-5=15. 
23.  3(.T+1)  =  30.  24.  5(a:-2)=45.  25.  A(a;-1)=3. 

15.  Axioms  used  in  Solving  Equations.  If  two  numbers  are 
equal,  what  is  the  result  when  the  same  number  is  added 
to  each  ? 

Thus,  if  x=G,  what  is  x-\-2  equal  to  ? 

What  is  the  result  when  the  same  number  is  subtracted 
from  two  equal  numbers  ;  or  when  each  is  multiphed  by  the 
same  number  ;   or  when  each  is  divided  by  the  same  number  ? 

Thus,  if  a;  =  10,  what  is  cc  — 4  equal  to  ?  What  is  3a;  equal  to  ?  What 
is  ^x  equal  to  ? 

The  preceding  conclusions  may  be  stated  thus  : 

(1)  //  the  same  number  be  added  to  equal  numbers,  the  sums 
are  equal. 

(2)  //  the  same  number  be  subtracted  from  equal  numbers, 
the  remainders  are  equal. 


SIMPLE   EQUATIONS  19 

(3)  //  equal  numbers  be  multiplied  by  the  same  number,  the 
products  are  equal. 

(4)  //  equal  numbers  be  divided  by  the  same  number,  the 
quotients  are  equal. 

These  statements  are  called  axioms,  or  self-evident  truths, 
and  are  used  in  solving  equations.  The  method  is  illustrated 
by  the  following  examples  : 

Ex.  1.— Solve  3a;— 7=35. 

Add  7  to  each  side,  .-.  Sx  — 7+7  =  35  +  7,  axiom  (1), 

.-.    3a; =42. 
Divide  each  side  by  3,         .".      x  =  ^  =  \4:,  axiom  (4). 

Ex.  2.— Solve  \x+2=M. 

Subtract  2  from  each  side,  .-.  ix+2  — 2  =  34— 2,  axiom  (2), 

ia;=32. 
Mviltiply  each  side  by  2,     .■.  a;=64,  axiom  (3). 

Ex.  3.— Solve  5a:— 3=2x+12. 

Add  3  to  each  side,  .'.  5x=2x-\-\5. 

Subtract  2x  from  each  side,  .'.  5x  —  2x=\5, 

3x=15. 
Divide  each  side  by  3,  .'.  x=5. 

The  object  of  the  changes  which  hav^e  been  made  in  these 
equations  is  to  get  the  quantities  containing  the  unknown 
{x)  to  one  side  and  the  remaining  quantities  to  the  other  side. 

The  unknown  quantities  are  usually  transferred  to  the  left 
side,  but  sometimes  it  is  better  to  transfer  them  to  the  right. 

Ex.  4.— Solve  3m+20=5m— 16. 

Add  16  to  each  side,  .'.  3m+36  =  5m, 

Subtract  3?^  from  each  side,  .".  3m+36  — 3m=5m  — 3m, 

36  =  2m, 
18  =  m  or  »n.=  18. 

16.  Verifying  the  Result.  If  we  substitute  18  for  m  in  the 
first  side  of  the  last  equation  we  get 

3m+20=3  X  18+20=74. 

If  we  substitute  in  the  second  side  we  get 

5m- 16=5x18-16=74. 

c  2 


20  ALGEBRA 

This  process  is  called  verifying  or  testing  the  correctness 
of  the  result.  If  the  root  obtained  is  the  correct  one,  the 
two  sides  of  the  equation  should  be  equal  to  the  same  number 
when  the  value  found  for  the  unknown  is  substituted. 

The  equation  is  then  said  to  be  satisfied. 

The  beginner  is  advised  to  verify  the  result  in  every  case. 
Verify  the  results  obtained  in  Ex.'s  1,  2  and  3. 

EXBRCISB  8  (Oral) 

1.  If  3a;=15,  what  does  x  equal  ?     What  axiom  is  used  ? 

2.  If  5a:-|-2:=17,  what  does  5x  equal  ?     What  axiom   is  used  ? 
What  does  x  equal  ?     What  axiom  is  used  ? 

3.  If  2^— 3  =  13,  what  does  2/ equal  ?     What  two  axioms  are  used  ? 

4.  If  hx-A—Q,  what  does  hx  equal  ?  What  does  x  equal  ?  What 
two  axioms  are  used  ? 

5.  If  ix=6,  what  does  }x  equal  ?  What  does  x  equal  ?  What 
two  axioms  are  used  ? 

What  is  the  value  of  x  in  the  following  equations  : 

6.  2a;=18.            7.     6.r=72.            8.     5.i=-16.  9.  3,r=6-9. 
10.     a;+20=-25.  11.     2x+l  =  15.   12.     3x-l=20.  13.  6.x+5==29. 
14.     ^x=8.           15.     |a;=-I2.         16.     ix^2h.  17.  ^x-==15. 

EXBRCISB  9 

Solve  the  following  equations,  giving  full  statements  of  the  methods. 
In  each  case  verify  the  result : 

2a;+5=27.  3.     4a;-5=51. 

4a;=a:+21.  6.     ^y=2y+80. 

|a;+5=50.  9.     6.x- +42= 9a;. 

Qa-3a=a-\-5.        12.     10.r+20=20. 
14.     20+6.r+5=50-3a;+ll. 
IG.     764.r-9=680;r+12. 
;hts  (iv)  together  with  a  20-gram  weight 
are  balanced  by  weights  of  50  grams  and  10  grams.     Express  this  by 
an  equation  and  find  the  weight  of  each  block. 

18.  If  17a:;— 11  is  equal  in  value  to  5a;+121,  what  is  the  value  of  x  ? 

19.  What  value  of  y  will  make  11?/+ 60  equal  to  20^-30  ?  J 


1. 

3a;+ 11=47.                  2 

4. 

3a;- 10=65.                  5 

7. 

7a-=60+3a;.                  8 

10. 

10a;+3  =  3a;+66.       11, 

^13. 

8to=36— 4>n. 

15. 

12a; -652= 7a; +428. 

17. 

Nine  blocks  of  equal  w( 

SIMPLE   EQUATIONS  21 

17.  As  we  have  already  sliowii,  an  equation  is  merely  the 
statement  in  algebraic  form  of  the  condition  given  in  an 
arithmetical  problem. 

The  solution  of  the  problem  is  thus  obtained  hy  solving  the 
equation. 

E3XERCISB  10 

State  the  condition  in  each  of  the  following  problems  in  the  form  of 
an  equation  : 

1.  What  must  be  added  to  33  to  make  50  ? 

2.  What  must  be  taken  from  90  to  leave  40  ? 

3.  What  is  the  number  which  when  doubled  is  36  ? 

4.  Five  times  a  certain  number  is  45.     What  is  the  number  ? 

5.  If  a  number  is  doubled  and  3  added,  the  result  is  25.  What 
is  the  number  ? 

6.  What  number  is  doubled  by  adding  27  ? 

7.  What  number  is  halved  by  subtracting  20  ? 

8.  If  8  is  subtracted  from  |  of  a  certain  number,  the  result  is  7. 
What  is  the  number  ? 

9.  Solve  the  equation  in  each  of  the  preceding  examples. 

18.  Problems  Solved  by  Equations.  The  following  examples 
will  illustrate  the  method  of  solving  problems  by  means  of 
equations  : 

Ex.  1. — When  I  double  a  certain  number  and  add  16,  the 
result  is  40.     What  is  the  number  ? 

Let  X  represent  the  required  number. 
Then  2x  is  the  double  of  the  number. 
Then  2x+16  is  the  double  with  16  added. 
But  the  problem  states  that  this  is  40, 

.-.  2a;+ 16  =  40, 
2a;  =  24, 

3^  =  12. 

Therefore  the  required   lumber  is  12. 

The  result  should  be  verified  by  showing  that  the  number 
obtained  satisfies  the  given  problem. 

Verifi6ation :  When  12  is  doubled  I  get  24  and  when  16  is  added 
I  get  40.     Therefore  tbc  result  is  correct. 


22  ALGEBRA 

Note  til  at  the  substitution  is  made  in  the  original  problem, 
not  in  the  equation.  There  might  be  an  error  in  writing  down 
the  equation  and  then  the  solution  obtained  might  satisfy 
the  equation,  but  would  not  necessarily  satisfy  the  given 
problem. 

Ex.  2. — The  number  of  pupils  in  a  class  is  33,  and  the 
number  of  boys  is  7  greater  than  the  number  of  girls.  J'ind 
the  number  of  each. 

Let  a;  =  the  number  of  girls, 

a;+7  =  the  number  of  boys, 
.'.  x+x-i-l  =  the  total  number, 
.-.  a;+x+7  =  33, 

2x=33-7  =  26, 
x=13,     .-.     a;+7  =  20, 
.'.  the  number  of  girls  is  13  and  the  number  of  boys  is  20. 
Verification  :     20+ 13  =  33,  20- 13  =  7. 

Ex.  3. — Divide  $100  among  A,  B  and  C,  so  that  B  may 
receive  3  times  as  much  as  A,  and  C  $30  more  than  B. 

Let  x=  the  number  of  dollars  A  receives, 

3x=      „         „         „  B 

:.   3a;+30=     „         „         „  C  „ 

.'.  they  all  receive  (rc+3a;+3a;+30)  dollars, 
.-.  a;+3a;+3a;+30=100, 
7a;+30=100, 
7ic=70, 
.-c=10, 
.-.  A  receives  $10,  B  $30  and  C  $60. 
Verify  this  result. 

19.  Steps  in  the  Solution  of  a  Problem.  The  examples 
which  have  been  given  will  show  that  in  solving  a  problem 
the  steps  in  the  work  are  usually  in  the  following  order  : 

(1)  Read  the  problem  carefully  to  see  what  quantity  is  to  he 
found. 

(2)  Represent  this  unknoivn  by  a  letter. 

(3)  //  there  be  more  than  one  quantity  to  be  found,  represent 
the  others  in  terms  of  the  same  letter. 


SIMPLE   EQUATIONS  23 

(4)  Express  the  condUion  stated  in  the  problem  in  the  form 
of  an  equation. 

(5)  Solve  the  equation  and  draiv  the  conclusion. 

(6)  Verify  the  solution  by  substitution  in  the  problem. 

On  referring  to  Ex.  1,  we  see  that  there  was  only  one  quantity  to 
be  found,  and  therefore  step  (3)  did  not  appear  in  the  sohition.  In 
Ex.  2  there  were  two  quantities  to  be  found,  and  when  we  represented 
the  number  of  girls  by  x,  we  could  represent  the  number  of  boys  by 

a;+7. 

The  pupil  is  advised  to  make  full  statements,  in  plain 
English,  as  to  what  the  unknown  represents. 

Thus,  in  Ex.  3  to  say,  let  x=^A,  or  let  a;=^'s  monej',  will 
only  lead  to  difficulties. 

Note. — The  examples  in  the  following  exercise  are  to  be  solved  by 
means  of  the  equation  and  the  results  should  be  verified  in  every 
case.  Although  the  answers  to  many  of  them  may  be  given  mentally, 
the  pupil  is  advised  to  give  complete  solutions,  so  that  he  may  become 
familiar  with  algebraic  methods. 


EXERCISES   11 

1.  If  37  is  added  to  a  certain  number,  the  sum  is  53.     What  is 
the  number  ? 

2.  If  27  is  subtracted  from  a  number,  the  result  is  5.     What  is 
the  number  ? 

3.  A  number  was  doubled  and  the  result  was  increased  by  27. 
If  the  sum  is  now  73,  what  was  the  number  ? 

4.  When  a  number  is  multiplied  by  7,  and  25  subtracted  from  the 
product,  the  result  is  59.     Find  the  number. 

5.  If  five  times  a  number  be  increased  by  6,  the  sum  is  the  same 
as  if  twice  the  number  were  increased  by  15.     Find  the  number. 

6.  What  number  if  trebled  and  the  result  diminished  by  36  gives 
twice  the  original  number  ? 

7.  If  you  add  19  to  a  certain  number  the  sum  is  the  same  as  if 
you  add  7  to  twice  the  number.     Find  the  number. 

8.  Five  times  a  number,  plus  19,  equals  nine  times  the  number, 
minus  41.     What  is  the  number  ? 


24  ALGEBRA 

9.     Two  miinhers  differ  by   11   and  their  sum  is   51.      Find    the 
tuimbers. 

10.  The  sum  of  two  numbers  is  47  and  one  exceeds  the  other  by 
15-     What  are  the  numbers  ? 

11.  ^'s  salary  is  three  times  5's  and  the  difference  of  their  salaries 
is  §1500.     Find  the  salary  of  each. 

12.  A  horse  and  carriage  are  worth  $.360.  The  carriage  is  worth 
twice  as  much  as  the  horse.     Find  the  value  of  each. 

13.  Divide  93  into  two  parts  so  that  one  part  wiU  be  27  less  than  the 
other. 

14.  The  length  of  a  rectangle  is  three  times  the  width.  The  peri- 
meter is  72  feet.     Find  the  sides. 

15.  ^  is  tmce  as  old  as  B.  In  10  j^ears  the  sum  of  their  ages  will 
be  41  years.     What  are  their  ages  ? 

16.  Divide  8500  between  A  and  B  so  that  A  wiU  receive  $20  more 
than  twice  what  B  will  receive. 

17.  The  sum  of  two  consecutive  numbers  is  59.  What  are  the 
numbers?  (Let  x  be  the  smaller  number,  then  .c+1  will  be  the 
greater. ) 

18.  Find  three  consecutive  numbers  whose  sum  is  150. 

19.  ^'s  age  is  twice  5's  and  C  is  7  years  older  than  .4.  The  sum  of 
their  ages  is  67  }-ears.     Find  the  age  of  each. 

20.  The  difference  betw-een  the  length  and  width  of  a  rectangle  is 
10  feet  and  the  perimeter  is  68  feet.     Find  the  sides. 

21.  Divide  $468  among  A,  B  and  C,  so  that  B  may  get  twice  as 
much  as  A,  and  C  three  times  as  much  as  B. 

22 .  A  railway  train  travels  m  miles  per  hour.  If  it  goes  from  Toronto 
to  Montreal,  a  distance  of  333  mUes,  in  9  hours  15  minutes,  what  is  the 
value  of  m  ? 

23.  A  line  20  inches  long  is  divided  into  two  parts.  The  length 
of  the  longer  part  is  J  inch  more  than  double  the  shorter  one.  Find 
the  lengths  of  the  parts. 

24.  What  value  of  x  will  make  5.r+6  equal  to  3x  -{-40  ? 

25.  If  5%  of  a  .sum  is  $48,  what  is  the  sum  ? 

26.  An  article  sold  for  $2-61  the  loss  being  10%.  What  was  the 
cost  ? 


SIMPLE   EQUATIONS  25 

27.  Divide  S145)(i  among  .-I,  B  and  C,  so  tliat  /)' will  get  three  times 
A's  share  and  C  will  get  SlOO  more  than  ^4  and  B  together. 

28.  A  has  five  times  as  much  money  as  B.  After  A  has  spent  S63 
he  has  only  twice  as  much  as  B.     How  much  has  B  ? 

29.  If  S20  less  than  |  of  a  sum  of  money  is  SIO  more  than  \  of  it, 
what  is  the  sum  ? 

30.  Three  bojs  sold  42  papers.  The  first  sold  \  as  many  as  the  third 
and  the  second  sold  \  as  many  as  the  third.     How  many  did  each  sell  T 

31.  The  sum  of  J-  of  a  number  and  i  of  the  same  number  is  55. 
What  is  the  number  ? 

32.  A  man  paid  a  debt  of  S4500  m  4  months,  paying  each  month 
twice  as  much  as  the  month  before.  How  much  did  he  pay  the  first 
month  ? 

33.  The  half,  third  and  fourth  parts  of  a  certain  number  together 
make  52.     Find  the  numbei-. 

34.  Divide  72  into  three  parts  so  that  the  first  part  is  h  of  the 
second  and  ^  of  the  third. 

35.  What  number  is  that  to  which  if  you  add  its  half  and  take 
away  its  third,  the  remainder  will  be  98  ? 

36.  If  3a=46c, 

(1)  Find  a,  when  6=10,  c=15. 

(2)  Find  b,  when  a=12,  c=  3. 

(3)  Find  c,  when  a=  8,  b=  \. 


EXERCISE  12  (Review  of  Chapter  II) 

1.  State  the  four  axioms  which  are  used  in  solving  equations. 

2.  Show    that    x^^lS    is    the    correct    solution    of    the    equation 
3x-7  =  2x+ll. 

3.  Determine  if  8  is  a  root  of  3(x+6)  =  .5(a;— 1). 

4.  Solve  (a)  5a;+3  =  2a;+9;    (fo)  l+2x  =  9-2x  ;    (c)  3a;-7  =  8-2a; ; 
{d)  7a;+l  =  9x-9;    (e)  lla;-l  =  5a;+l. 

5.  My  hotise  and  lot  cost  816,800,  the  house  costing  five  times  as 
much  as  the  lot.     Find  the  cost  of  each. 

6.  A  horse  and  carriage  cost  $520.     If  the  carriage  cost  $60  more 
than  the  horse,  what  did  the  horse  cost  ? 


26  ALGEBRA 

7.  Three  farmers  together  raised  2700  bushels  of  wheat.  A  raised 
three  times  as  much  as  B,  and  G  raised  twice  as  much  as  A.  How 
much  did  each  raise  ? 

8.  What  value  of  x  will  make  136— 3x  equal  to  172  — 9a;  ? 

9.  Where  r  is  the  radius  of  a  cii'cle  and  c  is  the  circumference, 
c  =  27rr,  where  7r  =  3y. 

(a)  Find  c,  when  r=   7  ;  when  r=42. 
(6)  Find  r,  when  c  =  88  ;  when  c=ll. 

10.  If  s=\ft-,  find  s  when  <  =  4  and/=32;  when  t=\Q  and /=  32-2. 

11.  In  a  company  of  98  persons,  there  are  twice  as  many  women 
as  men,  and  twice  as  many  children  as  women.  How  many  children 
are  there  ? 

12.  Six  boys  and  15  men  earn  $264  a  week.  If  each  man  earns 
four  times  as  much  as  each  boy,  how  mucli  does  a  boy  earn  in  a  week  ? 

13.  Five  times  a  certain  number,  increased  by  47  is  equal  to  eight 
times  the  number,  diminished  by  43.     What  is  the  number  ? 

14.  An  agent  charges  3  %  commission  for  collecting  an  accoimt. 
If  his  charge  is  $11-13,  what  was  the  amount  of  the  account  ? 

15.  Solve   (a)  •05x  =  4  ;  (b)  x+-04a;  =  208  ;         (c)  a;--06a;  =  235  ; 

(d)  x+5%a;  =  630. 

16.  If  6  is  the  base  of  a  triangle  and  h  is  its  height,  the  area  (o)  is 
given  by  the  formula  a=^bh. 

(i)  Find  a,  when  b=  8,  /i=  4. 
(ii)  Find  b,  when  a=  36,  /i=12. 
(iii)  Find  h,  when  a=176,  6  =  22. 

17.  The  sum  of  the  unequal  sides  of  a  rectangle  is  65  feet  and  their 
difference  is  15  feet     Find  the  area  of  the  rectangle. 

18.  If  6x  —  y  =  2x-\-y,  what  is  the  value  of  ^J  if  x  =  6  ? 

19.  For  what  ninnber  does  the  question  mark  stand,  if 

5x  +  i  =  3a;+? 
is  satisfied  when  a;  =  3  ? 

20.  If  4  %  of  a;  together  with  3  °;'  of  x  is  equal  to  35,  find  x. 

21.  State  a  problem  the  condition  of  which  is  expressed  by  the 
equation  3.1;  — 20  =  . r. 

22.  B  has  $10  more  than  A,  and  C  has  $20  more  than  B.  Together 
they  have  $190.     How  much  has  each  ? 

23.  A  turkey  costs  as  much  as  three  chickens.  If  2  turkeys  and 
3  chickens  cost  $7-20,  find  the  cost  of  a  chicken. 


SIMPLE    EQUATIONS  27 

24.  What  number  increased  by  f  of  its(ilf  is  ocjual  to  60  ? 

25.  Divide  $6400  among  A,  B  and  C,  so  tliat  B  will  get  $120 
more  than  A,  and  G  $160  more  than  A. 

26.  The  net  income  from  an  enterprise  doubled  each  year  for  five 
years.  If  the  total  net  income  for  the  five  years  was  $7750,  what  was 
the  income  for  the  first  year  ? 

27.  If  2ab  =  37nn, 

(1)  Find  a,  when  6  =  15,  m—  6,  n=    5. 

(2)  Find  b,  when  a=12,  m=  2,  n=     2. 

(3)  Find  m,  when  a=    ^,  6  =  6,  n  =     J. 

(4)  Find  n,  when  a=  ■d,b=-6,  m=-12. 

28.  Show  that  6  is  a  root  of  the  equation 

2(a;-l)(a;+2)  =  4(a;+3){x-5)  +  (a;-2)(.r  +  5). 

29.  The  area  of  the  United  States  is  4000  square  miles  more  than 
seventy  times  the  area  of  England.  If  the  area  of  the  United  States 
is  3,560,000  square  miles,  find  the  area  of  England. 

30.  Solve  and  verify  : 

(1)  6850+a;  =  27a;+350. 

(2)  lx+lx+ix  =  Sd80. 

(3)  1607a;+20=  1762a;- 11. 


CHAPTER  III 
POSITIVE  AND  NEGATIVE  NUMBERS 


20.  Arithmetical  Numbers.  In  the  diagram  the  hours  from 
12  noon  to   12  midnight  are  represented  on   the   horizontal 

hne,  and  the  temperature  at  each 
hour  is  shown  by  the  position 
of  a  point  on  the  corresponding 
vertical  Une. 

Thus  at  3  P.M.  the  tempera- 
ture was  61°,  at  7  P.M.  53°  and  at 
11  P.M.  55-5°. 

The  points  which  show  the  tem- 
perature for  each  hour  are  con- 
nected by  a  curve.  This  curve 
gives  a  picture  of  the  changes  in  temperature  during  these 
twelve  hours. 

These  cnanges  might  be  shown  by  a  column  of  figures, 
but  the  curve  exhibits  the  variations  in  temperature  more 
readily  to  the  eye.  We  can  see  at  a  glance  when  the  tempera- 
ture was  rising  and  when  falling,  at  what  hours  it  was  the 
same,  that  it  rose  or  fell  more  rapidly  during  certain  hours 
than  during  others. 

Here  we  say  that  we  have  represented  graphically  the 
changes  in  temperature,  and  the  curve  shown  is  called  a  graph. 


12  1    2    3    4    5    6    7    8    9  10  11  12 


BXE3RCISB   13  (1-8,  Oral) 
Using  tlie  diagram,  answer  questions  1-8. 

1.  What  was  the  temperature  at  1  p.m.,  at  4  p.m.,  at  10  p.m.  ? 

2.  At  what  hours  was  the  temperature  the  same  ? 

28 


POSITIVE    AND    NEGATIVE    NU31BENS 


29 


3.  What  was  the  highest  temperature  ?     What  the  lowest  ? 

4.  What  was  the  range  of  temperature  ? 

5.  Between  what  liours  was  it  rising  ? 

6.  How  much  did  it  rise  between  10  and  11  ?     How  much  did  it 
faU  between  6  and  7  ? 

7.  When  was  it  60°,  58°,  55°  ? 

8.  Between  what  hours  did  it  rise  most  rapidly  ?     When  did  it 
fall  most  rapidly  ? 

9.  The  percentage  of  games  won  by  a  baseball  team,  up  to  the 
beginning  of  each  month  of  the  playing  season,  was  as  follows  : 

June,  66;   July,  63  ;    Aug.,  60-5  ;    Sept.,  62  ;    Oct.,  61-5. 

Draw  a  graph  showing  these  changes. 

10.  A  boy's  height  in  inches,  for  each  year  from  the  age  of  7  to  thfe 
age  of  14,  was  44,  47,  50,  51,  52-5,  54,  56-5,  58.  Draw  a  graph  to 
illustrate  the  variations  in  his  height. 

21.  Negative  Numbers.  This  diagram  shows  the  average 
temperature  for  a  week. 

Thus  on  Monday  it  was  25° 
above  zero,  while  on  Thursday  it 
was  15°  below  zero. 

We  might  express  this  algebra- 
ically by  saying  that  on  Monday 
the  temperature  was  +25°,  and  on 
Thursday  it  was  —15°. 

The  number  ^-25  is  called  a 
positive  number  and  is  read  "  posi- 
tive "  25  or  "  plus "  25,  while 
number,  and  is  read  "  negative  "  25  or  "  minus  "  25 

A  negative  number  is  therefore  one  which  is  measured  on 
the  opposite  side  of  zero  from  a  positive  number. 


25   is   called   a   negative 


BXBRCISB   14 

1.  Using  algebraic  signs,  write  down  the  temperature  for  each  day 
in  the  diagram.     Also  read  the  temperature. 

2.  On  what  days  was  the  temperature  negative  ? 


30  ALGEBRA 

3.  How  much  higher  was  it  on  Monday  than' on  Thursday  ?  How 
much  lower  on  Tuesday  than  on  Saturday  ? 

4.  Tf  the  temperature  is  —30°  and  it  rises  40°,  how  much  will  it 
be  then  ?     If  it  had  fallen  10°,  how  much  would  it  have  been  then  ? 

5.  The  temperature  at  which  mercury  freezes  is  — 39°C.  What  does 
that  mean  ?  How  much  lower  is  it  than  the  normal  temperature 
of  the  blood  which  is  +37°C.  ? 

6.  Tf  the  price  at  which  a  certain  stock  sells  above  par  is  positive 
and  the  price  below  par  is  negative,  make  a  diagram  similar  to  the 
preceding,  showing  the  prices  of  a  certain  stock  for  a  week,  when  the 
record  was  as  follows  : 

Mon.,  4  above  par  ;    Tues.,  2  below  ;    Wed.,  1  above, 
Thurs.,  at  par;  Fri.,  3  below;  Sat.,  Ih  below. 

22.     Distances  measured  on  a  Horizontal  Line. 


P  B  O  AC 

I \ 1 \ 1 1 \ \ 1 1 1 

-5       -4       -3       -2        -1  0       +1       +2       +3       +4       +5 

On  this  diagram  the  distance  between  each  successive 
marking  represents  one  foot. 

What  is  the  length  of  OA  ?  of  OB  ?  In  what  respect  does  OA 
differ  from  OB  ?     How  might  we  use  signs  to  show  this  difference  ? 

It  is  usual  to  consider  measurements  made  to  the  right  as 
positive  and  to  the  left  as  negative. 

What  point  is  +5  feet  from  O  ?  What  one  is  —5  feet  from  0  ? 
If  a  point  moves  from  O,  4  feet  to  the  right  and  then  7  feet  to  the  left, 
how  far  is  it  then  from  0  1     Is  the  distance  positive  or  negative  ? 

We  thus  see  that  in  addition  to  the  numbers  of  arithmetic 
which  begin  with  zero  and  extend  indefinitely  in  one  direction, 
we  now  have  another  series  of  numbers  which  also  begin 
with  zero  and  extend  indefinitely  in  the  opposite  direction. 
In  each  series  all  integral  and  fractional  numbers  are  included. 

23.     Further  Examples  of  Negative  Numbers. 

(1)  A  man  has  property  worth  $100,  and  debts  amounting 


POSITIVE    AND   NEGATIVE   NUMBERS  31 

to  $60.  When  he  has  paid  his  debts  he  will  have  property 
worth  $40. 

Thus,  $100- $60=  $40. 

If,  however,  he  has  debts  amounting  to  $100,  when  these 
are  paid  he  will  have  nothing  left. 

Thus,  $100— $100=  $0. 

If  he  has  debts  amounting  to  $140,  when  he  has  paid  all 
he  can  he  will  still  owe  $40.  We  express  this  algebraically 
thus:  $100— $140=  —$40. 

In  the  first  case  we  say  that  his  net  assets  are  $40,  in  the 
second  they  are  zero,  and  in  the  third  they  are  minus  $40. 
When  we  say  his  assets  are  —  $40,  we  mean  he  is  $40  in  debt. 

It  will  be  seen  that  the  difference  in  meaning  between 
+40  and  —40  when  referring  to  dollars  is  practically  the 
same  as  the  difference  between  +40  and  —40,  when 
referring  to  degrees  of  temperature,  as  in  art.  21,  or  to  distances 
measured  in  opposite  directions  on  a  horizontal  hne,  as  in 
art.  22. 

(2)  If  a  man  gains  $20  on  one  transaction  and  loses  $15 
on  another,  what  is  the  net  result  ?  If  he  had  lost  $25  on 
the  second  transaction  what  would  have  been  the  net  result  ? 

If  we  attach  a  joins  sign  to  the  result  when  it  is  a  gain,  how 
may  we  indicate  a  loss  ? 

If  O  represents  a  sum  gained  and  L  a  sum  lost,  state  the  result  in 
each  of  the  following,  attaching  the  proper  sign  : 

1.    $30  G+  S20  G.         2.  $30  G+  $20  L.  3.    $30  L+  $20  L. 

4.   $30i+$20G.         5.   $40G+U0L.         6.    $20  (7+ $60  L. 

(3)  If  a  game  won  is  represented  by  +1,  then  —I  would 
represent  a  game  lost. 

In  a  series  of  games  I  find  that  my  record  is  :  won,  lost, 
lost,  won,  lost,  won,  lost,  won,  won. 

This  might  be  represented  thus  : 

+l-l-l  +  l-l  +  l-l  +  l  +  l  =  +5-4=  +  l. 

What  does  this  result  mean  ? 

Write  in  a  similar  way  the  following  record  :  lost,  lost,  won,  lost, 
won,  lost,  lost,  won.     Also  the  following  :   won,  lost,  drawn,  won,  won. 


32  ALGEBRA 

(4)  In  locating  points  on  the  earth's  surface,  the  distance 
in  degrees  north  of  the  equator  (north  latitude)  is  said  to  be 
positive,  and  south  of  the  equator  negative. 

Thus,  the  latitude  of  Toronto  is  -f  44°  and  of  Rio  de  Janeiro  is  —  23°. 
What  is  the  distance  in  degrees  of  latitude  between  these  two  cities  ? 

The  preceding  illustrations  show  that  a  positive  number 
differs  from  a  negative  number  in  direction  or  quality. 

Thus,  if  + 10  means  10  yards  measured  to  the  right  ;  or  *10°  east 
longitude  ;  or  10  games  won  ;  or  10  miles  a  boat  goes  up  stream  ; 
or  10  minutes  a  clock  is  fast ;  or  SIO  in  my  bank  balance  ;  or  10 
pounds  lifted  by  a  balloon  ;  what  would  — 10  mean  in  the  corres- 
ponding cases  ? 

24.  Signs  of  Operation  and  Signs  of  Quality.  The  numbers 
+25  and  —25  are  the  same  in  magnitude,  but  differ  in 
direction  or  quality. 

When  a  number  is  preceded  by  the  sign  +,  it  means  that 
the  number  is  taken  in  the  positive  direction  or  sense,  and 
when  preceded  by  the  sign  — ,  that  it  is  taken  in  the 
negative  direction. 

It  will  thus  be  seen  that  we  use  the  signs  +  and  —  with 
two  different  significations.  When  they  are  used  to  indicate 
the  operations  of  addition  or  subtraction,  they  are  called 
signs  of  operation.  When  they  are  used  to  indicate  direction 
or  quality,  they  are  sometimes  called  signs  of  quality. 

The  beginner  might  think  that  this  ambiguity  would  lead 
to  confusion,  but  he  will  find  that  such  is  not  the  case. 

When  we  read  a  quantity  like  —25,  we  should  say  "  negative 
25,"  but  this  is  not  followed  in  practice,  as  it  is  usually  read 
"  minus  25." 

When  no  sign  precedes  a  number,  it  is  understood  to  be  a 
positive  number. 

25.  Absolute  Value.     The   absolute   value   of   a  number  is 

its  value  without  regard  to  sign. 

Thus,  +  8  and  —  8  have  the  same  absolute  value. 


POSITIVE   AND   NEGATIVE   NUMBERS  33 


EXERCISE   15  (1-15,  Oral) 

1.  What  is  the  net  property  of  a  man  who,  {a)  has  $60  and  owes 
$47,  {b)  has  $40  and  owes  $50,  (c)  has  $65  and  owes  $65  ? 

2.  What  is  the  value  of,  (a)  $40-  $30,  (b)  $40-  $60,  (c)  $30-  $20, 
(d)  $20- $30,  (e)  $10- $0,  (/)  $0-S10  ? 

3.  The  temperature  was  —10°  at  6  p.m.  and  4°  at  10  p.m.  How 
many  degrees  did  it  rise  in  the  interval  ? 

4.  A  Uquid  whose  temperature  is  20°  is  cooled  through  30°.  What 
is  the  final  temperature  ? 

5.  A  vessel  sailed  on  a  meridian  from  latitude  15°  to  latitude  —5°. 
How  many  degrees  did  it  sail  and  in  what  direction  ? 

6.  What  is  the  distance  between  a  place  90  miles  due  east  of 
Toronto  and  another  60  miles  due  west  ? 

7.  I  am  overdrawn  at  the  bank  $20.  What  must  I  deposit  to 
make  my  balance  $100  ? 

If  — 20+x=100,  what  is  x  ? 

8.  What  would  a  negative  number  mean  in  stating  the  height  of 
a  tree  above  the  window  of  a  house  ?  The  height  above  sea  level  of  the 
bottom  of  a  weU  ? 

9.  A  man  buys  a  horse  for  $100  and  sells  him  for  $80.  What  is 
his  gain  and  his  gain  %  ? 

10.  A  man  travels  20  miles  from  A  and  his  friend  travels  —10  miles 
from  A.     How  far  are  they  apart  ? 

11.  What  is  the  rise  in  temperature  from  —30°  to  —10°  ? 
If  — 30+x=  — 10,  what  is  xl 

12.  What  is  the  distance  between  two  places  which  are  a  mUes 
and  b  miles  west  of  Montreal,  (1)  if  a  is  greater  than  b,  (2)  if  b  is 
greater  than  a  ? 

13.  Denoting  a  date  a.b.  by  +  and  B.C.  by  — ,  state  the  number 
of  years  between  these  pairs  of  dates : 

(1)  +1815  to  +1915.      (2)  -20  to  +75.  (3)  -65  to  -37. 

(4)  -120  to +60.  (5)  -200  to +200.       (6)  +1900  to  +1800. 

14.  Augustus  was  Roman  Emperor  from  —31  to  +14.  How  many 
years  was  he  Emperor  ?     What  is  the  difference  between  14  and  —  31  ? 

D 


34 


ALGEBRA 


15.  The  First  Punic  War  lasted  from  —264  to  —241.  How  Jong 
did  it  last  ?     What  is  the  difference  between  —241  and  —264  ? 

16.  A  boy  adds  15  marbles  to  his  supply,  gives  away  10,  buys  5 
and  gives  away  12.     How  many  has  he  thus  added  to  his  supply  ? 

17.  I  have  $a  in  the  bank.  If  I  issue  a  cheque  for  $6.  what  is  my 
balance  when  the  cheque  is  p^id  ?  If  a =40  and  6=50,  how  do  you 
interpret  the  result  ? 

18.  A  has  $50  and  B  has  $20.  A  owes  B  $10  and  B  owes  A  $40. 
How  much  will  each  have  when  his  debts  are  paid  ? 

19.  The  weights  of  two  pieces  of  iron  are  65  lb.  and  147  lb.  If  they 
are  attached  to  a  balloon  with  an  upward  pull  of  239  lb.,  how  would 
you  represent  the  combined  weight  ? 

20.  Represent  graphically  the  following  changes  in  the  price  of 
a  stock  : 


Month. 

July. 

Aug. 

Sep. 

Oct. 

Nov. 

Dec. 

Jan. 

Feb. 

Mar. 

Apr. 

Amount  above  par  . . . 

6 

1 

4 

5 

2 

3 

Amount  below  par  . . . 

2 

3 

1 

4 

EXERCISE   16  (Review  of  Chapter  III) 

1.  Using  signs,  express  the  results  of  the  following  transactions  : 

(1)  A  gain  of  $10  followed  by  a  loss  of  $15. 

(2)  A  loss  of  $12  followed  by  a  loss  of  $4 

(3)  A  loss  of  $8  followed  by  a  gain  of  $10. 

2.  What  is  the  difference  between  40°  and  -3°  ? 

3.  If  an  vipward  force  or  pull  is  positive  and  a  downward  force  is 
negative,  what  single  force  is  equal  in  effect  to  these  pairs  of  forces  : 

(1)   101b.,  -3  1b.  (2)  8  1b.,  -12  1b.  (3)   -7  1b.,  -2  1b. 

(4)   -9  1b.,  3  1b.  (5)  61b.,  -6  1b.  (6)  2a  lb.,  -alb.? 

4.  In  firing  at  a. target  each  hit  counts  5  and  each  miss  —3.  If  I 
fire  10  times  and  make  0  hits,  what  is  my  score  ?  If  I  make  only  2  hits 
what  is  my  score  ? 

5.  What  is  the  fall  in  temperature  from  27°  to  -  1 1°  ? 
If  27— «=  —11,  what  is  a;  ? 


POSITIVE    AND   NEGATIVE   NUMBERS  35 

6.  In  a  100  yards  handicap  race  A  has  3  yards  start  and  B  has 

—  3  yards  start.     What  do  these  mean  ?     How  far  has  each  to  run  ? 

7.  In  solving  a  problem  in  which  it  is  required  to  find  in  how 
many  years  A  will  be  twice  as  old  as  B,  I  get  the  answer  — 10.  What 
does  this  answer  mean  ? 

8.  Find  the  average  noon  tesoperature  for  a  week  in  which  the 
noon  temperatures  were  :  20°,  10°,  15°,  0°,  4°,  —6",  —15°. 

9.  A  train  was  due  at  10  minutes  to  3.  How  many  minutes  before 
three  did  it  arrive  if  it  was  half  an  hour  late  ? 

10.  A  man  travels   8   miles,   then     —6  miles,   then   4  miles,   then 

—  II  miles.       How  far  has    he  travelled  ?      How  far  is   he   from  the 
starting  point  and  in  what  direction  (positive  or  negative)  from  it  ? 

11.  Egjrpt  was  a  Roman  province  from  —30  to  616.  How  many 
years  was  this  ?     What  is  the  difference  between  616  and  —  30  ? 

12.  The  daily  average  temperature  for  14  days  were  :  6°,  5°,  0°, 
-4°,  2°,  -6°,  -2°,  0°,  5°,  1°,  -1°,  -6°,  -3°,  3°.  Show  these 
variations  by  means  of  a  graph. 

13.  If  a  gain  of  a  dollar  be  the  positive  unit,  what  will  represent 
a  loss  of  83-50  ? 

14.  The  record  of  a  patient's  temperature  for  each  hour  beginning 
at  12  noon  was  :  100°,  100-5°,  102°,  101°,  104°,  101-5°,  99-5°,  98°, 
97- 5"^,  97°.  Represent  these  changes  graphically,  taking  two  spaces  on 
the  vertical  line  to  represent  one  degree. 

15.  If  the  normal  temperature  of  the  body  is  98-5°,  write  the  record 
in  the  preceding  question  vising  positive  and  negative  signs. 

16.  The  minimum  temperatures  for  the  first  15  days  of  December 
were:  26°,  22°,  14°,  25°,  21°,  18-5°,  13°,  7-5°,  11°,  6°,  -4°,  -6°,  -1°, 
10°    12-5°.     Make  a  chart  to  show  these  variations. 


d2 


CHAPTER  IV 

ADDITION  AND  SUBTRACTION 

26.  Addition  of  Positive  Quantities.  What  is  the  result  of 
combining  : 

(1)  A  gain  of  $20  with  another  gain  of  $10  ? 

(2)  A  measurement  of  5  feet  to  the  right  with  another  of 
3  feet  to  the  right  ? 

(3)  A  rise  in  temperature  of  10°  with  a  rise  of  8°  ? 

(4)  6  points  won  with  4  points  won  ? 

As  explained  in  Chapter  III.,  we  will  consider  all  of  these 
to  be  positive  quantities,  and  we  might  show  this  by  attaching 
the  positive  sign  to  each. 

We  might  write  these  four  questions  as  problems  in 
addition,  thus  : 

+  $20  +5  feet  +10°  +   6  points 

+  $10  +3  feet  +8°  +   4  points 


+  $30  +8  feet  +18°  +10  points 

Similarly,  the  sum  of  6a  and  4a  is  10a,  and  the  sum  of  2x^,  5x^  and 
6a;*  is  13x*. 

Here  we  have  not  prefixed  any  sign,  and  when  that  is  the  case  the 
positive  sign  is  imderstood. 

We  see  then  that  the  sum.  of  any  number  of  positive  quantities 
is  always  positive. 

27.  Addition  of  Negative  Quantities.  W^e  might  change  the 
data  of  the  four  qv;estions  in  the  preceding  article  so  that 
all  the  qviantities  would  be  negative. 


ADDITION    AND   SUBTRACTION  37 

Thus,   the  first  might  be  changed  to   — .     "  What  is  the 
result  of  combining  a  loss  of  S20  Avith  a  loss  of  $10  ? 

Read  the  other  three  questions  making  similar  changes. 
What  would  now  be  the  answer  to  each  question  ? 
As  problems  in  addition  they  would  now  appear  thus  : 

-S20  -5  feet  -10°  -   6  points 

-SIO  -3  feet  —    8°  -   4  points 


-  S30  -  8  feet  -  18°  -  10  points 

Similarly,  the  sum  of  —7x  and  —5x  is  —12a;,  and  the  suna  of  —5a', 
—  2a',  —a'  and  — 6a'  is  —14a'. 

Thus,  the  sum  of  any  number  of  negative  quantities  is  negative, 
and  is  found  by  adding  their  absolute  values  and  prefixing  the 
negative  sign  to  the  result. 


BXEROISB  17  (Oral) 
State  the  results  of  the  following  additions  : 


1. 

+  $3 

-  7 
-13 

Add  10a:, 
Add  ^m, 

2.     -SIO 

-S  8 

10. 
12 

3.     -12° 
-10° 

4. 

8. 

—  )n'^, 
-Sy, 

3  yd. 
5  yd. 

5. 

6.     4a2 

12x,  I5z. 

fm,  |m,  |to. 

7.     -lOx'-y 

—  5x^y 

—^abc 
—  'labc 

9. 
11. 

Add  -3m\ 
Add  —I2y, 

-7m  2. 

-ny. 

28.  Compound  Expressions.  An  expression  of  one  term  is 
frequently  called  a  simple  expression,  while  one  of  more  than 
one  term  is  called  a  compound  expression. 

Thus,  2a,  3x^y,  labc  are  simple  expressions,  and  2a  +  Zb,  5x  — 3m  +  a* 
are  compound  expressions. 

29.  Addition  of  Compound  Expressions.  In  arithmetic  if  we 
wish  to  add  two  or  more  com]:)ound  expressions,  we  write 
them  under  each  other,  with  the  like  denominations  in  the 
same  column. 


38  ALGEBRA 

We  proceed  in  a  similar  way  in  algebra,  writing  like  terms 
in  the  same  column. 

In  arithmetic.  In  algebra. 

2  yd.  1  ft.     6  in.  2a+  6+  6c 

3  vd.  1  ft.     4  in.  3a+  5+  4c 


5  yd.  2  ft.   10  in.  5a+26+10c. 

If  the  like  terms  are  not  in  the  same  order,  they  must  be 
properly  arranged  for  addition. 

Ex.— Add  5x+Sy-2z,  4y—5z-{-3x,  —^z-\-4x+y. 

Here  the  problem  might  be  written  thus  : 
5x+32/—   2z 
3x+42/—   5z 
4x+  y—  3z 


Smn=12a;+82/— lOz. 

EXERCISE   18  (1-6,  Oral) 
Add: 
1.     3  ft.  2  in.  2.     3a;+22/  3.     5a— 116 

5  ft.  3  in.  5x+Zy  2a-  3b 


6hr.  lOmin.  11  sec.       5.     5a— 36+2c  6.     —  a—3b+7c 

5  hr.  12  min.    3  sec.  2a— 46+3c  —3a—  6+4c 

2  hr.  15  min.  20  sec.  5a—  6+  c  —5a— 26+  c 


7.*  a+b—c,  26  — 3c-fa,  36+5a  — lie. 

8.  5x^—lx+Q,  3-5x+x^-,  -2x+ix-. 

9.  2a— 36,  3a— 26,  4a,  -6,  a— 6. 

10.  a+26,  6+2c,  c+2a,  a+6+c. 

11.  a-26+c— 3d,  c— 56— d+2a,  -6+c— c^+a. 

12.  5x—3y,  —2y+z,  iz—y+3x. 

13.  ia-lb,  fa- 16,  ^a— #6,  |a— 16. 

14.  a2+262-3c2,  562-c2+2a2,  Za^+b^-2c~. 

15.  a+6+c,  6+c+c?,  c-\-d+a,  d+a+6. 

16.  ^^x+ly-lz,  §x+3y-|z,  x+y-fz. 


ADDITION   AND   SUBTRACTION 

30.     Addition  of  Quantities  with  Unlike  Signs. 

What  is  the  result  of  combining  : 

(1)  A  gain  of  $20  with  a  loss  of  $10  ? 

(2)  A  gain  of  $5  with  a  loss  of  ^15  ? 

(3)  A  loss  of  $8  with  a  gain  of  $6  ? 

(4)  A  loss  of  $7  with  a  gain  of  $12  ? 

These  might  be  written  as  problems  in  addition,  thus  . 

+  $20  +$5  -$8  -$  7 

—  $10  —$15  +$6  +$12 


39 


+  $10  -$10  -$2  +$5 

It  is  thus  seen,  that  when  we  add  two  quantities  differing 
in  sign,  the  sum  is  sometimes  positive  and  sometimes  negative. 

When  is  it  positive  and  when  is  it  negative  ? 

How  is  the  numerical  part  of  the  sum  found  when  the 
signs  are  alike  ?     How  is  it  found  when  the  signs  are  different  ? 

The  answers  to  these  questions  might  be  combined  mto 
the  following  rule  : 

When  the  signs  are  alike,  the  sum  is  found  by  arithmetical 
addition,  and  the  common  sign  is  affixed  ;  when  the  signs  are 
different,  the  sum  is  found  hy  arithmetical  subtraction,  and  the, 
sign  of  the  greater  is  affixed. 

Ex.  1. — Find  the  sum  of  6  and  —8. 

Here  the  result  is  —2,  since  the  difference  between  8  and  6  is  2  and 
the  one  with  the  greater  absolute  value  is  negative. 

If  there  is  doubt  in  any  case,  it  is  advisable  to  make  the  problem 
concrete  by  substituting  for  +6,  a  gain  of  $6  and  for  —8,  a  loss  of  $8, 
when  the  result  will  at  once  be  evident. 

Ex.  2. — Find  the  sum  of  5a,  —8a,  —7a,  6a,  —2a, 
The  sum  of  the  positive  quantities  is  11a. 
The  sum  of  the  negative  quantities  is  —17a. 
The  sum  of  11a  and  —17a  is  —6a, 
.'.  the  requii'ed  sum  is  —6a. 

They  might  also  be  added  in  the  order  in  which  they  coma. 
Thus,  the  sum  of  5a  and  —8a  is  —3a,  of  —3a  and  —7a  is   —10a, 
of  —  10a  and  6a  is  —4a,  of  —4a  and  —2a  is  —6a. 


40 


ALGEBRA 


Ex.  3.— Add  3a— 116+5C,  66— 5a,  56— c+a. 

Write  the  expressions  in  colunuis  as  already  explained. 

a=6=c=l 
3&-116  +  5C      I      =  -3 


-■5a+   66 
a+   56—  c 


=  +1 

=  +5 


-a  +4c  =  +3 

The  sum  is  —  a+4c  or  4c— a,  the  sum  of  the  second  column  being 
zero. 

We  may  check  the  result  by  substituting  particular  numbers 
for  the  letters.  Thus,  if  we  substitute  unity  for  each  letter 
the  first  quantity  becomes  3—11+5  or  —3,  the  second  is  +1, 
the  third  is  +5,  and  the  sum  (— a+4c)  is  +3.  Since  the  sum 
of  —3,  +1  and  +5  is  +3,  we  assume  that  the  work  is  correct. 


1. 

+6  ft 
-3  ft. 

2. 

6. 

10. 

EXERCISE 

-$10 

+  $27 

19  (1- 
3. 

7. 
11. 

12,  Oral) 

+  10  lb. 
-151b. 

4. 

8. 
12. 

-50° 

+40° 

5. 

-3 

7 

3a— 26 
5a +36 

5a2 
-3a- 

2x-ly 
-^x+2y 

-  Ix 
Ux 

— 8a6 
8a6 

9. 

a+66 
a— 66 

3a+26— c 
2a— 26+c 

13.  3j;+5t/,  2x—Zy,  —^x-y,  Qx-4.y.     (Check.) 

14.  5m,  — Gw,  —7m,  8m,  —9m,  10m. 

15.  2a+36— 5c,  6a— 46+c,  3a+26+4c.     (Check.) 

16.  3a— 56,  46-3c,  4c— 3a,  a+6+c.     (Check) 

17.  lx+y—\z,  ^x—ly+\z,  x-\-ly-{-z.     (Check.) 
18.*  a— 26+3c,  6— 2c+3rf,  c—M+2a,  6— 2c— 3a. 

19.  Zx+5y—2z,  2.r— 3y+42,  4.x+y—5z,  6.r+2?/+32. 

20.  3a6— 4ac+56c,  5ac— 46c— 2a6,  36c— a6— oc. 

21.  6a2-5a6+62,  3a2+7a6-262,  a~-ab+b\ 

22.  2a;2-3?/2+422,  5y^-%z^^x^,  2z^+2y^-3x\ 

23.  ^a-lb+lc  U-^b-^c. 


ADDITION    AND   SUBTRACTION  41 

24.  If  the  sum  of  13x— 7,  2x-+5  and  6— 4x  is  48,  find  the  value  of 
X  and  verify. 

25.  If  the  sum  of  x— 6,  3x— 6  and  5x—Q  is  the  same  as  the  sum  of 
12— a;,  12— 3.r  and  12—5.1%  find  x  and  verify. 

31.  Indicated  Additions.  If  we  wish  to  use  the  sign  of 
addition  to  indicate  that  6  is  to  be  added  to  a,  we  ^\Tite  it 
thus  :    a-\-b. 

Similarly,  if  we  wish  to  indicate  the  sum  when  —7  is  added 
to  11,  we  write  it  ll+(  — 7),  the  negative  quantity  being 
enclosed  m  brackets. 

To  find  the  value  of  ll+(— 7),  we  must  add  11  and  —7, 
which  is  done  by  subtracting  their  absolute  values  and 
prefixmg  the  positive  sign. 

Thus,  ll  +  (-7)  =  ll-7  =  4. 

Similarly,  6a+(  — 3a)  =  6a— 3a=3a, 

5m-|-(  — 3m)4-(  — m)  =  5m— 3m— m  =  m, 
and  a+{  —  b)=a—b. 

We  thus  see  that,  to  add  a  negative  quantity  is  the  same  as 
to  subtract  a  positive  quantity  of  the  same  absolute  value. 
If  we  'oish  to  simphfy  a  quantity  like 

(3a-2&)+(2a-36), 

we  may  write  2a— 3b  under  3a— 26,  and  add  in  the  usual  way, 
or  we  may  remove  the  brackets  and  say  that  the  quantity 

=3a— 26+2a— 36, 

=5a— 56,  when  the  like  terms  are  collected. 

EXERCISE  20  (1-12,  Oral) 
Simplify : 

1.     -3+4.  2.     10+(-6).  3.     3+(-4). 

4.     (-2)+(-3).  5.     5a4-(-4a).  6.     76+(-46). 

7.     — 8a+(+7a).  8.     —5ab+{—2ab).  9.     9x-+{  —  3x^). 

10.     —p+{-3p).  11.     (-3m)+(-8wi). 

12.     _a+(_2a)  +  (-3a). 

13.*  l0xy+(-Zxy)  +  {-4^y)-xy+i-5^y)- 


42  ALGEBRA 

14.  -6  +  (-26)  +  (-36)  +  (-46)  +  106 

15.  -5/j2-|-3p2_^(_2^2)_^8^2, 

16.  (2/rt+3H.)  +  (5//i— ;0  +  (3m-5«). 

17.  (6.i--f3(/-43)+(a;+2^-;:)  +  (^+,--7a:). 

18.  a  +  (-6)4-&  +  (-c)+c+(-«). 

19.  a;+(a— 26+c)  +  (6— 2c+a)+(c-2aH-6). 

20.  When  —20  is  subtracted  from  10,  the  difference  is  30.  Show 
that  this  is  true  by  adding  the  difference  to  the  quantity  which  was 
subtracted. 

21.  Show  by  addition,  that  when  2a — &+5c  is  subtracted  from 
3a— 46 -|- 3c  the  remainder  is  a  — 36— 2c. 

22.  Solve  (2x+3)  +  (3a;-5)-|-(5.r-l)=57.     (Verify.) 

23.  Solve   (8x-7)+(-4x-3)  =  (-5.x--7)+(7a;-2).     (Verify). 

32.  Subtraction  is  the  Inverse  of  Addition.  To  subtract  4 
from  7  is  equivalent  to  finding  the  number  which  added  to 
4  will  make  7. 

Thus  every  problem  in  subtraction  may  be  changed  into  a 
corresponding  problem  in  addition. 

If  we  wish  to  subtract  —4  from  7,  we  enquire  what  number 
added  to  —4  will  make  7.  We  might  make  the  problem 
concrete  by  finding  what  must  be  added  to  a  loss  of  $4  to 
result  in  a  sain  of  S7,  and  the  answer  is  evidently  a  gain  of 
$11. 

.'.   when  —4  is  subtracted  from  7  the  remainder  is  11. 

Thus,     7  less— 4=11,      because  —4+ 11  =  7. 

-  10  less  -3  =   -7,  because  -3  + (-7)=  —10. 
66  less  -46=106,    because  —46+106  =  66. 


EXERCISE  21   (Oral) 
What  must  be  added  to 

1.  A  gain  of  SIO  to  give  a  gain  of  $15  ? 

2.  A  gain  of  $8  to  give  a  gain  of  $3  ? 

3.  A  gain  of  $5  to  give  a  loss  of  $4  ? 


ADDITION    AND    SUBTRACTION  43 

4.  A  loss  of  S6  to  give  a  gain  of  $3  ? 

5.  A  loss  of  $20  to  give  a  loss  of  $15  ? 

6.  A  loss  of  $5  to  give  a  loss  of  $8  ? 

7.  8  to  give  12.  8.  10  to  give  3. 

9.  —8  to  give  2.  10.  —8  to  give  —2. 

11.  7x  to  give  3a:.  12.  3a  to  give  —5a. 

13.  5a;-  to  give  —2x^.  14.  —8a;  to  give  —10a;. 

15.  3abc  to  give  2abc.  16.  —  6y-  to  give  —9y-. 

17.  —5t  to  give  —41.  18.  a  to  give  —a. 

33.     Rule    for  Subtraction.  Examine    the    following    sub- 
tractions : 

(1)     9  (2)       9  (3)  -  9          (4)  -9 

3  -3  3                 -3 


6  12  -12  -6 

In  (I)  the  result  6  might  have  been  found  equally  well  by 
adding  —3  to  9,  in  (2)  by  adding  +3  to  9,.  in  (3)  by  adding 
—  3  to  —9,  and  in  (4)  by  adding  -\-3  to  —9. 

Thus  we  see  that  the  problems  might  have  been  re-written 
as  problems  in  addition  by  changing  the  sign  of  the  quantity 
to  be  subtracted. 

We  would  then  have  these  problems  in  addition  : 

(1)       9        (2)        9       (3)  -  9        (4)  -9 
-3  +3  -  3  +3 


6  12  -12  -6 

We    have    therefore    the    following   rule    for    subtraction : 
Change  the  sign  of  the  quantity  to  be  subtracted  and  add. 

To  subtract  compound  expressions,  apply  the  rule  to  the 
like  terms  of  the  expressions. 

Thus,  to  subtract     6a— 36  + 2c     change  to     6a  — 36  + 2c     and  add 
3a+46-6c  -3a-46  +  6c 

3a -76  + 8c. 


44 


ALGEBRA 


EXERCISE  22 

Re-write  the   following    problems   in   subtraction   as   problems   in 
addition  and  find  the  result : 


1. 


5. 


25 

2. 

-11 

13 

+  3 

16a2 

6. 

56 

20a2 

86 

10a 

■  8a 


4. 


20a; 
■  4x 


-3x^y 


8.     -23m2 


9.     3a+76        10.         6x—Sy       11.     5TO+4yi      12.     6a;2— 5x+2 
2a_26  — 2a;+4y  5m— An  3x2— 2x— 3 


13.  Subtract  a— 26  +  3c  from  3a— 46+c. 

14.  Subtract  5a;2-llx+4  from  3a;2-2j;+5. 

As  soon  as  possible  the  pupil  should  learn  to  subtract, 
without  actually  changing  the  signs,  but  by  making  the 
change  mentally. 


EXERCISE 

23  (1-8,  Oral) 

Subtract : 

1.         8 

2. 

-9 

3. 

-12 

4. 

-7a 

-3 

4 

-  6 

2a 

5.     -8a; 

6. 

—7m 

7, 

—  4a6c 

8. 

-9i 

-3a; 

10. 

—2m 

11. 

lla6c 

12. 

-H 

9.     3a;+42/ 

2a -36 

7a:2-5 

6m— 3n 

x—2y 

3a +26 

3a;2-7 

-3m+6Ji 

13.  2m+4n— 3^  from  5m—3n+6p,  and  verify  the  work  by  addition. 

14.  Find    the    remainder    when    6a— 46  — 5c    is    subtracted    from 
2a-}-36-llc.     (Verify.) 

15.*  Subtract  5a  — 36  from  the  sum  of  2rt— 6  and  6a— 46. 

16.     Subtract  a— 26+c  from   6a— 6  + 5c   and   from   the  remainder 
subtract  3a+6— c. 


ADDITION    AND   SUBTRACTION  45 

17.  Subtract  the  sum  of  Sx-— 5a;+6and5a;2+4a;— 3  from  5x^—x-\-Z. 
Check  when  x=l. 

18.  What  must  be  added  to  2??i+3n— 4p  to  give  5m  —  n—2p  1 
(Verify.) 

19.  By   how   much   is   Ta^— 15a— 11    greater   than   Sa^— lla4-4? 

20.  By  three  subtractions  simplify 

(6a+106-c)-(3a+46-2c)-(a-36+4c)-(2a+76-3c). 

21.  Subtract  the  sum  of  2p—5q—2r,  —p-\-Zq—2r  and  4^+65'— 4r 
from  the  sum  of  3^— 4g'+5r,  3g'— 4r4-5/>  and  3r— 4p+55'. 

22.  Subtract  2a  — 36+ 5c  from  zero. 

23.  What  is  the  excess  of  15  over  10  ?     8  over  —  2  ?     —4  over 

—  11  ?     a+6  over  a— 26  ?     2x-—bx+2  over  2a;2— lla;+7  ? 

24.  Add  a2+2a— 5  to  the  excess  of  2a2_4a+3  over  a^- 3a+10. 

25.  Subtract  the  sum  of  a— 36+c,  6— 3c+a,  and  c— 3a4-6  from 
zero. 

26.  What  must  be  added  to  the  sum  of  x^—5x,  2x-— 3a;+4  and 
6x— 3a;2  so  that  the  result  will  be  unity  ? 

27.  From  2x—Zx^-^5—x'^  take  3-ll.r3— 5a;2— 6.r. 

34.     Indicated   Subtractions.     If   we   wish   to   indicate  that 

—  3  is  to  be  subtracted  from  9  we  write  it:  9— (—3).     From 
the  preceding  this  is  at  once  seen  to  be  equal  to  9+(+3)  =  12. 

Also,  _8-(-5)=  _8  +  (  +  5)=  -3. 

—  3a— (  — 5a)=  —  3a+5a=2a. 
a-(-6)=   a  +  {  +  h)  =  a-{-b. 

We  thus  see  tliat  to  subtract  a  negative  quantity  is  the  same 
as  to  add  a  positive  quantity  of  the  same  absolute  value. 

Similarly,  (3a+46)-(2o-36)  =  (3a+46)  +  (-2o+36), 

=  3a+46  — 2a  +  3fe, 

=  a+7b. 
Also  a  — (6  +  c)=a  +  (  — 6— c)=a— 6— c, 

and  o  — (6  — c)  =  a  +  (  — 6  +  c)  =  a— 6  +  c. 

Thus,  brackets  which  are  preceded  by  a  minus  sign  may  be 
removed  if  the  signs  of  all  the  quantities  within  the  brackets  be 
changed. 


46  ALGEBRA 

Ex.— Simplify  5x-3y+4z- {Sx—2y+2z). 

5x  —  3y-\-  42 
We    may    consider   this    as    an   ordinary  problem  3x  —  2y4-2z 

in  subtraction  and  proceed  in  the  usual  way 

2x-   y  +  2z 

We  may,  however,  remove    the    brackets   using  the  rule  and  then 
collect  the  like  terms,  thus  : 

The  expression  —5x—3y-\-4:Z—3x-\-2y  —  2z, 

=  2x-y  +  2z. 

EXERCISE  24  (1-10,  Oral) 
Simplify 

1.  io_(_3).  2.     -5-(-6).  3.     -la-i-ia). 

4.  8x-{-3x).  5.     {-2ni)-(-3m).         6.     -(-6)  +  6 

7.  8— (—4)  — (  —  2).  8.     8a6— 10a6— (  — 7a6). 

9.  m-(-3m)-(-5m).  10.     -4x^+{-3x^)-{-lx^). 

11.*  {5x-2y)-{2x-4:y).  12.     3a-116-{5a-86). 

13.  2a-36f5c-(a-46+5c). 

14.  (a+6)  +  (2a-36)-(4a-36). 

15.  a+6—c—(6+c— «)  +  («+&— c). 

IG.     {Qx~-3x+5)  +  {2x--5x-e)-{5x^-8x-}-2). 

17.  Find  the  value  of  5a+&,  when  a=2,  b=  —  3. 

18.  Find  the  value  of  2a-\-3b—c,  when  a=\,  6=2,  c=  — 3. 

19.  By  two  different  methods  find  the  value  of  a— (6— c),  when 
a=20,  6  =  10,  ,c=7. 

20.  Solve  and  verify 

(1)  2x-3-(a;-4)=8. 

(2)  3a;-l-(a:-3)-(a;-|-7)=40. 

(3)  l-{4:-x)-{5-x)-(Q-x)=52. 

21.  What  value  of  x  will  make  5x-— 6  exceed  3.r— 11  by  70  ? 

22.  Find  the  value  of  3x^-2x+5-{2x^-\-x-l),  whenx=0, 1, 2,  3,4. 


ADDITION    AND    SUBTRACTION  47 

23,     Remove  the  brackets  and  simplify  : 

(1)  (a+3b-Uc)-{b+3c-8a)-{c+5a-2b). 

(2)  {a-3b)~{b-3c)  +  {c-3d)-{d-3a). 

(3)  -{3x-y+2z)-{2x-Sy+4z)-i3y-6z-5x) 

(4)  -{a-b)-{b-c)-{c-d)-{d-a). 

EXERCISE  25  (Review  of  Chapter  IV) 
1.*  Find  the  sum  of  oa,  —3a,  la  and  —8a. 

2.  Find  the  sum  of  four  consecutive  integers  of  which  x  is  the 
least. 

3.  Find  the  svun  of  five  consecutive  integers  of  which  n  is  the  middle 
one. 

4.  Add  3a-26  +  7c,  56-3c-2a  and  c-a-36. 

5.  Subtract  —106  from  —66,  —3a  from  5a. 

6.  From  4a— 36  + 5c  subtract  2a  — 56  — c. 

7.  Subtract  5a;— 3,v  +  42  from  4:X—2y  —  z. 

8.  What  must  be  added  to  3a  — 56  + 6c  to  give  6a  — 76  + 4c  ? 

9.  If  cc  +  2/=10  and  x  —  y  =  4:,  what  is  the  value  of  2x  ?     What  is 
the  value  of  2y  ? 

10.  What  is  the  sum  of  the  coefficients  in  6a— 116  +  c  — 3f/  ? 

11.  What  must  be  added  to  x  —  y  to  give  0  ? 

12.  When  a;=3  and  2/  =  4,  what  is  the  remainder  when  x'^  —  y^  is 
subtracted  from  2xy  ? 

13.  Wliat  is  the  difference  between  2a  — 6  — c  and  a  — 6  +  c  ?      Give 
two  answers. 

14.  To  the  sum  of  3m  — in  and  2m  — Sn  add  the  sum  of  ?w.  +  7n  and 

3m  —  2n.  —    ^t  ^  j,    -  ~  - 

15.  From     4a;^  +  3a;— 7     subtract     the     sum     of     2x^  +  73;— 5     and 
2a;2-8a;+7. 

16.  By  how  much  is  3a;— 7  greater  than  2a;+o  ?      For  what  value 
of  X  would  they  be  equal  ? 

17.  Simplify  a+(26-3c)-(c+a). 

18.  If  x=a  +  26  — 3c,  y=:6  +  2c-.3a  and  2=c  +  26-3a,  find  the  value 
oi{l)  x  +  (y  +  z),  (■2)x-(y-z),  {3)  x-(y  +  z). 


48  ALGEBRA 

19.  From  the  sum  of  •5x-\--4:y  and  -Sx—^y  subtract  the  sum  of 
■2x—l-ly  and  •4x+l-32/. 

20.  By  how  much  is  3x2  — 5a;+ll  greater  than  Sx^  — 8x+17? 
What  is  the  meaning  of  the  result  when  x  =  2  ?   when  a;=  1  ? 

21 .  From  |a  —  J6  +  ^c  take  ^a—^b  —  hfi. 

22.  If  a+6  +  c  =  0  when  a=3x— 4^  and  b  =  4:y  —  5z,  what  is  the 
value  of  c  ? 

23.  Solve  5a;-3-(x-4)-(a;-2)  =  27.     (Verify.) 

24.  What  value  of  x  will  make  3x—2  exceed  x— 7  by  63  ? 

25.  Whena=l,  6  =  2,  c  =  3,  the  sum  of  x+a— 36  +  4c,  2x+6-3c  +  4a 
and  3x+c— a— 6  is  124,     Find  the  value  of  x. 

26.  Using  brackets,  indicate  that  the  sum  of  a  and  b  is  to  be 
diminished  by  the  sum  of  c  and  d.  If  a  =  2x— 3,  6  =  5  — 3x,  c  =  3x— ^, 
d  =  I  —  6x,  what  is  the  result  ? 

27.  Solve  2-(x-^)-(i-3x)  =  16-25. 

28.  Subtract  2w— 7n  — 4x  from  zero. 

29.  What  must  be  added  to  a  — (1  — 6)  — (1— c)  to  produce  unity  ? 

30.  Subtract  the  sum  of  2a  +  36  — 4c  +  d  and  a—'lb  —  c  —  d  from  the 
-excess  of  4o— 6  +  c  over  a-\-b-\-c. 


CHAPTER  V 


MULTIPLICATION  AND  DIVISION 


35.     Multiplication  of  Simple  Positive  Quajitities. 

of  a  product  may  be  taken  in  any  order. 


The  factors 


Thus, 
Also 


3x5x4  =  3x4x5  =  5x4x3  =  etc. 
axbxc=axc  x6  =  6  XaXc=etc. 


ab 

ab 

ab 

ab 

ab 

ab 

The  latter  product  may  be  written  abc,  acb,  etc. 
Sax  26  =  3X0x2x6, 
=  3x2xax6, 
=  6ab. 

Make  a  diagram  to  show  that  4:XX2y=Sooy. 

Similarly,  3abx5cd=3xaxbx5xcxd, 

=  3x5xaxbxcxd, 
= 1 Sabcd. 


Thus,  the  coefficient  of  the  product  is  obtained  by  multiplying 
the  coefficients  of  the  factors,  and  the  literal  part  of  the  product 
by  multiplying  the  literal  parts  of  the  factors. 


36.     The  Index  Law  for  Multiplication. 

a^xa^  =axaxaxaxa  =a^ 
2xx3x^=6xxxxxx        =Qx^. 
m^Xm^^m  .m,  .m,  .mxm  .m  .m—m"^. 


Similarly, 
and 


3y^x4y^=l2y», 


Thus,  the  index  of  the  product  of  powers  of  the  same  quantity 
is  found  by  adding  the  indices  of  the  several  factors. 

49  E 


50  ALGEBRA 

EXBRCISS  26  (Oral) 

Find  the  product  of  : 


1. 

2x,  Zij. 

2. 

4?w,  5n. 

3. 

\x,  4i/. 

4. 

3x,  4a;. 

5. 

%x,  Ix. 

6. 

3a6,  4a;2/. 

7. 

a^  a. 

8. 

32/2,  2^/2. 

9. 

ab,  ac. 

10. 

2x^,  4a;^. 

11. 

5j>2,  4p4. 

12. 

5a:  2,  3x2/. 

13. 

(4x2)2. 

14. 

Sax,  2aa;, 

15. 

<2,  t\  t*. 

16. 

a6,  ac,  a. 

17. 

5a2,  3a,  2. 

18. 

(4a)3. 

19. 

fa,  ih,  |c. 

20. 

(36)2,  (26)2. 

21. 

f  m,  6w,  5mn. 

37.  Multiplication  by  a  Negative  Quantity. 

4x3  is  a  short  way  of  writing  4+4+4=12. 
—4x3  is  a  short  way  of  writing  (— 4)  +  (— 4)  +  (— 4)  =  — 12. 
Hence  multiplication  by  a  positive  integer  means  that  the 
multiplicand  is  taken  as  an  addend  as  many  times  as  there 
are  units  in  the  multiplier. 

Also,  we  shall  define  multiplication  by  a  negative  integer 
as  meaning  that  the  multiplicand  is  taken  as  a  subtrahend  as 
many  times  as  there  are  units  in  the  multipher. 
According  to  this  definition  then 

4x-3  =  -4-4-4=-12, 
_4x-3=-(-4)-(-4)-(-4), 
=+4+4+4=  +  12. 

We  may  state  these  results  in  algebraic  symbols,  thus  : 

(+rt)x(+fe)=+<*6, 
{—a)x{^b)=—ah, 
{■i-a)x{  —  b)=—ab, 
{  —  a)x{  —  b)=^ab. 

38.  Rule  of  Signs  for  Multiplication.  Examine  the  preceding 
statements  and  state  when  the  product  has  a  j)Ositive  sign, 
and  when  it  has  a  negative  sign. 

The  rule  of  signs  for  multiplication  may  be  stated  in  the 
form  : 

The  product  of  two  factors  with  like  signs  is  positive,  and  of 
two  factors  with  unlike  signs  is  negative. 


MULTIPLICATION    AND    DIVISION  51 

EXBRCISE3  27  (Oral) 


State  the  product  of  ; 

1. 

6,  -7. 

2. 

-4,3. 

3. 

-4,  -5. 

4. 

h  -5. 

5. 

3a;,  -2. 

6. 

—x,  y. 

7. 

-X,  -2y. 

8. 

2m,  —3n. 

9. 

—X,  -~3xy. 

10. 

-a(-6). 

11. 

(-6)2. 

12. 

—ay{—x). 

13. 

— 2nit'(— t'). 

14. 

-\x,  -12a;. 

15. 

—2x~x—3xK 

16. 

a;-,  —a:. 

17. 

-2a2,  _3a. 

18. 

—abx—3cd. 

19. 

—5xy,  2x-. 

20. 

— a^,  —a. 

21. 

3a%x—ab. 

22. 

5x^y^,  —xy^. 

23. 

3a263c*,  -ab 

2c. 

24. 

—x^yz^,  5x^y~z. 

25. 

Since  —4x3  =  - 

-12,  - 

-4x2=-8, 

-4X1  =  - 

4,   what  would 

you  expect  —  4  x  0  to  be  equal 

to  ?     Also  - 

■4X- 

-1  and 

. -4x-2? 

39.     Multiplication  of  several  Simple  Factors. 

Ex.  1.— Multiply  2a,  —36,  —4Mb,  —b. 
The  product  of  2a  and  —  36  is  —  Gab. 
„  „        ,,    —6ab  and  —  4a6  is  24a*6^. 

„    24a262  and  -6  is  —  24a26^ 
.'.  the  reqmred  product  is  —  24a^6^. 

Of  course  the  factors  may  be  multipHed  in  any  order  %ve  choose. 
If  we  mtdtiply  all  the  negativ^e  factors  first,  what  sign  will  their  product 
have  ? 

What  sign  will  the  product  have  if  we  multiply  four  negative  factors  ? 
Five  negative  factors  ?     Twenty  negative  factors  ? 

The  product  will  be  negative  when  the  number  of  negative 
factors  is  odd  and  will  be  positive  when  the  number  of  negative 
factors  is  even.  Any  number  of  positive  factors  will  evidently 
not  aflfect  the  sign  of  the  product. 

Ex.  2.— Multiply  3.t,  —5xy,  —6x^,  —y,  6y^. 

(1)  TTxe  sign  of  the  product  is  — ,  since  there  is  an  odd  nmnber  of 
negative  factors. 

(2)  The  numerical  coefficient  =  3x  5x  6x  6  =  540. 

(3)  The  literal  part  of  the  product  =  a'.  a;?/,  x- .3/.  2/'  =  a;*i/^, 
.".  the  complete  product  =  —o4:0x*y^. 

E  2 


52  ALGEBRA 

Ex.  3.— Find  the  values  of  {-2f,  (-2)^,  {-2)K 

In  (  —  2)3  the  number  of  factors  is  odd, 

•     (_2)3=— 23=-8. 
Similarly,  (-2)*=16  and  (-2)^= -32. 

SXSRCISE  28  (1-18  Oral) 

Find  the  product  of  : 

1.  3,  4,  -5.  2.    3,  -4,  -5.  3.     -3,  -4,  -5. 

4.  —a,  —b,  —c.  5.    2a,  3a,  —4a.  6.    —J,  —J,  12. 

7.  3:r,  -2a:?/,  -?/.  8.    -3,  -3,  -3.        9.    (-4)3. 

10.  -2,-3,-4,-5.    11.     (-2)2x(-3)2.     12.    -2a;, -2x,-2a;. 

13.  —a,  —2a,  —3a,  —4a,  —5a.        14.     2a:,  —3x,  4(/,  —y. 

16.  5xy,  —3xij,  —2x,  —2y.  16.     —1,  —2,  —3,  —J,  —J. 

17.  What  is  the  square  of  —2a,  of  —Sxy,  of  — 4a26c  ? 

18.  What  is  the  cube  of  —5,  of  —x,  of  —2x^  ? 
19.*  If  a:=  — 1  and  y=—2,  find  the  values  of  : 

x^,  y^,  x^-{-y^,  x^ — y^,  x^,  y^,  x^-\-y^,  x^ — y^. 

20.  Find  the  value  of  3.r-+2a:— 5  when  x=—2;  when  a;=— 3; 
when  0;:=— 4. 

21.  Write  without  the  brackets : 

{-a)\  {-a)\  {-2)\  (-l)^  (-1)«,  (-3)*,  (-l)3x(-2)^ 

22.  Find  the  sum  of  the  squares  of  —2,  —3,  —4.     Find  also  the 
square  of  their  sum. 

23.  Find  the  value  of  (a— 6)2+(6— c)2+(c— o)^  when  a=6,  6=4, 
c=2. 

24.  When  a=2,  6=1,  c=  — 3,  show  that  a^-^b^-[-c'^='iabc. 

25.  When  a:=3  and  y=  —  2,   how  much  greater  is  (x—y)^  than 
x^ — y^  1 

If  a  =  — 1,  6  =  — 2,  c-=  — 3,  d=— 4,  find  the  value  of: 

26.  3a  +  26+c-4rf.  27.     02+62+02+^2. 
28.     ab+ac+bc+cd.  29.     aH^—bW 

30.     abc+bcd^cda-\-dab,  31.     flS+fes+c^+rfS. 


MULTIPLICATION    AND    DIVISION 


53 


40.  Compound  Multiplication.  We  multiply  a  compound 
quantitj'  by  a  simjjle  one  in  a  manner  similar  to  the  method 
in  arithmetic. 


In  arithmetic. 
3yd.  Ift.  4in.     23  =  2.  10  +  3 
2  2     2 


6yd.  2ft.  Sin.     46  =  4.  10  +  6 


In  algebra. 
3a  +  46-    5c 

2 

6a+86-10c 


If  we  wish  to  indicate  the  product  of  x  and  y  +  z,  we  write  it  in  the 
form  x{y  +  z),  which  we  see  is  equal  to  xy  +  xz. 

Tlie  diagram  shows  how  this  product  may  be  illus- 
trated geometrically. 

Make  a  similar  diagram  to  show  that 

a(b  +  c  +  d)  =  ab-\-ac  +  ad. 
Similarly,        x{y  —  z)  =  xy  —  xz. 
Can  you   see  that  the  diagram  is  a  geometrical 
illustration  of  this  ? 

Ex.— SimpUfy  3(a-6)-4(6-c)-2(a-6+c). 

The  expression  =(3a-36)-(46-4c)-(2a-26  +  2c), 
=  3a-36-46  +  4c-2a+26-2c, 
=a-56  +  2c. 


y 

:: 

.xy 

xz 

y-s          s 

.x(y.z) 

xz 

EXERCISE! 

29 

Copy  and  supply  the 

products : 

1.     2a+6 

2.     3a— 26 

3. 

2m— 5n. 

4 

7 

6. 

-6 

4,     4x-3«/ 

5.     3a;— 4?/ 

2a+56— c 

2x 

-2y 
8.     3(5a;-2y). 

9. 

—a 

7.     2(3x--ll). 

-2(3x-y). 

10.     3a;(a;2+5a;-2). 

11.     hxy{1x'^—x\j). 

12. 

3mp{5—mp) 

Simplify  : 

13.*  3(a+6)+4(6+c)+5(c+a). 

14.     2(x— 2«/)+3(a;- 

y)_(4.r-3y/). 

15.     3(2m.— 3n)— 5(m-7!)+2(m+2H). 

54 


ALGEBRA 


16. 
17. 

18. 
19. 
20, 
21. 


4(a-26+c)-3(6-2c+a)-2(5c-4a-56). 

i(2a-36)+i(2a+56)+|(5a+6). 

x{x-l)+2x{x-3)+3x{x+5). 

a(a2_a4-l)+3(a2+a_2)-2(a2+2a-3). 

3a;(a;2-2a;+2)-2a;(3x2+4a;-5)+a;(4x2+5-c-6). 

— 2a(6— c+d)— 3a(c— c?+6)— a(ci— 6— c). 
Solve  and  verify  : 

22.     3(a;-l)=2(a;+4).  23.     5(a;-2)-2(a;+2)=70. 

24.     6(2a;-3)-3(a;-3)=0.  25.     2{5a;-9)+4(x-ll)=36. 

26.  3(a;+2)+5(a;-3)==2(a;-4)+4(x-l)  +  13. 

27.  Find  the  sum  of  x{x+l),  3a-(a;— 2),  2a:(a;— 5). 

28.  Subtract  a{2a^-a+l)  from  2a(a2+3a-2). 

29.  If  a  stands  for  x^-\-xy~\-y^  and  b  for  x^—xy+y"^,  find  the  values 
of  a—b,  2a+b,  3a— 2b. 

41.     Multiplication  by  a  Compound  Quantity. 

The  measures  of  the  sides  of  the  large  rectangle  are  a-\-b 
X         y       and  x-\-y.     The  measure  of  the  area  is  the 
prodiict  of  a+6  and  x^-y,  which  is  seen  to 
be  ax^ayAr^x^hy, 

:.  {a-^b)[Jc+y)=ax-\-ay-\-hx-\-hy. 

This  diagram  shows  how  to  find  the  product  of 
a; +  3  anda;+2.  What  does  it  show  the  product  to 
be  ? 

Make  a  similar  figure  which  will  show  the  product 
of  a  +  6  and  a-{-h,  and  thus  find  the  value  of  {a-\-bY, 
or  the  square  of  a +  6. 

The  method  of  obtaining  the  product  without  the  diagram 
is  similar  to  that  used  in  arithmetic. 


ax 

ny 

bx 

by 

2 

X 

3.r 

2x 

6 

In  arithmetic. 
12  = 
23  = 


1.10+2 
2.10  +  3 


12x    3=   36=  3.10  +  6 

12x20  =  240  =  2.  102  +  4.  10 


12x23  =  276  =  2.  10^  +  7.  10  +  6 


In  algebra. 

x+2 
2.-C  +  3 


3a;  +  6  = 

2a;2+4a;        = 


3(a;+2) 
2a;(x  +  2) 


2a;2  +  7a;+6  =  (2x+3)(.r+2) 


MULTIPLICATION    AND    DIVISION 


55 


Thus,  the  product  of  any  two  expressions  is  obtained  by 
multiplying  each  term  of  the  multiplicand  by  each  term  of  the 
multiplier.  The  proper  signs  are  attached  to  these  partial 
products,  and  the  sum  of  the  partial  products  is  then  taken. 

In  multiplying  in  arithmetic  we  begin  at  the  right,  but  in 
algebra  it  is  usual,  but  not  necessary,  to  begin  at  the  left. 

Ex.— Multiply  (1)  2ft-36  by  3a-26. 
(2)  3a;— 5?/  by  ^x-\-y. 

Check 


Check 

o  =  6=l 
-1 
\ 

(1) 
2a  -    3b 
3a  -    26 

-1 

6a2-   9ab 

-   4a6  +  662 

6a2-13a6  +  662 

-2 
5 

10 


(2) 
3x  —    5y 

4:x  +     y 

\2x^-2(ixy 
+   ^xy- 


■5y' 


12a;*—  nxy  —  5y^ 


42.  Checking  Results.  In  Chapter  II.  we  saw  how  to 
verify  the  root  which  we  obtained  in  solving  an  equation. 
We  might  verify  our  work  in  subtraction  by  addition.  As 
in  addition,  the  work  in  multiphcation  is  easily  checked  by 
substituting  particular  numbers  for  the  letters  involved. 

Thvis  to  check  the  work  in  the  first  example  in  tlie  preceding  article, 
we  might  substitute  1  for  each  letter  involved. 

Whena=6=l,  2a-36  =  2-3=  -1, 

3a-26  =  3-2=l, 

and  6a2-13a6  +  662  =  6-13  +  6=  -1. 

Since  the  product  of  —  1  and  1  is  —1,  the  work  is  likely  correct. 
A  convenient  way  of  exhibiting  the  test  is  shown.  Of  course  any 
numbers  might  be  used  in  cJiecking,  but  we  naturally  choose  the 
simplest  ones. 


EXERCISE  30 

^ind  the  product  of  the  following  and  check : 

1.     x+3              2.     2x+7             .  3.       x+5 

4. 

3a;+4 

x+4:                       x+l                      2x+2 

2a;+3 

li     7   ^  'i  -f 

56 


6. 

a— 3 
a-4: 

2x-3 

2x+S 

6. 
10. 

a— I 

a— 5 
a+3 

ALGEBRA 
7. 

11. 

14. 
16. 

6- 
26- 

-4 
-3 

8. 
12. 

-4(Z). 

3a— 5 

2a+5 

2x- 
bx- 

-3a 

-  a 

9. 

x+y 
x—y 

56).      ■ 

3a— 7c 

3a+7c 

13. 
15. 

{3a+Ab){2 

{X- 

-5y)(2x- 
-36)(3c- 

17.  Find  the  square  of  x—y  by  multiplying  it  by  x—y.     What  is 
{x—yY  equal  to  ? 

18.  Find  the  squares  of  2a -6,  2a -36,  4a +5,  3a +46.     Check  by 
putting  a =3,  6=1. 

19.*  Simplify  (a:+l)(.T+ 2) +  (.T-2)(x+ 3).  ^  cC- 

20.  Simplify  3(a+2)(a-2)+2(a-5)(a+l).  xTj-CJl^J^lSr 

21.  When  Sa;^— 2a;— 15  is  divided  by  2a;— 3  the  quotient  is  ^x-\-57~ 
Prove  that  this  is  correct. 

22.  Show  that  (6a;-8)(2a;-3)  =  (4a;-6)(3a;-4). 

23.  m{x-\-y)^mx-\-my.     Fmd  the  value  of  mx-\-my  when  m=2-14, 
.i;=43-7,  y=5Q-3. 

24.  If  a  train  goes  2a— 36  miles  per  hour,  how  many  miles  will  it 
go  in  2a +36  hours  ? 

25.  Simplify   (.r+7/)2+(a;-2/)2 ;     {x+y)^-ix-y)^. 

26.  Simplify  2(a-6)(2a+6)-3(a+6)(a-26). 

27.  Subtract  (a;+2)(a;-9)  from  (a;+3)(a;+4). 

28.  Multiply  3(a;+3)-2(a;+4)  by  2(a;-5)-(a;-3). 

29.  Subtract   (a;+3)(a;+7)    from    (a;+l)(a;+ll).     For   what   value 
of  X  are  these  quantities  equal  ?     (Verify.) 

30.  Show  that  there  is  no  value  of  x  which  will  make  (a;— 10)(.'C— 1) 
equal  to  (a;— 3)(a;— 8). 

31.  Subtract  the  sum  of  (3a;+2)(2.r+3)  and  (3a;-2)(2.T— 3)  from 
the  sum  of  (4a;+3)(3a;+4)  and  (4a;-3)(3a,--4). 

32.  Simplify  (a;-3)2+(a;-2)(,r+2)  +  (.r+l)(a;+5). 

33.  Subtract  the  product  of  2a— 5  and  3a +2  from  the  product 
of  3a +5  and  2a— 2. 


(     )) 


MULTIPLICATION    AND   DIVISION  57 

34.  Find  the  sum  of  the  squares  of  x+l,  x-\-2,  x+3.  Check  by 
putting,  x—2. 

Solve  and  verify : 

35.  {x+5){x-l)={x-5){x+2).        36.     {x-l)^={x-5){x-\-2). 

37.  (2x--l)(3x-l)  =  (a;-2)(6a;+4). 

38.  (a;+ll)(a;-2)=(a;-7)(a;-l)  +  107. 

39.  xix+l)-ir{x+l){x+2)=2{x+l)(x+3). 

40.  (x+iy-+{x+2)^+(x+3y-=3{x-2)^+U. 

43.     Division  by  a  Simple  Positive  Quantity. 

To  divide  24  by  6  is  the  same  as  to  find  the  number  by 
which  6  must  be  multiplied  to  produce  24. 

Thus  division  is  the  inverse  of  multiphcation  as  subtraction 
is  the  inverse  of  addition. 

Smce  axb=ah, 

.'.  ab'^a=b  and  ab^b=a. 

If  we  wish  to  divide  6xy  by  2x,  we  must  find  what  2x  must 
be  multiplied  by  to  produce  Qxy. 

(1)  2  must  be  multiplied  by  3  to  produce  6, 

(2)  X     „       „  „         „   y  „         „        xy. 

.'.    6xy  -^  2x  =  3y. 
Similarly,  I5a6c-:-36c  =  5a. 

A  problem  in  division  may  be  viritten  in  the  fractional 
form,  the  dividend  being  the  niimerator  of  the  fraction  and 
the  divisor  the  denominator. 

m,  24      .      ab     ,       Qxy     „ 

Thus,  -5- =  4,     —=b,     -^  =  3y. 

o  a  2x 

As  in  arithmetic,  we  may  remove  or  cancel  from  the  dividend 
and  divisor  any  factor  which  is  common  to  both. 

Thus,  — X5— =  4a,  on  removing;  the  factors  3  and  b. 
36  * 

Similarly,  —-^^  =  3ay  and  ~^'^^  =2p. 


68 


ALGEBRA 


44.     Index  Law  for  Division. 

Since  a^xa^^a^  by  the  index  law  for  multiplication, 
.*,  a^-^a'^=^a^  or  a^~a^=a", 


or 


^ .^ .a .a .a 


=  a^  and 


a^      ^  .^  .i^  .a 


=  a' 


Thus,  the  index  of  the  quotient  of  powers  of  the  same  quantity 
is  found  hy  subtracting  the  index  of  the  divisor  from  the  index 
of  the  dividend. 

Thus, 

Similarly, 


I5a363 


3a26 


:5a3-263-i  =  5a62. 


The  work  in  division  may  be  verified  by  multiplication. 
Thus  the  preceding  division  is  seen  to  be  correct,  since 
5a&2x3a26=15a363. 


EXERCISE  31 


Copy  and  supply   the  quotients,   verifying  the  results  by  mental 
nultiplication  : 


5.    ^rl 


9. 
13. 


X 

2a 


2. 


abc 

Qxi/^2x. 
16.     10a;5^2a;^ 
19.     22a*b^^lla~b' 


10. 


5abc 
ab 

Ix 

18^3^2 


11. 


24w« 
3n 

2a2 
4a3 


4. 


— .  8. 


12. 


25xyz 
~5z" 
65m% 
13mn 

\2x^y^ 


14. 
17. 


10a3-^5a. 
16a36— 4a6. 


Ixy 
15.     \mv'^-^\v. 

\bx^yH'^^2x'^yz^. 


18 


45.     Rule  of  Signs  for  Division. 

Since   (+a)x (+6)  = +«('->,    (- a)  x  (+6)=— a&,    (+a)x(— 6) 
-—ah,  {—a)x{—b)  =  -\-ab,  it  follows  that 
+a6       ,  ,      — a6 


■■+b, 


-b,    +^^=-6. 
—a 


When  is  the  sign  of  the  quotient  +   and  when  is  it  —  ? 
What  then  is  the  rule  of  signs  for  division? 
Compare  it  with  the  rule  of  signs  for  multiplication  (art.  38). 


MULTIPLICATION    AND   DIVISION  59 

Ex.— Divide  —lOxhj^  by  —2xij". 

(1)  Wliat  is  the  sign  of  the  quotient  ? 

(2)  What  is  the  numerical  coefficient  ? 

(3)  What  is  the  Hteral  part  ? 

(4)  What  is  the  complete  quotient  ? 


EXERCISE  32  (Oral) 
Perform  the  indicated  divisions  : 


1. 

12^-3. 

4. 

-7^7. 

7. 

0-^5. 

10. 

ab-. — a. 

13. 

I0a^^-2a 

16. 

-I2m^n^- 

-6>nn. 


2, 

-12^-4. 

3. 

-10^2. 

5. 

—2a^—a. 

0. 

-12^2-^-3. 

8. 

0^-5. 

9. 

Ga-^-2a. 

11. 

axy-. — X. 

12. 

45^-5^-3 

14. 

-6a^^3a. 

15. 

27a;*^-3.T2. 

17. 

x^y^z-. — xyz. 

18. 

-4a5^-2a3. 

20. 

—4:mn^ 

21. 

6-4.r3 
-2x' 

in  the 

following  : 

(2) 

(3) 

(4) 

(5) 

-10x2 

— lOabc 

357n^n 

-5c 
6a 

—  5m 

-   2.r 

2ac 

19.  -pir 

-pq 


(1) 
Dividend :       6«^ 
Divisor :           2a 
Quotient  :        

46.     Division  of  a  Compound  Quantity  by  a  Simple  One. 
If  we  divide  6  ft.  4  in.  by  2  we  get  3  ft.  2  in.,  or  12  lb.  6  oz. 
by  6  we  get  2  lb.  1  oz. 

Similarly, 

3)9  ft.  6  in.  4)  16  lb.  8  oz.  2)6  tens +  8  units  ^ 

3  ftTfmT  Tib.  2  oz.  "  3  tens +  4  units* 

3)9/+6i  _  4)16a-|-^86  _  2)6^  +  8 

3/4- 2^'  4a+26  ■  3<4-4 ' 

a)ab  +  ac  ^  3x)C)x^-3x^  - ab):ia^b^  —  2ab 

T+  c  *  2x^-^x'  '  -  Sab    +  2~  ' 

Thus  it  is  seen,  that  we  divide  a  compound  expression  by  a 
simple  one  by  dividing  each  term  of  (he  dividend  by  the  divisor, 
attaching  the  proper  sign  to  each  term  of  the  quotient. 


60  ~  ALGEBRA 


EXERCISE  33  (1-15,  Oral) 
Divide  the  first  quantity  by  the  second  : 

1.     9a2-f6a,  3.  2.     6x^+4:X^-2x,  x.     3.     15a;2-10a;,  5a:. 

4.     16m^— 4m,  4m.        5.     a;22/+^2/^  ^y-  6.     12a^— 4a6,  —2a. 

7.     — aa;+ay,  — «.         8.     ct^+a^— a,  a.  9.     6x^—4:xy,  2x. 

10.     — 6a6— 6a,  —3a.  11.     6a3_8a2^4a,  —2a. 

12.     a^b^-a%'^,ab''-.  13.     -5a*-10a^  -Sa^. 

14.     — 4a;+10x2-6x-3,  — 2.r.  15.     3y^—2}j\  \y. 

Simplify : 

.„       3.r-|-6  ,    10a;— 15  ._      ab^ac  ,   bc4-ab   ,   ac-\-bc 

16.      — +• 17. [- ! + ' 

3^5  a      ^      b      ^      c 

^g^   a^+3a      3a^-\-6a  ^^      (a;+2)(.T-2)  +  (a;-2)(a;-4) 

a  3a      '  '  2 

2^^     x^+xy   ^   y^-xy  ^i.     («+2)(a+3)-(a-3)(a-2) 

X  y  '  2a 

_„      ab—ac  ,   be— ah  „„      a'^—a'^  .   a^—a     , 

^Ji. h -— '  Jd.     H ]-l. 

— a  — 0  a  a 

24.  Subtract    (a;+3)(a;— 8)    from    (2x— 4)(a;-(-6)    and    divide    the 
remainder  by  x. 

■     ^      „  1  1         *     a;2-10a;  ,   Sx^+JSx  ,   10a;-15      .„ 

25.  Solve  and  verify    +    — \- =40. 

a;  3a;  5 


EXERCISE  34  (Review  of  Chapter  V) 

1.     State  the  rule  of  signs  for  multiplication  and  for  division. 
2.*  If  a=3  and  b=   —4,  find  the  values  of  : 

a2,  62,  ab,  a'^  +  b%  a^-b^,  a^,  b^,  a^-b^. 

3.  What  are  the  values  of  (-1)2,  (-l)^  (-1)^",  (-2)^  (-3)3  ? 

4.  Simplify  Sa^x    -^b^X   — 2a64-6a6*. 

5.  Simplify  2a(a-[-3)  +  3a(2a-6). 

6.  What  is  the  area  in  square  feet  of  a  rectangle  which  is  (a +6)  feet 
long  and  (a  —  b)  yards  wide  ? 

7.  Make  a  diagram  to  show  that  3a;x4x=12a;^ 


MULTIPLICATION    AND    DIVISION  61 

8.  A  merchant  bought  a  pieces  of  silk  at  GO  cents  a  yard  and  b 
pieces  at  80  cents  a  yard.  If  each  piece  contained  50  yards,  find  the 
total  cost  in  dollars. 

9.  To  the  product  of  3a;— 2  and  2x  — 3  add  the  product  of  3x-4-2 
and  2x+3. 

10.  From  the  product  of   ox—Sy  and  2.f  +  y  subtract  the  product 
of  3x—2y  and  2x-3y. 

11.  Make  a  diagram  to  show  that  the  product  of  a+3  and  a+1  is 
a2+4a+3. 

12.  Divide  4o^  — 6a  2  — 8a  by  —2a  and  verify. 

13.  To  the  square  of  2)n  —  3n  add  the  square  of  3m— 2n. 

14.  Prove  that  when    lox^—8xy—l2y^  is  divided   by   5x—6y    the 
quotient  is  3x+2y. 

15.  Find  the  product  of  a  —  b,  a-\-b  and   a--\-b^.     Check  by  sub- 
stituting 3  for  a  and  2  for  b. 

16.  SunpMy  ^ + 

17.  Solve  (2x+3)(3a;+2)  =  (6a;-l)(a;+3).     (Verify.) 

18.  Simplify  {2a-36)(a+6)  +  (a-6)(3a+6). 

19.  What  value  of  x  will  make  (a;+3)(a;+9)  equal  to  {a;+5)(x+6)  ? 
Could  (a;  +  3)(x+9)  be  equal  to  (x  +  4)(x+8)  ? 

20.  Find  the  sum  of  (a-l)^,  (a-2)2  and  (a-3y. 

21.  Subtract  the  product  of  2x — 3y  and  3x-\-2y  from  the  product 
Zx—iy  and  4a;  +  3«/. 

22.  Simplify— 2p"+(3  +  2a)(l-a). 

23.  Find  the  value  of  2x^  +  30;- 1,  when  x=  —3;    when  a;=  —4. 

24.  Find  the  product  of  x  —  2,  x+2  and  x^  +  'i. 

25.  If  x  =  a^  —  3a-Ti  and  y  =  2a^  —  a—l,  find  the  values  of  2x  +  3^, 

4:X  —  2y,  —^-^  . 

26.  If  a; = 2a +  6  and  y  =  a~r-2b,  find  in  terms  of  a  and  b  the  values  of 
ax  — by      4x—2y     x^  —  y^ 

^       '         Sa^'         3 

27.  If  a;=36  — 2c  and  y  =  2b  —  3c,  find  the  value  of  {2x—y){3x—2y). 

28.  If  a;=2,  y=2,  z=  —4,  find  the  value  of  x^+y^+z^  —  3xyz. 


CHAPTER  VI 
J     i  SIMPLE  EQUATIONS  {continued  from  Chapter  II.) 

47.  Definition.  An  equation  is  the  statement  of  the 
equahty  of  two  algebraic  expressions. 

Thus,  2a;+3=13  is  an  equation,  and  the  solving  of  it 
consists  in  finding  a  value  of  x  which  will  make  the  statement 
true. 

The  beginner  should  clearly  see  the  difference  between  the 
value  of  X  in  an  expression  like  2a;+3  and  the  value  of  a;  in 
an  equation  like  2.e4-3=13. 

In  the  expression  2a:+3,  x  may  represent  any  number, 
and  for  different  values  of  x  the  expression  has  different 
values.  But  in  the  equation  2;c+3=13,  x  can  not  represent 
any  number  we  please,  but  some  particular  number,  in  this 
case  5,  which  when  substituted  for  x  will  make  2x-\-3  have 
the  value  13. 

48.  Identity.  The  statement  4(a;— 2)=4a;— 8  is  an  equation 
according  to  the  definition  we  have  given. 

If  the  first  side  of  this  equation  be  simplified  by  multi- 
plication, we  obtain  4a; — 8,  which  is  identically  the  same  as 
the  second  side.  It  is  at  once  seen  that  this  equation  is  true 
for  aU  values  of  x. 

An  equation  which  is  true  for  all  values  of  the  letters 
involved  is  called  an  identical  equation  or  briefly  an  identity, 
while  an  equation  which  is  true  only  for  certain  values  of  the 
letters  involved  is  called  a  conditional  equation.  The  usual 
method,  however,  is  to  call  all  conditional  equations  simply 
"  equations,"  and  all  identical  equations,  "  identities." 

C2 


SIMPLE   EQUATIONS  63 

Thus,  5a;— 2  =  3x+ 10,  is  an  equation, 

and  {x+3){x  —  3)  =  x^  —  9,     is  an  identity. 

We  cannot  always  see  mentally  whether  a  given  statement  is  an 
equation  or  an  identity. 

Thus,  (a;+2)(x+3)  =  (a;— l)(a;— 3)  +  3(3a;+l)  might  appear  to  be  an 
equation,  but  if  we  simplify  each  side,  we  find  that  each  becomes 
x^-\-5x+6,  and  this  statement  is  therefore  an  identity. 


EXERCISES  35 

Which  of  the  following  statements  are  equations  and  which  are 
identities  ? 

1.  8(a;+3)=4.'c+4(.r+6).        ^-^-^v^^ 

2.  3x{x+l)=x{x+l)-\-2x{x+5)-{-10.  '^ 

3.  (a;-3)2-5=.r(a;-6)+4. 

4.  (2x-4)(a;-5)-H->C--2)(a;-3)  =  (3x--2)(a;-7)+40.       ^  ^'" 

5.  {x-\-a)(x^-{-a^)=x^-]-ax(x+a)-\-a^. 

6.  {x+2){x-B)=x{x+5)-\-3{x-l). 

49.  Transposition  of  Terms.  In  Chapter  II.  the  method 
of  solving  easy  equations  was  dealt  with. 

The  method  depended  almost  entirely  on  the  proper  use 
of  the  four  axioms  of  art.  15. 

The  following  examples  will  show  how  the  methods  of 
Chapter  II.  may  be  abbreviated. 

Ex.  1.— Solve  7x-6=4a:+12. 
Add  6  to  each  side,  7x==^x+12  +  6. 

Subtract  4x  from  each,  7a;— 4a;  =  12  +  6. 
Collect  terms  on  each  side,  3a;  =18. 

Divide  each  side  by  3,  x  =  6. 

Here  we  added  6  to  each  side  with  the  object  of  causing 
the  —6  to  disappear  from  the  first  side  of  the  equation,  so 
that  we  might  have  only  unknown  quantities  on  that  side. 
But  the  addition  of  6  to  the  second  side  caused  +6  to  appear 
on  that  side. 

We  might  say  then,  that  the  —6  was  transposed  from 
the   first  side  and  written  on  the  other  side  with  its  sign 


64  ALGEBRA 

changed,  and  similarly,  that  the  4a*  was  transposed  from  the 

second  side  to  the  first,  with  its  sign  changed. 
We  therefore  have  the  following  rule  : 
IV  Any  quantity  may  he  transposed  from  one  side  of  an  eqvMion 
^mthe  other  if  the  sign  of  the  quantity  he  changed. 


Using  the  rule,  the  solution  of  Ex.  1  might  appear  thus  : 

7a;-6  =  4a;+12. 

Transposing  terms, 

7a;-4a;=12  +  6, 

.-.  3x=18, 

.".     x  =  6. 

Verify  this  result. 

Ex.  2.— Solve 

2{^x-5)  +  ^x-b)=l{x-l). 

Removing  brackets. 

6a;- 10  + 3a;- 15  =  7a;- 7, 

Transposing  terms, 

6a;+3a;-7a;=10+15-7, 

.-.  2a;  =18, 

.-.     a; =9. 

Verification,  when  a; =9 

first  side 

=  2x22+3x4  =  56, 

second  side 

=  7x8                =56. 

Ex.  3.— Solve 

3(2/-2)-5(2/-3)  =  17. 

Removing  brackets, 

3y_6-52/+15=17. 

Transposing  terms, 

3y-5v  =  6-15+17, 

.-.   -22/ =  8, 

8 
..       2/  =  —  =  -4. 

Verification  :  first  side 

=  3(-6)-5(-7) 

=  -18+35=17. 

Ex.  4.— Solve  (2a:-l)2-(a;-3)(a:-2)=3{.'K-2)2-4. 

Here  the  indicated  multiplications  are  first  performed. 

(2a;-l)2  =  4a;2-4a;+l, 
(x-3)(a;-2)  =  a;2  — 5a;+6, 
(a;-2)2  =  a;2  — 4a;  +  4, 
4x2-4a;+l-(x2-5a;+6)  =  3(a;2-4a;  +  4)-4, 
4x2-4a;+l-a;2  +  5a;-6  =  3a;2-12a;+12-4, 
.-.   4a;2-a;2-3x2-4a;+5a;+12x=  12-4- 1  +  6, 
.-.  13a;=13, 
.-.       a;=l. 


SIMPLE   EQUATIONS  65 

Here  the  product  of  a;  — 3  and  a;  — 2  is  first  found  and  enclosed  in 
brackets  with  the  minus  sign  preceding.  In  the  next  line  the  brackets 
are  removed  and  the  signs  changed. 

In  ^{x—'2)-,  the  x—2  must  first  be  squared  and  the  product  multiplied 
by  3. 

Note. — The  beginner  should  not  attempt  to  perform  these  double 
operations  together. 


EXSRCISB  36 

Solve  and  verify : 

1.  4a;— 4=2a;+8.  2.     3a;— 7=8-2a;. 

3.  3-3a;=9— 5a;.  4.     2(a;— 5)=a;+20. 

5.  5(?/-2)=3(?/+4).  6.     10(a;-3)=8(a;-2). 

7.  ll(4x--5)  =  7(6a;-5).  8.     7a;-ll+4a;-7=3x-8. 

9.  14+5a;=9a;-ll+3.  10.     3(5a;-6)-9a;=30. 

11.  7(x-3)==9(a;-fl)-38.  12.     5(a;-7)+63=9.r. 

13.  72(a;-5)=63(5-a;).  14.     28(a;+9)=27(46-a-). 

15.  7(4.r-5)=8(3a;-5)+9.  10.     4(a;+2)=3-3(2a;-5). 

17.  (.-c+7)(a;-3)  =  (a;-l)(a;+l).     18,     (a;-8)(a;+12)=(a;+l)(a;-6). 

19.  20(a;-4)-12(a;-5)=a;-6.      20.     5(2a;-l)-3(4a;-6)=7. 

21.  (2m-5)(4m-7)=8TO^+52.     22.     5(3/i  +  ])-7A-3(A-7)=6. 

23.  (a;+5)2-(a;-f3)2=40.  24.     (a;+5)2-(4-a;)2=21.a;. 

25.  4(22/-7)-3(42/-8)=2^-7. 

26.  (a;+4)(a;-3)-(a;+2)(a;+l)=42. 

27.  (2a;-7)(a;-f5)=(2a;-9)(a;-4)+229. 

28.  (a;+l)2+(a;+2)2+{a;+3)2=3(a;+l)(.r+4)-7. 
~--29.  2(a;-l)2-3(a;-2)(a;+3)=32-(a:-3)(.r-4). 

30.  What  value  of  .r  will  make  10a;+ll  equal  to  5a;— 9  ? 

31.  Prove    that    3(a;— 2)+4(3a;— 5)=5(3a;— 6)+4    is    true    for    all 
values  of  x. 

32.  What  value  of  a  will  make  5(a— 3)  exceed  3(a— 7)  by  28  ? 

33.  For  what  value  of  x  will  the  sum  of  \2-\-lx,  4a;+3  and  9— 5a; 
be  zero  ? 


66  ALOEBRA 

34.  If  a; =2  is  a  solution  of  the  equation 

find  the  value  of  k. 

35.  Prove  that  10  is  a  root  of  the  equation 

(x+3)(a;+4)  +  (a;+5)(a:+6)=422. 

36.  When  (3:c+2)(4a;— 5)  is  subtracted  from  (2x+7)(6a;+3)  the 
remainder  is  141.     Find  x. 

37.  What  value  of  y  will  make  («/— 3)(?/+3)  exceed  {y-[-^){y—l) 
by  40? 

38.  What  value  of  k  will  make  (5  — 3^)(7-  2h)  equal  to 

(11-6A')(3-A-)  ? 

39.  What  is  peculiar  about  the  equation 

(a;-5)2-(a;-3)(a;-7)=0  ? 

40.  Under  what  condition  is  the  square  of  x+3  equal  to  the  product 
of  X — 1  and  a; +6  ? 

41.  If  3(2a:— 1)  is  greater  than  12(a;— 3)  by  the  same  amount  that 
6x  is  greater  fhan  22,  find  x. 

42.  If  i:dx—4^n'^=ax-\-2a^,  what  is  the  value  oi  x  1 

43.  The  lever  in  the  diagram  is  balanced  by  the  weights  P  and  Q, 

when   Pa=Qb.     The  point  of   support  F  is  called 
a       F    ^  the  fulcrum.     If  P=10  lb.,  ^=15  lb.  and  a=12  in., 

[  I        what  is  the  length  of  6  ? 

P  Q  44.     Two   boys   balance   on   a   teeter  16  feet  in 

length.  The  heavier  boy  weighs  85  lb.  and  the  point 
of  support  is  6  feet  from  his  end  of  the  teeter.  Find  the  weight  of  the 
other  boy. 

45.  How  far  from  the  larger  weight  must  the  fulcrum  be  placed, 
if  weights  of  8  lb.  and  16  lb.  balance  at  opposite  ends  of  a  lever  12  feet 
long? 

46.  The  formula  C=f,{F— 32)  is  used  to  change  Fahrenheit  readings 
of  a  thermometer  to  Centigrade  readings.  If  F^lT,  find  the  value  of 
C. 

47.  Change  the  following  readings  to  Fahrenheit  readings  : 

0°C.,  40°C.,  100°C.,  -10°C.,  -50°C. 

48.  What  is  the  temperature  when  the  two  scales  indicate  equal 
numbers  ? 


SIMPLE   EQUATIONS  67 

50.     Equations  with  Fractional  CoefBcients. 
Ex.  1.— Solve  ^x-}-^x=20. 

Since  i+i=4,  .-.    ia;  =  20, 

.-.     a;  =  20^f  =  24. 

Instead  of  adding  the  fractions,  we  might  get  rid  of  them  by  multiply- 
ing each  term  of  the  equation  by  6. 

Then  ^xx  6  +  ^a;x  6  =  20  X  6, 

.-.  3a;+2a;=120, 
5a;  =120, 
a;  =  24. 

Verify  by  substituting  in  the  original  equation. 

Ex.  2.— Solve      |(a;+l)  +  i(x-+2)=i(a;+14). 

Multiply  each  quantity  by  12  (the  L.C.M.  of  2,  3,  4), 

.-.  h(x+  1)  X  12  +  i(^+2)  X  12  =  i(a;+  14)  x  12, 
6(a;+l)  +  4(x  +  2)  =  3(a;+14). 

Complete  the  solution  and  verify. 


Ex.  3.— Solve 


x—2      x—3      x—l 


5  6  10 


Multiply  by  30,     .'.   ^y^  x  30  -  ^^-^  x  30  =  ^^^  x  30, 

.-.    6(.-r-2)-5(x-3)  =  3(a;-7), 
.-.      6x-12-5a;+15  =  3a;-21, 

6a;-5a;-3a:=  12- 15-21, 
.-.  -2a;  =-24, 
a;=12. 
Verification  :  first  side  =  i^  —  »  =  2  -  1  ^  =  ^. 

second  side  =tV       =1- 

Note. — In  this  solution  the  beginner  is  advised  not  to  attempt  to 
omit  the  line  with  the  brackets.  He  may,  however,  omit  the  preceding 
line  when  he  feels  that  he  can  safely  do  so. 

51.  Steps  in  the  Solution  of  an  Equation.  In  solving  an 
equation  the  steps  in  the  work  are  : 

(1)  Clear  the  equation  of  fractions  by  multiplying  each  term 
by  the  L.C.M.  of  the  denominators  of  the  fractions. 

(2)  Remove  any  brackets  which  appear. 

F  2 


68  ALGEBRA 

(3)  Transpose  all  the  unknown  quantities  to  one  side  and  the 
known  quantities  to  the  other. 

(4)  Simplify  each  side  by  collecting  like  terms. 

(5)  Divide  each  side  by  the  coefficient  of  the  unknown. 

(6)  Verify  the  result  by  substituting  the  root  obtained  in  the 
original  equation. 


EXERCISE 

37 

Solve  and  verify : 

1.     %x=x-\-5. 

2. 

ix=^x+2. 

3.     ^x—^x=10. 

4. 

Ja;+ix+i^=26. 

5.     fa;+|-a;=a;+5. 

6. 

1         2x 

7.   iy=hy+h 

8. 

XXX 

8  +  4  +  2  =  ^-4. 

XXX 

«•     2-5  =  4  +  1- 

10. 

3m      Im 

X  X  4:X  ,  ,  „ 

13.     |  +  2  =  li  +  ^-|-  14.    ix-i+7x^3x+li. 

15.     |-j  =  2|.  16.    i{.T-3)=20. 

7a;+2      4a;— 1  x+l 

17.     ^-  =  -^--  18.    ^-3=^0. 

x      x—8  x—1      a:+3 

19.     5  +  --J— =  5.  20.     ——  +  -^  =  8. 
3         4  4  o 

21.     l{x-3)+^{x-5)=0.  22.     L(x-6)=U^+5)i-l(x-lS). 


Sx—l       5       a:      2a;+l         „^      a;+2  ,  ^      .r+4  ,   x+6 
"3"  +  12  =  4  +  "T" 

x—3      2a;-4      3rc-5 


23-     -^+12  =  4  +  -^-       2*-     -3-  +  2  =  ^+     7 


^25.     ^-  =  -5- +  -8--  26.     i(2/_3)-i(2/-5)=l. 


SIMPLE  EQUATIONS                                69 

27.  -±--—  =  1.  28.  ^--rr=o. 

z—2      x+2      x—3  x+1       1_         2a:— 1 

29.     —2---^=     3     •  3^-2     ~"i~'^~~"3~" 

x      5.T+9  _   2x— 9  X—  1  _  x—2  _  3— a; 

31.     --— g— --— g— .  32.     6 --2-      -3             ^• 

33.     5(a;-2)=3-65.  34.    2-34=4(a;+l-5). 

35.     "Sx— 3=-25x+-2a;.  36.     -2(0;— l)+-5(x-— 9)=3. 

3a;— 9      x+1      3a,— 14  ,           x+6      3a;— 16             a;+3 


Y 


2—x      3—x      4— X      5—x      3 


a;— 1      2— a;      2x— 1      2— 3x  " 

-f^'    -9--^ 14-  +  ^0-^^- 

52.  Problems  leading  to  Simple  Equations.  In  Chapter  II. 
we  saw  how  certain  arithmetical  problems  might  be  solved 
by  means  of  equations.  The  steps  in  the  solution  of  such 
problems  are  stated  in  art.  19,  to  which  the  pupil  should 
now  refer. 

The  beginner  will  find  his  chief  difficulty  with  step  4, 
in  which  he  is  required  to  translate  the  statements  given  in 
ordinary  language  into  algebraic  language. 

Some  examples  are  now  given  to  illustrate  how  this  trans- 
lation is  effected. 

Ex.  1. — Find  three  consecutive  numbers  whose  sura  is  63. 

If  we  let  x  represent  the  smallest  one,  what  would   represent  the 
others  ?     How  wo\ild  you  now  express  that  the  siun  is  63  1 
We  thus  obtain  the  equation  : 

a;  +  (a;+l)  +  (a;+2)  =  63. 

Write  out  the  full  solution  of  this  example  and  verify  the  result. 


70  ALGEBRA 

Ex.  2. — A  is  3  times  as  old  as  5  ;  2  years  ago  A  was  5 
times  as  old  as  B  was  4  years  ago.     Find  their  ages. 

Let  X  years  represent  B's  age. 
What  will  now  represent  -4's  age  ? 
What  will  represent  ^'s  age,  2  years  ago  ? 
What  will  represent  B's  age,  4  years  ago  ? 
Now  express  that  3rc  — 2  is  5  times  a;— 4. 
The  complete  solution  might  appear  thus  : 
Let  X  years  =  B's  age, 

3a;     ,,      =^'s  age, 
.'.    (Sx  — 2)     „      =  4 's  age,  2  years  ago, 
.".     (a;— 4)      ,,     =B's  age,  4  years  ago, 
.-.  3a;-2  =  5(a;-4), 
.-.  3x-2  =  5a;-20, 
18  =  2x, 
a;  =  9. 
.*.  jB's  age  is  9  years  and  ^'s  is  27  years. 

Ex.  3.— How  do  you  represent  3%  of  130  ?     4%  of  $27  ? 

5%  of  %x  ?     21%  of  $(a;+50)  ? 

Solve  the  problem  :  "  Divide  $620  into  two  parts  so  that 
5%  of  the  first  part  together  with  6%  of  the  other  part  will 
make   $34." 

Let  $a;=the  first  part, 

$(620  — a;)  =  the  other  part, 
r^TT  of  $a;=5%  of  the  first  part, 
,-.    -xU  of  $(620-x)  =  6%  of  the  other  part, 
•••  Tl«^+TfT(620-.T)  =  34, 

5.T  + 6(620 -a;)  =  3400. 
Complete  the  solution  and  verify  the  result. 

Ex.  4.— What  is  the  excess  of  73  over  50  ?  What  is  the 
defect  of  30  from  50  ?  What  is  the  excess  of  x  over  50  ? 
The  defect  of  x  from  89  ? 

Solve  the  problem  :  "  The  excess  of  a  number  over  50 
is  11  greater  than  its  defect  from  89.     Find  the  number." 


SIMPLE   EQUATIONS  71 

Let  x  =  the  number, 

then  a;  — 50  =  its  excess  over  50,  • 

and  89  — x  =  its  defect  from  89 

.-.    a;-50  =  89-a;+ll. 

Complete  the  solution  and  verify. 

Ex.  5. — The  value  of   73   coins   consisting  of    10c.   pieces 
and  5c.  pieces  is  $5.     How  many  are  there  of  each  ? 

Let  .r  =  the  number  of  10c.  pieces, 

.".    73  — x=    ,,  „         ,,      5c.        ,, 

The  value  of  the  10c.  pieces  =  lOx  cents. 
The  value  of  the  oc.  pieces  =  5( 73  — x)  cents, 
.-.     10x+5(73-a;)  =  500. 

Complete  the  solution  and  verify. 

The  pupil  should  be  careful  to  express  each   term  of  the 
equation  in  the  same  denomination. 

Why  would  it  be  incorrect  to  say  that 
10a:+5(73-x)-5  ? 

SXBRCISE   38 

All  results  should  be  verified. 

1.  A  number  is  multiplied  by  23  and   117    is   then   added.     The 
result  is  232.     Find  the  number. 

2.  From  the  double  of  a  number  7  is  taken.      The  remainder  is 
95.     Find  the  number. 

3.  Three   times  a  number  is  subtracted  from  235  and  the  result 
is  217.     Find  the  number. 

4.  Five  times  a  number  with  33  added  is  equal  to  7  times  the 
number  with  18  added.     Find  the  number. 

5.  Find  a  number  such  that  the  sum  of  its  third  and  fourth  parts 
may  be  35. 

6.  A  has  SIO  more  than  3  times  as  much  as  B,  and  they  together 
have  §250.     How  much  has  each  ? 

7.  The  sum  of  two  numbers  is  81.      The  greater  exceeds  6  times 
the  less  by  4.     Find  the  numbers. 

8.  Find  a  number  whose   seventh  part  exceeds  its  eighth  part 
by  2. 


72  ALGEBRA 

9.     The  excess  of  a  number  over  42  is  the  same  as  its  defect  from 
59.     Find  the  number. 

10.  Find  3  consecutive  numbers  whose  sum  is  129. 

11.  Divide  114  into  three  parts  so  that  the  first  exceeds  the  second 
by  15  and  the  third  exceeds  the  first  by  21. 

12.  Divide   $176  among  A,  B  and  C  so  that  B  may  have  $16  less 
than  A  and  $8  more  than  C. 

13.  A  man  sold  a  lot  for  $2280  and  gamed  14%  of  the  cost.     What 
did  the  lot  cost  ? 

14.  Divide  420  into  3  parts  so  that  the  second  is  double  the  first 
and  the  third  is  the  sum  of  the  other  two. 

15.  A  man  buys  8  horses  at  $x  each,  5  at  ${x-\-5)  each  and  3  at 
${x+25)  each.     The  total  cost  is  $2020.     Fmd  x. 

16.  Find  a  number  which  exceeds  31  by  the  same  amount  that 
J  of  the  number  exceeds  1. 

17.  Fmd  a  number  which  when  multiplied  by  6  exceeds  35  by  as 
much  as  35  exceeds  the  number. 

18.  A  farmer  sells  7  cows  and  17  pigs  for  $754.     Each  cow  sells 
for  $70  more  than  each  pig.     What  is  the  price  of  each  cow  ? 

19.  If  10  be  subtracted  from  a  number,  40  more  than  J  the  remainder 
is  30  less  than  the  number.     Find  the  number. 

20.  Find  two  consecutive  numbers  such  that  the  sum  of  J  of  the 
less  and  J  of  the  greater  is  44. 

21.  Divide  46  into  two  parts  so  that  if  the  greater  part  is  divided 
by  7  and  the  other  by  3,  the  sum  of  the  quotients  is  10. 

22.  Divide  237  into  two  parts  so  that  one  part  may  be  contained 
in  the  other  IJ  times. 

23.  A  horse  was  sold  for  $116-25  at  a  loss  of  7%.     What  did  he 
cost? 

24.  The  difference  between  the  squares  of  two  consecutive  numbers 
is  17.  Find  the  numbers. 

\^  25i*  A  box  contains  two  equal  sums  of  money,  one  in  half-dollars 
and  the  other  in  quarters.  If  the  number  of  coins  is  30,  how  much 
money  is  in  the  box  ? 

r  26.  A  is  35  years  old  ;  B  is  1  years  old.  In  how  many  years  wiU 
A  he  twice  as  old  as  £  ? 


-t 


SIMPLE   EQUATIONS  73 

'Ti^   27.     My  age  in  20  years  will  be  double  what  it  was  10  years  ago. 
What  is  my  age  ? 

S<^  28.     J.  is  35,  £  is  7  and  C  is  5  years  old.     How  long  v.'ill  it  be  before 
A^s  age  is  the  sum  of  the  ages  of  B  and  C  ? 

29.  Find  three  consecutive  even  numbers  such  that  the  sum  of 
a  fourth  of  the  first,  a  half  of  the  second  and  a  fifth  of  the  third  is  17. 

L.       30.     A's  share  of  S705  is  i  of  iS's  and  B's  is  f  of  C's.     What  is  the 

share  of  each  ? 

31.  The  simple  interest  on  a  sum  at  2%  together  with  the  interest 
^^     on  a  sum  twice  as  large  at   3|%  is  $135  per  annum.     What  are  the 

sums  ? 

32.  Three  %  of  a  certain  sum  together  with  4%  of  a  sum  which 
is  850  greater  is  $12'50.     Find  the  sums. 

^33.  The  value  of  52  coins  made  up  of  quarters  and  ten-cent  pieces 
is  SIO.     How  many  are  there  of  each  ? 

34.  A  square  floor  has  a  margin  2  feet  wide  aU  around  a  square 
carpet.  The  area  of  the  margin  is  160  sq.  ft.  Find  the  dimensions 
of  the  room. 

35.  In  an}'  triangle  the  sum  of  the  angles  is  180°.  The  greatest 
angle  is  35°  larger  than  the  smallest  angle  and  10°  larger  than  the  other 
angle.     Find  the  angles. 

36.  The  length  of  a  room  exceeds  the  width  by  4  feet.  If  each 
dimension  be  increased  by  2  feet  the  area  will  be  increased  by  52  sq.  ft. 
Find  the  length. 

37.  If  I  walk  m  mUes  at  4  miles  per  hom*  and  7W+2  miles  at  3  miles 
per  hour,  the  whole  journe}-  will  take  15  minutes  longer  than  if  I  walked 
at  the  imiform  rate  of  3|  miles  per  hour.     Fmd  the  length  of  the  journey. 

38.  A  and  B  together  have  $65,  B  and  C  have  $100,  C  and  .1  have 
$95.     How  much  has  each  ? 

39.  State  problems  which  will  give  rise  to  the  following  equations : 

(1)  5a;-10=60.  (2)  4a;-a;=24. 

(3)  ?  +  |=a;-10.  (4)  23-5a;=4a:-4 

40.  A  fruit  dealer  buys  apples  at  the  rate  of  5  for  3  cents  and  sells 
them  at  the  rate  of  3  for  5  cents.  How  many  must  he  sell  to  gain 
SI -28  ? 


74  ALGEBRA 

41.  The  sum  of  two  numbers  is  147  and  J  of  the  less  is  9  greater 
than  \  of  the  other.     Find  the  numbers. 

42.  John  has  i  as  much  money  as  his  brother,  but  when  each  has 
spent  25  cents,  John  has  only  f  as  much  as  his  brother.  How  much 
has  each  ? 

53.     Algebraic   Statements  of  Arithmetical  Theorems.     If  we 

take  any  two  numbers,  say  23  and  13,  and  add  together 
their  sum  and  their  difference,  we  will  find  the  result  is  twice 
the  larger  number. 

Thus,  23+13=36  and  23-13=10, 

and  36+10=46,  which  is  twice  23. 

We  see  that  it  is  true  for  the  numbers  23  and  13,  and  we 
would  find  it  true  for  other  pairs  of  numbers,  but  we  are  not 
sure  it  is  true  for  all  pairs  of  numbers. 

By  the  use  of  algebraic  symbols  and  methods,  we  may  show 
that  the  statement  is  true  for  every  two  numbers. 

Let  the  larger  number  be  a  and  the  smaller  b. 
Their  sum  is  a  +  &  and  their  difference  is  a  —  b. 
But  (rt+6)  +  (a-6)  =  a+6  +  a-6  =  2a, 

and  2a  is  twice  the  larger  number. 

Thus  the  statement  (a+6)+(a— 6)=2a  represents  in  a 
brief  form  the  theorem  stated  at  the  beginning  of  this  article. 
Besides  stating  it  in  a  concise  form  it  shows  that  it  is  true 
generally. 

SXBRCISS  89 

Show  that  the  following  statements  are  true  for  all  numbers  : 

1.  The  sum  of  two  numbers  is  equal  to  their  difference  increased 
by  twice  the  smaller  number. 

2.  The  difference  between  the  sum  of  two  numbers  and  the 
difference  of  the  same  two  numbers  is  twice  the  smaller  number. 

3.  Half  of  the  sum  of  two  numbers  increased  by  half  of  their 
difference  is  equal  to  the  larger  number. 

4.  The  sum  of  two  numbers  multiplied  by  one  of  them  is  equal 
to  the  square  of  that  one,  plus  their  product. 


SIMPLE   EQUATIONS  75 

5.  The  square  of  the  sum  of  two  numbers  is  equal  to  the  square 
of  their  difference  increased  by  four  times  their  product. 

6.  The  sum  of  three  consecutive  numbers  is  equal  to  three  times 
the  middle  one. 

7.  If  two  integers  differ  by  2,  twice  the  square  of  the  integer 
between  them  is  less  by  2  than  the  sum  of  the  squares  of  the  two 
integers. 

8.  Read  the  statement  {a+b)^+{a—b)^=2{a^+b^)  without  using 
symbols  and  prove  that  it  is  true. 

EXERCISE  40  (Review  of  Chapter  VI) 

1.  What  is  an  equation  ?     An  identity  ? 

2.  Wliat  rule  is  followed  in  transposing  terms  ? 

3.  Solve  and  verify:   6x(2x+3)  =  (3x+2)(4:X+^). 

4.  Is  — ^ —  =  — -. —  an  equation  or  an  identity  ? 

5.  What  value  of  x  will  make  5{x  —  3)  —  4(x ~ 2)  equal  to  zero? 

_,  ,      x—7   ,   a;— 10      a;— 11    ,    ^ 

6.  Solve  — g-  H ^  =  — g h  2. 

7.  The  sum  of  two  numbers  is  50.  If  5  times  the  less  exceeds  3 
times  the  greater  by  10,  what  are  the  numbers  ? 

-J.  _L  3       5^;  -1-6      X 2 

8.  Show  that  x  —  I  -\ ^  =  — ^ 1-  —»-   is  true  for  all  values 

of  X. 

9.  What  value  of  x  will  make  the  product  of  5  — 3x  and  7  — 2a; 
equal  to  the  product  of  11  — 6a;  and  3  — a;  ? 

2x 3       3a; 4 

10.  If = 1-  -262,  find  x  correct  to  two  decimal  places. 

2-5  12-5 

11.  A  and  B  invested  equal  siims.  A  gained  S200  and  B  gained 
$2600.  If  B  then  had  3  times  as  much  as  A,  how  much  did  each 
invest  ? 

12.  From  a  cask  which  is  |ths  full,  36  gallons  are  drawn  and  it  is 
then  half  full.     How  much  will  the  cask  hold  ? 

13.  Show  that  a; =6  is  a  root  of 

(a;-l)(a;-2)(a;-3)  =  2a;(a;-5)(2a;-7). 

14.  A  man  has  $115  in  $2  bills  and  $5  bills.  If  he  has  35  bills 
altogether,  how  many  of  each  has  he  ? 


76  ALGEBRA 

^K      Tf  3a;— 20a      5a;— 6a  ,    c   j 

15.  If ^        H r —  =  31  and  a  =  ^,  find  x. 

16.  In  a  stairway  there  are  45  steps  of  equal  heights.  If  they  had 
been  one  inch  higher,  there  would  have  been  only  40  steps.  How  high 
is  each  step  ? 

in      c  1      ^—4      a;  — 5      x  —  2 

17.  Solve— --g-  =  —  . 

18.  Divide  150  into  two  parts  such  that  if  the  smaller  be  divided 
by  23  and  the  other  by  27  the  sum  of  the  quotients  will  be  6. 

19.  The  difference  between  the  squares  of  two  consecutive  numbers 
is  51.     Find  the  numbers. 

20.  A  father  is  30  years  older  than  his  son ;  five  years  ago  he  was 
four  times  as  old.     Find  the  son's  present  age. 

2a;  4-3  a;  4-5 

21.  If  the  sum  of  the  fractions      — —  and  — —    is  9,  what  is  the 

numerical  value  of  each  fraction  ? 

22.  Show  that  the  difference  between  the  squares  of  any  two 
consecutive  nmnbers  is  equal  to  the  sum  of  the  numbers.  Show  also 
that  the  sum  of  their  squares  is  one  more  than  twice  their  product. 

23.  Solve  2-(a;-4  +  3a;-5)  =  10-a;. 

24.  If  the  product  of  a; +  2  and  2a; +5  is  greater  than  the  product 

of  2a;-f  1  and  a:  +  3  by   127,  find  x. 

25.  Solve  |(2-3a;)-|(a;-4)==|-(a;-5). 

26.  Divide  -75  into  two  parts  so  that  three  times  the  greater 
exceeds  six  times  the  less  by  -75. 

„„       o  ,       a;-3   ,   2  +  a;       l-2a;       ^ 

27.  Solve  —;, h  ~ r?-  =  <>• 

o  3  15 

28.  A  man  walked  a  certain  distance  at  3  miles  per  hour  and  returned 
by  train  at  33  miles  per  hour.  His  whole  time  was  4  hours.  How  far 
did  he  walk  ? 

29.  Prove  the  accuracy  of  the  following  statement  :  "  Take  any 
number,  double  it,  add  12,  halve  the  result,  subtract  the  original 
number,  and  6  will  remain." 

30.  Solve  I' +  ^^  -  ^^  =  a;  -  8. 

31.  How  many  minutes  is  it  to  10  o'clock  if  three-quarters  of  an 
hour  ago  it  was  twice  as  many  minutes  past  8  ? 

32.  What  value  of  a  will  make  2(6a;-fa)-3(2a;+a)  =  4(Hx-6) 
an  identity  ? 


SIMPLE   EQUATIONS         '  77 

33.  Solve  (6a;-2)(2a;-l)-(4x-2)(3x-2)  =  4. 

34.  A  rectangular  grass-plot  has  its  length  5  yards  longer  than  its 
width.  A  second  plot,  of  equal  area,  is  5  yards  longer  and  3  yards 
narrower  than  the  first.     Find  the  dimensions  of  the  first. 

35.  Solve  (a;+l)(a;+2)  +  (a;+3)(a;+4)  =  2a;(a;+ 12). 

36.  A  man  leaves  his  property  amounting  to  $7500  to  be  divided 
among  his  wife,  two  sons  and  three  daughters.  A  son  is  to  have  twice 
as  much  as  a  daughter,  and  the  wife  §500  more  than  all  the  children 
together.     Find  the  share  of  each. 

37.  Solve—  +-g ^  =  0. 

38.  Find  an  integer  whose  square  is  less  than  the  square  of  the  next 
higher  integer  by  37. 

39.  if  — - —  exceeds  — : —  bv  — -— ,  find  x. 

3  4         -^       6 

40.  How  far  can  I  walk  at  3  miles  per  hour  and  return  on  a  bicycle 
at  10  mUes  per  hour  and  be  absent  6  hours  4  minutes  ? 

41.  A  man  invested  J  of  his  money  at  3%,  j  at  4%,  i  at  5%  and  the 
remainder  at  6%.  If  he  receives  an  annual  income  of  §516,  how  much 
did  he  invest  ? 

42.  Prove  that  the  product  obtained  by  multiplying  the  sum  of 
any  two  numbers  by  their  difference  is  equal  to  the  difference  of  their 
squares. 


CHAPTER  VTI 
SIMULTANEOUS  EQUATIONS 

54.     Equations  with  two  Unknowns. 

The  sum  of  two  numbers  is  10.     What  are  the  numbers  ? 

It  is  evident  that  there  are  many  different  answers  to  this 
problem.  The  numbers  might  be  1  and  9,  2  and  8,  3  and  7, 
etc.,  or  J  and  9|,  —3  and  13,  etc. 

If  we  are  also  given  that  the  difference  of  the  numbers 
is  4,  then  only  one  of  these  answers  will  satisfy  this  new 
condition.     The  numbers  would  evidently  be  7  and  3. 

If  we  follow  the  method  previously  adopted  and  represent 
the  required  numbers  by  x  and  y,  where  x  is  the  greater,  the 
first  condition  would  be  expressed  by  the  equation 

x-\-y=10. 

As  stated,  any  number  of  pairs  of  values  of  x  and  y  will 
satisfy  this  equation. 

If  the  second  condition  be  expressed  in  terms  of  the  same 
unknowns,  we  have  another  equation 

x—y=4t. 

It  is  now  required  to  find  a  pair  of  values  of  x  and  y  which 
will  satisfy 

x-\-y=lO, 
and  x—y=  4. 

If  we  add  the  corresponding  sides  of  the  equations  we  get : 

2a'=14,  .•.  a;=7  and  /.  y—3, 

.'.  7  and  3  are  the  required  numbers. 


SIMULTANEOUS   EQUATIONS  79 

55.  Simultaneous  Equations.  Any  equations  which  are  satis- 
fied by  the  same  vakies  of  the  unknowns  are  called 
simultaneous  equations. 

Thus,  x=^l,  y=3  satisf}-  both  of  the  equations 

x-\-y^lO  and  x—y=4. 

To  find  a  definite  pair  of  values  of  x  and  y  it  is  seen  that 
we  must  have  two  equations  contaming  these  letters.  To  solve 
any  problem  \Ahere  two  numbers  are  to  be  found  we  must 
have  two  conditions  given,  from  which  the  required  equations 
may  be  obtamed. 

Ex.  1. — If  5  men  and  4  boys  earn  $43  in  a  day,  and  3  men 
and  4  boys  earn  $29  in  a  day,  what  sum  does  each  earn  in 
a  day  ? 

Why  do  the  first  set  of  workers  earn  more  than  the  second  ?  How 
much  more  do  they  earn  ?  How  much  then  does  one  man  earn  ? 
How  can  we  now  find  how  much  a  boy  earns  ? 

We  might  solve  this  problem  algebraically,  thus  : 

Let  $a;=the  wages  of  a  man  for  a  day, 

and  Sy  =  the  wages  of  a  boy  for  a  day. 

The  conditions  of  the  problem  would  now  be  expressed  algebraically 
by  the  equations  : 

$5x+  Uy=  $43, 
$3a;+  Uy=  $29. 
Or,  omitting  the  $  sign  and  using  only  the  numbers, 

5a;+4i/  =  43, 
3a; +42/ =  29. 

Subtract  the  terms  of  the  second  equation  from  the  corresponding 
terms  of  the  first, 

.-.    2a;=14, 
.-.     x=    7. 

Substitute  a;  =7  in  the  first  equation  and 

35 +  41/ =  43, 

.-.    41/=    8, 

.-.    y=  2. 
.".  the  roots  of  the  equations  are  x  =  l,  y  =  2, 
.'.  a  man  earns  $7  and  a  boy  $2  per  day. 

Verify  by  showing  that  these  results  satisfy  the  conditions  of  the 
given  problem. 


80  ALGEBRA 

Ex.  2.— For  3  lb.  of  tea  and  2  lb.  of  sugar  I  pay  $1-30, 
and  for  5  lb.  of  tea  and  4  lb.  of  sugar  I  pay  $2-20.  What  is 
the  price  of  one  pound  of  each  ? 

How  does  this  problem  differ  from  the  preceding  ? 

What  change  might  we  make  in  the  first  statement  so  that  the 
number  of  pomids  of  sugar  would  be  the  same  as  in  the  second 
statement  ? 

Let  X  cents = the  price  of  a  lb.  of  tea, 

and  y  cents  =  the  price  of  a  lb.  of  sugar. 

Then  Zx  +  2y=\Z0,  (1) 

and  5a;+42/  =  220.  (2) 

Multiply  the  first  equation  by  2  and  we  get 

6a;+42/=260,  (3) 

5a;+4i/  =  220.  (2) 

Now  solve  (2)  and  (3)  as  in  the  preceding  example  and  verify  the 
results  you  get. 

Ex.  3.— Solve  3a;+42/=39,  (1) 

4a;+3?/=38.  (2) 

Multiply  (1)  by  4  and  (2)  by  3  and  we  get 

12a;+162/=156, 
12a;+   9?/=  114. 

Complete  the  solution  and  verify. 

Ex.  4.— Solve  5:r-22/=44,  (1) 

3a-+42/=42.  (2) 

Multiply  (1)  by  2,  10x-42/  =  88.  (3) 

To  get  rid  of  the  term  containing  y,  we  must  now  add  instead  of 


subtract.     When  we  do  so 

13x=130 

.-.     x=    10, 

Substitute  a;=10  in  (1)  and 

2/=      3. 

56.  Elimination.  In  all  of  the  preceding  examples  the 
object  has  been  to  get  rid  of  one  of  the  unknowns,  so 
that  we  might  have  an  equation  with  only  one  unknown. 
The  process  by  which  this  is  done  is  called  elimination. 


SIMULTAI^EOUS   EQUATIONS 


81 


Thus  in  Ex.  4  we  eliminated  the  y.  We  might  have  eUminated  the 
X  equally  well. 

Solve  Ex.  4  by  first  eliminating  the  x. 

After  performing  the  necessary  multiplications,  when  do  we  add 
and  when  do  we  subtract  to  eliminate  the  unknown  ? 


EXERCISE  41 
Solve  for  x  and  y  and  verify  1-21  : 


1.     x+2y=8, 
x+  y=.5. 

4.     2x+3y=25, 
2x—3y=  7. 

7.     3x+5y=lS, 
2x+3y=l2. 

10.     x-{-y=4:, 
x—y=3. 

13.     3x-4y=16, 

7a; +3?/ =62. 

16.     3x=2y+  7, 
2x=3y-12. 

19.       2x+13y=  275, 
14x-17?/-1385. 


2.     3x+5y=l3, 
3x+2y=  7. 

5.     5x—2ij=18, 

2x—  y=^  7. 

8.     5x—&y^3\, 
Qx-3y=33. 

11.     3.T+  42/=  5, 
6a:+12?/=13. 

14.     2x+5?/=  0, 
3x-^y=23. 

17.     a;=3?/+20, 
i/=2.r-20. 


3.     6.r+5?/=23, 
3x+2?/=lL 

6.     5a;+2?/=24, 
2x+3y=U. 

9.     3x— 22/=24, 
2.r-3«/=ll. 

12.     3x+2?/=24, 
-2.T+3i/=10. 

15.     2y-3a:=-22, 
2x+3i/=32. 

18.     3x=2y, 

2x-5?/=-33. 


20.  2a;+3j/=  5a:-?/=17. 

21.  4a;— 52/=10y— 14a;=  — 10. 


22.*  If  bx—y=S  and  5^/— a;=20,  find  the  values  of  x^-y  and  a;— i/ 

23.  If  2a;-52/-31=62/-9a;+57=0,   find  the  value  of   \^x-\-\3y. 

24.  Solve  a;+3=4-2j/,  7(a;-l)  +  ll?/=6. 

25.  If  ax+by  equals  39  when  a  is  3  and  b  is  4,  and  equals  13  when 
a  is  5  and  6  is  —2,  find  x  and  y. 

26.  What  values  of  x  and  y  will  make  16x— //  and  4:X-\-2y  each 
equal  to  6  ? 


27.     Solve  2(x-y)+3(x+y)=3l,  3(2x-y)+5{x-2y)=53. 


82  ALGEBRA 

57.  Fractional  Equations  in  two  Unknowns.  If  the  equations 
contain  fractional  coefficients  of  x  or  y.  the  fractions  may  be 
removed  by  multiphcation. 

Ex.— Solve  h^-{-iy=  8,  (1) 

ia;+|y=32.  (2) 

Multiply  (1)  by  6,  3x+   2y=   48. 

Multiply  (2)  by  4,  a;+  10?/=  128. 

Complete  the  solution  and  verify. 


BXERCISS  42 
Solve  and  verify  1-20  : 


1. 

ix+ly=3, 

2, 

,  h^  +  y=  6, 

3. 

i{x+y)=9. 

x+  2/=7. 

.  +  1=14. 

l(a;-2/)=4. 

4. 

^  +  ^  =  14, 

5. 

ia;+3y=2, 

6. 

-  +  ^=15, 

8^3 

?  +  ^  =  24. 
9^2 

a;+4y==0. 

^      2/_    4 
4       5 

7. 

^^-iy=  i» 

8. 

^  +  ^  =  41, 
3^8 

9. 

^4-9v=    91. 
9         "^ 

fx+f2/=26. 

3x—4:y=  0, 

Qx  +  ^^  167. 

10. 

l6  +  24  ^  '' 

11. 

.=ii/. 

12. 

ia;+-li/=6, 

^       2/  _i 
4      12 

9y-nx=S0. 

2/-i(.-2/)=7. 

13.  •3x+-5^=-23,        14.  •lx+    3?/=  2-6,      15.  ■05x+-03y=29. 
6x-+ 5^=2-6.  a;-l-6y=10-2.  -OSx-'Oiy^  0. 

16.  a:  =  ^:il  =  ^=^.    17.  ?_^  =  ^-^  =  3.  18.  ^=.^^6-y 
3  2  5376  25  ^ 

19.  a;  +  |  =  2/  +  |  =  7.  20.  ^  _  |  =  3x  +  7^  +  26  -  6. 

o  o  o       4 


SIMULTANEOVJ    EQUATIONS  83 

21.*   ^  +  ^=2^,  22.  x+y-^=y+^{x+y), 

^  +  ^=  4^.  2y-x+l=i{2x+y+3). 

23.  x+y=y-2,  24.  5{x+y)-l{x-y)=26, 

y+lx=x+6.  (3x+7^)-^4-(6a:-2/)H-3. 

^„    cc+1      3y— 5      a;— « 
25.  8a;-72/=  12,  26.  ^  =  -^  -  -^^  • 

x—2y      2x—y  _  ^ 

4      "^      3 

27.  ly-\x+24.=fy+\x+ll=0. 


SXERCISE  48 

Solve,  by  using  two  unknowns,  and  verify : 
1.     The  sum  of  two  numbers  is  40  and  their  difference  is  12.     Find 
the  numbers. 

'   2.     The  sum  of  two  numbers  is  19.     The  sum  of  3  times  the  first 
and  4  times  the  second  is  64,     Find  the  numbers. 

3.  If  4  lb.  of  tea  and  7  lb.  of  sugar  cost  $2-42,  and  5  lb.  of  tea 
and  3  lb.  of  sugar  cost  $2*68,  find  the  cost  of  each  per  lb. 

4.  Find  two  numbers  such  that  7  times  the  first  is  greater  than 
twice  the  second  by  23,  and  5  times  the  first  and  3  times  the  second 
make  136. 

^5.  If  5  horses  and  6  cows  cost  $840,  and  3  horses  and  2  cows  cost 
$440,  find  the  cost  of  a  horse. 

6.  If  either  9  tables  and  7  chairs,  or  10  tables  and  2  chairs,  can  be 
bought  for  8156,  what  is  the  cost  of  each  ? 

7.  If  3  men  and  4  women  earn  $164  in  4  days  and  5  men  and  2 
women  earn  $135  in  3  days,  find  the  daUy  wages  of  a  man  and  of  a 
woman. 

8.  Find  two  numbers  such  that  \  of  the  first  and  J  of  the  second 
is  26,  and  ^  of  the  first  and  |  of  the  second  is  8. 

^9.  Three  bushels  of  wheat  cost  20  cents  more  than  5  bushels  of 
corn,  and  2  bushels  of  wheat  and  1  bushel  of  corn  cost  $2*30.  What  is 
the  price  of  each  per  bushel  ? 

G  2 


84  ALGEBRA 

10.  In  10  years  a  man  will  be  twice  as  old  as  his  son,  but  8  years 
ago  the  man  was  8  times  as  old  as  his  son.      Find  their  present  ages. 

"  11.  If  the  sum  of  two  numbers  be  added  to  3  times  their  difference 
the  result  is  18  ;  if  twice  the  sum  be  added  to  their  difference  the  result 
is  26.     Find  them. 

12.  A  merchant  seUs  33  suits,  some  at  $35  each  and  the  others 
at  $25,  and  receives  $945.     How  many  did  he  sell  at  each  price  2 

13.  Find  two  numbers  such  that  5%  of  the  first  is  greater  than  6% 
of  the  second  by  3,  and  7%  of  the  second  is  greater  than  4%  of  the  first 
by  7-5. 

14.  If  3  algebras  and  4  arithmetics  cost  $2*95,  and  2  algebras  and 
3  arithmetics  cost  $2*10,  find  the  cost  of  6  algebras  and  2  arithmetics. 

■~  15.  A  bull's  eye  counts  5  and  an  inner  4.  In  10  shots  a  marks- 
man scores  46  points,  each  shot  being  either  a  bull's  eye  or  an  inner. 
How  many  of  each  kind  did  he  make  i 

--.16.  A  classroom  has  25  seats,  some  double  and  some  single.  If 
there  is  seating  accommodation  for  42  pupils,  how  many  double  Seats 
are  there  ? 

.  17.  A  man  bought  8  cows  and  50  sheep  for  $900.  He  sold  the  cows 
at  a  gain  of  20%  and  the  sheep  at  a  gain  of  10%,  and  received  in  all 
Find  the  cost  of  a  cow  ? 


18.  If  10  men  and  8  boys  receive  $37,  and  4  men  receive  $1  more 
than  6  boys,  how  much  does  each  boy  receive  ? 

19.  A  man  bought  20  bushels  of  wheat  and  15  bushels  of  corn  for 
$36  and  15  bushels  of  wheat  and  25  bushels  of  corn,  at  the  same  rat«, 
for  $32-50.     How  much  did  he  pay  per  bushel  for  each  ? 

20.  Find  two  numbers  such  that,  if  the  first  be  increased  by  8  it 
wiU  be  twice  the  second,  and  if  the  second  be  increased  by  31  it  will 
be  three  times  the  first- 

21.  A  farmer  bought  100  acres  of  land  for  $4220,  part  at  $37  and 
the  rest  at  $45  per  acre.     How  many  acres  were  there  of  each  kind  ? 

22.  Find  two  numbers  such  that  7  times  the  greater  and  5  times 
the  less  together  make  332,  and  51  times  their  difference  is  408. 

23.  The  quotient  is  20  when  the  sum  of  two  numbers  is  divided 
by  3,  and  the  quotient  is  7  when  their  difference  is  divided  by  2.  Find 
the  numbers. 


SIMULTANEOUS   EQUATIONS  85 

/^  24.  A  grocer  bought  tea  at  60c.  a  lb.  and  coffee  at  30c.,  the  total 
cost  being  $96.  He  sold  the  tea  at  75c.  a  lb.  and  the  coffee  at  35c., 
and  gained  $21.     How  many  lb.  of  each  did  he  buy  ? 

25.  Three  times  the  greater  of  two  numbers  exceeds  twice  the  less 
by  90,  and  twice  the  greater  together  with  three  times  the  less  is  255. 
Find  the  numbers. 

26.  The  sum  of  two  fractions  whose  denominators  are  2  and  5 
respectively  is  2-9.  If  the  numerators  be  interchanged  the  sum  would 
be  4"1.     Find  the  fractions?     "  "^'- 

27.  Divide  142  into  two  parts  so  that  when  the  larger  part  is  divided 
by  17  and  the  other  by  19  the  sum  of  the  quotients  will  be  8. 

28.  A  farm  was  rented  for  $650,  part  of  it  at  $6  and  the  rest  at  $8 
per  acre.  If  the  rates  had  been  interchanged  the  rental  would  have 
been  $750.     How  many  acres  were  in  the  farm  ? 

29.  A's  age  3  years  ago  was  half  of  5's  present  age.  In  7  years 
the  sum  of  their  ages  will  be  77  years.     Find  their  present  ages. 

30.  A  man  travelled  240  miles  in  4  days,  diminishing  his  rate  each 
day  by  the  same  distance.  The  first  two  days  he  went  136  miles. 
How  far  did  he  go  each  day  ? 


EXERCISE  44  (Review  of  Chapter  VII) 

1.  Solve  2x+3y  =  38,  3x+2.y  =  37. 

2.  I  fire  20  shots  at  a  target.  If  a  hit  counts  5  and  a  miss  counts 
—  2,  how  many  hits  did  I  make  if  my  net  score  is  51  ? 

3.  Solve  7a: -2?/ =13,  2a; +3y  =  43. 

4.  The  average  marks  of  those  who  passed  an  examination  was 
65,  and  of  those  who  failed  was  25.  If  there  were  1000  candidates 
in  all  and  their  average  was  53,  how  many  passed  ? 

5.  Solve  2(a;-2/)  =  3(a;-4?/),  U{x+y)  =  ll(x+S). 

6.  At  an  election  ^'s  majority  was  384,  which  was  f",  of  the  whole 
number  of  votes.     How  many  votes  did  A  receive  ? 

7.  Solve  i(x+5)-5  =  J(2/  +  2),  ^{y  +  8)-S=l{x-S). 

8.  Divide  S5600  into  two  parts,  so  that  the  income  from  one  part 
at  3%  may  be  equal  to  the  income  on  the  other  part  at  4%. 

9.  Solve  I  +  I  =  3x  -  72/  -  37  =  0. 


86  ALGEBRA 

10.  Two  numbers  differ  by  11,  and  J  of  the  larger  is  1  more  than 
I  of  the  smaller.     Find  the  numbers. 

11.  If  px  +  qy  is  74  when  p  =  5  and  q  =  3,  and  is  76  when  p  =  6  and 
q  =  2,  find  x  and  2/. 

12.  If  3%  of  ^'s salary  plus  4%  of  B's  salary  is  $93,  and  5%  of  ^'s 
plus  3%  of  i?'s  is  $111,  find  their  salaries. 

13.  Solve  2l2/  +  20x=165,  77?/ -30a;  =  295. 

14.  Divide  100  into  two  parts  so  that  |  of  the  greater  part  exceeds 
^  of  the  less  by  2. 

15.  Solve  5x  —  22/=7cc+22/=a;+2/+ 11. 

16.  If  3  men  and  4  boys  earn  $26,  and  5  men  and  2  boys  earn  $34, 
what  would  7  men  and  3  boys  earn  ? 

17.  Solve  i(x+l)-i(2/+2)  =  3,  ^x  +  2)  +  i{y  +  3)  =  4. 

18.  If  3x~4:=ax-\-b  when  x=2  and  when  x=5,  show  that  a=3  and 
6=-4. 

19.  I  bought  a  horse  and  carriage  for  $400.  I  sold  the  horse  at 
a  profit  of  20%  and  the  carriage  at  a  loss  of  4%,  and  on  the  whole 
transaction  I  gained  5%.     What  did  each  cost  ? 

,     20.     Solve  ~~2y  =  2x-^-^  =  7. 

21.  A  man   pays  a  debt  of  $52   in  $5  bills  and   $1  bills.     If  the 
'^'number  of  bills  is   24,  how  many  are  there  of  each  ? 

22.  Solve  19a;- 21?/ =  100,  21a;- 19?/=  140. 

23.  ^'s  wages  are  half  as  high  again  as  B's,  but  A  spends  twice  as 
much  as  B.  If  A  saves  $5  and  B  $10  per  week,  what  are  the  wages 
of  each  per  week  ? 

24.  If  23a;+l%  =  91,  and  y  is  50%  more  than  x,  find  x  and  y. 

'  25.  When  a  man  was  married  his  age  was  A  more  than  his  wife's 
age.  His  age  8  years  afterwards  was  }  more  than  his  wife's  age.  How 
old  was  he  when  he  was  married  ? 

26.  If  3(5a;-22/)  =  2(3a;+62/),  find  a;  in  terms  of  y. 

27.  A  man  has  two  farms  rented  at  $5  per  acre  and  the  total  rent 
is  $1100.  When  the  rent  of  the  first  is  reduced  20%  and  the  second 
is  increased  20%,  the  total  rent  is  $1120.  How  many  acres  are  there 
in  each  ? 


28, 


SIMULTANEOUS   EQUATIONS  .     87 


If  ^  +  ^  =  ?  +  i^  =  9,  find  the  value  of  t  +  '{ ■ 
3       4       6        Id  I       z 


29,     Seven  years  ago  B  was  three  times  as  old  as  A,  but  in  5  years 
he  will  be  only  twice  as  old.     What  are  their  present  ages  ? 

31.     Solve  ?^  =  5  +  ^-±i,    €±?  ^  3  +  2±-^ 


CHAPTER  VIII 

TYPE  PRODUCTS  AND  SIMPLE  FACTORING 

58.  Factor.  When  a  quantity  is  the  product  of  two  or 
more  quantities,  each  of  the  latter  is  called  a  factor  of  the 
given  quantity. 

Thus,  the  factors  of  36c  are  3,  b  and  c. 
The  product  oi  b-\-c  and  a  is  ah  +  ac, 
.'.  the  factors  of  ab-\-ac  are  a  and  fe  +  c, 
or  ab-\-ac  =  a{b-\-c). 

Similarly,  ab  —  ac  =  a{b  —  c). 

When  a:-(-?/+2  is  multiplied  by  a,  the  product  ax-f-ay-\-az 
contains  the  factor  a  in  each  term. 

If  we  wish  to  factor  ax-\-ay^az,  we  recog- 
vm,Q  that  since  a  is  a  factor  of  each  term,  it    ")"^+"y+°^ 
must    be   a    factor   of   the  whole   expression. 
The  remaining  factor  is  the  quotient  found  by  dividing  the 
expression  by  a. 

Then  ax+ay+az=^a{dc-\-y-\-z). 

This  is  seen  to  be  similar  to  the  method  in  arithmetic. 
If  we  wish  to  factor  485,  we  see  that  5  is  a  factor.  How  do 
we  obtain  the  other  factor  ? 

Ex.— Factor  4a2_6a6. 

Here  we  see  that  2  and  a  are  factors  of  each  term  and  therefore  2a 
is  a  factor.     On  division  the  other  factor  is  2a  — 36. 
.-.   4o2-6a6  =  2a(2a-36). 
Similarly,  36a;4-6ca;=3a;(  ). 

ab—a^—a^—  a{  ). 

The  result  of  the  factoring  may  be  verified  by  multiplication 
and  this  may  usually  be  done  mentally. 


TYPE   PRODUCTS   AND   SIMPLE    FACTORING         89 

EXERCISE  45 
Fill  in  the  blanks  in  the  following  : 
1.     4x+6=2{     ).  2.     3a-9=3(     ). 

4.     ax+^x=^x{     ).        5.     bx—bi/=b(     ). 
7.     Tp^-6p=p{     ).      8.     6//2+3«/=3i/( 
Factor  the  following  and  verify  : 
10.     2y+4.  11.     6«t-12. 

13.     ab-i-ac.  14.     am—bm. 

16.     mx-^-my—mz.        17.     x'^—lx. 
19.     4a;3+6x-2+2a;.        20.     a^^+a^y-aS. 
22.     2aa:— 4a</+6az.      23.     a:2— 3a:2;/+x?/2. 

25.     {x+y)a+{x+y)b.  26.     a-(a-6)+2/(a-6). 

27.     2a;(6— c)— 2(6— c). 

59.  Definition.  An  algebraic  expression  containing  only 
one  term  is  called  a  monomial,  one  of  two  terms  is  called  a 
binomial,  one  of  three  terms  a  trinomial,  and  one  of  more 
than  three  terms  a  multinomial  or  polynomial. 

Thus,  2x — 5  is  a  binomial  and  o.^+3ct  +  7  is  a  trinomial. 

60.  Product  of  two  Binomials.     The  pupil  should   be   able 
to  write  down  mentally  the  product  of  two  simple 
binomials  hke  x+2  and  a;4-3.  x  +3 


3. 

5.r-10?/-5(     ). 

6. 

a;2-j-.r=a;(     ). 

9. 

8a;3-2.r2=2.r2(    ) 

12. 

3.r--15. 

15. 

ab-\-ac-{-a. 

18. 

5a2+10a6. 

2J. 

\5x^~\0xy. 

24. 

4a6+6a262_8ff6c 

What  is  the  source  of  the  first  term  {x^)  in  the  product  ? 
What  is  the  source  of  the  last  term   (6)  ?       What   two      ^^  +  2a; 
quantities   were   added   to   give  the    middle    term   (5a;)  ?  '        "*" 

How  were  these  two  quantities  obtaiiaed  ?  x^ 4- 5x4-6 

In  the  product  of  a;+ 1  and  x+1,  what  would  be  the 
first  term,  the  last  term,  the   middle  term  ?      What   is  the  complete 
product  ? 

In  the  product  of  x—2  and  x— 3,  what  is  the  first  term,  last  term, 
middle  term  ?  How  does  the  product  differ  from  the  product  of  x+2 
and  a;+3  ? 

Ex. — Multiply  x—5  by  .r+3. 

Why  is  the  last  term  negative  ?     The  middle  term  is  the  sum  of 
+  3x  and  —  5a;  or  —  2a;.     What  is  the  complete  product  ? 
What  is  the  middle  term  in  the  product  of  a; +5  and  x— 3  ? 


90 


ALGEBRA 


The  middle  term  in  every  case  is  seen  to  be  the 
sum  of  the  two  cross  products,  each  taken  wdth 
the  proper  sign. 

EXERCISE]  46  (1-22,  Oral) 
State  the  products  of : 
1.     x+1  2.     x+  5  3.     x—3  4. 

x+2  x+n  jc— 4 


x  +  5 


13. 


y—6 

y+5 

x+4:y 
x—3y 

ab-1 
ab—3 


a;+ll 

6. 

m—2 

m-\-4i 

10, 

y-5x 

y-\-5x 

14. 

xy—1 

xy+7 

a+  b 
a+2b 


11. 

P~  6g 
p+Uq 

15. 

pq-r 
pq+r 

12. 


X—  5 
a;-12 

x—Zy 
x-2y 

a— 2 
a—h 


16.     ax—2by 
ax—3bu 


17.     (a+2)(a+l).  18.     {x-y)(x-4.y). 

20.     {x—Zy){x+2y).      21.     (?H  +  4?j.)(m— 5/t) 

Remove  the  brackets,  simplify  and  check : 

23.     3(x+2)+2(3.r-l)-(x-3). 

{x+\)(x+2)  +  {x+2){x+Z). 

(2/+3)(2/-2)  +  (.v-5)(i/+4). 

(x+\)^+{x-\){x+\)+{x+\)(x-2). 

2(TO+l)('m+2)  +  3(TO— l)(m-2). 

4(a;+3)(a;+l)-(x-+l)(a:+12). 


19.     ip+q){p-q). 
22.     (b-3){b-^). 


24. 
25. 
26. 
27. 

28. 


61.  Factors  of  Trinomials.  The  product  of  two  binomials, 
like  those  in  the  preceding  exercise,  is  seen  to  be  a  trinomial. 

To  find  the  factors  of  a  trinomial  we  must  reverse  the 
process  of  muItipHcation. 

Ex.  1.— Factor  a;2+6.r+8. 

Since  the  last  term  is  positive,  the  last  terms  in  the  factors  must  have 
like  signs,  and  since  the  middle  term  is  positive,  the  signs  must  both 
be  plus. 

.•.  the  factors  are  of  the  form  {x-\-      )(x+      ). 


TYPE  PRODUCTS  AND  SIMPLE  FACTORING  91 

The  last  terms  in  the  factors  must  be  factors  of  8,  so  they  must  be 
1  and  8  or  2  and  4. 

x+l  x+2 

X+8  X+4: 

Which  of  these  when  multiplied  will  give  the  proper  middle  term  ? 
What  are  the  factors  of  a;^  —  6a;  +  8  ? 

The  factors  of  a;^  — 9x+14  must  be  of  the  form  {x—  ){x~  ). 
What  are  the  factors  ? 

Ex.  2.— Factor  a;2_2a;— 15. 

Here  the  factors  must  be  of  the  form  (x—  )(x-\-  ),  since  —15 
must  be  the  product  of  two  numbers  differing  in  sign.  The  possible 
combinations  are  : 

a;— 15  a;+15  a;— 5  a;  +  5 

a;+    1  a;—    1  a;  +  3  a;     3 


Which  of  these  sets  of  factors  is  tlie  correct  one  ? 

In  factoring  a  trinomial   like  a;^  — 8a;+15,  we   require    two   factors 
of  15  whose  algebraic  smn  is  —8.     They  are  evidently  —5  and  —3. 

.-.   a;2-8x+15  =  (a;-5)(.r-3). 

In    factoring   a;^  — 4.r— 21,    we   require   two    factors    of    —21    whose 
algebraic  sum  is  —  4,  and  they  are  evidently  —  7  and  3. 
.-.  a;2-4a;-21  =  (a;-7){a;+3). 

The  pupil  is  advised  to  write  the  factors  under  each  other, 
below  the  expression  he  is  attempting  to  factor. 


Thus, 


a;*  — 6a;— 16 


X           +2 

.-.  a;*-6a;-16  =  (a;- 

8)(x+2) 

] 

Factor : 

iJXERCI 

1.     a;2+8a;+7. 

2.     i 

4.     a^+22a+2l. 

5.     . 

7.     a^+3ab+2b^. 

8.     1 

10.     x2-5.r+6. 

11,     c 

13.     x^-4xy+3y^. 

14. 

a;^+  \\xy  —  ^2y^ 
X  -\-  14i/ 

X  —   3?/ 

.-.  a;2+llxz/-422/«=(a;+14i/)(x-3y). 


6.r+5.  3.  2/2+8?/+ 15. 

8a;+12.  6.  h^+\Qh-^24t. 

w2+77nn+10?i2.    9.  i/2  +  40.r?/+39a;2. 

a;2-7a;+6.  12.  a;2-12.c+ll. 

o2_lla6+2862.    15.  m^-lmn+\2n\ 


92  ALGEBRA 

16.     x^-x-2Q.  17.     ir-y-ZQ.  18.  a^+a-SO. 

19.     a;2— 5a;— 14.  20.     m^—Gm— 40.        21.  x^— lOx— 24. 

22.     a262+8a6  +  15.      23.     .tV-11^»/+30.   24.  a;«-10a;2+9. 

25.     a2_f_6a+9.  26.     a;2-14;i-+49.         27.  ?/«- 122/2+36. 

Use  factoring  to  simplify  the  following : 

^  a2+5a+4      a24,4a_5  ^^      m^— 5m+6  _  w^— 7m+12 

a+4  a+5  *  m— 3  w— 4 

(a;2+3a:+2)(a:-5)  Sx^-Ba;      2a:3-4x2      a:2-5x+4 

^2_3^_10       •        ^  •         3a;      ^      2x2      ^      a._l      • 

32.  What  factor  is  common  to 

(1)  a;2— a;-30  and  .r2— 2a;— 35  ? 

(2)  a^+ab  and  a2+3a6+262  ? 

Find  three  factors  of  : 

33.  2.T2-10.r+12.       34.     3a2+3a_36.         36.     a;3-8a;2+7a;. 

36.  If  the  expression  a;2+TOa;— 6  has  two  binomial  factors  with 
integral  coefficients,  what  are  all  the  possible  values  of  m  ? 

37.  Is  the  expression  ;<;2— 3a;— 10  factored  when  it  is  written  in  the 
forma;(a;-3)-10? 

62.  Square  Root  of  a  Monomial..  When  a  number  is  the 
product  of  two  equal  factors,  each  factor  is  called  a  square 
root  of  the  number. 

Thus,  16  =  4x4,  therefore  a  square  root  of  16  is  4. 
But  16=— 4x  —4,  therefore  a  square  root  of  16  is  also  —4. 
Similarly,  the  square  root  of  25  is  +5  or  —5, 

and  the  square  root  of  9a  ^  is  +3a  or  —3a. 

Thus  it  is  seen  that  every  number  has  two  square  roots 
differing  only  in  sign. 

It  is  customary  to  call  the  positive  square  root  of  a  number 
the  principal  square  root. 

63.  Radical  Sign.  The  symbol  y/~ ,  called  the  radical 
sign  or  root  sign,  is  used  to  indicate  the  principal  square 
root    of  a  number. 

Thus,  V25  =  5,  A/a^  =  a,  V'9¥V  =  3xy. 


TYPE  PRODUCTS  AND  SIMPLE  FACTORING  93 

When  both  the  positive  and  negative  square  roots  are 
considered,  both  signs  must  precede  the  radical  sign. 

Thus,  ^9  =  3  not  -3;  —V'9=  —3  not  +3,  but  ±V'9=±3,  and 
is  read  "  plus  or  minus  the  square  root  of  9  equals  plus  or  minus  3." 

Thus,  V4+V9=2+3  =  5, 

but  ±\/4±V'9=±2±3=±5or  ±1. 

If  we  represent  the  square  root  of  16  by  x,  then  x^=16. 
To  solve  this  equation,  take  the  square  root  of  each  side, 

.'.  x~±4. 
We  might  have  said    ±  x~  ±  4,   which  includes   the  four 
statements  : 

+a;=+4,  -{-X——4,  — x=+4,  — a;=— 4. 

If  both  terms  of  the  last  two  be  multiplied  by  — 1,  the 
statements  become  the  same  as  the  first  two.  which  are 
represented  by  a;— ±4. 

We  see  then,  that  it  is  necessary  to  attach  the  double  sign 
to  the  square  root  of  only  one  side  of  the  equation. 

Ex.— Solve  (.r-f  1)2=25. 

Take  the  square  root  of  each  side, 

.-.     a;+l  =  ±5, 

a;=±5-l  =  5-l  or  -5-1, 
=  4  or  -6. 

Show  by  substitution  tliat  each  root  satisfies  the  given  equation. 


E3XERCISE 

48  (1-16,  Oral) 

T 

State  the  two  square  roots  of : 

1.     36.                 2,     81. 

3. 

121. 

4. 

2i. 

5.     y^.                  6.     b^c^. 

7. 

25a2. 

8. 

UxhjK 

9.     ia2.             10.     Imhi^. 

11. 

25?*- 

12. 

6ix\ 

Solve  the  following  equations  : 

13.     x2=9.         14.     3x2^75. 

15. 

x2=^4a2. 

16. 

x^=a%\ 

17.*  (a;+2)2=81.                18.     ( 

[x-Sf. 

=49. 

19. 

(a;-5)2=0 

94  ALGEBRA 

20.  If  the  area  of  a  square  is  100  square  inches,  find  the  length  of 
its  side. 

21.  If  r  is  the  radius  of  a  circle  the  area  is  -rrr^,  where  7r=31 
approximately.  If  the  area  of  a  circle  is  154  square  inches,  what  is  the 
radius,  or  what  is  the  value  of  r,  if  3}r^=154:  ? 

22.  Find  the  radius  of  a  circle  whose  area  is  616  sq.  in. 

23.  If  y  is  the  radius  of  a  sphere  the  area  of  its  surface  is  given  by 
the  formula,  area  =47rr^.  If  the  area  of  the  surface  of  a  sphere  is 
154  sq.  in.,  what  is  the  radius  ? 

64.  Squares  of  Binomials.  If  we  multiply  x-^-y  by  x-}-y, 
the  result,  which  will  be  the  square  of  x-\-y,  is  x'^-\-2xy-\-y^. 

The    diagram    shows    a   geometrical  illus- 
tration of  this  identity. 

The  first  and  last  terms  in  x^-\-2xy-\-y^ 
are  the  squares  of  the  terms  of   x-\-y,  and 
the  middle  term  is    twice  the  product  of  x 
and  y. 
Therefore,  the  square  of  the  sum  of  two  numbers  is  equal  to 
the  sum  of  the  squares  of  the  numbers,  increased  by  twice  their 
product. 
Also  {jc—y)^=^oc^—2xy-{-y^. 

Therefore,  the  square  of  the  difference  of  two  numbers  is  equal 
to  the  sum  of  the  squares  of  the  numbers  decreased  by  twice  their 
product. 

In  the  square  of  a  sum  all  the  terms  are  positive,  and  in 
the  square  of  a  difference  the  middle  term  is  negative. 

Thus,  (3a  +  26)2  =  (.3a)2  +  2(3a)(26)  +  (26)2, 

=  9o2  +  12ab  +462. 
(5x-37/)2  =  (5x)2-2(5x){32/)  +  (3y)^ 

=  25a;='-  30xy  +9y^. 
(^x-4y"~)  =  {^xr^-2(hx){4y)  +  (4y)\ 


.r= 

.VJ 

XJ' 

y' 

TYPE  PRODUCTS  AND  SIMPLE  FACTORING  95 


EXERCISE  49  (1-16,  Oral) 

What  are  the  squares  of : 

1.  a+l.  2.     y+2.  3.  to— 1.  4.     x— 4. 

5,  2a+l.  6.     l-3x.  7.  p— 9.  8.     2a;-l-3. 

9.  2a— 3.         10,     m— 2tt.  11.  3x— 2^.  12.     4x— 3a. 

13.  |— X-.  14.     2^—^.  15.  3x— |.  16.     —a;— 2. 

Simplify : 

17.*  {x+l)^+ix-l)K  18.  (a-i)2-f(a+6)2. 

19.  {2x+iy-+{x~2)\  20.  (a+6)2-(a-6)2. 

21.  (3to— n)2-(2TO+/?)2.  22.  i3x+2y)^-{2x-3y)^ 

23.  (a;+l)2+(a;+2)2+(.r^3)2.      24.  {x-iy^+{x-2)^-{x-3)". 

25.  2(a+l)2+3(a-l)2-5(a-2)2. 

26.  Find  the  value  of  a--{-b^-\-c^  when  a=x—y,  b=^x-\-y,  c=x—2y 

27.  SimpUfy  {x+iy-+(x-2)^+{x-3)^-3{x-4:y-. 

28.  From  the  sum  of  the  squares  of  x-\-2,  x+3,  a; 4-4,  subtract  the 
sum  of  the  squares  of  a;— 2,  x—3,  x— 4. 

29.  SimpUfy  (2a-36)2-f  (3a+26)2-(2a4-26)2. 

30.  If  two  numbers  differ  by  2,  show  that  the  difference  of  their 

squares  is  equal  to  twice  their  sum. 

2 

31.  Bv  how  much  does  the  square  of  a; -f  -  exceed  the  square  of 

2, 

X • 

x 

32.  Show  that  the  sum  of  the  squares  of  three  consecutive  numbers 
is  greater  by  two  than  three  times  the  square  of  the  middle  number. 

33.  The  square  of  1234  Is  1,522,756.     Find  the  square  of  1235. 

34.  The  square  of  2^=2x3+^^6};  the  square  of  5^=5x6+}= 
30},  etc.  In  the  same  way  find  the  squares  of  6|,  8|,  20^.  Prove 
that  this  method  may  be  used  to  find  the  square  of  any  number  ending 
in  |.     (Let  the  number  be  %+i.) 

65.     Square  Roots  of  Trinomials.      Any   trinomial   which   is 
of  the  form  a'^-\-2ab-{-b'-  or  a'"—2ab'\-b^  is  a  perfect  square. 
In  order  that  a  trinomial  may  be  a  square,  the  first  and 


96  ALGEBRA 

last  terms  must  each  be  a  square  and  the  middle  terra  must 
be  twice  the  product  of  the  quantities  which  were  squared 
to  produce  the  first  and  last  terms. 

Thus,  Qx-  +  2'ixy-\-  \Qy^  is  a  square,  because 

(1)  9a;-  is  the  square  of  3rc, 

(2)  162/^  is  the  square  of  4^, 

(3)  24:xy  is  twice  the  product  of  3a;  and  ^y. 
:.    9x2  +  24x2/+ 162/2  =  (3a;  +  4?/)2. 

.'.   the  square  root  of  9a;2  +  24a;2/+162/2  is  3x-{-4y. 
Is  4m2— 12mn+9n2  a  perfect  square  ?      What  is  it  the  square  of  ? 
What  is  its  square  root  ? 

Similarly,  25a;2- 10a;+ l  =  (5a;- 1)2, 

36a;2  +  24a;  +  4  =  (  )2, 

a262_6a6  +  9  =  (  )2. 

Why  is  CT2  +  5afe  +  25&2  not  a  square  ?  Is  it  the  square  of  a  +  56  ? 
How  would  you  change  it  so  that  it  would  be  a  square  ? 

The  square  root  of  a^-^2ab-\-b^  is  a+6,  but  —{a-\-b)  or 
—a—h  is  also  a  squai'e  root,  since 

(-a-&)2=:a2^2a6+&2. 

It  is  customary,  however,  in  stating  the  square  root  of  a 
trinomial  to  give  only  that  one  which  has  its  first  term 
positive. 

BXSRCISE!  50  (1-24,  Oral) 

Express  as  squares : 

1.     x^-^2xy+y\  2.  y^—2y+l. 

4,     4a2+20a+25.  5.  9a2_24a+16. 

7.     a%'^—2ah+\.  8.  \—&y+%if. 

10.  a%'C'—2abc+\.      11.  x^+x+J. 
What  is  the  square  root  of  : 
13.     9a2+12a+4.            14.  a;^— 4a;?/+42/2. 
16.     4a262_20a6+25.     17.  4m2+2m+i. 
19.     4— 4a+a2.               20.  9-12a:+4a;2. 
Supply  the  missing  terms,  so  that  the  following   will    be  perfect 

squares : 

11.  a2+.  .  .+6'^.         23.     .^2— .  .  .+42/2.      24.     x^+^x  .      .  . 

25.     4m2— .  .  .+9.       26.     9a2+18a.  ...     27.     .  .  .—^xy+'^yK 


3. 

4a;2+4a;+l. 

6. 

16a;2-8a;+l. 

9. 

9a;2— 18a;2/+92/2. 

12. 

y^—xy+lx\ 

15. 

1— 6a;+9.r2. 

18. 

a2_i4a6+4962. 

21. 

9a;2-30a;y+252/' 

TYPE  PRODUCTS  A  AW  SIMPLE   FACTORING 


97 


a  +b 
a  —b 

a^  -\-  ab 
-ab-b^ 


28.  If  16a2— ma+4  is  a  perfect  square,  what  is  the  vakie  of  m  ? 
Give  two  answers  and  verify  each. 

29.  What  is  the  square  root  of  9a;^-(-6x'+l  ?     Check  by  putting 
a;=10. 

30.  Solve  the  equations  and  verify  : 

(1)  Vx'^+2x+l  +  Vx-+10x+25=^U. 

(2)  SVx^—4:X+4:—2Vx^+6x+9=  —2. 

(3)  V'9x2+6x+l  +  \/4x2+4x+i  +  v'a;2— 2x+l  =  13. 

31.  Show  that 
2Va^—6a+9—Va^-4a+4:^SVa^—2a+l  —  Via^+'ia+l. 

66.  Product  of   the   Sum   and   Difference.     The   product  of 
a-\-b  and  a  —  h  is  a^—h", 

:.    {a-Vh){a-h)=a^-b^. 

Here  the  two  factors  multipHed  are  the  sum 
and  difference  of  the  same  two  quantities  a 
and  h,  and  the  product  is  the  difference  of  the 
squares  of  a  and  h. 

Therefore,  the  product  of  the  sum  and  dijference  of  the  same 
two  quantities  is  equal  to  the  difference  of  their  squares. 

Thus,  {x-\-y){x—y)  =  x'^  —  y^. 

(2a+36)(2a-36)  =  (2a)2-(36)2  =  4a2-962. 
(3a2-6)(3a2+6)  =  (3a2)2-62  =  9a4_{,2, 

(i  +  3a;)(J-3x)  =  (i)2-(3x)2=i-9x2. 

67.  Factors  of  the  Difference  of  Two  Squares.  ^.^ 

Since  a'^—h'^=^{a^h){a—h),  the  factors  of 
the  difference  of  two  squares  are  the  sum  and 
the  difference  of  the  quantities  squared. 

The  diagram  shows  how  this  identity  may 
be  ilhistrated  geometrically. 

Thus,  9x2- 251/2  =  (3a;)2-(5i/)2,  which  shows  that 
it  is  the  difference  of  the  squares  of  3x  and  5y. 

Therefore  one  factor  of  9x'  —  25y^  is  the  sum  of  3x 
and  5y,  and  the  other  is  the  difference  of  3x  and  5y. 

That   is,  9x'^-25y^  =  {dx  +  5y){3x-5y). 

Similarly,  16?H2-9  =  (4m)2-32  =  (4TO+3)(4m-3). 


(--  -  1 
I   -v    ] 

X        1 

h  I 


98  ALGEBRA 

If  we  wish  to  factor  8x^—2?/^,  we  should  recognize  that  2 
is  a  factor  of  each  term. 

.-.     8x^-2y^  =  2(4:x'^-y^)  =  2(2x  +  y)i2x-y). 


EXERCISE  51  (1-24,  Oral) 

State  the  products  of  : 

1.     m-{-n,  m—n.  2.    p—q,  p-^q-  3.     ct~\-2,  a— 2. 

4.     a:— 5,  a;+5.  5.     2a-f-l,  2a— 1.  6.     3x—2,3x+2. 

7.     {2a-3x){2a-\-3x).  8.     {4:X+5y){ix-5y). 

9.     {x+i){x-i).  10.     {x'—2y){x'~+2y). 

11.     {5x+ab){5x-ab).  12.     ( 2.r  -  | j  ( 2x  +  1). 

State  the  factors  of  : 

13.     x^-1.  14.     ?/2— 4.         15.     a2_4j2  j^^     4w2— w2, 

17.     4j92_9^2_        -i8_     x2— |.         19,     9-xK  20.     1  — 16a262^ 

21.     25-49a;2.       22.     a*-25.       23.     a262-49.       24.     992-982. 

Simplify  : 

25.*  (a-2)(a+2)+(2a-l)(2a+l). 

26.  {2a-3b)(2a+3b)-{a+b)(a-b). 

27.  2{x-3y){x+3yn2(3y~x){Sy+x). 

28.  2(p-qr^  +  3(p+q)ip-q)-5{p  +  2q)ip-2q). 

29.  Find  the  product  of  x—a,  x^a  and  a:2-|-a2_ 

30.  From  the  product  oi  x—l,  x-\-\  and  x^-\-\,  subtract  the  product 

of  x—2,  x-^2  and  a;2-|-4. 

Find  three  factors  of : 

31.  3x2— 3?/2.  32.     5x2—20.  33.     a^—a. 
34.     mx^—ma^.             35.     5— 45^^.  36.     x*—y*. 

37.     7r7?2_7rr2.  38.     a{x-—l)-Jrb{x^—l). 

39.     Why  is  the  difference  between  the  squares  of  any  two  consecutive 
numbers  always  equal  to  their  sum  ? 


TYPE  PRODUCTS  AND  SIMPLE  FACTORING  99 

40.  Simplify  (a"-b^){a--5ab^%b-)^{a^~Zab+2b'^). 

41.  SimpUfv  ""-^^  +  t^y^  and  ""^^^  -  ^!ll?- 

x—y         x-\-y  a-— 4         x+S 

42.  Solve  ^i!rd  +  •:^!l^  ^  10  ;  2(x--5)(a;+5)=15+(.r-I)(a;+l). 

x+l        a: — o 

68.     Numerical  Applications  of  Products  and  Factors. 

In  this  Chapter  we  have  developed  certain  formulaj 
concerning  products  and  factors. 

(1)  (a-6)2  =a2-2a6+6?. 

(2)  (a+6)2  =a2^2a6+62. 

(3)  {a+h){a-h)  =a^-b^. 

These  formulse  are  true  for  all  values  of  the  letters  involved. 
By  substituting  particular  numbers  for  the  letters  we  will 
see  how  some  arithmetical  operations  might  be  simplified. 

(1)  Since  {a-by  =  a^-2ab  +  b\ 

992=   (100-1)2=10000-200+1  =  10001-200  =  9801. 
372=     (40-3)2=    1600-240+9=    1609-240=1369. 
9982  =  (1000-2)2=  =  = 

892=     (90-1)2=  =  = 

(2)  Since  (a  +  6)2  =  a2  +  2a6  +  62, 

922=   (9o_^2)2=   8100+360+4=   8464. 
1212  =  (120+  1)2=  14400+240+  1  =  14641. 
752=   (70+5)2=  = 

(3)  Since  {a+b){a-b)  =  a"--b\ 

92x88  =  (90+2)(90-2)  =  902-22  =  8100-   4  =  8096. 
65x75  =  (70-5)(70+5)  =  702-52  =  4900-25  =  4875. 

27x23  =  (25  +  2)(25-2)=  =  = 

87x93  =  (  )(  )=  =  = 

(4)  Since  a^-b'  =  {a-Jrb){a-b), 

532-   522  =  (53  +  52)(53-52)  =  105x    1  =  105. 
412-   3i2  =  (4i-|-3i)(4i_3i)=   72x10=720. 
.272-6272  =  (  )(  )=  = 

672-   332  =  (  )(  )=  ^ 

H  2 


a 


100  ALGEBRA 

69.     Some  Geometrical  Applications. 

(1)  If  a  is  the  length  of  the  side  of  the  large  square  and 
h  the  side  of  the  small  square,  the  area  of  the  shaded  portion 
is  evidently  a^—b^. 

If  we  wish  to  find  the  area  of  the  shaded  part  when 
a  =  ll  and  6  =  23,  we  have 

a2- 62  =  772- 232  =  (77  + 23)(77- 23)  =100x54  =  5400. 

If  a  =  225  and  6=125,  find  the  difference  in  the  areas 
of  the  two  squares. 

(2)  The  radius  of  the  large  circle  is  R  and  of  the  small 
circle  is  r.  The  area  of  the  large  circle  is  -y-iJ^  and  of  the 
small  one  is  -y-r^, 

.'.   the  area  of  the  shaded  part  is  '^f-{R^—r'^). 

If  i?  =  39  and  r  =  31,  find  the  area  of  the  ring. 
The  area  =-V-(i?2_r2)  =  ^^(392-312) 

=  -V{39  +  31)(39-31)  =  -V-X  70x8=1760. 
If  i?  =  89  and  r=82,  show  that  the  area  of  the  ring  is  3762. 

(3)  In  the  right-angled  triangle  in  the  figure  it 
^  is  shown  in  geometry  that 

62-f  c^^a^  or  b^=a^—c^  or  c^=a^—b". 

If  a  =  41  and  c  =  40,  find  the  length  of  6. 

62  =  a2_c2  =  4i2_402  =  81xl  =  81, 

.-.    b=Vsi  =  9. 

If  a  =  61  and  6=11,  show  that  c  =  60. 

BXERCISE  52 

Use  short  methods  in  the  following  : 

1.  Find  the  squares  of  98,  999,  119,  58,  799. 

2.  Find  the  products  of  91  x  89,  61  X  59,  47  X  53,  203  x  197. 

3.  Find  the  values  of  522-48^,  79^-782,  215^-2052,  7252-2752, 
6732-5732. 

4.  If  x^=b^—c\  find  x  when  6=13,  c=12  ;   when  6=25,  0=24. 

5.  If  7a;2=642-572,  find  the  value  of  x. 


TYPE  PRODUCTS  AND  SIMPLE  FACTORING        101 


6.     Find  the  difference  of  the  areas  of  squares  whose  sides  are 
a  and  b  for  the  following  values : 


a— 

41 

13         29 

83 

15m 

2-85 

b= 

40 

12 

21 

17 

14to 

2-15 

a^~b^^ 

7.     Find  the  difference  in  the  areas  of  circles  whose  radii  are  R  and 
/•  for  the  following  values  : 


R= 

4 

14 

25 

51 

19a 

3-25 

r— 

3 

7 

24 

44 

5a 

2-35 

3l{R'--r'~)= 

Factor 

1. 

4. 

7. 
10. 
13. 
16. 


EXERCISE  53  (Review  of  Chapter  VIII) 


18. 


19. 


3x+6y. 
6ac  —  36c. 
4a2_9:c2. 
2a^-lS. 

a^-b*.  17. 

Write  down  the  squares  of : 

2a;— 3,  Sx— 6,  4x-:iij,  a-\,  bc  —  h 
What     are     the     square     roots  of  :     a^  +  Ga  +  d,     p^—8p-]-l6. 


2.     4m-12n. 

3.     ax  —  bx. 

5.     a;2  +  4x+4. 

6.     a2-2o+l. 

8.     x^-3x+2. 

9.     2/2-2/-110. 

11,       100/32- 81r/2. 

12.     a2-19a-20. 

14.     300-3x2. 

15.     (x  +  2/)2-l. 

Solve  x2=  100;  x^-  = 

\;  9x^  =  4;  5x2=1- 

m2n2-10mn  +  25,  a'^-a-{-\,  \Qx--^Qxy  +  -25y^  ? 

20.  How  much  must  be  added  to  the  middle  term  of    4:a--\-Za-{-Q 
to  make  it  the  square  of  2a  4- 3  ? 

21.  What    middle    term    must    be    inserted    in   Ox"  .  .   .    +25?/2  to 
make  it  a  complete  square  ?     Give  two  answers. 


102  ALGEBRA 

22.*  Find  three  factors  of  x^  —  x,  3a;*— 12,  a^  —  3a^-\-2a. 

23.  Solve  (x-3)2  =  25;  4(x-^)2  =  9. 

24.  If  a  =  irr^,  find  r  when  a=  12-56,  7r  =  314. 

25.  Find  the  values  of  9972,  8752-75^,  97x103,  81x81,  86x94, 
using  algebraic  methods. 

26.  Find   four   factors    of    2a;^-32,    a*-13a2  +  36,    2m3-18TO   and 
o2(a;2-2/2)-62(a;2-2/2). 

27.  Simplify  {b+l)^  +  {b-iy  +  (c+iy  +  {c-l)^. 

28.  Simplify  {x  +  y)(x~-y)  +  (x  +  2y)(x-y)  +  (x  +  y)(x-2y). 

29.  If  a  =  92  and  6  =  88,  find  the  values  of  ab,  a^-b^,  a^  +  b^,  using 
algebraic  methods. 

30.  Simplify  (a-26)(a  +  26)  +  (a-46)(a  +  4fe)-2(o-36)2. 

31.  What  are  all  the  possible  values  of  6,  if  x^-\-bx-\-4:2  is  the  product 
of  two  factors  with  positive  integral  coefficients  ? 

32.  Simplify  ^^^^'  +  ^^'  +  "'-^^'  - 


x—y         x~2y  x—3y 

33.     If  the  square  of  426  is  181476,  find  the  squares  of  427  and  425. 


CHAPTER  TX 

SIMPLE  APPLICATIONS  OF  FACTORING 

70.  Highest  Common  Factor.  \^nien  a  factor  divides  two 
or  more  expressions  it  is  called  a  common  factor  of  those 
expressions. 

Thus,  4  is  a  common  factor  of  8,  12  and  20, 

and  a  is  a  common  factor  of  a^,  2a  and  Sab. 

As  in  arithmetic,  the  highest  common  factor  (H.C.F.)  is 
the  product  of  all  the  simple  common  factors. 

Thus,  the  simple  common  factors  of  3a -6,  606^  and  9abc  are  3,  a  and  b, 
and  therefore  the  H.C.F.  is  Sab. 

In  the  case  of  monomials  the  H.C.F.  may  be  written  down 
by  inspection. 

Ex.  1.— Find  the  H.C.F.  of  Qm^n,  Urnhi^  and  dmhi^. 

(1)  The  H.C.F.  of  6,  12  and  9  is  3. 

(2)  The  highest  power  of  m  which  is  common  is  m^. 

(3)  The  highest  power  of  n  which  is  common  is  n. 

:.  the  H.C.F.  is  SxOT^xn  or  Sm^n. 

If  the  expressions  are  not  monomials  they  must  be  factored 
when  possible,  after  which  the  H.C.F.  may  be  written  down 
by  inspection. 

Ex.  2.— Find  the  H.C.F.  of  a^+ab,  ab+b^,  a^-\-3abi-2b2. 

a--\-ab=^a{a+b), 

ab  +  b^  =  b{a  +  b), 

a^  +  3ab+2b^  =  {a  +  b){a  +  2b), 

:.     the  H.C.F.  =  a+6. 

103 


104  ALGEBRA 

BXEROISE  54  (1-12,  Oral) 

Find  the  H.C.F.  of : 
1.     3,9,12.  2.     16,24,40.  3.     2«,  46,  8c. 

4.     3x,  6x,  \2x.  5.     4.ax,  Qax,  2x^.  6.     a%,  ab\  a%\ 

7.     3x2,  4^.3^  5^4  8^     5^2^  loa^,  15a.        9.     llx^y-,  Mxhj,  blx^. 

10.     2a,  a2+a6.  11.     Gx^,  4x2+2x.  12.     (a+6)2,  a^— 62. 

13.*  2a+46,  3a+66.  14.  a^-h"^,  ah-h\ 

15.     m2— n2,  7n^—2nin-\-n^.  16.  x^-\-xy,  xi/^i/^,  {x-{-y)^. 

17.     mn+2n,  m2+3m+2.  18.  a2— 3a+2,  a2— 5a+6. 

19.     a;2-9,  :c2-7x+12,x2-4x+3.        20.  y'^-ir2ij-^,rj^+y-2. 

21.     a2-{-2a6+62,  2a2-262.  22.  x2-10.r+25,  3x2-75. 

23.     6a6+462,  6a2+4a6.  24.  a3-2a2+a,  a^-\.a^-2a. 

25.  If  a-|-6  is  a  common  factor  of  a"-\-mab-\rb-  and  a2+?ia6-|-262, 
what  are  the  values  of  m  and  n  ? 

26.  The  H.C.F  of  a26  and  ab'^  is  ab.  Find  the  greatest  common 
measure  of  the  numbers  to  which  a^b  and  ab"  are  equal  when  a=2  and 
6=4,  and  compare  the  result  with  the  value  of  a6  when  a=2  and  6=4. 

71.  Algebraic  Fractions.  A  fraction  has  the  same  meaning 
in  algebra  as  it  has  in  arithmetic. 

Thus,  I  means  3  of  the  4  equals  parts  of  a  unit,  or  the 
quotient  of  3-^4. 

Similarly,  7-  means  a  of  the  h  equal  parts  of  a  unit,  or  the 

quotient  of  a-^h. 

The  fraction  y  is  read  "  a  divided  by  h  "  or  "  a  over  6." 

b  -^ 

72.  Changes  in  the  Terms  of  a  Fraction.  As  in  arithmetic, 
both  terms  of  a  fraction  may  be  muItipUed  or  divided  by  the 
same  quantity  (zero  being  excepted)  without  altering  the 
value  of  the  fraction. 


SIMPLE  APPLICATIONS  OF  FACTORING  105 

Thus,  f|=|  =  J.o,=  3o_etc. 

Similarly, 


a 
b~ 

ac 
be 

OCX 

bcx 

= 

etc.. 

a^b^ 

ab^ 

62 

6 

a^bc 

abc 

be 

c 

and 


73.  Lowest  Terms.  A  fraction  is  said  to  be  in  its  lowest 
terms  when  its  numerator  and  denominator  have  no  common 
factor.  If  it  is  not  in  its  lowest  terms,  it  may  be  reduced  by 
dividing  both  terms  by  all  the  common  factors. 

Examples. 

18  _  18^6  _  3 
42  "42-4-6^  T 

Iba^b  _  I5a^b^5ab  _  3a* 
25a62  ~  25ab^^5ab  ~   56  * 

g  x'-y'       ^  {x+y){x-y)  ^  x—y  _ 

x^  +  2xy+y'       {x+y)ix+y)       x+y' 

^      x^  +  5xy  +  4y^  ^  {x  +  y){x  +  4:y)  ^  x  +  y  _ 
x^  +  Sxy-'iy^       (x-y)(x  +  4:y)       x  —  y' 

The  attention  of  the  pupil  is  drawn  to  the  fact  that  it  is 
factors  and  not  terms  which  are  cancelled  from  the  numerator  and 
denominator. 

7  +  2 
ThvLS,  in  the  fraction  ~ — ^  we  cannot  cancel  the  twos  and  say  that  the 

9  +  2  ^ 

fraction  is  equal  to  I,  for  the  value  of  the  fraction  is  j\,  which  does  not 

7x2 
equal  J.      But  if  the  fraction  is  - — ^  we  can  now  cancel  the  twos  and 

the  resulting  fraction  is  l- 

Similarly,  —  =  -  after  cancelhng,  or  div-iding  by  the  common  factor  a. 

_      a+6 .  b 

But  IS  not  equal  to  -. 

a-\-c  c 

It  is  thus  seen,  that  no  cancelling  can  be  done  until  both 
terms  of  the  fraction  are  expressed  as  products. 


106 


ALGEBRA 


EXERCISE  55  (6-21,  Oral) 
Fill  in  the  blanks  in  the  followins; : 


2. 


4. 


5. 


15 

20 

ax 
bx 


30 


Gxy 


12      4a 

hm 


Sam 


h      b{m-{-n) 
6a^x       a^x 


662 


12a  2^:2 


( 


a^-2ab+b'^        ( 
Reduce  to  lowest  terms 


2x2 


6. 


14 
21 


3x 
6 


10.     — 


14. 


18. 


25. 


28. 


2a+4 
6 

x{x—l) 
a;2-l 


11. 


15. 


19. 


L5a 
"25 


lOm^n 


9.     ^ 


35a26 


74.  Multiplication  and  Division  of  Fractions.  The  methods 
by  which  fractions  are  multipHed  and  divided  in  algebra 
are  the  same  as  in  arithmetic. 


SIMPLE  APPLICATIONS  OF   FACTORING  107 


Examples. 

3       5_  3xo_  15 
4^7~4x7~28* 

10       7^^_10       7       15_25^ 
^-     21  ^  3   •    15~  21  ^  3  ^   4  ~  6  ^' 

a       c      ay.c       ac 
b      d  ~  bxd      bd 

ab       x^y   .    x^       a6       x'^y        a  __b 

4 .  X  IT    T"  ^  X  7C    X  5    ^ 

xy        a^        a        xy        a^        x^       x 
a^  +  ab       cd  +  d^  _  a{a  +  b)       djc  +  d)  _  ad 
^'     c'^+cd  ^  a6  +  62  -   c{c  +  d)  ^  b(a  +  b}  ~  be' 


EXBRCISE  56 

Simplify : 

,453  ^       2^5.3  2a      96 

'•     5^6^4-  ^'     15^7-^14  '•     36^4^ 

4.     ^X^-X^  5.     ^xf.  6.     1-^1^ 

6      c      a  xy      be  ob      So 

7      «^^.  8.     ?^^^1^\  9.     ^Vl2«. 

'     b  -  d  14?/-  ■     7y  56 

in*      ^•^'         ^~^  11      4a +66  lOx-  x-+a:y  ,  xy+y^ 

3^^^l0~'  5a;         2a+36'     "'    a2+a6a6+62* 

x^-l      a;2-5a;+6     . .    a^-3a+2      a^-'7a+l2      a2-4a+3 
^'-     ^^iHi  ^  a;2_4a;+3"         "  a2-5a+6       a^-Ga+S       a2-5a+4 

a2_62  a2^2a6-862      a2+3a6-462 

-IK  Y -^ • 

a2-3a6+262      a2_2a6-362  '  a2-4a6+362 

75.  Lowest  Common  Multiple.  A  product  is  a  multiple  of 
any  of  its  factors. 

Thus,  3xy  is  a  multiple  of  3,  of  x,  of  y,  of  3.T,  of  'Sy,  of  xy,  and  a*  is  a 
multiple  of  a,  of  a*,  of  a'. 

When    an    expression     is    a    multiple    of    two    or    more 
expressions  it  is  a  common  multiple  of  those  expressions. 
Thus,  I2a^b'  is  a  common  multiple  of  2a'-  and  3a62. 


108  ALGEBRA 

The  lowest  common  multiple  (L.C.M.)  of  two  or  more 
expressions  is  the  expression  containing  the  smallest  number 
of  factors  which  is  a  multiple  of  each  of  the  given  expressions. 

Ex.  1.— Find  the  L.C.M.  of  Qx-y,  9xy^  and  I2xy^. 

The  numerical  coefficient  of  the  L.C.M.  is  evidently  the  L.C.M. 
of  6,  9  and  12  or  36. 

The  highest  power  of  x  in  any  of  the  given  expressions  is  cc'  and  of 
y  is  2/',  so  that  the  L.C.M.  must  contain  the  factors  x^  and  y^. 

:.   tlie  L.C.M.  =  36  xa;2xj/3  =  36x22/3. 

Ex.  2.— Find  the  L.C.M.  of  a^-b^  and  a^-2ab-\-b^. 

o2-62  =  (a-6)(a  +  fe). 
a2_2a6  +  &2  =  (a_6)2. 

.-.    the  L.C.M.  =(a-b)^a  +  b). 
Why  is  (a-b)(a  +  b)  or  (a-6)2(o  +  6)2  not  the  L.C.M.  ? 


EXERCISE  57  (1-9,  Oral) 
Find  the  L.C.M.  of  : 

1.     3,  4,  5.  2.  10,  15,  20.  3.  2a,  4a,  6a. 

4.     a,  ah,  a^.  5.  x\  xij,  y^.  6.  2ah,  Sac,  66c. 

7.     10a2,  15a2,  5a.         8.  Sa^,  2a-,  4a.  9.  M^-h,  4:ab\ 

10.*  a\  a^+a.  11.  3a;,  3x2+6^.  12.  ab+ac,  b^+bc. 

13.     2a;+2,  a;2-l.         14.  x^+xij,  {x^y)K    15.  x^—l,x^-—3x+2. 

16.     a^—ab,ab—b^.       17.  a"—b\a^—2ab-\-b^.  18.     x^—x,x^—x. 

19.     2x,  4x+4:,  2x2-2.  20.     ?/2-3?/+2,  y^--y-2,  y^-1. 

21.  Show  that  the  product  of  x'^+x—2  and  x^—x—6  is  equal  to 
the  product  of  their  H.C.F.  and  L.C.M. 

76.  Addition  and  Subtraction  of  Fractions.  If  we  wish 
to  add  or  subtract  fractions  we  must  reduce  them  to  a 
common  denominator.  As  in  arithmetic,  the  lowest  common 
denominator  is  the  L.C.M.  of  the  denominators. 


SIMPLE  APPLICATIONS  OF  FACTORING  109 


Examples. 

3       5       2       9        10        8       9+10-8       11 
■     4  "^6       3  "12"^  12       12"         12        "12* 

o      a       a  _ac       ab       ac  +  ab 
b       c       be       be  ~       be      ' 

3^_  £       _5  _  ^_  4a2        5a6        Sb^-4a^  +  5ab 

**•         „2  U2   'T'    „U  „2;,2  „2»,2   "T 


4. 


ab       a^b-       a'^b^       a^b^  a%^ 

b  a  b  ay  —  bx 


x^+xy      xy+y^      x{x+y)      y(x+y)      xy{x+y) 

2x 2 2x 2 2a:-2(x-2) 

a;2_4      x+2  ~  (x+2)(cc+2)  ~  a;+2  ~  (a;+2)(a;-2) 

2a;-2a;+4 


(^.+2)(a;-2)       (x  +  2)(.x--2) 

EXERCISE  58  (1-8,  Oral) 
Reduce  to  fractions  with  the  lowest  common  denominator : 


1. 

2     5 
3'  9' 

2 

3      a 
4'  46' 

3. 

1 
a' 

1 
a-' 

4.    1,A 

3a;     2x 

5 

3a     4a 

(3 

in     n 

7 

1 

b 

8.   2,  i^ 

a     a- 

4  '    3 

n     m 

a 

c 

9. 

2       5 

3 

x».    ?. 

b 

c 

11. 

2x2 

3bx      4 

3a'  4a2' 

2a  ^ 

c 

a 

6   '' 

'     2   '  36c* 

12      —     ^,  A.   13      ^,  X,  ^.  14.     ^Ltl,  "~^     «+2 

Sij-'  3xy'  2x         '     3ab'  26c'  4ac  '        a    '     2a    '     3a 

Perform  the  operations  indicated  : 

a      a                            a+b      a~b  a+4      5— a 

lo.  3+5.         16.  -2-+nr-  '"•  ^-+^- 

._      u.—^      u,             iQ         3            2  a  +  6  ,  6+c      a+c 

18.     -,; ,.             19.     -_^  +  ^— .  20.     __^-^+_-. 

21.     . +  T— •      22.     -^             g-.  23.     ^-  +  ^             g-- 

24.    :::^_ii:^^^±^.  25.    ^ ^. 

3      '      4  X— 2/      a;+2/ 


a 

—  X 

a 

3 

5 

1 

-,',. 

1 

—  X 

x 

-y 

x—y 

110  ALGEBRA 

26         4  4  8a  ^7      a+x      a—x      a^—x^ 


a+4      a— 4      a2_i6  a  x  ax 

28.     — ^ ^—  •  29. 


3a;+6      2a;+4  '     a^— a&      a6— 6- 

30.     = \ 1 (Check  when  a=  1.) 

a2+3a+2  ^  a2^5a+6  ^  a2^4a+3 


a2_i    '  a2^3Q^^2      a2_|_a_2 

77.  Mixed  Expressions.  An  expression  which  is  partly 
integral  and  partly  fractional  is  called  a  mixed  expression.  A 
mixed  expression  in  algebra  corresponds  to  a  mixed  number 
in  arithmetic. 

Thus,  3?  is  a  mixed  number  and  0+     is  a  mixed  exj^ression. 

Note  that  in  a  mixed  number  the  sign  of  addition  is 
omitted  and  3|  means  3+|.     But  in  algebra  the  sign  must 

be  inserted,  as  a-  would  mean  ax-  and  not  a  +  -. 
c  c  c 

78.  Reduction  of  a  Mixed  Expression  to  a  Fraction.  Since 
every  integral  quantity  may  be  written  as  a  fraction  whose 
denominator  is  unity,  it  follows  that  the  reduction  of  a 
mixed  expression  to  a  complete  fraction  is  a  problem  in 
addition  or  subtraction. 

Exj\.mples. 

1.  3S„3  +  |="'+?  =  y- 

000 

b       ac   ,    b       ac  +  b 

2.  a  +  -= \-  -  = 

c        c        c  c 

X  _  5y      X  _  5y—x 


4. 


y      y     y        y 

ac        a{b-^c)        ac    _  ab+ac—ac  _    ab 
b+c  ~    6+c         6+c  ^        b\.c        ~  6+c 


SIMPLE  APPLICATWNS  OF   FACTORING  111 

79.     Reduction   of    a  Fraction    to   a    Mixed    Expression.     To 

,      ab-\-bc   .  . 

separate    into   two   ira'ctions  we   merely   reverse   the 

operation  of  addition. 

n,,  ab-\-bc       ab       be       ,       be 

Thtis,  — = =  b  A , 

a  a        a  a 

,  ab  —  bd  —  cabbdc  c 


EXERCISES  59  (1-9,  Oral) 
Reduce  to  complete  fractions  : 

1.     2+t.  2.     1  +  ^.  3.     3  +  ?. 

2  y 

4.     a  +  ~.  5.     x  —  ±.  6.     a  —  -. 

•*  o  C 

7.     x-"^-  8.     2.r  +  ^.  9.     ah-^1. 

n  X  a 

10.     a-^.  11.     x  +  ^^.  12.     2a  +  M. 

o+c  x—y  a  —  b 

^o           11,2               .,               ,       a-\-h  x^-\-v^ 

13.     a;+l+  — .  14.     a-h 3^.  15.     a;  -  y  +      ^^ 


Separate  into  fractions  in  their  lowest  terms : 

6a+26  aa:+6x  5a;— 8?/ 

Id,      .  X ^  , —  •  Xo,      • 

4  a/>  10a 

19.     6«'-3^'  20.     "+'^^-^g .  21.     ?i^i?. 
3a6                         *         21a6  '         Qxy 

.^„      Smn—4n  „.,      6a6c— Qfec+c^  „,      (a— 6)2+a; 

JJ.        .  Jo. .  J'i.       • 

2n  36c  a— 6 


EXERCISE  60  (Review  of  Chapter  IX) 

1.     Define  highest  common  factor  and  lowest  common  multiple. 
2.*  Find  the  H.C.F.  and  L.C.M.  of  3a;- 6,  4a;- 8,  5a;- 10. 


112  ALGEBRA 

3.  Find  the  H.C.F.  and  L.C.M.  of  x^+xy,  xy+y^  and  x^y  +  xy^. 

4.  Find  the  H.C.F.  and  L.C.M.  of  x^-Tx+lO  and  x2+2a;-8. 
Show  that  the  product  of  these  expressions  is  equal  to  the  product 
of  their  H.C.F.  and  L.C.M. 

5.  Reduce  to  lowest  terms  : 

a^-{-ab         x^  6a*  — 9a6      abx—bx^ 

a^      '  x^  —  xy      8ab—l2b^     acx—cx^ 

T.T  li-  1      Sax      4cy      5bd 

6.  Multiply^,    si'    2^x' 

7.  Simplify   _x— -.g-,- 

8.  Reduce  to  lowest  terms  : 

x'^—2x        x^+4:X+4:  20;"- 18  a*— 6* 


x^-5x+~6'   x^  +  5x  +  6'    3ic2  +  3:c-18'    a^-2ab  +  b^ 

x—y  x-'ry  2ax      xy—y^ 

x^-\-2xy-\-y^      x^—2xy-{-y^        y         x^-\-xy 

10.  Divide  -^^-   by   ^,-^^-,  and   — ^   by   ^^-^  • 

_       „.       ...     a;-3   ,   a;+4  2x-l       8-4a; 

11.  Simphfy       g     H ^      and     — \ g—  • 


12.     Find  the  sum  of 


x—y     y—z     z—x 

xy    '      yz         zx 


^o      T.  .u  ,  3&  +  40        ,  &-6c      ,^      ^  a+6c 

13.  From  the  sum  of  — jp^—   and  —^ —  subtract  —; • 

2ab  26c  4ac 

14.  Simphfy    -3-  -    ^^-    and   — -^--- • 

15.  Express        „,„      as    the    difference    of    two    fractions   in    their 

^  a^b^ 

J,2 g2  c* a* 

lowest  terms.     Do  the  same  with   — ,  s-i     and  — s— s—  and  find  the  sum 

of  the  three  results. 

y  —  4  y —  5 

16.  By  how  much  does      -      exceed   ^-- —  ? 

42/  5y 

17.  Find  the  sum  of      ,-^,  -,    and   -^ — ,  ,• 

a  +  b    a  —  b  a^  —  b^ 


SIMPLE  APPLICATIONS  OF  FACTORING  113 

18.  By  what  must  be  multiplied  to  give    -5 — ij~~_^^•)   ^^  ^^® 
product  ? 

19.  Find  the  quotient  when        -    is  divided  by   ^ —         ^* 

20.  Solve  {a-b)x  =  {a^-b-){a  +  b). 

21.  Find  the  difference  between 

a      ,       6       ,      c  ,       a;      ,      X      ,      a; 

\-  T 1 and h  r \-      -- 

a— a;      0  — x      c~x  a  —  x      o  —  x      c—x 

by  first  subtracting  from  ,  etc. 

°  a—x  a~x 

22.  Find  the  missing  term  in  the  following  identity  : 

x^  —  5x-{-Q       x^-\-5x  .  .  .       x--\-2x—8 
x^-Zx-\  ^       a;2-9  ~  ^  x^-x-l2' 


CHAPTER  X 
REVIEV/  OF  THE  SIMPLE  RULES 

80.  In  this  Chapter  will  be  found  such  exercises  as  will 
furnish  a  review  of  the  elementary  rules.  In  it  is  also 
included  matter  which  it  was  not  thought  advisable  to 
present  to  beginners  in  the  subject  of  algebra. 

81.  Use  of  Brackets.     We  have  abeady  seen  that 

(1)  a+(6  +  c)=a+6+c, 

(2)  a-\-{b—c)=a-\-h-c, 

(3)  a—{b+c)=a—b—c, 

(4)  a—{b—c)  =  a—b-\-c. 

That  is,  when  brackets  are  preceded  by  the  negative  sign, 
as  in  (3)  and  (4),  the  brackets  may  be  removed  if  the  signs 
of  all  terms  within  the  brackets  be  changed ;  but  when  they  are 
preceded  by  the  positive  sign,  as  in  (1)  and  (2),  no  change 
is  made  in  the  signs  ivhen  the  bracJcets  are  removed. 

In  (3)  the  sign  of  b  in  a—{b-\-c)  is  positive  as  the 
expression  might  be  written  a—{-\-b-\-c).  When  the  brackets 
are  removed  we  follo\y  the  rule  and  change  -\-b  into  —b. 

Sometimes  we  find  more  than  one  pair  of  brackets  in  the 
same  expression. 

Ex.  1.— Simphfy  a-(3a— 26)  +  (5a-46). 

Following  the  rule,  this  expression  becomes 

a-3a  +  2b  +  5a  —  ib  or  3a-26. 

When  one  pair  of  brackets  appears  within  another,  it  is 
better  to  remove  the  brackets  singly,  and  the  pupil  is 
advised  to  remove  the  inner  brackets  first. 


BE  VIEW  OF  THE  SIMPLE  RULES  115 

Ex.  2.— Simplify  4x-{2x-{3^x)\. 
Removing  the  inner  brackets,  we  get 

4a;— [2a;  — 3  — x}. 
Removing  the  remaining  brackets,  we  get 

4x-2x+3+x  or  3a;+3. 

Ex.  3.— Simplify  3a—[a+b  —  {a—h—c—{a+b-c)\]. 

The  expression  =  3a  — [a  +  6—  {a  — 6  — c  — a  — 6  +  c|], 
=  3a  —  [ct-1-6  —  a  +  fo  +  c  +  a  +  fe  —  c], 
=  3a— a  — 6  +  a  — 6  — c  — a  — 6  +  c, 
=  2a-3b. 

After  removing  the  first  pair  of  brackets,  the  quantity 
a  —  b  —  c  —  a  —  b-'rc 

might   have   been   changed   into    the    simple   form    —26.     Work    the 
problem  again,  simplifying  at  each  step. 

When  brackets  are  used  to  indicate  multiplication,  the 
multipHcation  must  be  performed  if  the  brackets  are 
removed. 


Ex.  4.— Simplify  4a;-.3(x— 2^)+2a;— 4i/. 

The  expression  =4.r  — (3a;— 6y)  +  2a;— 8?/, 

=  4x~3x+6y  +  2x-8y, 
=  3x-2y. 

Note. — When  the  pupil  has  had  some   practice  he  should  be  able 
to  remove  the  brackets  and  perform  the  multiplication  in  a  single  step. 


EXBRCI3B 

61  (1-9,  Oral) 

Remove  the  brackets  from  : 

1.     {a—b)  +  {c-d). 

2. 

{a-b)-{c-d). 

3.     -ia-b)  +  {c-d). 

4. 

-{a-b)-{c-d). 

5.     a-{b-c)-{d-e). 

6. 

a-{-b)-{-c). 

7.    a-}-',b+{c-d)}. 

8. 

a+',b—{c—d)\. 

9.    a-{b+{c-dy,. 

10. 

a->b-{c-d):. 

11.    a-{-b-{c-d)\. 

12. 

-[a-ib-{c-dn 

T  2 

116  ALGEBRA 

Simplify : 

13.     4a-26-(2a-26).  14.     2(lx-3y)-3{2x-3y). 

16.  B{a—b+c)-2{a+b—c).  16.     2a- l3a+2(a— 26)}. 

17.  3(a+b—c)—2{a—b+c)+5{b—c+a). 
18.*  l5x-{4.-[3-5x-{3x-l)]\. 

19.  Add  3(a+b)-5(p+q),  -2{a+b)+{p+q)  and  Mp+q). 

20.  Add  l+rc— ?/,  1—x—y  and  1— a;+?/. 

21.  Add  3x-2{y-z),  3y-2{z—x),  3z-2(x—y). 

Remove  the  brackets  and  express  in  descending  powers  of  x : 

22.  3{5x~3^2x^)-2{x^-5+3x)-3{4.-5x-dx-^). 

23.  2a:(3a;-2)-5(a:-3)+6a;(a;-l)-2(a;2-5a;). 

24.  ^(4a;-3)-J(6-a;2)+J(a;2+8a;-12). 

Solve  for  x  and  verify  : 

26.     4(a;-3)-7(a;— 4)=6— x. 

26.  5a;-[8.f-3;l6-6a;-(4-5x)|]=6. 

27.  3(2a;-7)-(*'-14)-2(5a;+17)  =  6(5-8a;)  +  21a;+149. 

28.  ^(27-2x)=|-/5(7a;-54). 

29.  Simplify  a-[5b-{a-(3c-3b)+2c-{a-2b-c)\]. 

30      SimBlifv  3(a-6+c)+2(6-c+a)-(c-a+&) 

^     -^  5(a-26+c)-2(6-3c+2a)-(llc-2a-116  " 

31.     Solve  (7ix-2|)-[4i-|(3i-5x)]  =  18i. 

82.     Insertion    of    Quantities  in    Brackets.      The    trinomial 

a—h-'rc  may  be  changed  into  a  binomial  by  combining  two 

of  its  terms  into  a  single  term.  This  may  be  done  in  a  number 
of  ways. 

Thus,  a—b  +  c  =  (a  -b)  +  c  =  {a+c)  —  b 

=  a  —  {b  —  c)  =  a+{c—b). 

Remove  the  brackets  mentally  and  see  that  each  of  these  is  equal 
to  a—b-\-c. 


REVIEW   OF  THE  SIMPLE  RULES  117 

Ex.  1. — Express  a—h-\-c—d  as  a  binomial. 
As  we  have  seen,  this  may  be  done  in  many  ways  as  a  — (6  — c  +  d), 
(a-6)  +  (c-rf),  {a-{-c)-(b+d),  (a-d)-ib-c),  c-{b+d-a). 

Note. — The  pupil  must  exercise  particular  care  when  dealing  with 
brackets  which  are  preceded  by  the  negative  sign.  The  signs  of  all 
terms  inserted  in  such  brackets  must,  of  course,  be  changed.  He 
should,  in  every  case,  remove  the  brackets  mentally  to  test  the  accuracy 
of  the  work. 

Ex.  2. — Express  a-\-b  —  c  as  a  binomial  by  combining  the 
last  two  terms  within  brackets,  preceded  by  the  negative 
sign. 

a-\-b  —  c  =  a  —  {—b-\-c'' 
=  a~{c  —  b). 

Either  form  is  correct,  but  it  is  usual  to  make  the  first  term  within 
the  brackets  positive,  so  that  the  second  form  is  preferable. 

83.  Collecting  Coefficients.  Brackets  are  frequently  used 
for  the  purpose  of  collecting  the  coefficients  of  particular  letters 
in  an  expression. 

Thus,  ax-{-by—cx—dy=x{a—c)-\-y{b—d), 

and  ?nx  —  ny+px+qy  =  z(m-irp)  —  y(n  —  q), 

=  x{m+p)  +  y{q-n). 
Verify  these  by  removing  the  brackets. 

EXERCISE  62 

1.  Express  3a  — 26+ 4c  as  a  binomial  in  three  different  waj's  and 
verify  in  each  case. 

2.  Express  p—q—r-^s  as  a  trinomial  in  four  different  ways  and 
verify. 

3.  Express  x—y—z—k  as  a  binomial  in  four  different  ways  and 
verify. 

Collect  the  coefficients  of  x  and  y  : 

4.  ax— by— ex— dy. 

5.  mx—ny—px-\-qy—ax-{-by. 

6.  a{x-y)  +  b{2y-3x)+c{5x+2y). 

7.  x{a—b)+y{b—c)—d(x+y). 

8.  2ax—3by—l0x—5y+6bx-4:ay. 

9.  {a-3)y-{2-b)x+4y+2ax-{3x+by). 


118  ALGEBRA 

10.  Enclose  a—h—c—cl—e\-f  in  alphabetical  order  in  brackets, 
with  two  terms  in  each  ;   with  three  terms  in  each. 

Arrange  in  descending  powers  of  x : 

11.  a(a;2+4— 3a;)-6(3a;-5a;2)_c(l-4a;). 

12.  ax''-—hx-\-c—{2px^—Zqx+r)  —  {l(lx^+Zcx+f). 

84.  Multiplication  with  Detached  Coefficients.  The  method 
of  multiptying  two  binomials  has  already  been  shown  in 
Chapter  V.  The  same  method  is  followed  when  the  factors 
are  not  binomials. 

Ex.  1.— Multiply  :i;2— 3x-+4  by  x-2. 


(1) 

a;2-3a;+4 
X  -2 

-8 

(2) 
1-3  +  4 
1-2 

-8 

(3) 
Check 
a;=l 

.t3-3x-2+    4x 
-2x--\-   6a;- 

1-3+    4 

-2+    6- 

+  2 
-1 

x^  —  5x'^-\-  10a;- 

-8 

1-5+10- 

-8 

-T 

The  second  method  is  called  multipHcation  with  detached 
coefficients.  The  processes  in  the  two  methods  are  the 
same,  with  the  exception  that  the  letters  are  omitted  in  the 
second  method  and  the  coefficients  only  are  used. 

When  the  second  method  is  used  the  first  coefficient  in 
the  product  must  be  the  coefficient  of  the  product  of  .r^  and 
X,  that  is,  of  x^.  The  next  must  be  the  coefficient  of  x^  and 
the  next  of  x,  as  the  product  will  evidently  be  in  descending 
powers  of  x,  as  both  factors  multipHed  are  so  written. 

In  (3)  the  check  is  shown  as  explained  in  art.  42. 

Ex.  2.— Multiply  3.r3-7:r+2  by  x^-2x+^. 

Here  the  term  containing  x^  in  the  first  3  +  0  —  7+   2 

expression  is  missing  and    a    zero    is   sup-  1  —  2  +  3 

plied  in  order  to  bring  coefficients  of  like  ■ 

S-l-0— 74-    ^ 
powers  of  x  under  each  other  in  the  partial  <J-rv      < -r    - 

products.  -6-0+14-    4 

The  first  term  in  the  product  is  3a; «.  +9+   0-21  +  6 

Write  down  the  complete  product  and          3_6  +  2  +  16  — 25  +  6. 

check  the  work. 


REVIEW  OF  THE  SIMPLE  RULES  119 

Ex.  3. — Find  the  coefficient  of  x^  in  the  product  of 
dx-'—Qx^+3x-2  and  x^—2x^—3x-^4. 

Here  the  complete  product  is  not  required,  but  only  the  term  which 
contains  x^. 

The  partial  jDroducts  which  will  contain  x^  are  evidently  those  whicli 
we  obtain  by  multiplying  —2  by  x^,  3x  by  —2x'",  —6x-  by  —  3.r  and 
5x^  by  4. 

.-.    the  coefficient  of  x^  =  -  2  -  6  +  1 8  +  20  =  30. 

Ex.  4. — Multiply  ax--{-hx-\-c  by  mx—n. 


ax-    -\-bx  +c 

mx     —n 


Here   the  multiplication  is 

performed  in  the  usual  way. 

In    adding   the    partial    pro-  amx^-{-bmx''  -\-cmx 

ducts,  the  coefficients  of  the  —anx^  —bux  —en 

powers  of  x  are  collected.  

amx^  +  (bm  —  an  )x^  +  (cm  —  bn)x  —  en. 


EXERCISE  63 
Multiply  and  check  : 

1.  .T2-3.C+2,  x-2.  2.  2x2-5.c-3,  3.f-2. 

3.  x~—x-\-\,  a;+l.  4.  a'^-\-ab-\-b-,  a—b. 

5.  'X'—x-\-\,  X'^x-\-\.  6.  a^—5a'^—2,  a^+a— 1. 

7.  3x2-2a;-5,  .r2+a;-3.  S.  2a'—oab+Zb^,  2a2+5a6-362. 

9.  a+6— c,  a— 6+c.  10.  .T3+2a;2+4T+8,a;2— 4a;+4. 

11.  6--6  +  1,  62+6+1,  fe^-^Hl- 

1*2.  a;2— a;i/+«/2+a;+?/+l,  x+2/— 1.  .     • 

Use  detached  coefficients  to  multiply  ;    check  the  results : 

13.  3a;3— 4a;2+7x— 3  by  x''-—2x—\. 

14.  5a*— 6a3_2a2-a+2  by  2a2_3a+2. 

15.  4x^— ox— 2  by  4a;2— 3.^-1. 

16.  (x2-x-2)(2x---x-l)(3.r-2). 
Simplify : 

17.*  (.T-l)(x-2)+(.r-2){.T-3)+(rr-3)(.T-I). 


120  ALGEBRA 

18.  {a+x){b-c)+{b+x){c-a)  +  (c+x)ia-b). 

19.  (a^b){c+d)-{a-b){c-d). 

20.  {a+b-c){a-b)  +  {b+c—a){b—c)  +  {c+a—b){c—a). 

21.  (x+l){x+2){x+3)-(x-l){x-2){x~3) 

Find  the  product  of  : 

22.  (l-x){l+x)(l+x^){l+x'^).      23.     (.r-l)(.r-2)(.r-3)(a;-4). 
24.     (a;-l)(a;-3)(a;+l)(a;+3).         25.     («-l)(a2+a+l)(a3-|-l). 

Find  the  coefficient  of  x^  in  the  product  of : 

26.  3x2— 5x+ll  and  dx^+Qx-i. 

27.  a;3+4.r2— 5x+2  and  x--2x-3. 

28.  3x2— 12a;+15  and  2x2-7x-38. 

29.  Multiply  l+x+x^+x^  by  l+2x-|-3x2-f-4.r^  retaining  no  powers 
higher  than  the  third. 

30.  Add    together    (x-l)(x+2),    (x+2)(x-3),   (x+3)(x+4)(.r-l), 
(a;+4)(a;2-2x+3)  and  7— x^+Sx. 

Check  by  putting  x— 2. 

31.  Multiply  lx^—5xhj—xy"+Qij^  by  4x2+3xy— 2y2_ 

32.  Show  that  the  expression  x(x+l)(x+2)(x+3)  +  l  is  equal  to 
(x2+3x+l)2. 

33.  Find  the  first  four  terras  only  in  the  product  of : 

2  +  3x+4x2+5x3  and  l-2x+3x2-4x3 

34.  Find  the  coefficient  of  x*  in  the  product  of  : 

l+4a;+7a;2+l0x3+13x«  and  l+5x+9x2+ 13x3+17x4. 

35.  Solve  and  verify  : 

(x-2)(x-4)(x-6)(x-10)  =  (x-l)(x-5)(x-7)(x-9). 

36.  Multiply  ax^+bx+c  by  bx'^—cx+d.     Collect  the  coefficients 
of  X  and  write  in  descending  powers. 

37.  Multiply  px^—qx->rr  by  px+q,  and  (a— l)x2+ax— 1  by  ax+1. 

38.  Simplify    {ay^-by+c)imj+b)^{ay^-^bij-c){ay-b). 

39.  Subtract  the  product  of  x~+x{p+l)  —  l  and  x— 2/3  from   the 
product  of  x^- x(p— 1)+2  and  x+p. 


REVIEW  OF  THE  SIMPLE  RULES  121 

40.  Point  out  two  obvious  errors  in  each  of  the  following  statements  : 

(i)  ab{a+b){a-+b^)^a*b+a%+a^b^-ab\ 
(ii)  {2x+Syf=6x^+S6xy-54xy^+21y^. 
(iii)  x^-6x'-y-3xy^~+2y^^ix-2y)ix^^~4:x+y^). 

41.  Use  the  formula  {a-{-l){b-\-l)=ab+{a+b)  +  l  to  find  the 
product  of  2146  and  3526,  being  given  that  the  product  of  2145  and 
3525  is  7,561,125. 

85.  Division  by  a  Compound  Quantity.  The  method  of 
dividing  by  a  monomial  has  akeady  been  shown  in  Chapter  V. 

The  method  of  dividing  by  a  quantity  containing  two  or 
more  terms  is  in  many  ways  similar  to  long  division  in 
arithmetic. 

Divide  672  by  32. 

(1)  (2) 

.32)672(21  3  .  10  +  2)6  .  10^  +  7  .  10  +  2(2  .  lO  +  l 

64  6  .  102  +  4.  10 


32  3.10  +  2 

32  3  .  10  +  2 


In   (2)   the   divisor   is   expressed   in   the   equivalent    form 
3  .  10+2  and  the  dividend  6  .  10^+7  .  10+2. 
If  we  substitute  a;  for  10  the  problem  would  be  : 

Divide  Qx^-\-lx-\-2  by  3.r+2 

The  method  here   is  so  similar   to  the  3a;  +  2)6x2+7a;  +  2(2.T+ 1 

method  in  arithmetic,  that  little  explana-  dx^  +  ix 
tion   is  necessary.     The  first  term  in  the 

quotient  is  obtained  by  dividing  3x  into  ox-^^ 

6x2.     The  product  of  3a; +  2  by  2x  is  then  3x+2 
subtracted    from    the    dividend    and    the 

remainder  is  3x+2.  The  last  term  of  the  quotient  is  obtained  by 
dividing  the  first  term  of  the  remainder  (3x)  by  the  first  term  of  tlie 
divisor  (3x). 

In   more   complicated   examples   the   method   is   precisely 
the  same  as  here.     The  division  is  continued  until  there  is 


122  ALGEBRA 

no  remainder,  or  until  a  remainder  is  found  which  is  of  lower 
degree  than  the  divisor. 

86.  Verifying  Division.,  The  work  may  be  verified  as  in 
arithmetic,  by  multipUcation.  It  is  simpler,  however,  to 
test  by  substituting  a  particular  number  for  each  letter 
involved. 

Thus,  in  the  preceding  problem  if  we  let  x=l,  the  divisor  is  5,  the 
dividend  is  15  and  the  quotient  is  3,  which  shows  that  the  result  is 
veiy  likely  correct. 

If  on  substituting  particular  values  for  the  letters  involved,  the 
divisor  becomes  zero,  other  values  should  be  selected. 

87.  Division  with  Detached  Coefficients.  As  in  multipUcation 
the  method  of  detached  coefficients  may  be  used. 

Ex.— Divide  Ux^-x^- 29x^-^12  by  7;r2+3a;-6. 

7  +  3-6)14-1-29  +  0+12(2-1-2 
14  +  6-12 
Check 
x=  1 
4)-4(-l 


-7- 
-7- 

-17  +  0 

-    3  +  6 

- 

-14-6+12 
-14-6+12 

Here  the  first  term  in  the  quotient  is  2x^,  since  l4:X*-^lx^  =  2x^ 
The  complete  quotient  is  2a;^  — a;— 2. 
Divide  also  by  the  visual  method. 

EXERCISE  64  (1-6,  Oral) 
State  the  quotients  in  the  following  divisions: 


G. 


^      a;2+3x+2                ^ 

a2 

-3a -1-2 

a;+l 

a-I       ■ 

^      a;2-4 

4                                         5 

a2 

+2ab+b^ 

*     x+2 

a+b 

Divide  and  verify  : 

7,     ijx-+x-l5hy2x-3. 

8.    e 

9.     5a;2— 31a7/+6.y•-by.T- 

-6//. 

10.     S 

8.     6x^+xy—12y^hy3x—4y. 
9a2+6«6-3562  by  3a+7b. 


REVIEW  OF  THE  SIMPLE  RULES  123 

11.  7a;3+96a;2-28x-by7a;-2.        12.     lOOx-^-lSz^-S^by  25x-+3. 

13.  3  +  7a:— 6.c2by3— 2a;.  14.     6a-+3o— 31a  by  2a— 7. 

15.  a;3+13a;24-54a;+72bya-+6.    16.     2a3+7a2+5a+100by  a+5. 

17.  x^-\-^x-y+oxij'^-\-y^hyx+y.  18.  —x^-\-Zx-y—^xy^-]-y^hyx—y. 

19.  IGm^'— 46??«2-|-39„i_-9  by  S.vi— 3. 

20.  6.r3— 29a:22/+18a;?/2+35?/3  by  2a;-7?/. 

21.  o4+a3_^4ct2_j.3cj^9  by  a2-a-i-3. 

22.  a;«-a;3-6a;2+15a;-9  by  a:2+2a;-3. 

23.  5x4-4.r3+3a;2+22x+55  by  5a;2+llx+Il. 

24.  2.t-3— 8.r+x-*+12-7a;2  by  a;2+2— 3a;. 

25.  30— 12.r2+x*-.r  by  a;-5+a;2. 

Use  detached  coefficients  to  divide  : 

26.  a;3-3.rH3a;-l  by  a;2-2a;+l. 

27.  6a;'*-a;3-lla;2-10a;-2  by  2a;2-3.r;-l. 

28.  a5-5a3+7a2+6a+l  by  a2+3a  +  l. 

29.  4.r2+9+a;4+3x+a;3  by  2a:+3+a;2. 
30.*  (a;2_a--2)(2.r2+a;-l)  by  2x-2— 5a;+2. 

31.  10.i-3+17a-*— 2.1-5- lLr2_x-+l  by  2a;2+a;— 1. 

32.  Divide  a^- 1  by  a— 1  and  a^+1  by  a+1. 
x^-\-y^      .r^ — y^ 


33.  Simplify 

34.  Simplify 


.T+y         x-y 
ffl2+a"-fl"^a2-^+l' 


.,.      .,  ,      6x24-a;— 2      3a;2+8x-3     „ 

3o.     Solve — ^—        ^  —        =11. 

2a;— 1  3a.- 1 

36.  If  .r(3a2-a+l)  +  2  =  3a3-7a2-L3a,  find  x. 

37.  The  dividend  is  a*-i-6a^-r6a2— 9a-f  2,  the  quotient  is  a--\-2a—\. 
Find  the  divisor. 

38.  Divide  3fi-{-x'^y'^-ry^  by  x'^—x-y'^-\-y'^  and  divide  the  quotient 
by  x^—xy-\-y\ 


124  ALGEBRA 

39      Simplify  "^^^— ^(Q^+^^)+^<^  i  acx^+x{ad+bc)+bd 
bx—c  cx-\-d 

Without  removing  the  brackets  divide : 

40.  ax--\-{b-{-ac)x-\-bc  by  ax+b. 

41.  x^-\-{2p—l)x-'rp{p—l)  by  x-\-p. 

42.  a^x'^—2abx-{-b^—c^  by  ax—{b—c). 

43.  ahf+{2a^+a)ij^+{a'+2a)y+{a+l)  by  ay^+ay+1. 

88.  Inexact  Division.  As  in  arithmetic,  the  divisor  may 
not  divide  evenly  into  the  dividend,  and  so  there  may  be  a 
remainder. 

Thus,  34-^5  gives  a  quotient  6  and  a  remainder  4, 

.-.    :u  =  6+4  or  34  =  6  .  5  +  4. 

Similarly,  when  a--\-3a+5  is  divided  by  a+1,  the  quotient  is  a  +  2 
and  the  remainder  is  3. 

.     a^  +  3a  +  5^  3  * 

a+1  ^a+1' 

or  a2+3o  +  5  =  (a+l)(a  +  2)  +  3. 

That  is,  dividend = divisor  xquotient+ remainder. 

Ex. — Express  -^j — —   as  a  mixed  expression. 

Here  the  quotient  is  1— a;)l  +a;*(l  +  a;  +  x^  +  a;' 

\+x  +  x^+x^  ^-^ 

and  the  remainder  is  2x*.  +a; 

.      l+x*       ,    ,        ,      ,   ,      ,  ,     2a;«  +a;-x2 


1     a;  1     a;  ^^t 

Divide  1— x^  by  l  +  .r  and  show  tliat  -{-x^  —  x^ 

1  3-5  9~5  ■ 

\-^  =  l-x  +  x^-x^  +  x* -,-'''.  +a;3  +  x4 

l+x  l+X  J^x^  —  X* 

In   such   cases  the   division    mav.    of  

course,  be  contmued  to  any  number  of 
terms. 


REVIEW   OF  THE  SIMPLE  RULES  125 

EXERCISE   65 
Find  the  remainder  on  dividing  : 
1.*  X--— lO.r+25  by  a;-7.  2,     a2+20a  +  70  by  a+5. 

3.     x-3-4x2+5.r+20by  a;-l.         4.     if -ly-+?,y-l  hy  y^-y-^rl. 
5.     x^-\-y^  by  x—y.  6.     x'^—y^  by  x-'f-y- 

Express  as  mixed  quantities  : 

7      ^±?  8      "  +  ^^  9      2a -36         ^^      5x2+7.1-3 

x+1  '  *      a—b  '  '       a+b    '  '  x+2 

Find  four  terms  in  the  quotient  of : 

11.     l  +  (l_x).     12.     l+(l+.r).        13.     i±^'.      14.     l+«^+2«' 

1—x  1— a+a- 

15.  When  the  dividend  is  a-— 3a+7,  the  quotient  is  a  and  the 
remainder  is  7.     Find  the  divisor. 

16.  Divide  x-~5x+a  by  x—2  and  determine  for  what  value  of  a 
the  division  will  be  exact. 

17.  If  x^—mx-\-12  is  divisible  by  x—3,  what  must  the  quotient  be 
and  what  is  the  value  of  m  ? 

18.  By  division  show  that 

a2+62  26'-        a2+62  26^ 

a—b  a—b        a+b  a+b 


EXERCISE  66  (Review  of  Chapter  X) 

1.  Add  z^-2ax^-\-a^x  +  a^,  3x^  +  3ax^,  2a^-ax^-x\ 

2.  Add  ^a-J6,  |a-J6,  |a+J6. 

3.  Subtract  8a+2b~~5c  from   lla-26  +  5c-3rf. 

4.  Subtract  —  3a  +  46  — c  from  zero. 

5.  Subtract  ^a  — ^6-f  Jc  from  fa  +  f6  — fc. 

6.  How  much  must  be  added  to  ix  —  ^y-\-^z  to  produce  x  —  y-\-zl 

7.  Subtract  the  sum  of  3a +  26,  26  — 3c  and  3c  — a  from  the  sum  of 
-6,  6  — c  and  c  —  a. 

8.  Simplify  a-(36-4c)-(6  +  c-a)-2(a-c). 


126  ALOE  BRA 

9,     Subtract     4a;^  — 3a;^— 2;+2    from    lx^—6x^-\-2z—l    and    check 
by  substituting  2  for  x. 

10.  Find  the  value  of  a^  +  fo^  +  c^  — 3a6c  when  a  =  2,  6  =  3,  c=  —  5. 

11.  SimpHfy  (a+6  — c)  — (6  |-c  — a)  — (c  +  a— 6)  — (a  +  &  +  c). 

12.  Multiply  1-40;-  lOx^  by  1  -  6x  +  3x^. 

13.  Find  the  product  of  x+l,  x-f  2  and  re— 3. 

14.  Divide  the  product  of  x+2,  2a;— 3,  3a;— 2   by  3a;2  +  4a;  — 4  and 
check  when  a;=  1. 

15.*  Multiply  a^  +  b'^  +  c'^  —  ab  —  bc  —  ca  by  a  +  b  +  c. 

16.  Find  the  product  of  a-2,a  +  2,a2  +  4,a*+ 16. 

17.  Divide  a;* +  64  by  a;2  +  4a;+8. 

18.  Divide    a;*  +  a;*  — 24a;2  — 35a;  +  57   by  a;2  +  2x— 3,   using  detached 
coefficients  and  verify  by  multiplication  using  the  same  method. 

19.  Find  the   coefficient  of  x^  in  the  product  of  3x^  —  2x^  +  lx  —  2 

and  2a;3  +  5a;2+lla;+4. 

20.  The  expression   44a;*— 83a;3— 74a;24-89a;+56   is  the  product  of 
two  expressions  of  which  4a;2  — 5a;— 7  is  one.     Find  the  other. 

21.  Divide     x*  +  4:X^  +  6x^  +  ^x+l     by    a;2  +  2a;+l     and    check    by 
substituting  a;  =10. 

22.  Subtract  ax^  +  bx  +  c  from  cx^  +  clx+f,  collecting  the  coefficients 
of  powers  of  x  in  the  result. 

23.  Find  the  remainder  on  dividing  a;^+6  by  x'—l. 

24.  Show  that 

(a-6)(a;-a-6)  +  (6-c)(a;-6-c)  +  (c-a)(a;-c-a)  =  0. 

25.  Show  that  {a-b){b-c){c-a)  =  a{b^-c^)  +  b{c~-a-)  +  c{a^-b-). 

26.  If     a  =  x^  +  2xy  +  2y\     b=x'-2xij  +  2y^,    c=x*-?/S    find    the 
value  of  ab  —  c. 

27.  Subtract  2x-3(y  +  2z)  from  3»/-(82-3a;). 

28.  If  s  =  a  +  b  +  c,  find  in  terms  of  a,  b,  c  the  value  of 

a{s  —  a)  +  b{s  —  b)  +  c{s-c). 

29.  Arrange  in  descending  powers  of  x, 

c{ax  —  b)  —  x(a  —  b)  +  bx{x"  —  cx). 

30.  When  a  =  5,  find  the  value  of 

2a- |3a-(46  +  2a)}  +  5a-(46-a). 


REVIEW   OF  THE  SIMPLE  RULES  127 

31.  What  quantity  when  divided  by  a;'— 2a;+3  gives  x'^-\-2x—'i  as 
quotient  and  9  as  remainder  ? 

32.  If  a=a;2-3a;+2,  6  =  3a;-- 10x-+8,  c  =  4a;2-9a;  +  2,  find  the 
value  of  (a+26  — c)-h(a;  — 2). 

33.  Arrange  in  descending  order  of  magnitude  and  find  the  average 
of:    30,  -15,  27,  0,  3,   -10,   -2,  6,   -8. 

34.  What  number  must  be  added  to  5a;'— 13a;2  +  2a;— 1  so  that 
the  sum  may  be  divisible  bj'  x—2  ? 

35.  Find  the  coefficient  of  x^  when  \-\-x-\-x'^  is  divided  by  \~x  —  x-. 

36.  Divide  Zp''-lp{\-p'^)-{2-^p^)  by  (3p+l)(p+l). 

37.  If  a;2— Ox+c  is  divisible  by  a; +4,  find  c. 

38.  Find  the  sum  of  the  coefficients  in  the  sqviare  of  2a;-  — x  — 3. 

39.  Find  the  product  of  x-\-a,  x-\-b,  x-\-c.  Collect  the  coefficients 
of  the  powers  of  x  in  the  product.  From  the  result,  write  down  the 
product  of  a;+l,  x+2,  a;+3  and  of  a;— 1,  a;— 3,  a;+4. 

40.  Divide   a;«-2a;3+l   by   a;2-2a;+l. 

41.  When  o  =  3,  b  =  2\,  c=2,  find  the  vakie  of 

~  +  Vlab{2c^-ab)  -  (2a -36)^. 

42.  Prove  that  {l  +  xf{\  +  y-)-{l  +  x-){\+y)-  =  2{x-rj){\-xy). 

43.  If  p  =  X and  q  =  x^ ^,  show  that  ^^(^^  +  4)  =  q^. 

44.  Divide  a'  +  6'  +  c'  — 3a6c  by  a-\-h-^c.  What  are  the  factors 
of  a^-\-b^-\-c^  —  Zabc  ?     Compare  witli  Ex.  15. 

45.  Multiply  x'^-\-bx-\-c  by  x'^+px-[-q,  arranging  the  product  in 
descending  powers  of  x. 

46.  Divide  9a2-462-c2  +  46c  by  3a-2^  +  c. 

47.  Multiply  .i;^  — a;(a- 1)- 1  by  a;2  +  aa;+l. 

48.  Divide  a*-166«c*  by  a-26c. 

49.  Arrange  the  product  of  x  —  a,  x  — 6,  x~c  in  descending  powers 
of  X. 

50.  Divide  a^b-\-b'^c-\-c^a  —  ab^  —  bc^  —  ca'''  by  a  —  b,  and  divide  the 
quotient   by   a  —  c. 

51.  What  expression  will  give  a  quotient  of  x^-\-\  and  a  remainder 
of  2a-2  — 7a;-f  6  when  divided  by  3a;*—  10a;-  — 6  ? 

52.  Divide  x^  —  \j^+Q>y^-\2y~\-^  by  x-~y+2. 


CHAPTER  XI 
FACTORING  (continued) 

89.  In  Chapter  VIII.  we  have  already  dealt  with  the 
subject  of  factoring  m  simple  cases.  This  Chapter  will 
furnish  a  review  of  the  methods  already  used,  and  an 
extension  of  those  methods  to  more  difficult  examples. 

90.  Type  I.  Factors  common  to  every  Term.  When  every 
term  of  an  expression  contains  the  same  factor,  that  factor 
can  be  found  by  inspection  (art.  58). 

Thus,  2xy  is  a  factor  of  4:X^y  —  6xy^-\-2axy, 

.'.     4x^y  —  (ixy^-^2axy  =  2xy(2x~3y-\-a). 

Also,  x-\-y  is  a  factor  of  a{x-'t-y)-'t-b{x-\-y).  When  this  expression 
is  divided  hy  x-\-y,  the  quotient  is  a +  6, 

.-.     a{x  +  y)  +  b(x  +  y)  =  (x  +  ^j){a  +  b). 

91.  Type  II.  Factors  by  Grouping.  When  every  term  has 
not  a  common  factor,  if  the  number  of  terms  be  changed  by 
grouping,  we  may  sometimes  obtain  a  common  factor. 

Ex.  1. — Factor  mx-\-nx-\-my-\-ny 

(2) 


(1) 

mx  -\-  tix  +  7ny  +  ny, 
=  x{in-\-n)-\-y{'m-{-n), 
=  {m+n){x+y). 


mx  -\-  nx + my  +  ny, 
=m{x+y)-\-n{x-{-y), 
=  {x+y){m+7i). 


Here  we  changed  from  four  terms  to  two,  and  we  found  a  common 
factor  in  the  two  terms.  The  other  factor  of  the  expression  was  then 
found  by  division. 

The  two  solutions  show  that  different  methods  of  grouping  may  be 
employed.     If  the  first  method  tried  is  not  successful,  try  others. 

128 


FACTORING  129 

Usually  those  terms  are  grouped  which  contain  a  simple  common 
factor.  In  the  example  we  should  not  expect  to  be  successful  by 
grouping  mx  with  ny,  as  these  terms  have  not  a  common  factor. 

Ex.  2.— Factor  x^-\-x^-\-2x+2. 

Use  two  different  methods  of  grouping  and  obtain  the  factors  x-\-\ 
anda;2  +  2. 

Ex.  3. — Factor  {a—h)^—ax-\-hx. 

{a  —  h)^  —  ax-\-hx={a—b)''-  —  {ax  —  hx), 
=  (a~b)^-x{a-b), 
=  {a  —  b)(a  —  b  —  x). 

Note. — When  quantities  are  enclosed  in  brackets,  the  pupil  must 
not  forget  to  verify  by  mentally  removing  the  brackets. 


EXERCISE  67  (1-9.  Oral) 


Fac 
1. 

tor: 
3.r-27. 

4. 

&2_56. 

7. 

a{x+y)+b{x-iry). 

2. 

2ffl-6. 

5. 

3a2_i5o6. 

8. 

p{m—n)-\-{m—n). 

3. 

a  2- 3a. 

6. 

6x'-i/—l2xi/'~. 

9. 

x{a—b)—2y{a—b). 

Factor,   using   two  different   methods  of  grouping   and   verify  by 
multiplication  : 


10. 

ax-\-bx-\-ay-\-by. 

11. 

am—bm  \-nn—bn. 

12. 

x^—ax-\-bx—ab. 

13. 

bx —ax-\-ab— x ''. 

14. 

2ac  +  3ad-2bc-3b(l. 

15. 

x^-j-x^-JrX+1. 

16. 

a^-a^-3a+3. 

17. 

x-y-xy+1. 

18. 

x^+^x^-3x-12. 

19. 

a3-7a2-4a+28. 

20.  Factor  x^-\-r*—x^—x--{-x-\-l  by  making  three  groups  each 
containing  two  terms,  also  by  making  two  groups  each  containing  three 
terms. 

21.*  Find  three  factors  of  3x^—6x--{-3x—Q  and  of  axy—ay—ax-\-a. 

22.  Find  a  common  factor  of 

am-\-bm-{-av.+bn  and  ax-\-ay-{-bx-\-bi/, 
and  of  x^—x^+x—l  and  x^—x^-i-2x~2. 

23.  Factor   l0x^—5xy—Qxz+3yz   and   a^b+a^c—3a^^—3ab"C. 

K 


130  ALGEBRA 

24.  Find  a  common  factor  of  2x^ —6x^ —3x -\-9  and  x^—3x^-{-2x—6, 
and  show  that   it  is  a  factor   of  their  difference. 

25.  Factor  ■iSax—56ay-35bij+30bx. 

26.  Factor  (x+y)^+4:X+4:y  and  2(a-by—a+b. 

92.  Type  III.  Complete  Squares.  We  have  already  seen 
in  art.  64  how  the  square  of  a  binomial  may  be  written 
down.  We  have  also  seen  in  art.  65  how  the  square  root  of 
a  trinomial  may  be  found,  when  the  trinomial  is  a  perfect 
square. 

93.  Square   of  a  Trinomial.      A  trinomial  may  be  squared 

by  expressing  it  in  the  form  of  a  binomial  or 
hy  multiplication. 

Thus, 

(a  +  6  +  c)2=[a  +  (6  +  c)|2, 

=  a2_|_2a(6  +  c)  +  (6  +  c)2, 
=  a^  +  2ab  +  2ac  +  b^  +  2bc  +  c'^, 
:.  {ft  +  b  +  cy  =  a^  +  b^+c^  +  2ab  +  2ac+2bc. 

Multiply  a  +  6  +  c  by  a-^-b  +  c  in  the  ordinary  way  and  compare  the 
results.     Examine  the  diagram  and  see  that  the  same  result  is  obtained. 

Similarly,  (^a  +  b~c)^={a  +  {b-c)}'^, 

and  (a_6_|_c)2=[a-(6-c);2. 

Complete  these  two  in  a  manner  similar  to  the  one  worked  in  full. 

If  we  examine  these  products  we  see  that  they  consist  of 
two  kinds  of  terms,  squares  (a^,  6-,  c^)  and  double  products 
{2ah,  2ac,  26c). 

We  might  express  the  result  thus  : 

The  sq7iare  of  any  expression  is  equal  to  the  svm  of  the  squares 
of  each  of  its  terms,  together  ivith  twice  the  svm.  of  the  products 
of  each  pair  of  terms. 

In  writing  down  the  square,  care  must  be  taken  to  attach 
the  proper  sign  to  each  double  product. 

Ex.1.     (2.T-32/  +  42)2 

=  4x^  +  9y"+\6z^-\2xy+\  Gxz  -  24yz. 


2 

a 

ab 

ac 

ab 

b" 

be 

ac 

be 

c^ 

FACTORING 


131 


Ex.2.     (a-26+c-d)2 

=  a2  +  462  +  c2  +  d2-4a6  +  2ac-2ad-4fec  +  46rf-  2cd. 

Ex.  3,     Factor  a;^+4y-  +  22—4a:^  —  2.r2 4-41/2. 

This  is  evidently  the  square  of  an  expression  of  the  form  x+2j±z 
Which  of  these  when  squared  will  give  the  proper  arrangement  of  signs  ? 
Verify  by  writing  down  the  square. 


9. 

x+h 

13. 

a  —  h  —  c. 

10. 

2a -\. 

14. 

a  +  6+c+cZ. 

11. 

x+y—z. 

15. 

p—q—r-\-s. 

12. 

x~y+z. 

16. 

x-y^-z-\. 

^xy 

+  9;/2.        19 

.    4a;2+4a;+]. 

EXERCISE  68  (1-33,  Oral) 

What  are  the  squares  of  : 

1.  m-\-n.  5.     x—2y. 

2.  m—n.  6.     4x—y. 

3.  3a;+2.  7.     2a-3b. 

4.  .3a— 5.  8.     3?n—5n. 

Express  as  squares  : 

17.     a;«+4a-?/+4?/2  18.  a 

20,     4a2_20a6+2562.  21.  9a^-l2ab+ib^.    22.    mhi^-8mn  +  ie. 

23.     4rt2+2a+i.  24.  l-10a  +  25a2.       25.    a*+2a^b^+b*. 

What  are  the  square  roots  of  : 

26.  a;2?/2-Ul0.ry3+2oz2.  27. 

28.  4-20a2+25a*.  29, 

30.  m^+n^+p^+2mn+2mp+2np, 

31.  a^+b^+c^—2ab+2ac—2bc. 

32.  a;2+42/2+22+4a:2/+2xz+42/z.  ; 

33.  4a2+624-9c2— 4a6— 12ac+66c. 

Simplify : 

34.*  (3a;-2/)2+(a;-32/)2+(2x+3i/)2. 

35.  (a-6)2+(6-c)2+(c-a)2+(a+6-fc)*. 

36.  (x2+a;+l)2+(x2-x+l)2. 

37.  (a-6+c)2-l-(6-c+a)2+(c-a-f6)2. 

38.  (3a;-2?/+z)2-(a;-22/+32)2 

K  2 


16x2-24x!/-f  9j/2. 

(a+6)2-2c(rt+6)+c2. 


132  ALGEBRA 

Complete  the  squares  by  supplying  the  missing  terms : 

■^   39.     a;2— +25.  40.     4cX^+ +25y^ 

41.     a^+4:ab 42 —  Uimi+dn^ 

43.  a^+%"+ —6ab—2ac 

44.  9a;2+ + —  Qxy—I2xz 

^'45.     Find  three  factors  of  3a;2+6x+3  and  of  a^+ia^b+Aab^. 

46.  Factor 

(a+6)2+4c(a+6)+4c2  and   {a+b)^-2{ai-b){c+d)+{c+d)\ 

47.  Show  that  the  square  of  the  sum  of  any  two  consecutive  integers 
is  less  than  twice  the  sum  of  their  squares  by  unity. 

48.  Divide  the  sum  of  the  squares  of  a—2b-\-c,  6— 2c+a, 
by  the  sum  of  the  squares  of  a—b,  b—c,  c—a. 

49.  If  .r  +  -  =  4,  find  the  value  of  x^  -\ — -. 

X  x^ 

50.  Factor  {ax-\-hy)"-\-{bx—ayY-^c~{x'^-\-y'^). 

51.  Express  a^x-'+fe'^'+a^^/^+ft^x^  as   the  sum  of  two   squares. 

52.  Find    the    value    of    x^-\-y^-\-z~-\-2xy^2xZ'^2yz,    when 

x=a^2b—^c,  y=b+2c-3a,  z=c+2a—3b. 

94.  Type  IV.  The  Difference  of  Squares.  The  product  of 
the  sum  and  difference  of  the  same  two  quantities  is  equal  to 
the  difference  of  their  squares  (art.  66). 

Conversely,  the  difference  of  the  squares  of  two  quantities 
is  equal  to  the  product  of  their  sum  and  difference  (art.  67). 

Or,  in  symbols,     {a-{-b){a—b)^a^—b^, 
and  a^-b^={a-\-b){a-b). 

95.  The  formula  for  the  product  of  the  sum  and  difference 
may  sometimes  be  used  to  find  the  product  of  expressions  of 
more  than  two  terms. 

Ex.  1.— Multiply  2a—b-\-c  by  2a— b—c. 

Here  2a  —  6  +  c  is  the  sum  oi2a  —  b  and  c, 

and  2a  — b  —  c  is  the  difference  of  2a  —  b  and  c. 


•♦  $ 


FACTORING  133 


They  might  be  written  (2a  —  b)-\-c  and  (2a  — 6)  — c.  The  product 
is  therefore  the  difference  of  the  squares  of  2a— 6  and  c. 

.-.    42a-6  +  c)(2a-6-c)  =  (2a-6)2-c2, 

9  =4a-  — 4a6  +  ^"  — C-. 

Ex.  2. — Find  tl^  product  of  2x-\-y—z  and  2x-— y-j-z. 

Here  the  first  expression  =  2a; +(2/  —  3), 
and  the  second  =2x—{y~z), 

:.  the  product  ={2xY  —  {y  —  z)-, 

=  4a;^  —  (2/^  —  2?/2  -\-z^), 

=  4:X^  —  y^-\-2yz—z^. 

Verify  by  ordinary  multiplication. 

Ex.  3. — Multiply  a—b-\-c—d  by  a-\-b—c—d. 

Note  that  the  terms  with  the  same  signs  in  the  two  expressions  are 
a  and  —d.  These  should  be  grouped  to  form  the  first  term  in  each 
factor. 

a  —  b-\-c  —  d  —  (a  —  d)  —  (b  —  c), 

a  +  6  — c  — d  =  (a  — c/)  +  (6  — c), 
.".     the  product  =  (a— rf) 2  — (6  — c)^. 

Simplify  this  result  and  verify  by  multiplying  in  the  ordinary  way. 

Ex.  4. — Factor  ;:>-— 4^jq'+4g'2— a;2. 

Here  the  first  three  terms  -form  a  square  and  the  expression  may 
be  written  : 

(p2  _  ^pq  _j.  4^2)  _  ^2  _  ^p  _  2q)^  —  x^, 

=  {p  —  2q  +  x){p—2q-x). 

What  two  quantities  were  here  added  and  subtracted  to  obtain 
the  factors  ? 

Ex.  5.— Factor  a^-h'^+2hc-c'^. 

Here  the  last  three  terms  should  be  grouped  to  form  the  second 
square. 

.-.     a2-62  +  26c-c2  =  a2-(62-26c  +  c2), 

=  a2-(6-c)2, 

=  {a+{b-c)\\a-{b-c)\, 

=  {a-{-b  —  c){a~b-\-c). 
Verify  by  multiplication. 


134  ALGEBRA 

Ex.  6.— Factor  x^-^ij'^—a-—h^^2xy-^2al). 

Evidently  three  of  these  terms  form  one  square  and  the  remaining 
three  the  other  square. 

The  expression  ={x'^-\-2xy-{-y^)—{a^  —  2ab-{-b'^), 

=  {x  +  yY-{a-b)\ 
=  {x-\-y^a-h){x+y-a  +  b). 


EXERCISES  69  (1-10,  17-32,  Oral) 

Use  the  formula  to  obtain  the  following  products : 

^     1.  (2a4-3){2a-3).  2.     (4x-l)(4x4-l). 

3.  (a;y/+5)(x7/— 5).  4.     {ah~-c){ab^-c). 

5.  {2m^-\'^n)(2m'^—'in).  6.     {abc-\-x!/){abc—xy). 

7.  (x  +  iKx-l).  8.       (.x2-y2)(.,;2^_y2). 

9.  {x-'ry-\-z){x-\-y—z).  ).    10.     {a—b—c){a  —  b~[-c). 

11.*  {a+b-c){a-b^c).  12.     {2x+3y-5){2x  +  :\y+5). 

13.  {p-2q+3r){p+2q-3r).  14.     {l-X']-x-){l+x+x-). 

15.  {a+b—c+d){a—b—c—d). 

16.  (ri-26-fc-2rf)(n— 26-c+2(Z). 
Factor  and  verify  : 

17.  ^2-9. 
20.  a^b^—x^. 
23.  l-a~b^ 
26.  (x+?/)2-25. 
29.  (a-f6)2-(c-d)2. 
32.  a2_2aft_^62_c2_ 
35.  (4.r-f-3)2-16x-2. 
38.  a2-l-62-f2a&— c2- 
40.  a2_2a_^i_62^26c-c2. 
42.  4x2— 4x-i/2  4-4a?/-4a2-(-l.    43^     i_^a2_62_4c2^4jc_2a, 
44.  Find  three  factors  of  2x-2— 8,  a'^—a,  a*— x"*. 


18. 

4x2—25. 

19. 

a2-462. 

21. 

16x2-0?/2. 

22. 

932-42. 

24. 

25 -x*. 

25. 

(a-6)2-c2. 

27. 

c2-(a+6)2. 

28. 

x^—{y—z)K 

30. 

(a+26)2-4c2. 

31. 

x^~Ar2xy+y^—a\ 

33. 

a2_^,2_c2-26c. 

34. 

a^-b^-4:c^+ibc. 

.36. 

l-x2+2x//-2/2. 

37. 

a^-x^^2ay+y^. 

-d2- 

-2crf.      39.     a2-, 

&2  +  c2 

-d^-2ac-2bd. 

\bc- 

■c2.            41.      X*- 

x2-4 

-2xhf--4x+y*: 

FACTORING  135 

45.  Find  three  factors  of  5a^— 10a6+56-— 20c-  ami  of 

(.r-36)3-462.i;  +  1263. 

46.  Find  four  factors  of  a~b'^—a-c~—b-d'--{-cH-  and  of 

47.  Factor  a-x'— 6-(/-+2ac.c+c^  and  m^—9m^n'^-\-n^—2mn. 

48.  Find  the  simplest  factors  of  'ix^—2x'^~^x-\-2  and  of 

49.  Simplify  (a— 6-[-c)(a— 6— g)-(-(«+6— c)(a— 6+c). 

50.  AiTange  x%x^—a-)—y'^{y'-—a-)-\-2xy{x'^—y-)  so  as  to  show 
that  x^—y'^  is  a  factor  of  it,  and  thus  find  the  simplest  factors. 

51.  Use  factoring  to  simplify  : 

(1)  {a^~3ai-l)^-(a--'Sa)'. 

(2)  {x-2y+Szy'-{3z-x+2yf. 

(3)  (a2_3rt_4)2_(a2^4)2. 

(4)  {5x^—2xy-^y-r—(5x-'+2xy~yY". 

52.  Multiply  a-f 6+c  by  a  +  b—c  and  a— ft-fc  by  a—b—c  and  use 
the  results  to  obtain  the  product  of 

(a+6+c)(a4-6-c)(«— 6+c)(a-6-c). 

53.  Show  that  x{y"—z-)^y{z^—x~)-\-z{x''^—y-)  is  equal  to 

{'V-y){i/—z-)—{x-—y''-){y—z) 
and  then   find   the  factors  of  this  expression. 

54.  Arrange  a{b'-—c-)^b{c~—a-)-\-c{a-—b")  hi  the  form 

a(6-— c-)— 6c(6— c)— ft-(6— c) 
and  thus  obtain  the  factor  b—c.     Find  the  other  two  factors. 

96.  Type  V.  Incomplete  Squares.  We  have  already  factored 
many  expressions  which  were  seen  to  be  the  difference  of  two 
squares. 

Sometimes  the  two  squares  of  which  an  expression  is  the 
difference  are  not  so  easily  seen. 


136  ALGEBRA 

Ex.  1.— Factor  x'^-\-x^y^+y'^. 

This  expression  would  be  the  square  oi  x^-\-y'  ii  the  middle  term  ^W6re 
2x^y^.  We  will  therefore  add  x^y^  to  com.plete  the  square  and  also 
subtract  x^y^  to  preserve  the  value  of  the  expression. 

Then  x'^+x^y^+y*=x'^+2x^y^+y^-x^y^, 

=  (x^+y^)^  —  (xy)^, 
=  {x^  +  y''-\-xy)(x^-\-y^—xy). 

In  order  that  this  method  may  be  successful,  it  will  be 
seen  that  the  quantity  we  add  to  complete  the  square  must 
itself  be  a  square. 

Thus,  to  change  a^-{-ab-\-b^  into  a^-\-2ab-\-b^  —  ab  is  of  no  value  as 
06  is  not  an  algebraic  square. 

Ex.  2.— Factor  a'^-\-4:M. 

This  can  be  made  the  square  of  a^-\-2b^  by  adding  4a'b^. 

Complete  the  factoring  and  verify  by  multii^lication. 

Ex.  3. — Factor  im^— IGm^n^-f  9n*. 

What  must  be  added  to  make  it  the  square  of  2m^  —  3n^  ?  Complete 
the  factoring. 

Try  to  factor  it  by  making  it  the  square  of  2m^-\-3n~. 

Ex.  4.— Factor  a^-^b'^^c^-2a^b^-2b^c^-2cW'. 

How  does  this  expression  differ  from  the  square  of  a^  +  fe^  — c'  ; 
ExjDress  it  in  the  form  {a--\-b^  —  c^)-  —  4:a-b^. 

Write  down  the  two  factors  and  see  if  you  can  factor  each  of  them 
again  and  tlius  obtain  the  result 

{a+b  +  c){a  +  b-c)(a-b  +  c)(a-b-c). 

EXERCISE  70 
Factor  and  verify : 

1.     a*+a^-+l.  2.     .T*+.r2+25.  3.     x^+lx^+lG. 

4.     x*+2xhj^+Qy\       5.     4a*  +  l.  6.     dx*+8x~y^+iey*. 

7.     46^-1362+1.  8.     9a*- 15^2+ 1.       9.     da'^-52a%''+Ub\ 


FACTORING  137 

10.     25.r'*-89.r2j/2+64^«.      11.     x'^+y'^-\\xhj\      12.     x^-lx^+\, 
13.*  Find  three  factors  of  2a;*+8  and  x^+x^-\-x. 

14.  Find  four  factors  of  Qa'^-lQa^b'^+b'^. 

15.  Find  three  factors  t)f  a:®4-x*+l. 

16.  Find  four  factors  of  a'^+h'^-\-c'^—2a-b'—2b-c-—2c~a-  by 
completing  the  square  of  a^—b^-\-c^. 

17.  Factor  (a  +  l)H(«--l)-+(«-l)*. 

97.  Type  VI.  Trinomials.  We  have  already  dealt  with 
the  factoring  of  expressions  of  the  type  x'^-\-j)x-\-q,  where  the 
coefficient  of  the  first  term  is  unity  (art.  61). 

We  now  wish  to  factor  expressions  of  the  type  mx^+^xx'+g, 
where  m  is  not  necessarily  unity. 

98.  First  Method,  by  Cross  Multiplication. 

Ex.  1.— Factor  2x^+lxy+^^-. 

The  product  of  the  first  terms  of  the  factors  is  2x^,  and  therefore 
the  first  terms  must  be  2x  and  x  ;    similarly,  the  last  terms  must  be 
Zy  and  y  and  the  signs  are  evidently  all  positive. 
.'.  the  factors  must  be 

2x  +  Zy  2x+  y 

x-{-   y         or  x+Sy 


It  is  seen,  by  cross  multiplication,  that  the  coefficient  of  xy  in  the 
first  product  is  3  +  2  =  5,  and  in  the  second  is  1  +  6  =  7. 
.'.  the  correct  factors  are  {2x+y){x-\-3y). 

Ex.  2.— Factor  S.r^-T.r-G. 

Here  the  numerical  coefficients  of  the  first  terms  of  the  factors  must 
be  3  and  1,  and  of  the  last  terms  may  be  6  and  1  or  3  and  2. 

Since  the  third  term  is  negative,  the  signs  of  the  second  terms  of  the 
factors  must  be  different. 

The  possible  sets  of  factors,  omitting  the  signs,  are  : 

3x     3  3a;     2  3x     6  3a;     1 

x     2  x     3  X     I  X     G 


138  ALGEBRA 

Since  the  signs  are  different  for  the  last  terms,  when  we  cross 
multiply  to  find  the  coefficient  of  x  in  the  product,  the  partial 
products  must  be  subtracted. 

It  is  easily  seen  that  the  second  arrangement  is  the  only  one  from 
which  Ix  can  be  obtained. 

Since  the  middle  term  is  negative,  tlie  larger  of  the  cross  products 
must  be  negative. 

.'.  the  factors  are  (3a;+2)(a;— 3). 

This  method  is  Hable  to  be  found  tedious  when  the  coefficients 
have  a  number  of  pairs  of  factors,  but  in  ordinary  cases  the 
puj^il  will  find  little  difficulty  after  he  has  had  some  practice 
in  the  work. 

99.  Second  Method,  by  Decomposition.  In  the  process  of 
multiplying  two  binomials  like  2,<-l-3  and  3x-{-5,  we  have 

(2.r+3)(3^+5)-3,r(2.r+3)+5(2x+3), 
=-6a;2+9.r+10x+15, 
-6.r2+19:r+15. 

If  we  wish  to  factor  a  trinomial  like  6a;'^-t-19a;-|-15,  we 
may  do  so  by  reversing  the  process. 

Thus,  Qx^+  19.T+  15=6.rH9.r+  10a:+ 15, 

=  3a;(2.c+3)+5(2.r+3), 
=  (2a;+3)(3.c+5). 

The  only  difficulty  in  this  method  is  in  finding  the  two 
terms  into  which  the  middle  term,  19.r,  should  be  decom- 
posed.    This  difficulty  may  be  overcome  in  the  following  waj'^ : 

{ax-\-b)icx^d)=acx^+x{ad+bc)-\~bd. 

Note  that  the  product  of  the  two  terms  in  the  coefficient 
of  X,  ad  and  he,  is  the  same  as  the  ]iroduct  of  the  coefficient 
of  x^,  ac  and  the  absolute  term,  bd. 

In  the  trinomial  6;r2-|-19.r+15  above,  the  product  of  6  and 
15  is  90  and  the  two  factors  of  90  whose  sum  is  19  are  9 
and  10,  which  shows  that  the  middle  term,  19a;,  should  be 
decomposed  into  9x+10a;. 


FACTORING  139 

Ex.  1.— Factor  6,r2+13a:+6. 

The  product  of  the  coefficient  of  x^  and  the  absokite  term  is  36. 
The  two  factors  of  36  whose  sum  is  13  are  4  and  9. 

.-,      6a;2_^13.T+6  =  6:c2  +  4a;  +  9x+6, 

=  2a;(3a;+2)  +  3(3a;  +  2), 
=  (3a;  +  2)(2a;+3). 

Ex.  2.— Factor  \2x^—\lx-5. 

Here  we  require  two  factors  of   —60  vvliose  sum  is    —17,  and  tliey 
are  evidently    —20  and   3. 

.-.     12x2-17a;-5=12a;2-20a;  +  3a;-5, 

=  4a;(3a;-5)  +  (3x-5),  » 

=  (3a;-5)(4x+l). 


EXERCISE  71  (1-18,  Oral) 

Factor  and  verify  : 

1.  a;2+4a;+3.             2.    rtHll«+30.  3.  ?/2-f8«/-fl5. 

4.  a2— lla+18.         5.    x'^— 14a;+48.  G.  l+5a;+6a;2. 

7.  a;2-15.c+14.         8.    aW^-5ah^Q.  9.  a^-\5a+m. 

10.  l-21a;+38x2.     11.    x'^-Qxy^Sif.  12.  a2_i3f^ft_^3662. 

13.  a;2-4x-5.           14.    a2_ga_22.  15.  x--2Sx-2^. 

16.  ?/2_4y_2i.         17.     l-2a-15a2.  18.  a^-ay-2y\ 

19.  2a;2+5a;-f-3.        20,    4x-+8xy+3yK  21.  Oa^— l8a6  +  862. 

22.  8x2+a;— 9.          23.    3x'—x—2.  24.  Ga^— a— 2. 

25.  4x2+a;-5.           26.     1562-196-8.  27.  10a;2-23a;-5. 

28.  1062-896-9.     29.    {}x^-3lxy+12y\  30.  10a2-29a6-f-1062. 
Find  the  sir.  ■  lerj'^  factors  of  : 

31.*  3x2-3a:-21G.     32.    2a2+8a+6.  33.  x-Bx^+Qx"^. 

34.  a:«— 5x2+4.         .35.    a^-~\0a^+9a.  36.  9a*— 10^2+1. 

37.  (a;2+4x)2-2(.r2-|-4.r)-15.       38.     (.r2_9x)2+4(x2-9a;)-140. 

39.  Without  multiplying  show  that 

(x2-a;-2)(x2+2x--15)=(x2+6x+5)(a;2-5a:+6). 

40.  An  expression  is  divisible  by  x-—2,  the  quotient  being  x-—x—Q. 
Show  that  it  is  divisible  by  x-\-2  and  find  the  quotient. 


140  ALGEBRA 

V     41.     Show   that   the   product   of   6x^~ldx-\-6   and    2x~—lx-\-5   is 
divisible  by  3x^—5x-\-2  and  find  the  quotient. 

42.  If  3x^-\-ax—l4:  is  the  product  of  two  binomials  with  integral 
coefficients,  find  all  the  different  values  that  a  may  have. 

43.  By  factoring,  find  the  quotient  when  the  product  of 

6a2+7a6-2062  and  22a^-13ab-Ub^ 
is  divided  by  4a-— 4a6— 356'. 

44.  Factor  x''^-'r5xy-\-iy'^-^x-\-y. 

45.  Factor  3a^—ab—2b^+6a+4:b. 

J^[^  46.     Divide  the  product  of  x^-\-3x-\-2  and  x*—l   by  the  product 

of  a;24-2x+l  and  x'^+x—2. 

100.     Type  VII.     Sum  and  Difference  of  Cubes. 

Divide  x'-^-}-y^  by  x^y,  x+y)x^  +y^(x^-xy+y^ 

and  x^—y^  by  x—y.  ^  +^  ^ 

and  x^-if^{x-y){x^+i)cy-^y^).     ~^'^~^^'' 

Examine  carefully  the  signs  in  these  ^^^al^^-i 

factors. 


xy^^-y' 


It  is  thus  seen,  that  the  sum  of  the  cubes  of  tiuo  quantities 
is  divisible  by  their  sum,  and  the  difference  of  the  cubes  is 
divisible  by  their  difference. 

The  quotient  in  each  case  consists  of  the  square,  product  and 
square  of  the  terms  of  the  divisor,  with  the  proper  algebraic  signs. 

Ex.  1.— Factor  8a^+27b^. 

Here  8a^  =  {2a)^    and    2763  =  (36)3, 

.'.  the  expression  may  be  written  (2a)3-|-(36)3, 
.'.  the  first  factor  is  2a +  36  and  the  second  is 

(2a)2-(2a)(36)  +  (36)2  or  4a2-6a6  +  962. 
.'.     8o3  +  2763  =  (2«  _,_  36)(4a2-  6a6  +  962). 

Ex.  2. — Factor  a^x^—My^. 

a3a;3— 64?/*  =  (a.r)3  — (4!/")3, 

=  (ax—iy'^){a-x-  +  4:axy'^+lQf/*). 


FACTORING  141 

Ex.  S.^Factor  x^~y^. 

This  may  be  expressed  as  the  difference  of  two  squares  or  of  two 
cubes. 

•      .x-«-?/«  =  (x3)2- (1/3)2,  OT  {x-)^-{y-Y, 

=  {x^-\-y^){x^  —  y^),  or  {x-  —  y^)(x*-\-x-y^'\-y^). 

Complete  the  factoring  by  each  method  and  decide  which  you  will 
use,  if  you  have  the  choice,  as  here. 


EXERCISE  72  (1-12,  Oral) 
State  one  factor  of :  '       ^ 

1.     a'^+h^.  2.     .r3-L8.  3.     x'^-21.  4.     lOOO-a^. 

5.     x^-Uy^.  6.     27-fe3.  7.     Sa^Ar\25.       8.     \25a^-%b^. 

9.     1-27x3.  10.     343.1-3-8.  11.     {a+bf+cK 

12.  (a-fe)3-c3. 
Factor  and  verif}^  13-21  : 

13.  a3_^27.  14.     .r^-S^/^. 
16.     27.r3-64;/3.            17.     8-27a3. 
19.     a^+h^.  20.     x^-b^. 
22.*  2rt3-16.                 23.     81-t-3y3. 
25.     a36+6*.                  26.     a^+b^. 
28.     (.T-2)3+8.             29.     (a-6)3+ff3. 

31.  What  is  one  factor  of  {2x—y)^—{x—2y)'^  1 

32.  Show  that  (2a-Zbf^{^a-2bf  is  divisible  by  a-b. 

33.  Factor  (a2-26c)3+863c3  and  21xhfz-y^z*. 

34.  Find  six  factors  of  a^^ — 6 ^2. 

35.  Find  two  binomial  factors  of  (2.r-— 3x'-f-3)3— (.r^— 2.r-t-5)3. 

36.  If  X  +  1  =2,  find  the  value  of  x^  -}-  1 . 

x  x^ 

37.  By    factormg    show    that    {a+bY-^ab{a+b)^={a-^b){a^-^h^). 

101.     Type  VIII.     The  Factor  Theorem.     \Vliat  are  the  values 
of  0x5,  axO,  Ox(— 4),  1x0,  -1000x0? 


15. 

8a.3  +  l. 

18. 

1000.f3-2/3. 

21. 

a^— ^«. 

24. 

«*+«. 

27. 

{x^yf+a^ 

30. 

(a-6)3+(a-|-6)3 

142  ALGEBRA 

If  one  of   the  factors  of   a  product  be  zero,   the  product 
must  also  be  zero. 

If  the  product  of  two  numbers  be  zero,  what  can  Ave  infer  ? 

If  a6  =  0,  it  follows  that  either  a=0  or  6=0. 

If  (a;— 3)(a:— 4)=0,  then  either  a;— 3=0  or  a;— 4=0. 

Since        {x—2){x^—lx+\2)=x^-Qx^+2Qx-2i:, 
.'.  .r^— 9.r"^4-26.r— 24  must   be    equal  to  zero  when  .r=2,  for 
then  one  of  its  factors,  x—2,  is  zero. 

If  we  substitute  2  for  x,  we  see  that  this  is  true. 

x3-9a;2  +  26a;-24=23-9  .  22  +  26  .  2-24, 

=  8-36     +    52    -24  =  0. 

Conversely,  when  any  expression  becomes  zero  when  a;=a, 
then   x—a  is  a  factor  of  it. 

Substitute  .t  =  3  in  x^  —  Qx'^-\-\\x—%  and  it  becomes 
33-6  .  32+11  .  3-6  =  27-54  +  33-6  =  0, 
a;  — 3  is  a  factor  of  a;3  — 6x2+ 11a;— 6. 

Divide  it  by  x  — 3  and  the  other  factor  is  aj2  — 3a;  +  2. 

.-.      a;3-6a;2+lla;-6  =  (a;-3)(a;2-3a;+2), 

=  {x-3)(a;-2)(a;-l). 

If  a;+l  is  a  factor  of  an  expression,  the  expression  must  be  equal  to 
zero  when  x=   —  1 ,  for  then  a;  +  1  =  0. 

Thus,  a;+l  is  a  factor  of  a;'  — a;2— lOx— 8,  since 

(-l)3-(-l)2-10(-l)-8=-l-l  +  10-8  =  0. 
Divide  by  a;+  1  and  complete  the  factoring. 

Any  expression  is  divisible  by    x—a  if  it  vanishes  (becomes 
zero)  when  a  is  substituted  for  x. 
This  is  called  the  factor  theorem. 

Show  that  x  —  a  is  a  factor  of  .-c^- 7aa;2+  lOa^a;- 4a'. 
Show  that  x  +  a  is  a  factor  of  5x^~\-6x^a+\lxa^+l0a^. 

Ex.— Factor  .r^— 9a:+10. 

If  it  has  a  binomial  factor  it  must  be  of  the  form 

x+1,     a;±2,     x±5ora;+10. 
Testing  for  these  factors  we  find  that  a;  — 2  is  a  factor, 

.-.      x»-9x+10  =  (x-2)(x2  +  2x-5). 
The  factoring  is  complete  as  x^-\-2x—  5  has  no  simple  factors. 


FACTORING  143 

102.  Special  Case.  It  is  easy  to  see  when  x—  1  is  a  factor 
of  any  expression,  for  when  1  is  substituted  for  x,  the  vahie 
of  the  expression  becomes  equal  to  the  sum  of  its  coefficients. 

Thus,  ifa;=l,  a;3-2a;2- 19a;+20, 

=  1    -2     -19    +20  =  0, 

.'.  X—  1  is  a  factor.      Complete  the  factoring. 

Similarly,  x  —  a  is  a  factor  of  'ix^—\Q)X^a—lxa^-[-20a^,  since 
3-16-7  +  20  =  0,  and  a-b  is  a  factor  of  a^-Qa^b  +  Zah^  +  2b^,  since 
1-6  +  3  +  2  =  0. 

EXERCISE  73 

Each  of  these  expressions  is  divisible  by  x—\,  x—2  or  .r- 3.  Find 
all  the  factors  of  each  and  verify. 

3.r2-12x+14. 

4a;2+a;+6. 
4a-3-9.r2-10a;+3. 


1.     a;3— 10x2+29a;— 20. 

2, 

a;' 

3.     .r3+5x2-2a:-24. 

4. 

X 

6.     2.i-3-7.r2+7.T-2. 

6. 

4: 

Factor : 

7.*  2a;3-ll.r2+5a;+4. 

8. 

X 

9.     .x-3— 7.r+6. 

10. 

x^ 

11.     a3+a2— lOa+8. 

12. 

a 

-2x2— .T+ 2. 
19^+30. 
-3a62_263. 

13.  Show  that  x-\-2  is  a  factor  of  a;^— .r-— x+lO. 

14.  Show  that  x+o  is  a  factor  of  x^-\-lx^a-{-Qxa^Ar'ia^. 

15.  Show  that  a:+3,  a;+4  and  x—1  are  the  factors  of  x^— 37.r— 84. 

16.  If  x^— 10a;+a  is  divisible  by  x+2,  find  a. 

17.  Show  that  a—b  is  a  factor  of  a^-\-Aa%-\-ab'^—(Sb^,  and  find  all 
the  factors. 

18.  Noting    that   x^— 2x-3  =  (x+l)(a;— 3),    show    that   x^— 2.r— 3 
is  a  factor  of  x*— 4x3+2x-+4x— 3. 

19.  Show  that  a—b,  b—c  and  c—a  are  factors  of 

a(62_c2)  +  6(c2_a2)+c(a2-62). 

20.  If  x—1  and  x— 2  are  factors  of  x^- 5x2+a.T+6,  find  a  and  b. 

21.  If  px^—3x^+qx—\0  and  qx^-\-2x^—l'Jx+p  are  both  divisible 
by  X— 2,  find  p  and  q. 


144  ALGEBRA 

103.  Equations  Solved  by  Factoring.     We  have  seen  that  if 

(.r-3)(.c-4)  =  0, 
then  a;— 3=0  or  a;— 4=0. 

Thus  the  equation  (x— 3)(x-— 4)=0  is  equivalent  to  the  two 
simple  equations  x— 3=0  and  x— 4=0. 

But  if  a;— 3=0,  .r=3, 

and  if  .r— 4=0,  .t=4, 

.'.  the  roots  of  the  equation  (.r~3)(.r— 4)=0  are  3,  4. 

The  truth  of  this  may  be  seen  by  substitution. 
Ifa;  =  3,  (x-3)(x-4)  =  (3-3)(3-4)  =  0x-l  =  0. 

Ifa;  =  4,  (x-3)(a:-4)  =  (4-3)(4-4)  =  lx      0  =  0. 

Since  (a;-3)(x-4)  =  a;2_7a;-I-12, 

the  given  equation  may  be  written 

.T2-7.r+12  =  0. 

104.  Quadratic  Equation.  Any  equation  which  contains 
the  square  of  the  unknown  and  no  higher  power  is  called  a 
quadratic  equation  or  an  equation  of  the  second  degree. 

The  preceding  shows  that  if  we  wish  to  solve  a  quadratic 
equation  we  may  do  so  by  finding,  by  factoring,  the  simple 
equations  of  which  it  is  composed. 

The  simple  equations  when  solved  will  give  the  roots  of  the 
given  quadratic  equation. 

Ex.  1.— Solve  x2-6.r-7  =  0 

Factoring,  {x—7)(x+l)  =  0, 

:.     a;-7  =  0  or  a;+l  =  0, 
x  =  l  or  —  1. 
Verification:  if  a;  =7,       x2-6a;-7  =  49-42-7  =  0, 
iix=-l,x^-6x-7=    1+   6-7  =  0. 

Ex.  2.— Solve  3x^+1  x=6. 

Transpose  the  6  so  as  to  make  the  right-hand  side  zero,  as  io  the 
previous  problem 

3.T2  +  7.r-6  =  0, 
(3.'r-2)(a;+3)=0, 
.-.      3.T-2  =  0  or  a;+3  =  0, 
.T=5  or  —3. 
Verify  both  of  these  roots. 


FACTORING  145 

Ex.  3. — Form  the  equation  whose  roots  are  2  and  —5. 

The  required  equation  is  at  once  seen  to  be  a  combination  of  the 
two  simple  equations 

X— 2  =  0  and  a;+5  =  0, 

and  therefore  is  (a;— 2)(a;+5)  =  0, 

or  a;2+3a;-10  =  0. 

Ex.  4. — If  x=2  is  a  root  of  the  equation 

.T3+3a;2-16.T+12=0, 
€nd  the  other  roots. 

Since  x=2  is  a  root,  then  x  —  2  is  a  factor  of  a;^  +  3a;-— 16a;+12  and 
the  other  factor,  found  by  division,  is  x^-\-5x  —  Q. 

:.    x^  +  3x^-l6x+12={x-2){x-l){x+6)  =  0, 
«— 2  =  0  or  a;— 1  =  0  or  a;+6  =  0, 
a;  =  2  or  1  or  —6. 
.'.   the  other  roots  are  1  and  —  6. 

EXERCISE  74  (1-16,  Oral) 

To  what  equations  of    the    first  degree  is  each  of    the  following 
equivalent : 

1.  (a;-l)(a;-2)=0  2.  (a;-3)(.-c+5)=0. 

3.  a;(a;-5)=0.  4.  (a;-l)(a;-2)(a;-3)=0. 

5.  a;2— 4=0.  6.  a;^— 4a;+3=0. 

7.  a;2+5a;+6=0.  8.  a;2-a;-20=0. 

9.  x^+3ax+2a-=0.  10.  x^—bx—12b^=0. 

State  the  equations  whose  roots  are  : 

11.     2  and  3.  12.     4  and -5.  13.     -2  and -4. 

14.     a  and  6.  15.     2,  3  and  1.  16.     4,  5  and —6 

Solve  and  verify : 

17.     x'-8x+15=0.  18.     x2+8.r+15=0. 

19.     x2+2a;-15=0.  20.     a;2-2x-15=0. 

21.     3x2-8x+4=0.  22.     4a;2-2a:-2=0. 

23.     2x2+a;=15.        •  24.     a;(3a;— 1)  =  10. 

25.     a;3-a;=0.  26.     x^~ax+bx-ab=0. 


146  ALGEBRA 

27.  If  x=2  is  a  root  of  a;^— 19a;+30=0,  find  the  other  roots. 

28.  Solve  x^-6x^+Ux-Q=0  and  ix^~l2x'^  +  UxS^0. 

(Note  that  the  sum  of  the  coefficients  is  zero.) 

29.  The  sum  of  a  number  and  its  square  is  42.     Find  the  number. 

30.  The  sum  of  the  squares  of  two  consecutive  numbers  is  61. 
Find  them. 

31.  The  sides  of  a  right-angled  triangle  are  x,  x-\- 1  and  x-\-2.   Find  x. 

105.  Notes  concerning  Factoring.  The  subject  of  factoring 
is  one  of  the  important  parts  of  algebra,  as  it  enters  into  so 
many  other  processes.  We  have  already  had  examples  of 
its  use  in  solving  equations  and  m  performing  operations  on 
fractions. 

In  the  preceding  exercises,  in  this  Chapter,  the  expressions 
to  be  factored  have  been  classified  for  the  pupil.  In  the 
practical  use  of  factoring,  however,  he  must  determine  for 
himself  the  particular  method  to  be  used. 

This  is  usually  done  by  determining  the  type  or  form 
to  which  the  expression  belongs.  The  examples  in  the 
review  exercise  which  follows  will  give  the  required  practice. 

The  types  which  have  been  discussed  in  this  Chapter  are 
here  collected  for  reference  : 

I.  ax-\-ay.    (Common  factor  in  every  term.) 

II.  ax+ay-\-hx-\-hff.     (Factored  by  grouping,] 

III.  x^±2xy-\-y'^.     (Complete  squares.) 

IV.  a^—b^.     (Difference  of  two  squares.) 
V.  x^+x^y^^y^.    (Incomplete  squares.) 

VI.    ax^-{-bx-{-c.     (Trinomials.) 
VII.    x^  ±  y^.     (Sum  or  difference  of  cubes. ) 
VIII.    Factored  by  the  factor  theorem. 


FACTORING  147 

EXERCISE  75  (Review  of  Chapter  XI) 

1.  State  the  squares  of  a-\-b,  a  —  b,  x—3y,  2a;— 1,  3x— 5,  5a  +  26, 
3x—4y,  7a  — 3,  a^—l,  a-{ 

2.  State  the  squares  of  a-\-b-\-c,  x-{-y~z,  a  —  b  —  c. 

3.  Writedown  the  products  of  x{a  —  b),  a{a  —  b-\-c),  {x-\-\){x+l), 
(a;-3)(x-5),  (2a;-3)(2a;+3),  (ax -(S){ax -{-!). 

Use  short  methods  to  find,  in  the  simplest  form,  the  value  of  : 
4.*  (x+a+6)(a;+a-6)  +  (a;-a-6)(x-a  +  6). 

5.  (x2  +  a;+l)(x2  +  a;-l)-(x2-a;+l)(a;2-x-l). 

6.  (o  +  6  +  c)2  +  (a  +  6-c)2  +  (a-6  +  c)2  +  (&-6-c)2. 

7.  (2a+36-c)2  +  (3a+6-2c)2  +  (a-26  +  3c)2. 

8.  99992-99982.  9.  57432-42572. 

10.  503x497-502x498.  11.  (a  +  99)2-(a  +  98)2 

Find  the  simplest  factors  and  verify  12-29: 

12.  a;2-a;-42.  13.  a;3-3a;2-a;+3 

14.  a;3-4x.        ^  15.  a2  +  a-56. 

16.  x^-a+ax-x.  17.  27x2-127/2. 

18.  a;»  +  5a;2-4a;-20.  19.  a;=»-3x2+2a;. 

20.  15a2+32a  +  9.  21.  343-a;». 

22.  a;*-4x2.  23.  18a;2  +  48x+32. 

24.  (a;-3)2  +  (a;-3)(x+4).  25.  15a;2- 15^/2- 16a;2/. 

26.  l  +  2a6-a2_62.  27.  x{x-2)  +  y{x-2)-x-\-2. 

28.  abc^  +  a^cd->rabd^  +  b~cd.  29,  25x«2/-40a;»i/2+ 16.^2^/3. 

30.  4(x-2)2-x+2.  31.  24a*-3a63. 

32.  (a+26-3c)2-(3a+26-c)2.      33.  12x2-x-20. 

34.  108o*-500.  35.  x'^  +  x-y^-[-y. 

36.  x*-7x2-18.  37.  x^-y^-2x^y  +  2xy'^. 

38.  x2-x2/- 1327/2.  39.  (a  +  c)(a-c)-6(2a-6). 

40.  a;3^y3_j.  33,^(3,  _^^)  41_  ax2-x(3a6  +  2)  +  66. 

42.  a2-462_3a-66.  43,  2x{2x  +  a) -y{y-\- a). 

44,  a^  +  2ab  +  b^^ac-\-bc.  45.  a2-2a6-f  62-a+i;. 

T      '> 


148  ALGEBRA 

46,  x^+x^-\-x—y^—y^—y.  47.  a^b-a^b^-a^b^-^abK 

48.  4a2-2562+2a+56.  49.  8(a  +  6)3-(2a-6)3. 

50.  a;Hy*- 18x22/2.  51.  a«-o2-9-2a262_|_64^6a. 

52.  a;3-llx2+7a;+3.  53.  3a3-5a2-8a+ 10. 

54.  a;2c3— cs+a;2— 1.  55.  a'-a3  +  8a*-8. 

56.  Show  that  a  — 6  +  c  is  a  factor  of 

(2o-36  +  4c)3  +  (2a[-6)3. 

57.  Factor  4a4— 37a262-[-964,  (i)  by  cross  multiphcation,  (2)  by 
completing  the  square  of  2a~  —  Zb",  (3)  by  completing  the  square  of 
2a2  +  362. 

58.  Without    multiplication    show    that 

(x2-4a;+3)(a;2-12a;+35)  =  (x2-6x+5)(x2-10x+21). 

59.  Make  a  diagram  to  show  the  square  of  a  +  b-\-c-^d. 

60.  Factor  {a-b){h^-c'^)-{b-c){a^-b^). 

61.  Find  the  factors  of  6.^3  —  7x2— 16a;+ 12,  being  given  that  it 
vanishes  when  x=2. 

62.  Find  four  factors  of  (a;2-5.'c)2+ 10(a;2  — 5x)  +  24  and  of 

(a;2  —  6)2  —  4a;(a;2  —  6)  —  5^2. 

63.  Use  the  factor  theorem  to  solve 

x3-31a;+30  =  0  and  a;*-43a;2  +  42a;  =  0. 

64.  If  two  numbers  differ  by  6,  show  that  the  difference  of  their 
squares  is  equal  to  six  times  their  sum. 

65.  Find  the  quotient  when  the  product  of  x'^  —  {b  —  c)x  —  bc  and 
.c2  — (c  — a)x  — ca  is  divided  by  x^~\-{a-'b)x  —  ab. 

66.  Multiply  a2- 62 _c2  + 26c  by  |i^i^- 

67.  If  a;*+a;^  +  aa;2  +  6a;  — 3  is  divisible  by  x—  1  and  a;+3,  find  a  and 
b  and  the  remaining  factor. 

68.  Factor  2x^-ax-\-bx  —  ab  —  a'^. 

69.  Express  a262-f  c2cZ2  — a2c2  — 62^2  ^s  the  difference  of  two  squares 
in  two  different  ways. 

70.  Factor  a*  +  6*  +  c*- 20*6^  262c2  — 2c2a2  by  completing  the 
square  of  a2  — 62  — c*. 

71.  Find  four  factors  of  (a2-62-c2  +  d2)--4(ad-6c)=. 


CHAPTER  XII 
SIMULTANEOUS  EQUATIONS  {continued) 

106.  In  Chapter  VII.  the  sohation  of  simple  examples 
of  equations  in  two  unknowns  has  been  considered. 

The  method  there  followed  was  to  make  the  coefficients 
of  one  of  the  unknowns  numerically  equal  by  multiplication, 
and  then  that  unknown  was  eliminated  by  addition  or 
subtraction. 

Other  methods  of  eliminating  one  of  the  unknowns  are 
useful  in  certain  cases. 

107.  Elimination  by  Substitution. 

Ex.— Solve  x-2i/=  2,  (1) 

5x+ly=lS.  (2) 

From(l),  x  =  2  +  2y.  (3) 

Substituting  this  value  of  x  in  (2), 

5(2 +  2^/)  + 72/ =  78, 

.-.  \0+lOy  +  ly  =  lS, 

172/ =  68, 

y=   4. 

Substituting  2/  =  4in(3),  x=10. 

Here  we  eliminated  x  by  finding  the  value  of  x  in  terms 
of  y  from  (1)  and  substituting  that  value  in  (2).  We  thus 
obtained  an  equation  which  contained  only  the  luikiiown  y. 

This  is  called  the  method  of  elimination  by  substitution. 

We  might  take  the  value  of  x  from  (2)  and  substitute  in  (1). 
Thus  from  (2),  5x=18-7y,      :.    x  -       ~   ^- 


5 


Substituting  in  (1),  ^^~  -  2?/  =  2. 

Complete  the  solution  and  verify  the  roots. 

149 


150  ALGEBRA 

The  value  of  y  might  have  been  found  from  either  equation  and 
substituted  in  the  other. 

Thus  from  (1),  2ij  =  x~2,      :.    y  =  ^~^. 

Substituting  in  (2),  bx  +  '^~~^  =  78. 

Complete  the  solution. 

Solve  also  by  finding  y  from  (2)  and  substituting  in  (1). 

If  the  four  solutions  be  compared  it  will  be  seen  that,  in 
this  problem,  the  first  is  the  simplest. 

In  solving  equations  with  two  unknowns,  the  pupil 
should  examine  them  carefully  and  choose  the  unknown 
which  he  thinks  will  be  the  simpler  to  deal  with. 

108.  Elimination  by  Comparison. 

Ex.— Solve  2a:- 3^=  7.  (1) 

3x-\-5y=39.  (2) 

From  (1),  a;  =  ''-^.  and  from  (2),  x  =  ^^~^, 

T  +  3y  ^  39-% 
2  3      ' 

.-.     3{l  +  3y)  =  2(3d-5y). 
Complete  the  solution  and  verify  the  roots. 

Here  we  effected  the  elimination  of  x  by  comparing  the 
values  of  x  from  the  two  equations. 

This  is  called  the  method  of  elimination  by  comparison. 

We  might  have  compared  the  values  of  y  obtained  from 
the  two  equations.     Solve  it  that  way. 

109.  Three  Methods  of  Elimination.  We  have  illustrated 
three  methods  of  elimination,  by  addition  or  subtraction, 
by  substitution  and  by  comparison.  When  no  particular 
method  is  specified,  the  pupil  is  advised  to  use  the  first 
method  as  no  fractions  appear  in  it. 


SIMULTANEOUS  EQUATIONS  151 

EXERCISE  76  (1-6,  Oral) 
State  the  value  of  eacli  unknown  in  terms  of  the  other  in : 
1.     x+y=5.  2.     x—7j=3.  3.     .r+2^=ll. 

4.     3x—y=Q.  5.     2x+3^=12  6.     5.^—4^=19. 

Solve  by  substitution  and  verif\' : 

7.       x+2i/=lS,  8.     Sx+  y=  7,  9.     2x—  ?/=19, 

2a;+5«/=41.  4x+3?/=ll.  5a;-3^=46, 

10.     2x-3y=\4:,  11.     3x-4?/=10,  12.       8.r+  Qy=   7, 

x-5//=  0.  2x+6^=ll.  10.c+21(/=12. 

Solve  by  comparison  and  verify  : 

13.     x+2y=\Q,  14.     2x-\-y=2Q,  15.     3.r+42/=10, 

.T-j-52/=14.  3a:— ?/=14.  4;«:— 3?/=  5. 

Solve  by  any  method  and  verify  : 

16.     \x-\y=2,  17.     Zx=2y,  18.     y=lx+Q, 

|a;+f2/=9  i^=ii2/-2.  3^-l2/=3|. 

19.     -  =  y,  20.     ?^  +  '^  =  6,  21.     3v-7.r=T, 

2      3  5^3  • 

'      \2  ^  4  ^  2/ 

22.  3x  +  2--^  =  2w  +  ^^  =  10. 

11  -^  7 

23.  x-52/+3=2x-8?/+3  =  7a;-10?/+16. 

24.  (x-l)(?/-2)-(y-3)(.T+l)  =  17, 
(x-3)(y-5)-(.i--5)(2/-3)=-22. 

25.  •la;+-21?/+-52=.-01x-+-01i/+3=0. 

26.  x+5  =  3(.y-3),  ^^  +  2/  =  -~^+19. 

27.  ^^3^-1:5  ^  ^^._  23^  2^^-9^, 

4  ■^6  2 

28.  If  the  sum  of  two  numbers  is  i  of  the  greater  number,  the 
difference  of  the  numbers  is  how  many  times  the  less  ? 


152  ALOEBRA 

110.     Equations  with  three  Unknowns. 

Ex.— Solve  2.T+3J/— 4z=12,  (1) 

3:f-  y+2z=\5,  (2) 

4a;+  2/-32=19.  (3) 

This  system  of  equations  differs  from  the  preceding  by 
containing  three  unknown  quantities. 

If  we  can  obtain  from  these  three  equations,  two  equations 
containing  the  same  two  unknowns,  the  solution  can  be 
effected  by  preceding  methods. 

How  can  we  obtain  from  (1)  and  (2)  an  equation  containing 
X  and  z  only  ?  How  can  we  obtain  another  equation  from 
(2)  and  (3)  containing  x  and  z  only  ? 

Perform  these  two  eliminations  and  find  x  and  z  from  the 
resulting  equations. 

Now  find  y  by  substituting  in  any  one  of  the  given  equations 
and  verify  by  showing  that  the  values  you  have  found  for  x, 
y  and  z  will  satisfy  all  of  the  given  equations. 

The  solution  might  be  written  in  the  following  form  : 


liminate  y  from  (1)  and  (2), 

.-.   lla;+22  =  57. 

(4) 

„     „      (2)     „    (3), 

.-.     7a;- 

2  =  34. 

(5) 

z      „      (4)     „    (5), 

x=    5. 

ibstitute  x  =  5  in  (4), 

2=     1. 

,,           a;  =  5  and  2=1  in  (1), 

y=  2. 

.-.    x  =  5,  y  =  2,  2=1. 

Of  course  it  will  be  seen  that  any  other  unknown  might  have  been 
eliminated  twice  from  two  pairs  of  the  equations. 

Thus  we  might  have  eliminated  z  from  (1)  and  (2)  and  also  from 
(1)  and  (3),  and  thus  obtained  two  equations  in  x  and  y.  We  might 
then  have  completed  the  solution  as  before. 

Solve  the  equations  by  this  plan.  Also  solve  them  by  two 
eliminations   of   x. 

Which  letter  do  you  think  is  easiest  to  eliminate  twice  ? 

Note  that  the  solution  is  completed  only  when  the  values  of  all 
of  the  unknowns  have  been  found. 


SIMULTANEOUS  EQUATIONS  153 


BXBRCISB  77  (1-4.  Oral) 

1.  What  operation  will  eliminate  both  x  and  a;-fi/+2=35  (1) 
y  from  (1)  and  (2)  ?  What  is  the  value  of  z  ?  x+y—z=25  (2) 
of  x+yl  x-y-\-z=\5  (3) 

2.  What  operation  will  eliminate  both  y  and  z  from  (2)  and  (3)  ? 
What  is  the  value  of  a:  ?  oi  y~zt 

3.  How  can  you  eliminate  both  x  and  z  from  (1)  and  (3)  ?  What 
is  the  value  oiy  t   oix-\-zt 

4.  In  No.  5  below,  which  letter  is  simplest  to  eliminate  from  two 
pairs  of  the  equations  ?     Which  in  No.  6  ?     Which  in  No.  7  ? 

Solve  and  verify : 

5.  a;+2;/+  33=16,  6.  2x-y+^z=  7, 
a;+3^+  4z=24,  Zx+y-Az=  7, 
a;+4?/+10z=41.  %x-y+5z=2\. 

7.     4x-3;/+  z==10,  8.     a;+2/—  ^=16, 

6a;— 5(/+23=17,  a;— ?/+  2=  4, 

x-\-  y+  2=  8,  a;+2/+22=22. 

9.       x+2^+32=32,  10.     x+y^25, 

4:x-5y+6z=27,  y+z=15, 

lx+8y—9z=l4:.  2+a:=70. 

11.       x+2y=12,  12.     3(2-1)     =2(?/-l), 

3y+4:z=  2,  ^y+x)     =9z  -4, 

52_2.c=-21.  7(5a;-32)=2;/ -9. 

13.     3+a;=5-42/,  14.     ir+  J?/+  Jz  =36, 

2+x=3;/,  i^+i^s2/+2V2  =10, 

Ty=z+2.  lx+  ky+^z  -43. 

2^3^4      3^4^5      4^5^6 
16.*  If  x-\-2y—2o,  2/+3z=5o,  z-|-4a;=35,  find  the  value  of  x-{-y-\-z. 

17.  If  X— ?/+z=9,  2x--|-?/=8,  «/— 4z=5  and  x+y-{-z+io=\2, 
find  ?<?. 

18.  If  ax'^—hx-\-c  is  8  when  .<;=1,  8  when  a;— 2,  and  10  when  .T=3, 
find  a,  6  and  c. 


154  ALGEBRA 

19.  If  ax"-\-bx-\-c  is  9  when  x=\,  —3  when  x=  —  \,  18  when 
x=2,  find  its  value  when  .x=3. 

20.  Determine  three  numbers  whose  sum  is  9,  such  that  the  sum 
of  the  first,  twice  the  second  and  three  times  the  third  is  22,  and  the 
sum  of  the  first,  four  times  the  second  and  nine  times  the  third  is  58. 

21.  If  «+6=12,  6+c=15,  c4-rf=19.  find  a+d. 

111.  Special  Forms  of  Equations.  Two  equations  of  the 
first  degree  iii  x  and  y  will  usually  determine  the  values  of 
X  and  y. 

Consider  the  following  sets  of  equations 

(1)  2x-Zy=lO,  (2)  2x-^y=\0,  (3)  2a:-3y=10. 

4a;+5i/=42.  4.T-6?y=20.  4a;— 6!/=30. 

In  (1),  if  the  two  equations  are  solved  in  the  usual  way 
we  find  that  x— 8,  ?/=2  will  satisfy  both  of  the  equations, 
and  no  other  values  of  x  and  y  will  satisfy  them. 

We  therefore  say  that  these  equations  are  determinate,  that 
is,  the}'  determine  the  values  of  x  and  y. 

In  this  ease  the  second  equation  can  not  be  deduced  from 
the  first,  nor  the  first  from  the  second.  We  therefore  say 
that  the  equations  are  independent. 

In  (2),  the  second  equation  may  be  deduced  from  the  first 
by  multiplying  by  2.  These  equations  are  dependent  and 
not  independent  as  in  (1). 

Any  number  of  values  of  x  and  y  will  satisfy  both  equations, 
because  any  values  which  will  satisfy  the  first  will  also  satisfy 
the  second.     These  equations  are  therefore  indeterminate. 

In  (3),  if  the  first  equation  is  true,  the  second  can  not  be 
true.  They  are  therefore  said  to  be  inconsistent  or  impossible, 
and  no  values  of  x  and  y  can  be  found  to  satisfy  both  of  them. 

We  thus  see  that  two  equations  in  two  unknowns  can  have 
a  definite  solution  only  ivhen  the  equations  are  independent 
and  consistent. 

In   this   set    of    equations,    the    third    may    be  3x-\-'2y—   z=    5, 

obtained   by   adding   the   other   two.     They   are  4a;—   i/  +  32  =  20, 

therefore  dependent  equations-  and  consequently  lx+   y-\-2z  =  25. 
indeterminate. 


SIMULTANEOUS  EQUATIONS  155 


BXBROISB  78 

1.  Find  three  pairs  of  values  of  x  and  y  which  satisfy  the  equation 
2x-3y=l2. 

2.  Solve  2.r+3?/=13,  Sx— ?/=24.     Is  it  possible  that  2x+3y=\3, 
5x — y=24  and  4a;-|-5?/=19  can  be  true  at  the  same  time  ? 

3.  What  is  peculiar  about  the  equations  4.t-|-«/=17,  8x-\-2y=35  ? 
Also  about  8x+l2y=60,  6.r+%=45  ? 

4.  Shov/  that  the  equations 

x+z+4:  =  3y,  3x+z=2y+6,  2x+y  =  l0, 
are  indeterminate.     If  2=5,  solve  the  equations. 

a.     Find  two  solutions  of  the  simultaneous  equations  . 
xJi-y-\-z=lO,  3x—2y—z=l. 

For  what  values  of  a  will  the  following  sets  of  equations  be  consistent : 
6.*  3x—  y=  5,  7.       3x+2y=  1,  8.     9.r—  ay=Q, 

x-^2y=25,  10.r-4?/=   2,  3.c-     y=2, 

.r+4^y=  a.  3x'+rt?y=ll.  5.r— ^if;y=^-. 

9.     Show  that  these  equations  are  inconsistent : 

2x+3y— 3z=20,  3;r-L7//— 22=5,  a;+2y— 2=6. 


(1) 

(2) 

Here  we  coiild  obtain  the  solution  in  the  usual  way  by  removing 
the  fractional  forms,  by  multiplying  each  equation  by  xy.  See  if  you 
can  complete  the  solution  by  this  method. 

It  is  simpler,  however,  to  eliminate  y  from  the  equations  as  they 
stand. 


112. 

Special  Fractional  Equations. 

Ex.- 

-Solve 

X      y 

4      2 

^  +  -  =  21. 

X      y 

156  ALGEBRA 

Thus,  multiplying  (2)  by  4  and  adding 

—  =  95,      .-.    95a;  =  19,      .-. 

X 

1  8 

Substitute  a;  =  >  in  (1)  and  15 =  11. 

5        '  '  y 

y  ^ 

The  solution,  therefore,  is  x  =  i,  y  =  2. 
Verify  this  result. 


Solve  and  verify : 

EXERCISE  79 

1.    ^   +'-    =2, 
X        y 

24      21  _  J 

X        y 

2. 

'  +  '    =29, 
X      y 

5_6    _2. 

X      y 

3. 

?  +  '=19, 

a;      y 

4.     ^    _4    ^8, 
a;        2/ 

1?  +  '   =101. 

5. 

-  +  2^/  =  15, 
a; 

!-3,  =  0. 

6. 

1+2=11, 

X      y 

?-?=2, 

!(      » 

*-l=I7. 

Z         X 

7.     3y—5x=xy, 
2y-\-Sx=26xy. 

8.     ^  +  o^  = 

a;  ^2^ 

15 
2a: 

+^=^. 

^32/       6 

9      5      3  _  135      75  _ 

X      y        X         y 

30 

10.     3a;  H 1  =  12a;  + 

2/ 

5 

y 

+  14  =  i  -  2a;  -  14. 

y 

11.     ?  +  2_^^^1^4 
X      y                X      y 

=  ■ 

122 --  +  -  =  83 +  17. 
X      y 

113.  Problems  leading  to  Simultaneous  Equations.  In 
Chapter  VII.  we  have  had  illustrations  of  problems  which 
were  solved  by  using  equations  of  two  unknowns.  We  now 
give  some  further  examples  on  special  subjects  which  were 
not  then  considered. 


SIMULTANEOUS  EQUATIONS  157 

The  number  47  might  be  written  4  .  10  +  7.  What  is  the  sum  of  the 
digits  of  this  number  ?  What  number  would  be  formed  by  reversing 
the  digits  ?  What  is  the  sum  of  the  number  and  the  reversed  number  ? 
What  is  the  sum  of  the  digits  of  the  reversed  number  ? 

Ex.  1. — A  number  has  two  digits.  If  IS  is  added  to  it 
the  digits  are  reversed.  The  sum  of  the  two  nuinbers  is  88. 
Find  the  number. 

Let  a;  =  the  units  digit  and  y  the  tens  digit, 

.".  the  number  =lOy-i-x, 

and  the  reversed  number  =lOx+y. 

:.    I0y  +  x+l8=10x+y,  (1) 

and  I0y  +  x+10x  +  y  =  88.  (2) 

Simplifying  (1),  9a;  — 9?/=  18  or  a;  —  ?/ =  2, 

(2),  Ux+ny  =  88  or  x+y  =  8. 

Solving  x  =  5,  y  =  3. 

.".    the  required  number  is  35. 

Verification  :  35+18  =  53,35+53  =  88. 

Ex.  2. — If  4  be  added  to  the  numerator  of  a  fraction 
and  3  to  the  denominator,  the  fraction  becomes  h.  If  2 
had  been  subtracted  from  the  numerator  and  5  from  the 
denominator  the  result  would  have  been  ^.  Find  the 
fraction. 

Let  -  =  the  fraction, 

y 

x+i       I  ,        x-2       1 

~r5  =  o  and =  ^  • 

2/  +  3       2  y-5       6 

2a;+8  =  2/  +  3      and    6x—l2  =  y  —  5. 

2x  —  y=:    —5     and     dx—   y  =  l. 

Complete  the  solution  and  verify. 

Sometimes  the  solution  of  a  problem  may  be  simplified 
by  using  some  function  of  x  instead  of  x  to  represent  one 
of  the  imknowns. 

Thus,  if  two  numbers  are  in  the  ratio  of  7  to  6,  we  might 
represent  the  larger  number  by  x  and  then  the  smaller  ^ould 
be  yo:. 


158  ALOEBRA 

A  better  way,  however,  would  be  to  represent  the  larger 
by  Ix,  and  then  the  smaller  would  be  Qx.  By  domg  so  we 
get  rid  of  the  use  of  fractions. 

Ex.  3. — The  incomes  of  A  and  B  are  in  the  ratio  of  3  to 
2,  and  their  expenses  in  the  ratio  of  5  to  3.  Each  saves 
$400  a  year.     Find  their  incomes  and  expenses. 

Let  $3a;  =  .4's    income,    then    %2x  =  B's  income. 

Let  %5y  =  A's  expenses,  then  $3?/  =  Z?'s  expenses. 

.-.     3a;  -  52/  =  400  and  2x  -  3^/  =  400. 
Solving  X  =  800  and  y  =  400. 

.-.      .4's  income  =  %Zx=  $2400  and  B's  =  $1600. 

.-.     ^'s  expenses  =  $5y=  $2000  and  B's  =  $1200. 

Note. — In  solving  the  problems  in  the  exercise  following,  the  pupil 
will  find  that  he  frequently  has  the  choice  of  using  one,  two  or  more 
unknowns.  Except  in  special  cases,  he  is  advised  to  use  as  small  a 
number  of  unknowns  as  possible.  In  each  case  the  results  should  be 
verified. 

EXERCISE  80 

1.  If  10  men  and  4  boys,  or  7  men  and  10  boys,  earn  $96  in  a 
day,  find  a  man's  wages  per  day. 

2.  Two  numbers  are  in  the  ratio  of  5  to  7  and  their  difference  is 
10.     What  are  the  numbers  ? 

3.  The  sum  of  three  numbers  is  370.  The  sum  of  the  first  two 
is  70  more  than  the  third,  and  six  times  the  first  is  equal  to  four  times 
the  third.     Find  the  numbers. 

4.  Find  three  numbers  such  that  the  results  of  adding  them  two 
at  a  time  are  29,  33,  36. 

6.  Divide  429  into  three  parts  so  that  the  quotient  of  the  first 
by  7,  the  second  by  4  and  the  thu-d  by  2  will  all  be  equal. 

6,  A  workman  can  save  $200  a  year.  He  goes  to  another  town 
where  his  wages  are  10%  greater  and  his  expenses  are  5%  less,  and  he 
can  now  save  $310  a  year.     What  are  his  wages  now  ? 

7.  The  denominator  of  a  fraction  exceeds  the  numerator  by  3. 
If  2  is  subtracted  from  each  term,  the  fraction  reduces  to  |.  Find 
the  fraction, 


SIMULTANEOUS  EQUATIONS  159 

8.  Divide  120  into  three  parts,  so  that  f  of  the  first  part  is 
greater  than  the  second  by  5  and  J  of  the  second  part  is  greater  than 
the  third  by  10. 

9.  If  6  men  and  2  boys  earn  S56  in  2  days  and  7  men  and  5  boys 
earn  S57  in  1|  days,  how  long  wUl  it  take  3  men  and  4  boys  to  earn  S60  ? 

10.  A  number  between  10  and  100  is  8  times  the  sum  of  its  digits, 
and  if  45  be  subtracted  from  it,  the  digits  are  reversed.  Find  the 
number. 

11.  The  difference  of  the  two  digits  of  a  number  is  4.  The  sum  of 
the  number  and  the  reversed  number  is  110.     Find  the  number. 

12.  The  sum  of  the  two  digits  of  a  numbers  is  14,  and  when  18  is 
added  to  the  number  the  digits  are  reversed.     Find  the  number. 

13.  When  1  is  added  to  both  terms  of  a  fraction  the  result  is  ^.  If 
9  had  been  subtracted  from  the  denominator  only  the  result  would 
have  been  J.     Find  the  fraction. 

14.  A  number  consists  of  two  digits  whose  sum  is  II.  If  the  order 
of  the  digits  be  reversed,  the  number  thus  obtained  is  greater  by  7 
than  twice  the  original  number.     What  is  the  number  ? 

15.  The  difference  between  the  digits  of  a  number  less  than  100  is  6. 
Show  that  the  difference  between  the  number  and  the  number  formed 
by  reversing  the  digits  is  always  54. 

16.  The  sum  of  the  reciprocals  of  two  numbers  is  ^g.  Six  times 
the  reciprocal  of  the  first  is  greater  than  five  times  the  reciprocal  of 

the  second  by  J.     Plnd  the  numbers.     (The  reciprocal  of  x  is -•) 

17.  Divide  150  into  two  parts  such  that  the  quotient  obtained  by 
dividing  the  greater  by  the  less  is  3  and  the  remainder  is  2. 

18.  I  wish  to  obtain  100  lb.  of  tea  worth  34c.  per  lb.  by  mixhig 
tea  worth  30c.  per  lb.  with  tea  worth  40c.  per  lb.  How  much  of  each 
must  I  take  ? 

19.  Three  pounds  of  tea  and  10  of  sugar  cost  $2-40.  If  tea  is 
increased  10%  in  price  and  sugar  decreased  10%,  they  would  cost 
S2-52.     Find  the  price  of  each  per  lb. 

20.  Two  numbers  are  in  the  ratio  of  7  to  5.  What  quotient 
is  obtained  when  three  times  their  sum  is  divided  by  six  times  their 
difference  ? 


160  ALGEBRA 

21.  Show  that  the  sum  of  any  number  of  two  digits  and  the  number 
formed  by  reversing  the  digits  is  always  divisible  by  11  and  that  the 
difference  is  divisible  by  9. 

22.  A  number  has  three  digits,  the  middle  one  being  0.  If  396 
be  added  the  digits  are  reversed.  The  difference  between  the  number 
and  five  times  the  sum  of  the  digits  is  257.     What  is  the  number  ? 

23.  Divide  126  into  four  parts,  so  that  if  2  be  added  to  the  first, 
2  be  subtracted  from  the  second,  the  third  be  multiplied  by  2,  and  the 
fourth  be  divided  by  2,  the  results  wiU  aU  be  equal.     (Let  the  result=a;.) 

24.  There  are  three  numbers  such  that  when  each  is  added  to  twice 
the  sum  of  the  remaining  two  the  results  are  44,  42,  39.  Find  the 
numbers. 

25.  The  sum  of  the  three  digits  of  a  number  is  12.  If  the  units 
and  tens  digits  be  interchanged  the  number  is  increased  by  36,  and  if 
the  hundreds  and  units  be  interchanged  it  is  increased  by  198.  Find 
the  number. 

26.  Find  three  numbers  such  that  the  first  with  ^  of  the  sum  of 
the  other  two,  the  second  with  J  of  the  others,  and  the  third  with 
i  of  the  others,  shall  each  be  25. 

27.  A  piece  of  work  can  be  done  by  A  working  6  days  and  5  21 
days,  or  by  A  working  8  days  and  5  18  days.  In  what  time  could 
each  of  them  complete  it  alone  ? 

28.  Divide  84  into  four  parts,  so  that  the  first  is  to  the  second  as 
2  to  3,  the  second  to  the  third  as  3  to  4,  and  the  third  to  the  fourth  as 
4  to  5. 

29.  Of  what  three  numbers  is  it  true  that  the  sum  of  the  reciprocals 
of  the  first  and  second  is  J,  of  the  first  and  third  is  J  and  of  the  second 
and  third  is  J  ? 

30.  Two  numbers  consist  of  the  same  three  digits  but  in  inverted 
order.  The  sum  of  the  numbers  is  1029.  The  sum  of  the  digits  of 
each  is  15  and  the  difference  of  the  units  digits  is  5.     Find  the  numbers. 

31.  A  stream  flows  at  2  miles  per  hour.  A  man  rows  a  certain 
distance  up  stream  in  5  hours  and  returns  in  If  hours.  How  many 
miles  per  hour  could  he  row  in  still  water  ? 

32.  A  rancher  sold  50  head  of  horses,  part  at  §125  a  head  and  the 
balance  at  $150  a  head.  After  spending  $50  he  was  able  to  make  the 
first  payment  of  ^  of  the  purchase  price  of  1200  acres  of  land  at  $18 
per  acre.     How  many  horses  did  he  sell  at  $125  a  head  ? 


SIMULTANEOUS  EQUATIONS  161 

33.  A  number  consists  of  a  units  digit  and  a  tenths  digit,  the  units 
digit  being  the  greater  by  1.  The  sum  of  the  digits  is  less  than  twice 
the  number  by  2.     Find  the  number. 

34.  A  grocer  spent  $120  in  buying  tea  at  60c.  a  lb.,  and  100  lb. 
of  coffee.  He  sold  the  tea  at  an  advance  of  25%  on  cost  and  the  coffee 
at  an  advance  of  20%.  The  total  selling  price  was  $148.  Find  the 
number  of  lb.  of  tea  purchased. 

35.  When  2  is  subtracted  from  each  term  of  a  fraction  the  result 
is  equal  to  .^.     Show  that  the  result  would  have 
been  the  same  if  1  had  been  subtracted    from 
the  numerator  only. 

36.  It  is  shown  in  geometrj^  that  the  two 
tangents  drawn  to  a  circle  from  a  point  are 
equal.  Thus,  in  the  figure  AD=AE,  etc.  If 
AB^\b,  BC=14:,  CA  =  13,  find  x,  y  and  z. 

37.  If  the  sides  of  a  triangle  are  10,  15  and 
19,  where  will  the  inscribed  circle  touch  the 
sides  ?     (See  figure  of  preceding  example.) 

38.  li  A  can  do  a  piece  of  work  in  m  daj's  and  B  can  do  it  in  n  days, 
in  what  time  can  they  do  it  working  together  ? 

If  X  is,  the  number  of  days  required, 

1       1,1  *"« 

then   -  = 1 


m      n  m-\-n 

39.  Use  the  preceding  result  to  find  in  what  time  A  and  B  working 
together  can  do  a  piece  of  work  which  could  be  done  by  A  and  B 
separately  in  the  following  number  of  days  : 

(1)  yl  in  10,  £  in  15.     (2)  A  in  20,  S  in  5.     (3)  A  in  f,  B  in  1^-. 


EXERCISE  81   (Review  of  Chapter  XII) 
Solve  and  verify  : 


7a;- 

-8y= 

10, 

3x- 

-2y  = 

=  10. 

10 

X 

12 

=  14, 

7 

X 

4. 

=  16, 

4:x+7y=~l,  3.     73x+y=   75, 

3a;-   2/ =  3.  x+ldy^Ul. 

^       ^       3,  6.     12y~-8x  =  2xy, 


X      y 
2      25 


Sy-]-4:X  =  2xy. 

M 


162 

ALGEBRA 

7. 

"+'-7,    ,-^     =6. 

y            1+2/ 

8. 

X+l           03+3           ^ 

2/  +  2       2y+l 

9. 

x-y       2x  +  3y 
3^5 

10. 

x~2y       Zx-y 
3               7     ' 

^x+y)  =  8. 

11. 

3.      '^-^  =   6, 
5 

12. 

x— 1       2/  — 3      z— 5 
2     "~      4     ~     6    ' 

4y  +  ^-'  -  12. 

a;+2/+2=33. 

13. 

x+  y+  2=-3, 

14. 

2x+3i/-   2  =  5, 

x  +  2y+   2=-0, 

3x  — 42/  +  22=l, 

303+  2/  +  62  =  0. 

4a;-6y  +  5z  =  7. 

15. 

3  =  8  +  ^-2-^- 

16. 

a;-2       10-a;      y—\0 
5               3^4' 

a;  +  y  +  z  =  24. 

22/  +  4       2a;+2/      x-\-\Z 
3               8      ~      4 

a;— 1       y  +  5  ^  a;+2 
3  12    "'    60  ' 

{x^\l){y-n)  =  xy-5. 

18.  ia;-f2/  +  2+l  =  3(a3-2/)  +  52  +  4=a;+62/-22-9  =  0. 

_^      5x~3y        .         _         102  +  Ja; 

19.  — ^^  =  42/  -  22  = 2 =  ^• 

20.  x+y  =  5,  y  +  z  =  Z,  z+w=l,  x—iv=3. 

21.  31a;  +  282/=146,  28x+31?/=149.  (Add  and  subtract  the 
equations  and  remove  common  factors.) 

22.  97a;-592/  =  329,  59a;-97y=139. 

23.  What  values  of  x  and  y  will  make  ^  and  — ^  each  equal 
toa3-10  ? 

24.  Show  that  x+y  +  z=l2,  3a;  +  4?/ - 5z  =  - 22,  10x+ 122/-6z  =  4, 
are  indeterminate. 

25.  Divicfe  unity  into  two  parts  so  that  18  times  the  first  part  may 
exceed  12  times  the  second  by  13. 

26.  A  number  of  two  digits  is  four  times  the  sum  of  its  digits, 
and  if  18  be  added  to  the  number  the  digits  are  reversed.  What  is 
the  numlxT  ? 

27.  The  tens  digit  of  a  number  is  twice  the  units  digit.  When  the 
number  is  divided  by  the  sum  of  the  digits  what  must  the  quotient  be  ? 


SIMULTANEOUS  EQUATIONS  163 

28.  Find  a  fraction  equal  to  |  such  that  ^  of  the  denominator 
exceeds  f  of  the  nvunerator  by  8. 

29.  Two  persons  who  are  30  miles  apart  are  together  after  5  hoiu-s 
if  they  walk  in  opposite  directions,  but  are  not  together  for  15  hours 
if  they  walk  in  the  same  direction.     What  are  their  rates  ? 

30.  ^'s  age  is  equal  to  the  combined  ages  of  B  and  C.  Ten  years 
ago  A  was  twice  as  old  as  B.  Show  that  10  years  hence  ^  will  be  twice 
as  old  as  C. 

31.  A  biU  of  S19-50  was  paid  in  half-dollars  and  quarters  and  four 
times  the  number  of  quarters  exceeded  twice  the  number  of  half-dollars 
by  12.     How  many  of  each  were  used  ? 

32.  If  5  lb.  of  tea  and  8  lb.  of  coffee  cost  $5*80,  and  coffee  advances 
10%  in  price  and  tea  15%  and  they  now  cost  $6'53,  find  the  prices 
per  lb.  of  each  before  the  advance. 

33.  I  invest  a  certain  sum  at  4%  and  another  sum  at  6%  and 
receive  $42  interest.  If  the  sums  had  been  interchanged  I  would 
have  received  $8-50  more.     What  were  the  sums  ? 

34.  If  each  side  of  a  rectangle  is  increased  by  5  feet  the  area  is 
increased  by  275  square  feet.  If  each  side  is  decreased  by  5  feet  the 
area  is  decreased  by  225  square  feet.  Show  that  the  sides  can  not 
be  determined  from  these  conditions. 

35.  Solve  ^-^t|:=-^=^  =  4x-.=  l.      ■ 

36.  A  grocer  wishes  to  mix  tea  worth  30c.  a  lb.  with  tea  worth 
40c.  to  make  a  mixture  weighing  601b.  worth  36c.  a  lb.  How  many  lb. 
of  each  must  he  use  ? 

37.  If  3x^~2x-\-5=ax^-\-bx-\-c,  when  x=l  or  x=2  or  x=3,  show 
thata=3,  6=  — 2,  c  =  5. 

38.  The  tens  digit  of  a  number  exceeds  the  units  digit  by  3.  By 
how  much  is  the  number  decreased  by  inverting  the  digits  ? 

39.  A  train  is  27  minutes  late  when  it  makes  its  usual  trip  at  28 
miles  per  hour  and  is  42  minutes  late  when  it  runs  at  27  miles  pei 
hour.     What  is  the  distance  ? 

40.  A  piece  of  work  can  be  done  by  A  working  6  days  and  B  10  days 
or  by  A  working  9  days  and  B  14  days.  How  long  would  it  take  each 
alone  to  do  it  ? 

41.  A  number  has  three  digits,  the  units  being  J  of  the  tens  and  ^ 
of  the  hundreds.  If  396  be  subtracted  the  digits  are  reversed.  Find 
the  number. 

M  2 


164  ALGEBRA 

42.  When  the  greater  of  two  numbers  is  divided  by  the  less  the 
quotient  is  5  and  the  remainder  is  2.  When  12  times  the  less  is  divided 
by  the  greater  the  quotient  is  2  and  the  remainder  is  12.  Find  the 
nvunbers. 

43.  Find  four  numbers  such  that  when  each  is  added  to  twice 
the  sum  of  the  remaining  three,  the  results  are  46,  43,  41  and  38 
respectively. 

44.  If  the  sum  of  two  numbers  is  a  times  the  greater  and  the 
difference  is  b  times  the  smaller,  show  that  a—b  +  ab  =  2. 


CHAPTER  Xm 
GEOMETRICAL  REPRESENTATION  OF  NUMBER 

114.  Function  of  x.  The  value  of  the  expression  3x— 2 
depends  upon  the  vahie  of  x. 

Thus,  when  x=  4,  3,  2,  1,      0,  -1,  -2,     -3,     -4, 

3a;-2=10,  7,  4,  1,  -2,  -5,  -8,  -11,  -14. 

When  the  value  of  an  expression  depends  upon  the  value 
of  X,  the  expression  is  called  a  function  of  x. 

Thus,  2a;— 3,  5a;,  ^a;+  1,  are  functions  of  x. 

What  is  the  value  of  each  of  these  functions  when  a;  =  2,  1,  0, 
-1,  -2  ? 

Instead  of  repeating  the  words  "  the  expression  "  or  "  the 
function,"  we  might  represent  the  function  by  a  symbol, 
say  ij. 

Thus,  if  y  =  5x-\-\,  when  x=  1,  y  =  &  ;  a;=3,  y=  16. 
If  i/  =  ^a;+4,  what  are  the  values  of  y  when  x  has  the  values  6,  3,  0, 
-1,  -8  ? 

115.  Variables  and  Constants.  A  quantity  that  has  not 
always  the  same  value  is  called  a  variable,  while  a  quantity 
whose  value  does  not  change  is  called  a  constant. 

Thus,  the  population  of  a  city  and  the  height  of  the  barometer  are 
variables,  while  the  ntunber  of  days  in  a  week  and  the  length  denoted 
by  an  inch  are  constants. 

Note. — To  do  the  work  of  this  chapter  properly,  pupils  should  be 
suppUed  with  squared  paper.  Paper  ruled  in  tenths  or  eighths  of  an 
inch  will  be  found  most  satisfactory. 

165 


166 


ALGEBRA 


116.  Connected  Variables.  Two  variables  may  be  so 
connected  that  for  every  change  in  the  value  of  one  there  is 
a  corresponding  change  in  the  value  of  the  other. 

Thus,  if  y  =  2x  +  5,  for  each  value  of  x  there  is  a  corresponding 
value  of  y.     Here  x  and  y  are  variable  quantities,  but  5  is  a  constant. 

In  arts.  20  and  21  we  have  shown  how  the  changes  in  two 
variable  quantities  may  be  represented  by  a  diagram.  Those 
diagrams  show  that  for  each  variation  m  time  there  is  a 
corresponding  variation  in  temperature. 


117.  Graph.     A  line  so  drawn  as  to  exhibit  the  nature  of 
the  relation  of  two  variables  is  called  a  graph. 

118.  Arithmetical  Graphs.     The  solution  of  many  problems 
in  arithmetic  might  be  rej)resented  graphically  as  follows  : 

Ex.  1. — The  passenger  rate  on  a  railway  is  3  cents  per 
mile.     Represent   graphically   the   amount   charged  for   any 

number  of  miles  from  1  to  10. 


In  the  diagram  each  iinit  on  the  hori- 
zontal line  OX  represents  1  mile  and  each 
unit   on  the  vertical  line    OY   represents 

3  cents. 

The  point  A  shows  that  the  cost  for 

4  miles  is  12  cents.     What  does  the  point 
B  show  ?     The  point  C  ? 

Read  from  the  figure  the  cost  for  2 
miles,  5  miles,  9  miles.  How  far  can  I 
travel  for  9  cents,  21  cents,  27  cents  ? 


■~  15 
t--  12 


Y 

/ 

/ 

C 

/ 

/ 

/ 

/ 

B 

/ 

/ 

A 

/ 

/ 

^ 

X 

8    9  10 

No.  of  Miles 


Ex.  2. — Represent  graphically  the 
simple  interest  at  2%  on  |100  for  any  number  of  years  from 
1  to  6. 

Reading  from  the  diagram  (on  the  next  page),  what  is  the  interest  on 
SI 00  in  2  years  ?    In  5  years  ?     In  4^  years  ? 

What  does  the  point  A  show  ?  The  point  C  ?  The  point  D  ?  The 
point  half  way  between  C  and  D  ? 

In  how  many  years  is  the  interest  $8,  $6,  $5,  $11,  $6-50  ? 


GEOMETRICAL  REPRESENTATION  OF  NUMBER    167 


Place   a  ruler   on   the  points  marked   A,   B,   G,    D,   E,  F.     What 
peculiarity  do  you  notice  ? 

Make  a  similar  diagram,  on  sqviared  paper,  which  will  give  the 
interest  on  §200  at  4%  for  any  num- 
ber of  years  from  1  to  7.  So  that 
your  diagram  will  not  occupy  too 
much  space  vertically,  suppose  each 
unit  on  OY  to  represent  §4  instead 
of  $1. 

Read  from  your  diagram  the  in- 
terest for  3  years,  for  5  years.  In 
how  many  years  is  the  interest  $8, 
$24,  S4,  812,  S44  ? 


Q   8 

■5  ^ 

^   6 


Y 

/ 

/ 

F 

/ 

/ 

E 

/ 

/ 

D 

/ 

/ 

C 

/ 

/ 

B 

/ 

/ 

A 

d 

X 

Time  in  Years 


Ex.  3. — A  starts  at  9  a.m.  on 
a   bicj^cle   at    8   miles  per   hour. 
He  is  followed  at  noon  by  B  on  an  automobile  at  16  miles 
per  hour.     When  and  where  will  B  overtake  A  ? 

Each  space  on  the  horizontal  line  represents  1  hour,  and  on  the 
vertical  line  4  miles. 

At  the  end  of  successive  hours  A's  position  will  be  C,  D,  etc.,  and 
B"s  will  be  F,  G,  etc. 

The  line  OP  is  the  graph  of  A's  journey  and 
MQ  is  the  graph  of  B's. 

The  diagram  shows  that  B  overtakes  A  at 
the  point  R,  which  is  48  miles  from  the  start- 
ing point  and  that  the  time  is  3  p.m. 

How  far  is  A  ahead  at  12  o'clock  ?  at  1  ? 
at  2  ? 

Solve  the  problem  otherwise  and  compare 
the  results. 

Ex.  4. — A  starts  from  P  at  9  A.M.  to 
go  to  Q,  a  distance  of  60  miles,  travel- 
ling at  5  miles  per  hour.     He  stops  at 
12  for  one  hour  for  lunch.     B  starts  from  Q  at  11  A.M.  to  go 
to  P,  travelling  at  15  miles  per  hour.     An  accident  detains 
him  from  12  to  2.     Where  and  when  will  they  meet  ? 

The  graph  of  A's  journey  is  represented  by  the  upward  line  drawn 
from  P,  and  of  B's  by  the  downward  Hue  drawn  from  C. 

The  position  of  each  at  the  end  of  the  successive  hours  is  marked 
on  the  diagram.    (See  next  page.) 


K 

48 
M 
40 
36 
32 

za 

lA 
ZO 
16 
\l 
6 
4 

0// 

P 

^R 

r» 

/ 

9 

/ 

D 

/ 

1 

k 

/ 

c 

/ 

/ 

1 

/ 

h. 

k 

c 

)  10  II  12  1  2  3  A  5  6  1 

168 


ALGEBRA 


They  meet  at  M  at  about  3.15  p.m.  and  at  a  distance  from  P  of 
about  26  miles. 

How  far  are  they  apart  at  the  end  of  each  hour  from  11  to  5  ?     When 

did  B  reach  P  ? 

We  might  solve  the  problem 
algebraically. 

Suppose  they  are  together  x 
hours  after  9  a.bi. 

Tlien  A  has  travelled  x—l 
hours  at  5  miles  per  hour,  and 
B  x  — 4  hours  at  15  miles  per 
hour, 

.-.     5(a;-l)  +  15(cc-4)  =  60. 

Solve    and   compare  with  the 
■  9     io     11     12      1       2      3      4      5       results  obtained  graphically. 

119.  Graphical  Results  only  Approximate.  The  last  problem 
illustrates  the  fact  that  the  results  obtained  by  graphical 
methods  are  approximate  only.  When  the  problem  is  solved 
algebraically  we  find  that  they  will  meet  26J  miles  from  P 
at  3.15  P.M. 


Q 

o 

V 

P 

A 

s\ 

\ 

\, 

V 

V 

-> 

\ 

r^ 

\ 

-r 

A 

- 

^ 

^ 

V 

^ 

^ 

\ 

- 

> 

^ 

^ 

^ 

\ 

•^ 

^ 

\ 

<^ 

^ 

\s 

BXE3RCISE3  82 

1.  A  man  walks  at  the  rate  of  4  miles  per  hour.  Construct  a 
graph  to  show  the  distance  he  walks  in  any  number  of  hours  from  1  to 
10.  Read  from  the  graph  the  distance  walked  in  3,  5,  1\,  9^  hours. 
(Take  two  units  on  the  horizontal  line  to  represent  1  hour  and  one  unit 
on  the  vertical  line  to  represent  2  miles.) 

2.  In  Ex.  1,  if  he  rests  30  mmutes  after  walking  each  4  miles, 
how  long  will  it  take  him  to  walk  8,  12,  14,  7,  17  miles  ?  How  far 
will  he  have  gone  m  \\,  3|,  5|,  8|  hours  ? 

3.  A  starts  running  at  the  rate  of  6  yards  per  second,  and 
4  seconds  later  B  starts  from  the  same  place  at  9  yards  per  second. 
Construct  a  graph  to  show  when  and  where  B  will  overtake  A. 
How  far  apart  are  they  6  seconds  after  A  started  ?  When  was  A 
12  yards  ahead  oi  B  1 

4.  Oranges  sell  at  19  cents  per  dozen.  Make  a  graph  from  which 
you  can  read  off  the  price  to  the  nearest  cent  of  any  number  from  1 
to  12.  What  is  the  cost  of  2,  5,  7,  8,  10  oranges  ?  How  many  can  I 
buy  for  3,  5,  8,  13,  16  cents  ? 


GEOMETRICAL  REPRESENTATION  OF  NUMBER     169 

5.  If  8  kilometres  equal  5  miles,  construct  a  graph  which  will 
enable  you  to  change  into  miles  any  number  of  kilometres  up  to  20. 
Read  the  approximate  number  of  miles  in  3,  5,  11,  13,  16,  19,  20 
kilometres. 

6.*  A  starts  from  Toronto  at  12  miles  per  hour  to  motor  to  Hamilton, 
a  distance  of  40  miles.  An  hour  and  a  half  later  B  starts  from  Hamilton 
to  drive  to  Toronto  at  8  miles  per  hour.  By  means  of  a  graph,  find 
when  and  where  they  will  meet. 

7.  The  distance  from  yl  to  5  is  10  miles,  5  to  C  8,  C  to  D  8,  Z>  to 
E  10  miles.  A  mail  train,  which  leaves  A  at  10  a.m.,  arrives  at  B  at 
10.24,  C  at  10.48,  D  at  11.12,  E  at  11.40.  An  express  train  leaves  E 
at  10.24  and  without  stopping  reaches  A  at  11.28.  If  the  mail  train 
stops  4  minutes  at  each  station,  show  graphically  : 

(a)  when  and  at  what  point  they  pass  each  other, 
(6)  how  far  they  are  apart  at  10.30  and  at  11.12, 
(c)  when  the  express  passes  through  B. 


120.     The    Axes.      In   the   diagram  the  Hne  OX   is  called 
the  axis  of  x  and  0  Y  the  axis  of  //. 
We  will  call  the  measurement  along 
OX,  X,  and  along  OY,  y. 

For  the  point  A  the  x  measure- 
ment is  3,  and  the  y  measurement 
is  I.  What  are  the  x  and  y 
measurements  for  the  points  B, 
C,  D,  E^ 

Examine  the  x  and  y  measure- 
ments for  each  point  marked  on 
the  line  OP.  What  equation  con- 
nects the  values  oi  x  and  y  for 
each  point  on  the  line  OP  ?  For  each  point  on  OQ,  OR, 
OS^ 

OP  is  the  graph  of  the  equation  y=2x,  OQ  of  y^4:X,  OR 
of  y==x,  and  OS  of  y—\x. 

The  X  and  y  measurements  of  every  point  on  the  line  OP 
satisfy  the  equation  y=^2x.     This  equation  is  not  satisfied 


74^          7               Z^ 

^          t          J- 

fi     §t     p        -.Z 

^      4     T^^  -7^ 

^       2      L           Z 

t     ^                2^.F 

.   ^tJ5     ^Z  ^ 

^   I^^^      2     ^ 

^    i  t     7        s 

17     2             s 

9       tt^^                            =.1 

^-iJZTt       ^^^^ 

,itz     ^^^ 

^yTZ^^^  - 

Jt^                                 X 

Y       1       E      3      4       5      6. 

170 


ALGEBRA 


by  the  x  and  y  of  any  point  not  on  the  line  OP. 
hy  the  x  and  y  of  the  points  A,  C,  D,  E  "^ 


Is  it  satisfied 


121.  Equation  of  a  Line.  Since  the  equation  y—2x  is 
satisfied  by  the  values  of  x  and  y  for  each  point  on  the  line 
OP  and  by  no  other  points,  the  equation  y—2x  is  called 
the  equation  of  the  line  OP. 

What  is  the  equation  of  OQ  ?   oi  OR  \   of  OS  ? 

122.  To  Construct  the  Graph  of  a  given  Equation. 

Ex. — Construct  the  graph  of  y=\x. 


Here  when 


Y 

^ 

■ 

^^ 

.> 

,'' 

'! 

^ 

^ 

^ 

K 

4<- 

u 

— 

LL 

— 

x  =  2,  y=\, 

a;  =  6,  2/ =  3,  etc. 


To  find  the  point  where  x  =  2,  y=l, 
count  2  units  from  O  along  OX  and  then 
1  unit  upwards.  Find  in  a  sinxilar 
manner  the  points  where  a; =4,  y  =  2; 
x=6,  y  =  3  ;  a;=8,  2/  =  4  ;  a;=10,  y  =  5. 
Join  all  the  points  located.  They  are  all  seen  to  lie  on  the  same 
straight  line  passing  through  0. 

This  line  is  the  graph  of  tlie  equation  y  =  ^x. 

123.  The  Origin.  All  the  lines  we  have  so  far  considered 
have  been  drawn  through  the  point  0.  This  point  is  called 
the  origin. 

The  X  and  y  measurements  of  the  origin  are  x=0,  y=0. 
These  values  satisfy  the  equation  ?/=^x,  and  consequently 
the  graph  of  this  equation  should  pass  through  the  origin 
as  the  figure  shows. 


BXE3RCISE   83 

Construct  the  graphs  of  : 
1.    y=x.  2.      y=5x,  3.      y=^x. 

4.     y='^x.  5.     4y=x.  6.     5y—2x. 

124.  Coordinates.  In  the  diagram  0  is  the  origin,  XOX' 
is  the  axis  of  x  and  YO  Y'  is  the  axis  of  y.  The  x  measurement 
of  the  point  A  is  4,  and  the  y  measurement  is  2. 


GEOMETRICAL  REPRESENTATION  OF  NUMBER     171 


These  are  called  the  coordinates  of  the  point  A,  4  being 
the  oc  coordinate  and  2  the  y  coordinate.  The  coordinates  of 
the  point  A  are  written  (4,  2),  the 


,v 

p 

1- 

n 

u 

A 

7. 

K 

Y, 

Q 

"t  7- 

P 

p 

u 

'J 

;v; 

-4,  6). 


a;  coordinate  being  written  first. 

Similarly,  the  coordinates  of  B 
are  (3,  5)  and  of  C  (6,  7). 

So  far  all  our  measurements 
have  been  made  from  0  towards 
the  right  and  then  U2nvards.  We 
might  also  measure  from  0  to  the 
left  and  downwards.  When  we 
do  so  we  indicate  the  change  in 
direction  by  a  change .  in  algebraic 
sign. 

Thus,  to  reach  the  point  D  we  measure  2  units  to  the  left  and  then 
3  units  upwards.      Therefore  the  coordinates  of  D  are  (  —  2,  3). 

Similarly,  the  coordinates  of  £•  are  ( -  6,  5),  of  i^  ( -  3,  -  3),  of  G  (3,  -  5), 
and  of  H  (0,  -4). 

What  are  the  coordinates  of  P,  Q,  R,  S,  O  1 

Mark  on  the  diagram  the  points  whose  coordinates  are  (2,  2),  (■ 
(2,  -5),  (-1,  -3),  (0,  4),  (-3,  0),  (0,  -3). 

Using  squared  paper,  take  the  origin  at  the  intersection  of  two  lines. 
Mark  the  points  which  would  be  indicated  thus  :  (2,  —  3),  (5,  6),  ( —  3,  7), 
(  —  6,  —2),  (3,  0),  (0,  3).  Mark  any  other  four  points  and  show  how 
their  positions  would  be  indicated. 

125.  Quadrants.  That  part  of  the  plane  between  OX  and 
OF  is  called  the  first  quadrant,  between  OY  and  OX'  the 
second  quadrant,  between  OX'  and  OY'  the  thud  and  between 
OY'  and  OX  the  fourth. 

Thus,  the  points  A  and  B  are  in  the  first  cjuadrant,  D  and  E  in  the 
second,  F  and  S  in  the  third  and  P  and  G  in  tlie  fourth. 
In  which  quadrant  are  both  x  and  y  negative  ? 

126.  Plotting  Points.  When  we  represent  the  position  of 
a  point  with  respect  to  the  axes  XOX'  and  YOY',  we  are 
said  to  plot  the  point. 

When  two  i)oints  are  plotted  the  distance  between  them 
may  be  obtained  by  adjusting  the  points  of  the  compasses 


172 


ALGEBRA 


to  the  two  points  and  transferring  the  compasses  to  the  line 
OX,  or  any  other  Hne,  and  reading  off  the  distance. 

Plot  the  points  (3,  5)  and  (6,  1),  and  see  if  the  distance 
between  them  is  5. 


EXBRCISE  84 

1.  In  what  quadrants  are  the  points  (3,  4),  (4,    —1),  (—5,  3), 

(-1,-2)? 

2.  Plot  the  points  (1,  2),  (4,  -6),  (-3,  7),  (-5,  -2). 

3.  Plot    the    points   (5,    0)   and   (—3,0).     What  is  the  distance 
between  them  ? 

4.  Where  are  the  points  (0,  0),  (0,  2),  (-5,  0),  (4,  0)  situated  ? 
5.*  What  is  the  distance  between  the  points  (6,  4),  (1,  —8)  ? 

6.  What  kind  of  figure  is  formed  by  joining  the  points  (0,  0), 
(4,  0),  (4,  4),  (0,  4)  m  order  ?     What  is  its  area  ? 

7.  What  kind  of  a  triangle  is  formed  by  joining  the  pomts  (0,  2), 

(2,  6),  (2,  2)  ?     What  is  its  area  ? 

8.  Plot  the  points  (1,  1),  (1,  3),  (2,  1),  (3,  3),  (3,  1).     Join  them 
in  order.     What  letter  is  formed  ? 

9.  What  is  the  area  of  the  figure  formed  by  joining  (1,   —3). 
(-5,  -3),  (-5,  6),  (1,  6)  in  order  ? 

10.  The  angular  points  of  a  triangle  are  (6,  0),  (3,  4),  (—2,  0), 
Construct  the  triangle  and  find  its  area.  Measure  or  calculate  the 
lengths  of  the  sides. 

11.  What  is  the  length  of  the  perpendicular  from  the  point  (5,  8) 
to  the  line  joining  (3,  2)  and  (7,  2)  ? 

127.     Complete    Graphs.      In   art.    122   we   constructed  the 

graph  of  the  equation  y=hx, 
but  only  for  positive  values 
of  X  and  y. 

The  diagram,  which  is  here 
repeated,  shows  that  the  line 
also  passes  through  the  points 


Y 

/^l 

^ 

^ 

(6, 

3) 

■^ 

^ 

(4, 

2) 

X 

v 

^ 

(2, 

1) 

X 

^ 

^ 

^ 

"(- 

2, 

-1) 

^ 

'f- 

4,- 

2) 

^ 

<- 

6,- 

3) 

-Y 

GEOMETRICAL  REPRESENTATION  OF  NUMBER     173 


(—2,-1),     (-4,-2)  and    (-6,-3).     This    is    as   we   would 
expect  because 

x=  —  2,y=  —  l;    a;=— 4,  ?/=— 2;    x=-—Q,y=  —  3, 

all  satisfy  the  equation  y=^x. 


128.  Linear  Equation.  It  is  seen  that  the  graphs  of  all 
the  equations  so  far  constructed  have  been  straight  lines. 
This  is  true  concerning  all  equations  of  the  first  degree.  For 
this  reason  an  equation  of  the  first  degree  is  sometimes  called 
a  linear  equation. 

Since  a  straight  line  is  fixed  or  determined  when  any  two 
points  on  it  are  fixed,  it  follows  that  to  construct  the  graph 
of  an  equation  of  the  first  degree,  we  need  to  determine  only  two 
'points  on  it. 

129.  Lines  not  passing  through  the  Origin.  Every  equation 
of  the  form  y—mx  represents  a  straight  line  passing  through 
the  origin,  because  the  equation  is  satisfied  by  .t=0,  y=0. 

If  the  equation  contains  a  term  independent  of  x  and  y, 
it  represents  a  straight  line  which  does  not  pass  through  the 
origin. 

Thus,  y  =  2a;+  1  represents  a  straight  line  which  does  not  pass  through 
the  origin,  because  this  equation  is  not  satisfied  by  x  =  0,  y  =  0. 

Ex.  1. — Construct  the  graph  of  y=2x-\-\. 

The  coordinates  of  two  points  on  the  line 
are  x  =  0,  y=\  and  x=  1,  2/ =  3. 

Locate  these  two  points  and  draw  the  un- 
limited straight  line  which  joins  them.  This  is 
the  required  graph. 

The  diagram  shows  that  it  also  passes  through 
the  points  (2,  5),  (-1,  -1),  (-2,  -3),  (-3, 
—  5).  Do  the  coordinates  of  these  points  satisfy 
the  equation  ? 

In  constructing  the  graph  of   an  equa- 
tion  by   locating   two    points   on   it,  the 
pupil  should  try  and  determine  two  points  whose  coordinates 
are  integers. 


Y 

/ 

/ 

/ 

(1. 

3) 

/ 

X 

J 

5" 

,1) 

X 

A 

% 

y 

~ 

' 

-) 

/ 

/ 

/ 

^ 

174  ALGEBRA 

Ex.  2. — Construct  the  graph  of  3x4-4?/=  15. 

TT                 15  — 3a;         ,      ,         /x=l,  w  =  3\ 
Here  y  = ; —    and  when  ^  „  • 

Plot  the  points  (1,  3)  and  (5,  0)  and  join  them  giving  the  required 
graph. 

We  might  have  found  the  points  at  which  the  graph  cuts  the  axes. 
Thus,  when  x=0,  2/  =  3f  and  when  y  =  0,  x  —  5.  The  required  line  is 
then  found  by  joining  the  points  (0,  3|)  and  (5,  0). 

If  the  latter  method  is  followed  and  fractions  appear  in 
the  coordinates  of  either  of  the  points  found,  the  unit  of 
measurement  should  be  changed,  in  this  case,  by  taking  four 
spaces  as  the  unit  instead  of  one. 

When  the  unit  is  not  one  space,  it  should  be  clearly  shown 
on  the  diagram  what  the  selected  unit  is. 


EXERCISE  85 

1.  Find  two  pairs  of  values  of  x  and  y  which  satisfy  a;+?/=6. 
Plot  the  points  whose  coordinates  are  the  values  found  and  construct 
the  graph  of  the  equation  x-\-y^6. 

2.  What  are  the  coordinates  of  the  points  at  which  the  graph  in 
Ex.  1  cuts  the  axis  of  x,  the  axis  of  y  ? 

Construct  the  graphs  of  the  following  equations : 

3.  y=x+3.  4.  y=x—3.  5.  y=2x—3. 
6.  ?/=3,r— 2.  7.  x+2y=7.  8.  x—2y=l. 
9.     2a;+3?/=12.           10.     3a;— 4^=16.           11.  5a;+6y=17. 

12.  Construct  the  graph  which  cuts  off  4  units  from  the  axis  of  x 
and  6  miits  from  the  axis  of  y.  Find  the  area  of  the  triangle  which  this 
line  forms  with  the  axes. 

13.  On  the  same  sheet  construct  the  graphs  of  x—y=lO  and 
x+2y=7.  What  are  the  coordinates  of  the  point  at  which  they 
intersect  ?     Do  the  coordinates  of  this  point  satisfy  both  equations  ? 


GEOMETRICAL  REPRESENTATION   OF  NUMBER    175 


14.  Will  the  point  (3,  4)  lie  on  the  graph  of  the  equation  4x+3y=24  ? 
Which  of  the  following  points  lie  on  it :  (2,  6),  (0,  8),  (6,  0),  (9,  —4), 
(5,  2),  (  —  1,  9)  ?     Verify  by  constructing  the  graph. 

15.  By  constructing  the  graph  of  2a;+3?/=24,  find  three  sets  of 
positive  integral  values  of  x  and  y  which  satisfy  this  equation. 

16.  Why  is  there  an  unlimited  number  of  positive  integral  values 
of  X  and  y  which  wiU  satisfy  2,i-— 3^=24,  but  only  a  hmited  number 
which  will  satisfy  2x-\-3y=24:  ? 

130.  Graphical  Solution  of  Simultaneous  Equations.  In  this 
diagram  are  shown  the  graphs  of  the  equations  x-\-y=^5  and 
2x—3y=l5. 

The  coordinates  of  the  point  P,  at 
which  the  hnes  intersect,  must  satisfy 
both  equations. 

The  coordinates  of  P  are  (6,-1). 

.".  x=Q,  y=  —  l,  must  be  the  values 
of  X  and  y  which  satisfy  both  equa- 
tions. We  have  therefore  obtained 
the  sohition  of  these  two  equations 
graphically. 

Since  it  is  evident  that  two  straight 
lines  can  intersect  at  only  one   point, 
it   must   follow   that    there    is   only  one  pair  of  roots  of  two 
simultaneous  equations  of  the  first  degree. 

In  this  diagram   are  shown  the  graphs  of 

(a)  x-y=3,  (6)  2x-y=l,   (c)   Zx+y=-l. 

At  what  point  do  the  graphs  of  (a) 
and  (6)  intersect  ?  (a)  and  (c)  ?  (6) 
and  (c)  ? 

Is  there  any  point  which  is  common  to 
the  three  hnes  ?  Are  there  any  values 
of  X  and  y  which  will  satisfy  these  three 
equations  at  the  same  time  ? 

When  three  equations  in  x  and  y  are 
all  satisfied  by  the  same  values  of  x  and 
y,  what  peculiarity  will  appear  iri  their 
graphs  ? 


\ 

Y 

~ 

~ 

\ 

\ 

s.i 

\ 

X 

\ 

■^ 

^ 

X 

(•\ 

\ 

Xj 

\ 

y 

R 

k- 

\ 

y 

^ 

\ 

^ 

IOl^ 

.^ 

A 

h 

y 

>> 

^ 

1^ 

\ 

(L 

, 

V                    -i 

^\ 

t 

^ 

^ 

\             V 

X'   - 

ii  -L^  _^  X 

-A           ^L 

-^     zjr 

-  ^  y  le 

-   aZ.  J.\ 

0       ^^       tt 

^ 

V   \7^ 

A/ 

i    it 

^/ 

jZ 

/ 

-     l\' 

.  u  ^ 

176 


ALGEBRA 


131.     Special  Forms  of  Equations. 

(1)  In  this  diagram  are  drawn  the  graphs  ot 
a;+2?/=6  and  x+2y=2. 

The    hnes   which 


}>• 

^^^ 

•^1 

^ 

^ 

"^j;,^^ 

>, 

■^ 

r^ 

^'? 

s 

^ 

^-eu 

> 

^ 

V, 

<s 

X' 

r 

^ 

V 

^ 

y^  1 

s 

) 

"v 

N 

< 

N 

■v 

■V 

_^ 

y 

V 

N 

these  equa- 
tions represent  are  seen  to  be 
•parallel,  that  is,  there  is  no  point 
at  which  they  intersect.  This  is 
equivalent  to  saying  that  these 
equations  have  no  solution. 
Compare  with  art.  Ill,  where  inconsistent  equations  were 
discussed. 

(2)  If  we  draw  the  graphs  of  a;— 3^=10  and  2a:— 6?/=20 
on  the  same  sheet,  we  shall  find  that  they  represent  the  same 
straight  Ime,  so  that  any  points  which  lie  on  the  graph  of  one 
of  them  will  also  lie  on  the  graph  of  the  other. 

These  equations  are  indeterminate  (art.  111). 

(3)  An  equation  like  a:=^3  may  be  written  a;+0?/=3. 

This  equation  is  satisfied  by  a;=3  and  any  value  of  y. 

Thus,  a;=3,  y—\\  ir=3,  y=2  ;  a;=3,  ?/=10,  etc.,  satisfy 
the  equation.  If  we  plot  the  pomts  which  have  these  co- 
ordinates, we  see  that  .t=3  represents  a  straight  Ime  parallel 
to  the  axis  of  y  and  at  a  distance  3  to  the  right  of  it. 

Similarly,  x=  —3  represents  a  line  parallel  to  the  axis  of  y  and  at  a 
distance  3  to  the  left  of  it.  Also  y=  —4  represents  a  line  parallel 
to  the  axis  of  x  and  at  a  distance  4  below  it. 

Thus  an  equation  which  contains  only  one  of  the  variables 
X  and  y  represents  a  line  parallel  to  one  of  the  axes. 
What  Ime  does  ,t==0  represent  ?     y=0  ? 


EXERCISE  86  (1-4,  Oral) 

1.  What  is  the  graph  of  x=^  ?     Of  a;+3=-0  ? 

2.  What  is  the  graph  of  2/-4=0  ?     Of  2«/+3=0  ? 

3.  If  a;-)- 3?/=  11,  express  a;  as  a  function  of  y  and  y  as  a  function 


of  X. 


GEOMETRICAL  REPRESENTATION  OF  NUMBER     177 

4.  If  3x—2i/=6,  express  x  as  a  function  of  y  and  y  as  a  function 
of  X. 

Solve  graphically  and  verify  : 

5.  x+3y=9,  6.       a;+  y=  8,  7.       .r— 2y=  6, 
2a;+  t/=8.                       3x-4)/=10.  2.r-3i/=:ll. 

8,     2a;+3^=  6,  9.       2i/=  a;,  10.       x+4y=9, 

3x+2y=U.  10y=ix—2.  Zx—8y=-—3. 

11.  Show  by  graphs  that  the  equations  x+y=5,  2x--t-3//=12, 
3x—2y=o  have  a  common  pair  of  roots  and  find  them. 

12.  Show  graphically  that  2x-{-3y=lS  and  \x-\-^y=2  are  incon- 
sistent.    What  is  peculiar  about  the  graphs  of  these  equations  ? 

13.  Show  graphically  that  no  values  of  x  and  y  wiU  satisfy  all  of 
the  equations  a;+?/=4,  2x—y=\\,  4.c+?/=13.  What  values  satisfy 
the  first  and  second,  the  first  and  third,  the  second  and  third  ? 

14.  Show  the  coordinates  of  the  points  where  the  graph  of  y—2x+3 
cuts  (1)  the  axis  of  x,  (2)  the  axis  of  y,  (3)  the  graph  of  y=6—x. 

15.  Show  by  graphs  that  the  values  of  x  and  y  which  satisfy 
2x—3y+l—0  and  5x— 2?/— 14=0  will  also  satisfy  3x— 4^=0  and 
x-2y+2=0. 


BXBROISB  87  (Review  of  Chapter  XIII) 

1.  At  what  point  does  the  graph  of  x  +  y  =  5  cut  the  axis  of  a;  ? 
The  axis  of  y  ?  Construct  the  graph.  In  the  same  way  construct 
the  graph  of  x-{-4:y=  —4.     At  what  point  do  they  intersect  ? 

2.  How  does  it  appear  geometrically  that  two  equations  of  the 
first  degree  can  have  only  one  set  of  roots  ? 

3.*  Plot  the  points  (0,  0),  (-3,  4),  (3,  12),  (-2,  0).  What  is  the 
distance  between  each  consecutive  pair  of  these  points  ? 

4.  From  a  certain  point  a  man  walks  5  miles  E.,  then  4N.,  then 
2W.,  then  3N.,  then  3E.,  then  4S.  Using  squared  paper,  determine 
by  measurement  how  far  he  is  now  from  the  starting  point. 

5.  A  man  walks  8  miles  W.  and  then  5S.  Find  by  calculation 
how  far  he  must  now  walk  to  reach  a  point  4  miles  E.  of  his  starting 
point. 

N 


178  ALGEBRA 

6.  If  11  lb.  equal  5  kilogrammes,  make  a  graph  from  which  you 
can  express  any  number  of  kilogrammes  in  lb.  or  lb.  in  kilogrammes. 
Read  from  the  graph  3^  kilogrammes  in  lb.  and  8|  lb.  in 
kilogrammes. 

7.  What  is  the  perimeter  and  area  of  the  triangle  whose  angular 
points  are  (0,  0),  (5,  0),  (0,  12)  ? 

8.  How  do  you  show  that  the  point  (3,  —2)  lies  on  the  graph, 
of  5x  —  2y=l9  ?  Which  of  the  following  lie  on  it:  (6,  5),  (1,  —7). 
(_3,  _17),  (4,  1),  (_2,  -12),  (5,  3)  ? 

9.  Find  the  area  of  the  triangle  formed  by  joining  the  points  (5,  9), 
(8,  -6),  (-7,  -6). 

10.  Draw  the  triangle  whose  vertices  are  (2,  0),  (10,  0),  (5,  6)  and 
find  its  area.  Why  do  the  points  (2,  0),  (10,  0),  (8,  6)  determine  a 
triangle  of  the  same  area  ? 

Solve  graphically  and  verify  : 

11.  x+2y=l2,  12.     3x-4:y=  0,  13.     y-x=4, 
x-3y=   2.                       4:X-3y=-U.  x  =  2. 

14.     y-2x=-Z,  15.     2x+ly  =  52,  16.     2/  =  ix+4. 

x+2y=li.  3x-5y=l6.  y=^x  +  5. 

17.  What  is  the  area  of  the  figure  formed  by  the  lines  whose 
equations  are:    a;  =  4,  x=—2,  y=3,  ?/=  — 1  ? 

18.  What  are  the  coordinates  of  the  middle  point  of  the  line  joining 
the  points  (2,  3)  and  (6,  5)  ? 

19.  On  the  same  sheet  draw  the  graphs  of  the  equations 

y  =  x  +  4:,  y  =  4:X—2,  y  =  2x+2. 

What  peculiarity  is  presented  by  the  graphs  ?  What  conclusion 
do  you  draw  concerning  these  equations  ? 

20.  Draw  the  graphs  of  2x-\-3y  =  20,  4a; +6?/ =  35  on  the  same  sheet. 
What  do  you  conclude  as  to  the  solution  of  these  equations  ? 

Determine  graphically  whether  these  sets  of  equations  are  consistent 
or  inconsistent  : 

21.  X-  y=  4,  22.  x+2y=10,  23.  2x+  y=  8, 
4x+  y  =  26,  Sx-  y=  9,  Sx+2y  =  l3, 
2x-5y=   2.                     2x-  y=    1.  5x-3y=   9. 

24.  Describe  the  triangle  whose  sides  are  represented  by  the 
equations:  3x-\-2y=l4:,  5a;— 6?/=  — 14,  x+10y=  —  14:.  What  are  the 
coordinates  of  its  vertices  ?     (Verify  by  solving  the  equations  in  pairs.) 


GEOMETRICAL  REPRESENTATION  OF  NUMBER     179 

25.  At  what  point  do  the  graphs  of  2x-\-Zy=\2,  ^x—2y=5 
intersect  ?     At  what  angle  do  they  seem  to  intersect  ? 

26.  A  teacher's  salary  is  increased  by  S50  each  year.  His  salary 
for  the  first  year  is  $750.  Construct  a  graph  from  which  you  can 
read  off  his  salary  for  any  year.  What  is  his  salary  for  the  8th  year  ? 
In  what  year  would  his  salary  be  $1300  ? 

27.  In  the  process  of  solving  2a;  — 3^=1,  3x+2?/  =  8,  by  eliminating 
y  we  have 

2x—Zy=\,      I       4a;-6i/=   2,       1  x=2,      |  x  =  2, 

3a;+2?/  =  8,      |       9a;  +  6i/  =  24.        |      2x~Zy=\.       \  y=\. 

On  the  same  sheet  show  the  graphs  of  each  of  these  sets  of  equations, 
and  thus  show  that  they  all  determine  the  same  point  and  that  the 
four  sets  are  therefore  equivalent. 


N  2 


CHAPTER  XIV 

HIGHEST    COMMON    FACTOR    AND    LOWEST    COMMON 
MULTIPLE 

132.  In  Chapter  IX.  we  defined  the  terms  highest  common 
factor  and  lowest  common  multiple,  and  showed  how  they 
were  found  in  simple  cases. 

When  the  expressions  under  consideration  can  be  factored, 
the  H.C.F.  and  L.C.M.  can  at  once  be  written  down  from 
the  factored  results. 

A  few  examples  are  here  given  of  a  more  difficult  character 
than  those  previously  considered. 

Ex.  1.— Find  the  H.C.F.  and  L.C.M.  of 

x^y-\-lxy^-\-12y^  and  x^y—x^y^—l2xy^. 

x^y  +  7xy^+l2y^  =y{x^  +  lxy-\-12ij^)=y{x+4y){x+3y). 
x^y—x^y*—l2xy^=xy(x'  —  xy—12y^)=xy{x—4:y){x+3y). 

Here  the  common  factors  are  y  and  x+3y,  and  since  the  H.C.F 
is  the  product  of  all  the  common  factors, 

.-.    the  H.C.F.  =2/(a;+32/). 

The  L.CM.  is  the  expression  with  the  lowest  number  of  factors  which 
will  include  all  the  factors  of  each  expression, 

.-.    the  'L.C.M.  =xy{x  +  4y){x  +  3y){x  —  'iy) 

Ex.  2.— Find  the  L.C.M.  of 

x^—1,  x^-\-l,  x'^—x  and  rr^+x^+l. 
a;2-l  =  (a;+l)(a;-l). 
x^-\-\  =  (x-\-\){x^-x-\-\). 
x*—x—x{x^—  \)=x{x—  \){x^-\-x-\- 1). 
a;<+a;2+l  =  (a;2+l)2-a;2  =  (a;2+a;+l)(a;2-x+l), 
,*.     the  L.C.M.  =  .r(a;+  l)(a;-  \){x^+x^  \){x^-x-\-\). 

180 


H.C.F.   AND  L.C.M.  •  181 

If  the  multiplications  be  performed  the  L.C.M.  will  bo  found 
to  be  x'  —  x.  It  is  customary,  however,  to  leave  tlio  result  in  tlie 
factored  form,  as  it  is  in  this  form  that  it  is  usually  made  use  of. 


EXERCISE  88 
Find  the  H.C.F.  and  L.C.M.  of  • 
1.*  4.x^yH,  %xyh^,  \2axyH. 

2.  x^ — y^,  xy—y^,  x'^—xy. 

3.  a^-h\  ab+b\  a^+2ab+b^. 

4.  a;2-7a;+12,  .t2+2x-15,  x^-9. 

5.  a'+8a+15,  a--2a-35,  a^+3a-lO. 

6.  3z2-12a;+12,  3.r2-I2,  3x^-Bx-6. 

7.  x^—xy-\-xz—yz,  xy—y^. 

8.  m^— 8,  m*H.2— 4to^w^  4)?i^— l&»i -|-16. 

9.  6a3_663,  2a3+2a26+2a62. 

10.  a--\-ab—ac,  a^-{-b-~c^-\-2ab. 

11.  a^-b^-c^-2bc,  b^-c^-a^-2ca,  c^-a^-b^-2ab. 

12.  x^+y^,  x*+x^y^+y*. 

13.  3a;2+7a;-6,  3x^-Ux+6,  6a;2_  13:^+6. 

14.  10ax—2a+15cx—3c,  2oa;2— 1,  25a;2-10a;+l, 

15.  x^-5x^-^6x,  x^-3x^+5x-l5. 

16.  M*  —  U*,  M"'  —  V^,  U^  —  V^,  U  —  V. 

17.  a;3+2a;*-8.r— 16,  a'3+3a;2-8.r— 24. 

18.  Show  that  the  product  of  x-— 8.r+15  and  x^+a;— 12  is  equal 
to  the  product  of  their  H.C.F.  and  L.C.M. 

19.  The  L.C.M.  of  a^— 5a+6  and  a- —  6  is  a^—^a^—^a+\2 

Supply  the  missing  term. 

20.  Find  two  trinomials  whose  H.C.F.  is  x—2y  and  whose  L.C.M. 
is  x^—lxy^-\-%y^. 


182  ■  ALGEBRA 

Ex.  1.— Find  the  H.C.F.  and  L.C.M.  of 

x^-\-2x—3  and  a;^— 8a:+3. 

Here  a;2-f2x  — 3  is  readily  factored,  but  none  of  the  methods 
previously  given  will  apply  in  factoring  x'— 8x4-3,  except  by  using 
the  factor  theorem  of  art.   101. 

The  difficulty  is,  however,  easily  overcome  thus  : 

z^  +  2x-3  =  {x-l){x+3). 
If  the  expressions  have  a  common  factor  it  must  evidently  be  either 
x—l  or  x-{-3. 

By  using  the  factor  theorem,  find  if  x—l  or  x+3  is  a  factor  of 
x^-8x+3. 

When  .^-1  =  0  or  x=l,  .'c3_8a;+3=  1-8  +  3=  — 4, 

.'.     x—l  is  not  a  factor. 
When  x+3  =  0  or  a;=-3,  x3-8a;4-3= -27  +  24  +  3^0, 

x+3  is  a  factor. 
How  can  we  obtain  the  other  factor  of  x'  — 8x+3  ? 
We  now  have  x2  +  2x-3  =  (x— l)(x  +  3), 

and  x3-8x+3  =  (x+3)(x2-3x+l). 

.-.    theH.C.F.=x+3, 
and  the  L.C.M.  =  (x+3)(x- l)(x2-3x+l ). 

Ex.  2.— Find  the  H.C.F.  and  L.C.M.  of 

x^—lx-{-lO  and  a;^— 6a;2+lla:— 6. 

The  factors  of  x-  — 7x+10  are  (x  — 5)(x  — 2). 

Here  it  is  evident  that  x  —  5  is  not  a  factor  of  the  second  expression, 
since  its  last  term  is  —  6,  which  is  not  divisible  by  5. 
Is  X— 2  a  factor  of  x^  — 6x2+  llx— 6  ? 
Complete  the  solution. 


SXERCISEI  89 

Find  the  H.C.F.  and  L.C.M.  of  : 
1.*  x2-3x+2,  x3— 6.r2+8x-3. 

2.  a2-6a+5,  a^-19a^-\-na+l. 

3.  x3-2x2+4x-8,  2x3-7x2+12. 

5.     .t3+3x2-4x,  x3-7x+6. 


H.C.F.   AND  L.C.M.  -  183 


6.  If  x—2  is  a  common  factor  of 

a;H3a;--ar-2  and  .x-3-4.r2+3.r+2, 
find  their  L.C.M. 

7.  Reduce  to  lowest  terms : 


a3-19a62+3063  2x^-Ux'^—\2x 

8.     Find  two  expressions  of  the  third  degree  in  x,  whose  H.C.F. 
is  a;2— 5a;+6  and  whose  L.C.M.  is  a;*— 10,r3+3o.c--50x-+24. 

133.  Method  of  finding  the  H.C.F.  of  two  expressions  which 
can  not  be  factored  by  the  usual  methods.  From  the  preceding 
it  is  seen  that  the  chief  difficulty  in  finding  the  H.C.F.  of 
two  expressions  is  in  factoring  the  given  exj)ressions. 

If  neither  of  the  expressions  can  be  factored  by  the  usual 
methods,  another  method  may  be  used  which  depends 
upon  the  same  principle  as  that  of  finding  the  G.C.M.  of  two 
numbers  in  arithmetic. 

134.  Fundamental  Theorem.  This  method  of  finding  H.C.F. 
depends  upon  the  following  theorem  : 

If  X  is  a  common  factor  of  any  two  quantities,  then  ■dc,  is  also  a 
factor  of  the  sum  or  difference  of  any  multiples  of  those  quantities. 

Thus,  X  is  a  common  factor  of  mx  and  nx. 

Then  mx-[-nx,  mx—nx,  pmx-\-qnx,  rmx—snx,  are  each  the 
sum  or  difference  of  multiples  of  mx  and  nx. 

It  is  evident  that  each  of  these  is  divisible  by  x,  the 
quotient  in  each  case  being  found  by  division,  thus  : 

x\mx-\-nx  x\mx—nx  x\pmx-\-qnx  x\r^3:  —  snx 

m  +n  m  —n   '  pm  -\-qn  rtn  —sn 

The  way  in  which  the  theorem  is  applied  is  shoAvn  in  the 
following  examples. 

Ex.  1.— Find  the  H.C.F.  of 

.r3+4x2+4.r+3  and  x-^-f  3x2+4a;+12. 
Any  common  factor  of  these  is  a  factor  of  their  difference,   which 
is  x^  —  9. 

But  a;2-9  =  (a;-3)(a;+3), 

.-.     the  H.C.F.  is  a;- 3  or  03+3  or  (a;-3)(a;+3). 


184  ALGEBRA 

It  is  evident  that  x— 3  is  not  a  factor  of  either  expression,  since  their 
terms  are  all  positive.  Therefore  if  they  have  a  common  factor  it 
must  be  x-\-3. 

By  applying  the  factor  theorem,  or  by  division,  we  find  that  a;+3 
is  a  factor  of  each,  and  since  it  is  the  only  common  factor,  it  must  be 
the  H.C.F. 

Ex.  2.— Find  the  H.C.F.  of 

3x^-nx^~5x-\-10  and  3:^3-23^2+ 23a:— 6.      • 

Their  difference  =  6a;2  -  28a;+  16  =  2(a;-4)(3x-  2). 

Now  2  is  not  a  factor  of  either  and  may  be  discarded,  also  x  — 4  is 
not  a  factor,  since  4  is  not  a  factor  of  10  nor  of  6.  Therefore  if  there  is 
a  common  factor  it  must  be  3a;— 2. 

Divide  3a;— 2  into  one  of  them  and  see  if  it  divides  evenly.  If  it 
does  not  there  is  no  common  factor  but  unity. 

If  it  does  divide  evenly  into  one  of  them,  it  is  not  necessary  to  divide 
it  into  the  other,  for  if  it  is  a  factor  of  one  of  them  and  also  of  their 
difference  it  must  be  a  factor  of  the  other. 

Ex.  3.— Find  the  H.C.F.  of 

3a;3-13a;2+23a;-21  and  6x^-\-x^-44:X-{-21. 

Multiply  the  first  by  2  and  subtract  the  product  from  the  second 
and  we  get 

21x^~90x  +  G3  =  9{x-l)(Sx-l). 

Now  since  9(a;— l)(3a;— 7)  is  the  difference  of  two  multiples  of  the 
given  expressions,  it  must  contain  all  their  common  factors.  Which 
of  these  factors  may  be  discarded  ?     Complete  the  solution. 

We  nfiight  have  obtained  the  H.C.F.  thus  : 

The  sum  of  the  expressions  is 

9x^-12x'^-2lx=3xix+l){3x-l). 
This   expression   contains   all   the   common    factors    of    the    given 
expressions. 

Complete  the  solution  by  this  method. 

The  object  in  each  case  is  to  obtain  from  the  given 
expressions  an  expression  of  the  second  degree.  If  this 
expression  can  not  be  factored,  it  must  be  the  H.C.F.,  if 
there  is  any  common  factor  other  than  unity.  If  it  can  be 
factored  the  H.C.F.  can  then  be  found  either  by  the  factor 
theorem  or  by  ordinary  division. 


H.C.F.   AND  L.C.M.  185 

In  obtaining  the  expression  of  the  second  degree,  the  last 
problem  shows  that  it  is  sometimes  easier  to  ehminate  the 
last  terms  than  the  first  terms. 

Ex.  4.— Find  the  H.C.F.  and  L.C.M.  of 

6a:3_5a;2_8x+3  and  4a;3— 8a;2+a;+3. 

Eliminate  the  absolute  terms  and  show  that  2x  —  3  is  the  H.C.F. 
Since  2a;— 3  is  a  factor  of  each,  the  other  factors  may  be  found  by 
division,  then 

6a;3  -  5x2  -  8a;  +  3  =  ( 2a:  -  3)(  3a;2  +  2a;  - 1 ), 

4:x^-8x^+x+3  =  {2x-3)(2x^-x-l), 
:.     the  L.C.M.  =  (2a;-3)(3a;2  +  2a;-l)(2a;2-a;-I). 
Why  is  it  unnecessary  to  factoi-  Zx^-\-2x—  1  and  2a;^  — a;— 1  ? 

Ex.  5.— Find  the  H.C.F.  of 

a-4—4r3-l- 10x2-1  la;+ 10,  (1) 

and  x*-x^-^x^\-\^x-\5.  (2) 

Subtract  (1)  from  (2),  and  we  get 

3a;»-14a;2  +  30a;-2o.  (3) 

Multiply  (1)  by  3  and  (2)  by  2  and  add  to  eliminate  the  absolute 
terms.     Remove  the  factor  x  and  we  obtain 

5a:3-14a;2  +  22a;+5.  (4) 

The  common  factor  we  are  seeking  must  be  a  factor  of  both  (3) 
and  (4). 

Eliminate  the  absolute  terms  from  (3)  and  (4)  and  show  that  the 
H.C.F.  is  a;2-3a;  +  5. 

Find  also  the  L.C.M. 

Ex.  6.— Find  the  H.C.F.  of 

8:r4-f  4a;34-4a;2— 4x  and  Qx'^+^x^+^x^—^x. 

Here  4a;  is  a  factor  of  the  first  expression  and  2a;  of  the  second,  and 
therefore  2a;  is  a  common  factor.  Remove  these  simple  factors  and 
find    the    H.C.F.    of    the    quotients,    and    show    that    the    H.C.F.    is 

2a;(a;2  +  a;+l). 

135.     Product  of  the  H.C.F.  and  L.C.M. 

Suppose  that  x  is  the  H.C.F.  of  mx  and  nx,  so  that  m  and 
n  have  no  common  factor. 


186  ALGEBRA 

Then  the  L.C.M.  of  mx  and  nx  is  mnx. 

But  XX  mnx = mx  x  nx, 

therefore  the  'product  of  any  two  quantities  is  equal  to  the 
product  of  their  H.C.F.  and  L.C.M. 

Is  a  similar  theorem  true  concerning  any  three  quantities 
mx,  nx  and  px  ? 

If  the  H.C.F.  of  two  quantities  has  been  found,  we  might 
therefore  find  their  L.C.M.  by  dividing  their  product  by  the 
H.C.F. 

BXERCISB  90 
Find  the  H.C.F.  of  : 
1.*  x^—lx^^lZx-l^,  a;3— 6a:2+a;+20. 

2.  a3-10a2+33a-36,  a3-2a2-23a+60. 

3.  6a;3+ 10x2+8x4-4,  6x3-2x2-4. 

4.  2x3— 5x2— 20x+9,  2a;3+x2— 43x— 9. 

5.  263+562-86-15,  463-4&2_96+5. 

6.  3x3+ 17x2?/ -44x1/2 -282/3,  Qx^-^x^y-ZZxy^-\-2%y\ 

7.  2a3-3a2-4a+4,  3a*-4a3-i0a+4. 

8.  2x*-12x3+19x2-6x+9,  4x3- 18x2+ 19x-3. 

9.  18a^6-3a'»6-12a36-3a26,  12a5c-6a*c-9a3c+3a2c. 

10.  x3-x2-2x+2,  x*-3x3+2x2+x-l. 

Find  the  L.C.M.  of  : 

11.  x3— 7x— 6,  x3— 4x2+4x— 3. 

12.  x3+6x2+llx+6,  x3+7x2+14x+8,  x3+8x2+19a;+12. 

13.  2x3+9x2+7x-3,  3x3+5x2- 15x+4. 

14.  x3-6x2+llx-6,  x3-7x2+14x-8. 

15.  20x*+x2— 1,  25x«— 10x2+1,  25x*+5.r3— x— 1. 

16.  Find  a  value  of  x  which  will  make  x3  — 13x+12  and 
x3— 6x2— x+30  gach  equal  to  0. 

17.  The  L.C.M.  of  two  numbers  is  70  and  the  H.C.F.  is  7.  If  one 
of  the  numbers  is  14,  find  the  other. 


H.C.F.   AND  L.C.M.  187 

18.  The  H.C.F.  of  two  expressions  is  x—2,  the  L.C.M.  is  x-^— 39a; +70. 
If  one  of  the  expressions  is  a;^— 7a;+10,  find  the  other. 

19.  Two  integers  differ  by  11.     If  they  have  a  common  factor, 
other  than  unity,  what  must  it  be  ? 


EXERCISE  91  (Review  of  Chapter  XIV) 

Find  the  H.C.F.  and  L.C.M.  of 

1.*  a;2-20a;+99,  a;2-24x+143,  a;2-2Lr+110. 

2.  x^-l5x+36,  x^-21,x^-3x^-2x+6. 

3.  a^-b^,a^  —  2ab  +  b^,a^-b^. 

4.  x^  —  2x'^-l5x,x^  +  x^—l4:X—24:. 

5.  4a3-12a2-a  +  3,  2a3-|-a2-18a-9. 

6.  x^—ax—bx+ab,  x^  —  bx—cx+bc. 

7.  x^  —  6x^+Ux—6,x^  +  4x^+x-6. 

8.  a;*+3a;3  +  3x2  +  5a;-12,  a;*-4x3- 19x2+ 10a;+ 12. 

9.  2a4+15a3  +  39a2-(-40a+12,  2a«+9a3_2a2-39a- 18. 

10.  x*—6x^y+l3x^y^-12xy^  +  4y*,x*+2x^y-3xhj^-4:xy^  +  iy*. 

11.  x*  +  x'y'^+y\  x*—2x^y  +  3x'y'^~2xy^+y\ 

12.  Show    that    two    consecutive    integers    can    have    no    integral 
common  factor  except  unity. 

13.  Two  odd  integers  which   differ   by  6  have  a  common  factor 
other  than  unity.     What  must  it  be  ? 

14.  Find  the  H.C.F.  of  a;«  +  a«  and  x«  +  x*a*  +  a^. 

rth 

15.  If  the  H.C.F.  of  a  and  b  is  d,  show  that  the  L.C.M.  is  -7  • 

a 

16.  If  a  is  the  H.C.F.  and  b  is  the  L.C.M.  of  three  quantities,  show 
that  the  product  of  the  quantities  is  a^b. 

17.  For  what  common  values  of  x  will 

x^-3x^-x  +  3  and  a;^-4a;3  +  12a;-9 
both  vanish  ? 

18.  Find  two  expressions  of  the  second  degree  in  x,  whose  H.C.F. 
is  x—l  and  L.C.M.  is  a;'-8x2+17a;- 10. 

.„      „    ,  18x'-3a;*+2aj+8  .     , 

19.  Reduce  X2^3^8a?'-7x+12  *°  ^°^^^^  *®"^- 


CHAPTER  XV 
FRACTIONS 

In  Chapter  IX.  fractions  were  introduced  and  simple 
examples  of  operations  upon  fractions. 

In  this  Chapter  the  subject  is  extended  and  applications 
made  to  more  complicated  forms. 

136.  Changes  in  the  Form  of  a  Fraction.  Both  terms  of  a 
fraction  may  be  multiplied  or  divided  by  the  same  quantity 
without  altering  the  value  of  the  fraction.  As  previously 
stated,  the  only  exception  to  this  rule  is,  that  the  quantity 
by  which  we  multiply  or  divide  must  not  be  zero. 

The  rule  might  be  stated  in  the  symbolic  form : 
a  _  ma        na  _  a 
b  ~  inh        nb  ~  b 

The  case  in  which  the  terms  are  multiplied  or  divided 
by  ^  1  deserves  special  attention. 


From  the  rule  of 

sig: 

ns  for  division 

a 
-b 

is 

seen 

to  be 

the 

a 
same  as  —  7- , 
0 

so  also 

is' 

—a 
b 

—a 
b    ~ 

a 

-b^~ 

a 
b' 

Similarly, 

—a 
-b 

a 

-a 
b~  ~ 

=  — 

a 

It  is  thus  seen,  that  the  value  of  a  fraction  is  not  changed 
by  changing  the  signs  of  both  of  its  terms ;  or  by  changing  the 
sign  of  one  of  its  terms  and  at  the  same  time  changing  the  sign 
before  the  fraction. 


FBACTI0N8  189 

Since  (a— 6)x(  — 1)  =  — a+6  or  h—a,  it  is  seen  that  a—h 
and  h—a  differ  only  in  sign,  or  that  each  one  is  equal  to  the 
other  multiplied  by  — 1. 

That  is,  a—h=  —  {h—a)  and  h—a=  —  {u—h). 

rp,  a  —  b  _  (a—b)  x(— 1)  _  b  —  a  __      a  —  b  _      b  —  a 

c—d~{c—d)x{  —  i.)~d  —  c~      d—c~      c—d 
Also,  since  (  — a)  x  (  — 6)  =  (  +  a)  x  (  +  6)=a6, 

it  follows  that  {a  —  b){c  —  d)  =  {b  —  a)(d  —  c), 

m  _    m  _  —m  m 

(a-b){c~d)  ~  (b-aj(d^)  ~  (^'bj(d-c)  ^  ~{b-a)(^^f 

(^"Z^K^y]  =  (a^-q)(y-fc)  ^  {a-x)(y-b)  ^  ^^^ 
(b-xj{a-y)       {x-b){y-a)       (b-x)(y-a) 

EXERCISE]  92  (1-29,  Oral) 

Express  these  fractions  in  their  simplest  forms  with  no  negative 
signs  in  either  term  : 


-2 
^-    4    • 

4 
-  -2                    ^• 

-6 

-9' 

4. 

—  3a 
—a 

— a 
5. 

y 

5 
6. 7. 

—m 

-iab 
26 

8. 

—ax 
-bx 

-3x-5 
»•          7         • 

—axb 
10.      _^     .          11. 

—a.  —b 
—c 

12. 

—X.  —y 
—a.  —b 

Express  with  the  numerator  a—b , 

» 

b—a 
13.       3  . 

b—a 
14.   -  — r  •          15. 

—  0 

b—a 

x-y 

16. 

b-a 
—3{c—d)  ' 

Express  with  the  denominator  c—d: 

-5 

d—c 

—a;.  —V 

18.        , '  ■        19. 

rf— c 

a—b 
d—c 

20. 

—m{x—y) 
d-c^ 

Express  with  the  positive  sign  before  the  fraction 

-4 
21.-    , 

4 
22.   -  —  •            23. 

a 
~b' 

24. 

a—b 
c 

X 
25.    -  ;• 

a—b 

26.  -^-,-          27. 
o— 0 

x-2 
x—y 

28. 

c—d 

c+d 

29.     What  is  the  relation  between 

and 

x—y 

a              a+b       , 6+ 
»               V  and  , 
y—x           a—b          b— 

a          b—a 
a'          -3 

and  - 

7'' 

190  ALGEBRA 

(p — q)(q — f) 

30.  Write  -. r^ — -^  ,  in  four  equivalent  forms,  with  the  positive 

(x-y){y-z) 

sign  before  the  fraction. 

31.  Which  of  the  following  are  equal  in  value  : 

(a— 6)(6— c)(c— a),  {b—a){c—b){a—c),  {a—b){c—b){a^c), 
{u-b){b-c){a-c),  {b-a){b-c){a-c)  ? 

137.     Reduction  of  Fractions  to  Lowest  Terms.     The  formula 

,  -  =  7  may  be  used  to  reduce  a  fraction  to  its  lowest  terms, 
ox       o 

by  dividing  both  terms  by  all  the  common  factors. 

Ex.  1.— Reduce  -  ,  ,     , -l  ,     . 

x^-\-y^  =  {x-\-y){x^—ocy-\-y^). 
x'^  +  x'^y^+y^  =  {x^-\-y^)^  —  x^y^={x^+xy-\-y'^){x'^—xy-\-y^), 

x+y 


Ex.  2. — Reduce 


the  fraction  -     „  ,        ,     „ 

(»~+xy+y^ 


a2  +  62-c2  +  2a6  =  {a  +  6)2-c2  =  (a+6+c)(a+6-c). 
Complete  the  reduction. 

T.      o      T,    T  "a;2-lLr+28 

Ex.  3.— Reduce  ^r-^ — „  „  ,  _ rn, 

2x^—Qx^-{-Tx—Q0 

x^-nx+2S  =  {x-4)(x-  7). 

Which    of    these    factors    can   not   divide    into    the    denominator  ? 
Complete  the  reduction. 

Ex.  4.-Reduce     e.3+3,._5,+  i 

Here  the  factors  of  neither  term  can  be  readily  obtained,  so  the 
common  factor  must  be  found  by  the  method  of  art.  134. 
Eliminate  the  x^  and  we  obtain 

33x2  +  33a;-ll  or  ll(3x2  +  3x-  1). 

This  expression  must  contain  any  common  factor  of  both  terms. 
Since    3a;-  |-3a;~l    can    not   be   factored,   what    conclusion  can   be 
drawn  ?     Complete  the  reduction. 


FRACTIONS 


191 


EXERCISE  93 


Reduce  to  lowest  terms  : 
a-4-3a+2 

^'    4^2+ 8a; +3  ' 

a^-\-a'^b+ab^+b^ 

«3+2a2+2ar+l  " 

a2-4a+3 
4a3_9a2_l5a+i8' 

2x3-x-2+2a;-3 


7  * 


6. 


8. 


11. 


13. 


2.^3+3x2+4x4-3 

_  «^+a2-3a-3 

a5_a4_2a3+2a2_3(j_5 ' 


10. 


12. 


14. 


a;2+7xy— 8y2 
a;2+ 5x2/— 24^2 

x2+2xy+y2-za 

2-3.v-2y2 
4— 5?/— 6?/2 

x^— x2— 2x 
x3— 3x2+4  " 

3x2-3x-18 
6x5-12x4-18x3" 

3x3+4x2_6x-8 


36x3+27x2-40x-16 

2x«— 4x3— 2x2— 12x 
4x4+2x3+6x2— 4x  ' 


138.  Addition  and  Subtraction.  In  adding  or  subtracting 
fractions  we  should  be  careful  to  note  whether  any  of  the 
given  fractions  can  be  reduced  to  lower  terms.  When  the 
result  is  obtained  we  should  examine  it  to  see  if  it  can  be 
reduced. 

2y_  _  xy^-\-y^ 


Ex.  1.— Simplify  - J^  + 

^     -^       X         x—y 


The  expression 


x'^—xy 
yH^+y) 


x-y  ,  _2y ^ 

X         x—y      x(x—y){x+y)' 


x—y 


+ 


'■iy 


y 


x—y      x{x     y) 

=  (^^^0H-_2xy-^  _ 
X(x-y) 

_  X2  _        X 

x{x—y)       x—y 

The  form  of  the  last  fraction  in  the  given  expression  shovild  prompt 
the  pupil  to  examine  whether  it  can  be  reduced. 


192  ALGEBRA 

Ex.  2.— SimpUfy  —^  +  ^ 


x—2  '   a;2-3a;+2      a;^— 4x+3 
The  expression 


x~2   '   (x-l){x-2)       (a;-3)(x-l) 

{x-l){x-  S)+x-  3-  2(a:-2) 
(a;-2)(a;-r)(x-3) 

a;*— 5a;+4         _        (x— 4)(a;— 1)  a;— 4 


(a;-2){a;-l)(a;-3)       (a;-2)(a;-l)(a;-3)       (x-2)(x-3) 


Simplify ; 


EXERCISE  94 


l.*_^__L_L.  2.     -i L..        3.     a;+y      a;— y 


4. 


a-4      a-7  ^         2a^  2a  „         a^ 


a 


a-2      a-5  a^-b^      a+b  a-a^      l+a^ 


2x2  2a;2  a^— .V     ,1         <.       x^— 4?/^      x—2y 

x^—y^      x^-\-xy          '     x^—y^      2x—y       '     x^-}'2xy  x 

10  1 1  11      J^ 3?/        x^+y" 

*  a;2_[.9a;_^20      x2+12a;+35  '     x+?/      x—yx^—y^ 

a;2— 3x— 10_a;2+2.T— 3  a:— y        2.r        x^+x^y 

a;^— 8x+15      x^— 3x+2  '        y         x—y      x^y—y^ 

a^—ab-^-b"^      a^-\-ab-\-b^  x-\-  •         4xy    _y—x 

a—b                a-\-b  '     x—y      x'^—y^      x-\-y 

a-b          a+b    _  a^+b^  ^^      3x^-8  _     5x+7         _2_ 

*  2{a+b)      2{a—b)      a^—b^'  '     x^-l       a;2+x+l      a;— l' 

18  1       1    2a+96  1  ^g      Jl 2  1 

■     2a-36     4a2-962^2a+36  '    x-1      x-2"^x-3 

20.     — i_+           3  * 


21. 


x2-3x+2  '  x2-7x+10      x2-6x+5 

(a+6)(6+c)(c+a)  _  a-\-b  _  6+c  _  c-\-a 
abc  cab 


22. 
23. 


FRACTIONS  193 

a2-62+26c-c2  _  c2+2ca+a2_62 
62_c2+2ac'_:o-      62:p26c+c2— a2 ' 

(a+c)2-62      (6+a)2_c2  +  (^qi^pITaa 


g.        o^— 2ffl  3a  5a 


a2-a-2      6a-4  ^  6a2+2a— 4 


25.     -i— 1-and     ^  ^  ^^ 

26. 


a;-?/      a;+?/  a;-?/      x+y      x^-^-y^ 

1  1  22/  4?/3 


27. 


a;—?/      a;+«/      a;2+?/2      x*-i-y* 

_l 1_  _     2x 

3—x      3+a:~9+a;2' 

28  -^  4-  -^  ^    2a2  4a262 

29  ^ ^        1 ^  3^ 

4— 4.r      4+4a;      24-2a;2~iT^* 

30.     Solve   -^- 1 1^ ^^  _1      /Verifv-k 

139.     Special  Types   in   Addition  and    Subtraction. 

We  have  already  seen  that 

b—a=  —  {a—b)  and  a—b=  —  {b—a), 
or  (a— 6)-f(6— a)=  — 1. 
When  a—b  and  b—a  occur  in  the  factored  denominators 
of  different  fractions,  which  are  to  be  combined,  it  is  not 
necessary  to  inchide  both  of  them  in  the  L.C.D. 

Ex.  1.— Simplify  -^  +  -^  . 
^     -^   a—h      b—a 

Here   only   one   factor  is  required  in  the  L.C.D.   and   we  may   use 
either  a  —  b  or  b  —  a. 

If  we  decide  to  use  a-b,  then  it  is  better  to  change  the  second 

fraction  into  the  form . 

a  —  b 

Then  -^  +  ^  =  ^ ^  =  ?Z1^=1 

a  —  b       b  —  a       a  —  b       a—b       a  —  b 


194  ALGEBRA 

0,^4  3      ,    x—S 

Ex.  2.— Simplify  r +  :j .  • 

^         x—l      x-\-l       l—x^ 

The  denominator  of  the  last  fraction  should  be  changed  to  x^  —  1, 
so  as  to  be  the  product  of  a;—  1  and  a;+  1. 

4  3  x-3 


The  expression 


x—l      x+1      x^—V 
4(a;+l)-3(a;-l)-(a;-3)  10 


(x-l){x+l)  (a;-l)(a;+l) 

Ex.  3. — Simplify 

1,2  3 


{x-l){x-2)   '   {x-2){S-x)      {x-3){i-x) 

Here  there  are  only  three  factors  x—\,  x—2,  x—3,  required  in  the 
L.C.D. 

We  therefore  change  the  second  and  third  fractions  so  that  the  given 
expression 

+ 


{x-l){x-2)       {x-2)(x-3)   '    (a;-3)(a;-l) 
Complete  the  simplincation. 

140.     Cyclic  Order,     Suppose  we  wish  to  simplify 

b-\-c  c-\-a  a-\-b 

{a—h){a-c)  "^  {b-c){b-a)  "^  {c—a){c-b) ' 

The  L.C.D.  in  this  case  will  contain  three  factors  and  it 
might  be  written  in  different  forms  as 

(a— 6)(6— c)(c— a),  {a—b){a—c){b—c),  etc. 

The    pupil    is    advised    to   write  the   factors    in   what    is 
called  cyclic  order. 

If  we  arrange  the  letters  on  the  circumfer- 
ence of  a  circle,  as  in  the  diagram,  and  follow 
the  direction  of  the  arrows  we  see  that  a  is 
followed  hy  b,b  by  c,  and  c  by  a. 


Thus,   if  we  write   a  —  b    as   the   first    factor,  then 
changing  ato  b,  b  to  c,  and  c  to  a,  we  write  the  second 
factor  b  —  c  and  the  third  c  —  a. 
If  we  write  the  L.C.D.  as  (a— 6)(6  — o)(c  — a),  we  should  change  the 
fractions  so  that  these  factors  appear  in  the  denominators. 


FRACTIONS  195 


The  given  expression  then 

b+c  c+a  a-{-b 


ia-b){c-a)       (6-c)(o-6)       {c-a){b-c) 
-{b+c){b-c)-{c+a)(0-a)-{a+b)(a-b) 
{a-b){b-c){c-a) 


(a  —  b)(b  —  c)(c  —  a) 


=  0. 


„        CI-      IT             ^c                       ca  ^  ab 

Ex. — Simplify j-- +  -pr r-r :  + 


{a—h){a—c)      {b—c){b—a)       {c—a){c—b) 

Proceed  as  in  the  preceding  example  and  you  should  get  the  restilt 
—  b^c-'rbc^~c^a-\-ca^  —  a^b-\-ab^ 
(a — b)(b — c)(c — a) 
This  fraction  is  equal  to  unity,  for  the  numerator  is  equal  to  the 
denominator.     Prove  that  this  is  true. 


EXERCISE  95 


Simplify : 

1  1 


1  * 


a^+ax      x^^ax  2a— Zb      36— 2a 


„      x+3      x—S  .  ^        .       1 


x—2      2—x  x^—9y^      3?/— a; 

5  2  3  g      x+a      x'^-aP- 

a^—ab      62_^"  '    x—a      a^—ax' 

y      _2 3 x2-3  ^       _5 4 16 


x-l      x+1      l-a;2'  '    x-2      2+x^  4:-x^' 

3  3  24 


9      _^  -  J^  4-      y^  10 

x—y     x+y      y^—x^  a;+4      x— 4      16— a;^ 

11.     ^ ^  +  -^, 

^      3a+2a;  _  3a— 2a;  16a;2_ 

3a— 2«      3a+2x  "^  4a;^— 9a2 

j3      _1 4 8_  ^  3a;+7 


a;— 1       1— a;      1+x      a;^— 1 


02 


106  ALGEBRA 

a  c 

14.     — —  . 

c(a—b)      a{b—a) 

.5  1  I 1 

*     {a-b){c-a)  "^  (b-a)(c-6 

16.     , ^_^+  ^ 


{x—a){a—b)      {x—b){b—a) 

2  2  1 

17.     - + 


18. 


y^ — x^      x^ — 2xy-\-y^      x^-\-'xy 

3—x        3+x       l-lQx 
r=^      1+3^      9a;2-l ' 


2  2  4 

19 I _)_ 

'    x^-Sx+ 15  ^  a;2-4a;+3  ^  6a;-a;2-5 
20  ct+^         I  *^+c  c+a 


(6— c)(c— a)      (c— a)(a— 6)      (a— 6)(6— c; 


(a—b){a—c)      {b—c)ib—a)      (c— a)(c— 6) 

22.  ,-^! ,+,^  ^;    .+     '^^ 


(a— 6)(a— c)      tb—c){b—a)      {c—a)(c—b) 
23.     , -f, ,  +  , I ,+  ^ 


(a:— 2/)(x— 2)      {y—z){y—x)      (z-x){z—y) 
ax— be        ,        bx—ca        ,       cx—ab 

24.        7 tTT ^  +  /IT-   \/T ^  +  ' 


(a— 6)(a— c)      (6— c)(6— a)      (c— a)(c— 6) 

•  /_■>  19\/    .0  .9x    "t"    /to  -9\/l,'>  _9\       ' 


„           6c(a+(i)      ,      ca{b-\-d)  ab(c-\-d) 

^"'     ; —  iT^/ ^  ~r  77 —  \/£ ;  + 


27. 


(a— 6)(a— c)      (6— c)(6— a)      (c— a)(c— 6) 
a^      2(6+0.)      2(a2"+p)  ^  6*  -  a ' ' 
x-5      X+5J      \x+3 


'»•    J^  +  iis-^-^      (Check  when  .=2.) 


FRACTIONS  197 

141.  Multiplication  and  Division.  The  ordinary  cases  in 
multiplication  and  division  of  fractions  have  been  treated  in 
art.  74.  Some  special  forms  which  appear  are  illustrated 
in  the  following  examples. 

Ex.  1. — Multiply  a  -\ by   a , 

^  -^  a—x     "^  a-\-x 

Here  the  mixed  expressions  shovild  be  reduced  to  the  fractional 
form  before  multiplying. 

r_,            ,     ,          a^—ax+ax      a^-\-ax—ax  a* 

The  product     = X 


a-\-x  a^—x^ 

Ex.  2.— Multiply  ?+^  +  l    by   J+^-1. 

Multiply  this  in  the  ordinary  way,  by  multiplying  each  term  of  the 
one  by  each  term  of  the  other. 

We  should  recognize  that  the  first   expression  is  the  sum  of   7-  H — 
and  1  and  the  second  is  the  difference. 

The  p,„duct  ^  (» + 5)'_  .= »_v  2 + ^^:  _.=«_!+,+ ^^:. 

Or,  we  might  proceed  as  in  Ex.  1,  thus  : 


ah  ab  a^b-  a-b^ 

a^ 

fe2 


q2  f)2 

This  result  is  seen  to  be  the  same  as  ^  +  1  H — ~,  and  the  answer 


may  be  given  in  either  form. 

x^       1  r         1        1 

Ex.  3.— Divide  ^  +  -   by   4--  +  -. 
y6      X  y^      y      X 

The  dividend         =  ^!±^     the  divisor  =  ^^-^V+V^ 
xy^  xy^ 


the  quotient     =     ^3     x     j 


x^-\-y^  xy^         _  x-\-y 

xy^         x^—xy-\-y^         y 


Divide  in  the  ordinary  way  and  get  the  quotient  -  +  1. 

We  must  not  make  the  error  of  thinking  that  we  can  invert 
the     divisor,    or   take   the    reciprocal     of    it,    by   inverting 


198 


ALOEBRA 


each  term  of  it,  and  change  the  problem  to  one  in  multi- 
pUcation,  thus  : 

-^   is   a-\-h,    but   the    reciprocal    of 


The   reciprocal    of 

U--T-0' 

-  +  Y-  is  not  a-\-h. 
a      0 

_      1,1       a-\-h       J  .^         .  ,  .      ab 

For  -  +  r  =  — .—  and  its  reciprocal  is  — -y 
a      0        ab  a-\-b 


Miiltiply : 


EXERCISS  96 


1* 
3. 


-x—6 


c^-2x-S 


a;2+4a;+4'  x^-lx+12' 
a;^4-2a;-15  x^+lx-U 
x^+Sx-dB'   a;2+9a;+20 ' 

^'    1 n,'   ^  +  — A  • 

a-\-o  a—o 

7.    :^2+i+l     x2-l  +  i 


a2_2a6+62'   a^-^-ab' 


xy 


^     y 


2  —  3:8 


6.    y  +  -^^,  y-    ,   ,    2  ,    , 

1/— a;  a;+2/    ^z^  +  aj^ 


8.     a2  _(_  2  +  , 


r2' 


2  + 


9. 


J    .  hx     ^ 

6  +  —  ,    1 
a 


a        a 

a-\-x    bx 


Divide  : 

a:2_lla;+30 


11. 
13. 
14. 
15. 


by 


x^—6x 


a;2— 6a;+9      ''  x'^—Zx 
a2-j-62_c2_i_2a6       a+6+c 
^6+c— a' 
X      y 


c^—a^—b^+2ab 


X      y 

X 

X  .  y 


y^      x^      y      X       y      X 


y^     x'^       y     X 


Simplify : 

a+l 


17. 


a+2 


a^— 2a      a^—a  '  a^—a^ 


10. 


12. 


16. 


18. 


t3_53    (j+ft    a^^ab+b"^ 


as+fcs'   cj_6'   a2_^a6+62 


^ by  2' 

x^—xy       xy—y^ 


x^'—y" 
x*^—y* 


by 


x^—y^ 
x—y 


a2-4  ^  a^+2g 
aH-5a  ■  a2-25' 


FRACTIONS  199 


'    x~-4x+3  ^   6X-+X-2    •  3.r2-7x-6' 
20. —  +  -^ -  (a+a;  2. 

21.  ri+'Yi+-Ufi+?X'+f). 


22. 


a/\        aj  '   \        b, 
(a4-6)2-(c+<?)^  .  (a-c)2-((Z-6)2 


(a-j-C)2_(6+d)2    •    (o_6)2_(rf_c)2 


g2-64  a2+12g-64  _^  a2-16ft+64 

■     a2_f.24a+i28  ^       a3-64        "    a2+4a+16  ' 

„^      /   5a  26     \      /   2a         26— a 

24. 


a— 66      3a— 26/  "   Va+26      26— 3a 


142.  Complex  Fractions.  A  complex  fraction  is  one 
which  contains  fractional  forms  in  either  the  numerator  or 
denominator  or  both. 


Thus,  -  is  a  complex  fraction  and  is,  of  coxirse,  only  another  way  of 


writing  r  -i-  ;;  •     It  is  simplified  in  the  usual  way  by  changing  it  into 

a      d     ,  .  ,  .ad 

r  X  -  which  equals  r~  - 

A  complex  fraction   may  sometimes   be  easily  simplified 
by  multiplying  both  terms  by  the  same  quantity. 

Thus,  — ~-  =  —TV —  on  multiplying  each  term  by  4. 

a+26      a 

a+6  "*"  b     _  6(a+26)+o(a+6)  _  a^  +  2o6  +  26'  _  , 
0+26       ^«_  ~  (o+26)(a+6)-a6  "  a2+2o6  +  26*  " 
b  a-^b 

Here  both  terms  were  multiplied  by  b{a  -\-  b). 


200  ALGEBRA 

If   the   L.C.D.    is    not  the   same   for    both    terms    of   the 

fraction,  it  is  usually  better  to  simplify  the  terms  separately. 

x+l      x—l 

_,.      ...    x+2~^^ 
Ex.-Simplify  ^_^      ^_^^. 


The  numerator      = 
The  denominator  = 


x+2      x—3 

{x+l){x-2)-{x-l)(x+2)  -2x 


(x+2)(x-2)  (x+2)(x-2) 

(a;-2)(a;-3)-(a;+2)(a;  +  3)  _         -  lOa; 

(x+2){x-3)  ~  {x+2){x-3) 


-2x  {x+2)(x~3)         cc-S 

the  fraction      =  (^^2){x-2)  ^         -lOx        ==  5{^^ ' 


EXERCISE  97 
Simplify : 

6a  ^  1_1 

^  ^                  2       Qa_  g      g— 6                ^      :» y 

12c                     *      56  *     a-\-b  '     x—y 
12c 

1  11  ,  a;2+w2 

1+a;  ^      a+6'^a-6  „  x+y 

1+a;  a+6      a— 6  .a;+?/ 

„      a^— 5a 

''•  _3c^  •     (3x+2/)2-(3:i:-2/)2 

■^a-3 

10.        ,y-l     •  a  +  b-         ''         ' 


a^-ab+b^ 


c  d 


12.     ^±L_i±£.  ^  13.     i 


a  6 

6+c      c+a 


2x  +  3  - 


1__j!_ 

a;+6 


FRACTIONS  201 


14.  "!         +— ^ 15  ^"^^^ 


^!+i"_6      ^!±^^-a  A-Jl  +  L 

a+6  a+6  «^      «&      b^ 

16.  Find  the  value  of when  x= ~  ,  y—  — —  . 

x+y  a—b  a-\-b 

17.  Find  the  vahie  of  — ■- — ^  when  a  =  —-^,  b=  ) — --^. 

l+2a+6  x+y  {x^y)^ 


EXERCISE  98  (Review  of  Chapter  XV) 

Simplify  -. 

a 

^  *      a a_    ,      2a-  „      a  _  & 


o      __«^    _     y     _  jx-y)-  ,         2  _^ x__ 

"**     x+Sy       Sy-x      a;2-92/2"  '     1_1  j_^       j_? 

X      y  X  y 

1  1  2  4  ,  X  11 


{'+^}-('+'S0- 


(l+a;)2-{l-a;)2 

26c   \/,  26c  \/,       6c-c2\     62 


/,  26c   \/,  ,     26c  \/,       6c-c2\ 


62       h^+c^ 


^^'     x+1       a;«+3x+2"^  a;3  +  6x2+llx+6' 
o^-o*       a6  +  62  2a26 

-'■■'■"       _a       ta  ~r   _  a   ,    j,  a    i 


o«-6«  cH^  ct+6 


a2+2a6  +  62   -^    o2_52    -^    „4^.„2ft2  +  54 

ifi  «+a:  g-a;  2x» 

a*  +  oa;+x2  "•"  a^-aaj  +  x*"*"  o*  +  o2a.^_|_^4* 
14.     (l-o2)  ^  |(l-a)2  -  (a^-  1)  ^  (a  +  ^)}' 

._  6+c— g  c+g— 6  a+6— c 

•     (g-6)(g-c)  "•'  (6-c)(6-g)  "^  (c-g)(c-6)  * 


202  ALGEBRA 


16.      7^7 ,  +  , 1^  + 


(x-y)(x-z)       (y-z){y-x)       {z-x)(z-y) 
ix+e  3x—9  15a;- 10 


6a;2+5a;-6   '    2x^-Zx—^       9x2-12a;+4 
18.     Express  the  product  of 

8-6a;+a;2         ,  1  1 

and 


1+x  (a;-l)(2-a;)       (a;-2)(4-a;)       (cc-4)(l-a;) 

as  a  fraction  in  its  lowest  terms. 

19.  How  can  you  show  mentally  that  3  is  the  sum  of 

a;  XX  ^^        ah c_  ^ 

x-\-a      x-\-h      x+c  x+a      x-\-b      x-\-c  ' 

20.  Divide   -^  +  -^ ^,   by   -^  +       '^ 


x-\-y      x—y      x^—y^  x-\-y      x^—y^ 

22.  Divide^^  +  '-^+'-^-l    by   2-(l+Ul). 

a  b  c  \a       0       c/ 

23.  Show  that  (i_^^),_(^^y),  -  (i_^.)(i_y2)_4^  ' 

24.  If  a=2^^,  6^-24,.  c=^^,  d=^^,  prove  a=a:. 


25.     Find  the  product  of 

l+x 


and  l-{-x-\-- 


1  +  x^  1  ,     a; 

a;— 1 

26.     Subtract  -       ,^  from  r  and   determine  which   fraction  is  the 
6+10  b 

greater  if  a  is  greater  than  b  and  if  both  a  and  6  are  positive. 
(l+o6)(l  +  ac)   (l  +  fec){l  +  6a)   (l+ca)(l+cb) 


27.     Add 


(a— 6)(a— c)  '    (6  — c)(6  — o)  '    (c  — a)(c  — 6) 


28.  Show  that 1 r  has  the  same  value  when  a;=a+6  as  it 

x—a      x—o 

has  when  x—  — ~r  • 
a+b 

29.  Prove  that  the  product  of  any  two  quantities  is  equal  to  their 
sum  divided  by  the  sum  of  their  reciprocals. 


FRACTIONS  203 


30.  If  x= ,  y  =  — T — ,  2= ,  prove  that 

a  0  c 

ocyz-\-x-\-y-\-z  =  0. 

x—y    ,     y—z         z—x     ,        ^,    , 

31.  If  a=—~,  b=^-~—,  c=— — ,  show  that 

x+y       y+z       z+x 

(l_a)(l_6)(l-c)  =  (l+a)(l+6)(l+c). 

32.  If  a  and  b  are  positive,  which  is  the  greater 

a4-36        a+2b  „ 

— ■ or  — ' 

a+26         a  +  b   ' 

_1 1  a+Sb  463 

a-6       2(a+6)       2(0^+62)       a*— 6« 

x—y  y—z  z—x 


33.     Simphfy 


34.     Add 


■(a;-2/)2'  a;2-(2/  — 2)2'  ^/S— (z  — a;)2 


35»     Simphfy  -^ %  +  — ^ ^. 

36,     When  x= ,  find  the  value  of 

a-\-c 

x—2a       x  +  2a  ^ac 


x-\-2c       x—2c  ^  a;2-4c* 
y—x  y—x 


37.     Simphfy  \  — 4±^, ^^  I  -  f  ^  -  -Y 

^     x{y-x)      ^     y{y-x)l    •  \x      y! 


1+xy  l  —  ocy 


CHAPTER  XVI 
FRACTIONAL  EQUATIONS 

143  If  an  equation  involves  fractions,  the  fractions  may 
be  removed  by  multiplying  every  term  by  the  same  quantity. 

In  Chapter  VI.  simple  examples  of  fractional  equations 
were  given. 

The  case  in  which  two  fractions  are  equal  deserves  special 
attention. 

CI        c 

Thus,  a  T  =  -,  and  each  side  is  multipUed  by  bd  we  have 
0      a 

-  X  bd  =  J  X  bd. 
b  d 

ad=bc. 

144.  Cross  Multiplication.  It  is  thus  seen  that  when  two 
fractions  are  equal,  we  can  remove  the  fractions  by  multiplying 
the  numerator  of  each  fraction  by  the  denominator  of  the 
other  and  equating  the  results.  This  operation  is  sometimes 
called  cross  multiplication. 


Ex.  1.— Solve 


a;+3 
^9 


Cross  multiply  {x-5)(x  +  9)  =  {x+3)(x-l), 

.'.    x^-\-4x  —  45  =  x^  —  'ix  —  2\, 
8a; =24, 
x=3. 
Verify  by  substitution. 

This   method   is   applicable   only   when   a   single    fraction 
appears  on  each  side  of  the  equation. 


FRACTIONAL  EQUATIONS  205 

Ex.  2.— Solve  — hr-  =  7 t-  • 

x-\-3  X—4: 

Simplify  the  right-hand  member  and  we  have 
4x4-17  _  4a;— 18 
x+Z'  ~    x—4:  ' 

Now  cross  multiply,  complete  the  solution  and  verify. 


EXERCISE   99 

Solve  and  verify  : 

^      3a:+l_a:+12  2  3a:-5      3a--l 

2              3~"  *  5x-Z~  5x-\' 

*  3        II    ~           ■  *  x-S      .T-5"~ 

g      x+2_x-2_x-l  g  a;+l      a:-3_a:+30 

*  ~5         ~2~      ^7~'  '        2         ~3            13 

7      7a;-3         _  2a:+5  ^  x^+Tx-G  _a:+l 

2                      3'  '  a;-+5.r— 10  "x— l" 

9.     ^±^  =  2-^+1.  10.  -L+l^_l_. 

X— 1  X-  — 3  X— 1        .T        x+l 

x+2"^x-2      x-3"  *"  3x-2      6x-l 

13.     2x+38^6x+8_j  ^^^          2/-8       _       y-l2 


15. 


19. 


x+12       2x+l  2/2-8^+15      2/^-122/+30 

2x+7      8x+19_5a;+ll  j^.      x-1  •-?:— 5  _  ^ 

3          "12           7x+9             '*     .T-2  x-3 

6-8x      _3_  ^  ^        2x+7  3x-5  ^  5x-f9 

^^^^r^~    *                         '     x+l  x+2  ~  x+3  ' 

4x3+ 4x2-(-8x+J  ^  2x2-{-2x+l 
2x2+2x+3      ~'       x+1 


20.   ?^=:B=,^=f+i. 

X— 3       X— 5 

21      ^—^^  I  ^^  _  a;-2  _  2x— 3      „ - 
'       1-5    ^1-25      T8"~      9  ^'^' 


206  ALGEBRA 

22.  Solve  — - — '- = ^  by  first  reducing  each  fraction 

3a;— 4  4a;— 5 

to  a  mixed  expression. 

23.  Find  three  consecutive  numbers  so  that  the  sum  of  ^  of  the 
first,  i  of  the  second  and  \  of  the  third  may  be  30. 

24.  Divide  300  into  two  parts  so  that  if  one  be  divided  by  5  and 
the  other  by  7,  the  difference  of  the  quotients  will  be  18.  Give  two 
answers. 

25.  How  much  water  must  be  added  to  100  lb.  of  a  4%  solution 
of  salt  to  make  a  3%  solution  ? 

26.  A  pupil  was  told  to  add  3  to  a  number  and  to  divide  the  result 
by  5.  Instead  of  doing  so  he  subtracted  3  and  multiplied  by  5  and 
obtained  the  correct  answer.     What  was  the  number  1 

27.  A  man  bought  180  lb.  of  tea  and  560  lb.  of  coffee,  the  coffee 
costing  -^  as  much  as  the  tea  per  lb.  He  sold  the  tea  at  a  loss  of  25% 
and  the  coffee  at  a  gain  of  50%,  and  gained  $62-60  on  the  whole.  What 
did  the  tea  cost  per  lb.  ? 

28.  I  sold  some  butter  at  25c.  a  lb.  If  I  had  received  5c.  more 
for  1  lb.  less,  I  would  have  received  2c.  more  per  lb.  How  many  lb. 
did  I  seU  ? 

29.  If  I  walk  to  the  station  at  the  rate  of  11  yards  in  5  seconds 
I  have  7  minutes  to  spare  ;  if  I  walk  at  the  rate  of  13  yards  in  6 
seconds  I  am  3  minutes  late.     How  far  is  it  to  the  station  ? 

145.     Fractions  with  similar  Denominators. 

^      ,      ^  ,       a;+6      2x-18  ,  2x+3      16  ,  3.T+4 
Ex.  l.-Solve  — 3-  +  ^_  =  -  +  -^. 

Here  we  might  multiply  each  term  by  the  L.C.D.,  which  is  132. 
It  will  be  found  simpler,  however,  to  remove  all  the  fractions  but  the 
first,  to  the  same  side  of  the  equation,  as  they  are  easily  reduced  to  a 
common  denominator. 

g!+6  _  2a;- 18  _  2a;+3    ,16,   3a;+4 

11    ~       3  4      "^  3   "^     12     ' 

a;-f-6  _  4(2a;-18)-3(2x+3)  +  64+3a;+4 
■       11    ~  12 

Now  simplify,  cross  multiply  and  complete  the  solution.  The 
correct  answer  is  a; =5. 


FRACTIONAL  EQUATIONS  207 

This  problem  shows  that  the  denominators  of  certain 
fractions  are  such  that  these  fractions  can  be  conveniently 
combined  when  they  are  grouped  on  one  side  of  the  equation. 

Ex.2.-Solve  __^^— ^  +  __^. 

Since  4x+4  =  4(a;+l),  it  is  seen  that  it  is  simpler  to  combine  the 
second  fraction  with  the  first  than  with  the  third. 

2x+3      Jx+5_  _5x+A 
Ic+T  ~  4(a;+l)  ~  5x+  1  ' 

Subtract  the  first  two  fractions,  complete  the  solution  and  verify 
the  restilt. 

Ex.  3.— Solve  — _ H _  =  0. 

x—2      x—3      x—5      x—o 

Here  it  is  too  laborious  to  multiply  all  the  fractions  by  the 
L.C.D.  It  will  be  found  easier  to  change  the  equation  so  as  to  have  two 
fractions  on  each  side,  then  simplify  each  side  and  cross  multiply. 

Solve  by  transposing  the  last  two  fractions,  also  by  transposing 
the  second  and  fourth,  and  compare  the  results. 

Ex.  4.-Solve  -^„  +    h>  =  "L  +  -  Tq  * 

x-\-5      x-\-2      x-\-4      x-{-3 

AAA-  u    -A  2x4-7  2a:  +  7  ... 

Adding  on  each  side,  ,  ,  _ — -^rx  =    „  ,  „ — -^5 '  (1) 

2a;+7  2a;+7       _ 


x^  +  lx+lO      a;2+7a;+12 
.-.      {2x+l)  (^2^7^^  10  -  a.2+7a;+i2)  =  ^' 

x=-3^  or  x2  +  7a;+10=a;2  +  7a;+12. 

Since  the  equation  a;*  +  7a;-f  10  =  a;*  +  7x+ 12  is  impossible,  the 
only  root  of  the  given  equation  is  a;=  —  3i.     (Verify  this  root.) 

If  in  line  (1)  we  divide  each  side  of  the  equation  by  2x-\-l, 
an  impossible  equation  will  result.  It  is  not  allowable  to 
divide  both  sides  of  an  equation  by  a  common  factor  unless  we 
know  that  the  factor  is  not  zero.     Here  2a; +7  might  be  equal 


208  ALGEBRA 

to  zero,  and,  in  fact,  would  be  if  a;=— 3|.     If  x= — 3|  the 
equation  in  line  (1)  is  satisfied  as  each  side  becomes  zero. 
Solve  the  equation  by  writing  it  in  the  form 

^ ]__  J^ |_ 

x-\-5      x+4"~a;+3      x-\-2' 


EXERCISE   100 
Solve  and  verify : 

1      2a:+l      6a;— l_3.r-2  2x+3  _  x—l_  _  x-\-2 

5             15     "  Qx+3 '  '        4          6a;— 8  ~     2    ' 

3      a:+3      3a;+5_2a:+l  6a;+l      3a:— 1      2x—1_q 

7         6a;+2"~     14  *         4              2         3a;-2 

5.     ^  +  -A-  =  ^1+  '' 


8. 


10. 
11. 
12. 


14. 


a;-8      2a;-16      24      3a;-24 

A-L  5a;— 5  _6x+7 
12"^12a;+8~9a;+6" 
13a:-10      4a;+9      7a;-14  _  23a:-88 
36       "^     18  12      ~17a;-66" 

3a:— 4  _  1         6a;-5 
6a;-9  ~  12      8a;— 12  " 


„      5a:- 17  ,  2a;- 11      23      3a;- 7 


13— 4a;  14  42         21 

1        5a;-7       4a;-3 


10      10a;-5      4a;- 2 

J- 1___1 1_ 

a;— 1      a;— 2      a;— 3      a;— 4 

1111 


a;— 10      a:— 5      a;— 7      a:— 2 


13.     -^-+       1  3  1 


3a;+12      6a:+24      2a;+10      a;+6 

_3 ^__8 \_ 

x—5      x—1      1—x      5—x 


._       a:— 8    ,  a;— 4      a;— 5  ,  a;— 7  ,,  ,         . 


FRACTIONAL  EQUATIONS  209 

,„      2x-21  ,  x-1      x-\2  ,   2a;- 17 

lb, = 

a-U       a;-8      .r-13        x-9 

2x 27  .  1 

17.     Solve  the  preceding  example  by  changing  — -  into  2  -| 

and  making  similar  changes  in  the  other  fractions. 
4a:- 17       10a:- 13  _  8a:- 30      5a;-4 
'       a;-4    "^  ~2^^^  ~  '2a;-7  ^  r-l  ' 

5a;-64      2a;-ll       4a;-55      a:-6 


19. 


a:— 13         a;— 6         a:— 14       a;— 7 


20.  ^  +  ^i  +  ?z:?  =  3. 

a;+l      X — 2      X — 1 

c  1       x—a  ,   x—b  ,   x—c      „ 

21.  Solve 1 1 =  3. 

o  +  c      c-\-a      a-\-b 

22.  If  a-—b'-=a—h,  does  it  follow  that  a  must  be  equal  to  6  ? 
What  is  the  alternative  conclusion  ? 

146.  Literal  Equations  with  one  Unknown.  Equations  often 
occur  in  which  the  known  quantities  are  represented  by 
letters  instead  of  numbers. 

These  are  called  literal  equations. 

The  same  methods  are  used  in  solving  them  as  were  used 
in  solving  equations  with  numerical  coefificients. 


Ex.  1. — Solve  ax=bx-\-c. 

ax—bx=c, 
.'.  x{a—b)=c, 

_    c 
a  —  b 


Solve  8a:=3.r+20. 

8a;- 3a; =20, 
.-.   5a; =20, 

.-.      a3=\'^  =  4. 


Here  the  letters  a,  b,  c  represent  some  known  immbers 
whose  values,  however,  are  not  stated,  while  x  represents 
the  unknown  whose  value  is  to  be  found  in  terms  of  a,  b  and  c. 

Usually  the  earlier  letters  of  the  alphabet  are  used  to 
represent  known  quantities,  and  the  later  ones  x,  y,  z  to 
represent  unknown  ones. 

Compare  the  two  solutions  given.  They  are  practically 
identical.  When  we  work  with  numerical  coefficients  the 
result  can  usually  be  expressed  in  a  simpler  form. 

P 


210  ALGEBRA 

Note. — The  pupil  must  not  make  the  mistake  of  giving  a;=  ■ 

as  a  solution  of  ax  =  bx-j-c. 

This  statement  is  true,  but  it  is  not  a  solution,  since  it  does  not 
give  the  value  of  the  iinknown  in  terms  of  known  quantities  only. 

Ex.  2.— Solve  a{x—2)—b=a—2x. 

Removing  brackets,  ax—2a~b=a—2x. 
Transposing,  ax-{-2x=a-\-2a-^b, 

:.  x{a+2)  =  3a+b, 
_3a  +  b 

The  result  should  be  verified  by  substitution,  but  this  will 
frequently  be  found  more  troublesome  than  the  solution. 
When  it  is  not  verified  in  the  usual  way,  the  pupil  should 
review  his  work  to  ensure  accuracy. 

T-,      ,»      o,  1  ^—^       x—a 

Ex.3. — Solve  =-, 

a—x      b—x 

Cross  multiply,  bx—b^—x^-\-bx=ax—a'^  —  x^-\-ax, 
:.    2bx-2ax=b^-a^, 

_b^-a^  _  b  +  a 
^~2{b-a)~     2 

Verify  by  substitution, 

BXERCISE3  101 
Solve  for  x,  verify  1-12  : 

X 

1.     mx-}-a=b.  2      ax=bx+2.  3.     a-\--  =  c. 

0 

x-\-a  _1  x—a_^a  „      x—c a—h 

x—a      3  *     x+b      b  '     x-\-c      a-\-b 

_      a_        6  x—a_x—b  x—ax—b_, 

X      x—a-\-b.  '     x—b      x—a  '       2b  2a 

10.     ^-^  =  -_^.  11.     aix-a)+b{x-b)=0. 

b       a      a       c 

12.     l--^-=:--l.  13.*  {a+x){b+x)  =  {c-\-x){d+x). 


FRACTIONAL  EQUATIONS  211 

14.     {ax-h){hx+a)^a{hx^-a).        15.    ^fc^)  _  ^i^±£)  =a;. 

')  a 

a  b    _a—h 

x—a      x—b      x—c 

17.  x{x—a)-\-x{x—b)=2{x—a){x—b). 

18.  {x—a){x—b)^{x-a-b)^ 

19.  J^ L_^_i L_. 

x—a      x—2a      x—3a      x—4a 

20.  ix—a){x—b)—(x+a){x+b)=(a+b)K 

21.  (o+a;)(6+a:)-a(6+c)  =  ^^  +  a;2. 

0 

22.  {a+x){b-x)+x^^b{a+x)  —  —' 

23.  a2^+63+a6a;=a3_62^., 

24.  The  excess  of  a  number  over  a  is  three  times  its  excess  over  b. 
Find  the  ntimber. 

25.  Divide  the  number  a  into  two  parts  so  that  one  part  may 
contain  b  as  often  as  the  other  will  contain  c. 

26.  Divide  a  into  two  parts  so  that  m  times  the  greater  may  exceed 
n  times  the  less  by  b. 

27.  A  rectangle  is  a  feet  longer  and  b  feet  narrower  than  a 
square  of  the  same  area.     Find  the  side  of  the  square. 

28.  If  a  number  be  divided  by  a,  the  sum  of  the  divisor,  quotient 
and  one-third  of  the  number  is  6.     Find  the  number. 

29.  A  man  sells  a  acres  more  than  the  mth  part  of  his  farm  and 
has  b  acres  more  than  the  nth  part  left.  How  many  acres  were  in  the 
farm  ? 

30.  Solve  {a—x){b—x)=ic—x){d—x). 
Check  by  putting  a=l,  6=6,  c=2,  d—3. 

31.  If  5  =:  -  {ci-{-l),  solve  for  n  ;   for  a  ;  for  L 

I 

32.  If  5  = ,  solve  for  a  ;   for  I ;  for  r. 

r-1 


33.     If  s=at  +  ^gt-,  solve  for  a  ;  ior  g . 


p2 


212  ALGEBRA 

147.  Literal  Equations  with  two  Unknowns.  Every  simple 
equation  in  x  and  y  may  be  reduced  to  the  form  ax-\-by=C, 
where  a,  h,  and  c  represent  known  quantities. 

If  two  equations  in  x  and  y  with  Uteral  coefficients  be  given, 
the  equations  may  be  solved  by  the  same  methods  as  were 
used  with  equations  with  numerical  coefficients. 


Ex.  1.— Solve 

ax-\~by=c, 
ax—by=d. 

Adding, 

2ax= 

=c+d,     ..X       2„  • 

Subtracting, 

2by  = 

c—d 

-0     d,      .:   2/  -    26   • 

Verify  in  the  usual 

way. 

Ex.  2.— Solve 

ax-\-hy=c, 
mx-\-ny=k. 

Multiply  (1)  by  n, 
Multiply  (2)  by  b. 
Subtracting, 

nax-\-nby  =  cn, 

bmx-{-bny=kb. 
x(na—  bm) =cn— kb, 
cn  —  kb 
na  —  bm 

(1) 

(2) 


We  might  substitute  this  value  of  x  in  either  of  the  given  equations 

to  find  y,  but  it  is  simpler  to  solve  for  y  in  the  same  way  as  we  did 

for  X. 

,  „    ,          cm  —  ak        ak  —  cm 
Ehminate  x  from  the  two  equations  and  nnd  y  =  r or r — 

Ex.  3. — Solve  a^x-[-h{y=c^, 

a2X-\-h.yy~c.,. 

Here  the  symbols  aj,  Uo,  etc.,  are  used  to  represent  known  quan- 
tities. They  are  read  "  o  one,  a  two,  b  one,  etc."  There  is  no  relation 
in  value  between  a^  and  a^,  nor  Oj  and  6].  The  notation  is  used  to 
obviate  the  necessity  of  employing  many  different  letter  forms. 

Solve  these  equations  as  in  the  preceding  example  and  obtain 


afi<^—a^i '  b^a^—b^Oi 


FRACTIONAL  EQUATIONS 


213 


EXERCISE   102 

Solve  for  x  and  y,  verify  1-12  : 


1.     mx-{-ny=a,  2. 

'mx—ny^=h. 

4.     ax-\-hy=a'-\-b",      5. 
x-\ry=a+h. 

7.    ax-hy=2a'^+'ih\ 
hx-{-ay=—ah. 

9.     a"x^h'^y=a-—ab+h'^, 
ax—by=a—b. 


lx-\-m,y^m, 
mx-{-ly=l. 

ax-{-by=2ab. 


6. 


bx—ay=b^—a^. 

8.         ax—by=2ab, 
2bx+2ay=Sb'^—a 


px+qy=r, 
x+y^O. 

ax-{-by=2, 
a^x—b^y=a—b. 


10. 


a  b 
x_y 
a      b 


=  3, 
=  1. 


13.*  a^x-{-b^y=a^-¥ 
bx-\-ay—0. 


11. 


14. 


a      6 


ax—bi/=a-—h'^. 


a^x-{-biy=Ci,     15. 

.r+?/=l. 


2,  12.    a2-c_52y^c^3_f.i,3^ 

2/. 


^ ^  =  2 

a      6 


-  +  -  —  —  -J. 
a;      y 

3a      4& 


5A. 


// 


M.     (a+b)x-{a-b)y=a^~+b^ 
X — y=a — b. 


18.     x  + 


20. 


a?/ 


ax     ,  , 

0      a+o 


17. 


19 


a;        y 

a.,  ,   fto 

^  +  -=  =  c.. 
X       y 

l-5a  ,  3-26  _„^ 

X  y 

•5a  'ZSb  _  ,„_ 

a;  y 


If  ax-\-by=c  and  x— ?/=l,  prove  that  x(c— a)  =  ?/(6+c). 

21.  If  y=ax+b  and  x=py—q,  prove  that  y{q—bp)=x{aq—b). 

22.  If    6a;+a2/+c2=ac+6c,     ax+by=ac,    cy-\-dz=ad,     solve    for 
a;,  2/  and  z. 

23.  What  is  the  value  of  m,  in  terms  of  a  and  6,  if  the  following 
equations  are  consistent 

ax+Zby=a'^+Zb^, 

Zx+y=^a-\-b, 
4:X—3y=m  ? 


3a;- 2 
2x-3 

x+ll       ^ 
a;+10       ''■ 

2a;  +  7 

9a;- 8      a;- 11 

7 

11              2 

a;+l 

2 

3      X       5—x 
a;       3          6 

214  ALGEBRA 

BXERCISE!  108  (Review  of  Chapter  XVI) 

Solve  and  verify  1-24  : 

1.     §(7-a;)-4(ll-a;)  =  ^(a;-8).  2. 

"*•     2  ^  3       ^     3  ^  2       6. 

2a;- 3       3a;- 2       5a;g-29a;-4 
•      a;-4  +  a;-8  ~a;2-12a;+32'         * 

2  3  21  5     ■    ^"'     2a;+l  ^  3x  +  5~ -'• 

11.     a{x—a)—b{x—b)  =  {a+b){x—a—b),    12.     aa;+6  =  6a;+a. 

.„      8a;+5       9a;-3       4a;-3                        ^^       4-a;       2+4a;      ^ 
^^-      -10--7^+2  =  -5--  1*-     ^3 7-  =  ^-^- 

-,=      ««,   2/,     a;        ?/k  ^c      «'^+&      ca;+d 

a       b  3a       46  ax—b       cx—d 

5  2  7  1  -r  1  '>r 


3a;  '    5y         '    6a;  '    1%         '                    *     a-26       2  "^  2a-6 
19.     ox+%  =  2o6,  a2/-6a:=a2-62.         20.     ^ —  =  1  ^ 


x-\-b—a  x+b  —  c 

21      ?-±^  _  ^±^  =  ?±?  _  ^±^  22  3  2      ^  _5_ 

•     a;  +  3       x+4       x+6      a;  +  7'  *     a;+10"^a;-10      x-2' 

a;-l       a;-3       x-5      x—1      ^     „^       a;+l        1   ,      8x-3 


X--2      x-4      a;-6^a;-8  '     3a;-4       5  "^  16a;- 20 

25.*  (3a-a;)(o-6)  +  2aa;  =  46(a-fa;). 

26.     ^ +  2b{a-c)  +  ~  =  c(a+b)  +  ~' 

y   ,    2y-3x  Ix-Zy 

^^'     1-^1=  -'^'  ^x-2y^-\(iTy. 


FRACTIONAL  EQUATIONS  215 

30.  ^ +  ^=2^xy,  5y-2x=2^xy.  ^ 

31.  What  value  of  y  will  make 

x+5  ,    y—l  ,  ,     x—l      v+11  „ 

-T-+V   equal  to  ^-+^-±-? 

32.  Two  sums  of  money  are  together  equal  to  $1000,  and  5|%  of 
the  larger  exceeds  6^%  of  the  smaller  by  16  cents.     Find  the  sums. 

33.  Find  a  fraction  such  that  if  4  be  added  to  its  numerator  it 
becomes  equal  to  |,  but  if  4  be  added  to  its  denominator  it  becomes  f . 

34.  If  ax  +  b  =  cx  +  d,  give  the  argimient  which  leads  to  the  con- 
clusion that   X  =  ,  indicating  at  what  point  it  is  assumed  that  a 

a~c 

and  c  are  unequal. 

35.  Take  any  two  proper  fractions  whose  sum  is  unity.  Add 
unity  to  the  difference  between  their  squares.  Show  that  the  result 
is  always  twice  the  greater  fraction. 

36.  A  man  has  $30,000  invested,  part  at  4|%  and  the  rest  at  5J%. 
He  receives  $65  per  annum  more  income  from  the  former  than  from 
the  latter.     How  much  is  invested  at  each  rate  ? 

37.  The  sum  of  three  numbers  a,  b,  c  is  3036;  a  is  the  same 
multiple  of  7  that  b  is  of  4,  and  also  the  same  multiple  of  5  that 
c  is  of  2.     Find  the  numbers. 

38.  If  5  =  H(2a  +  nd  — d),  solve  for  ct ;    for  rf. 

39.  If  ax  —  by  =  a^  +  b'^,  x  —  y=2b  and  x^->ry'  =  c,  find  c  in  terms  of 
a  and  b. 

40.  A  man  can  walk  2J  miles  an  hour  up  hill  and  3|  miles  per  hour 
down  hill.  He  walks  56  miles  in  20  hours  on  a  road  no  part  of  which 
is  level.     How  much  of  it  is  up  hill  ? 

41.  A  farm  cost  3 J  times  as  much  as  a  house.  By  selling  the  farm 
*•■*  "77-%  gain  and  the  house  at  10%  loss,  $2754  was  received.  Find 
the  cost  of  each. 

42.  In  10  years  the  total  population  of  a  city  increased  11%.  The 
foreign  population,  which  was  originally  -^j  of  the  total,  decreased  by 
1160  and  the  native  population  increased  by  12%.  Find  the  total 
population  at  the  end  of  the  period. 


CHAPTER  X\T:I 
EXTRACTION  OF  ROOTS 

148.  Square  Roots  by  Inspection.  In  art.  65  we  have  seen 
that  the  square  root  of  any  trinomial,  which  is  a  perfect 
square,  may  be  written  down  by  inspection. 

We  have  also  seen  that  every  quantity  has  two  square 
roots  differing  only  in  sign. 

Thus,  the  square  root  of  a^-{-2ab-\-h^  is    ±{a-\-b), 
and  of  a'^—2ab-\-b^  is   ±{a—b). 

±  {a-\-h)=a-{-b  or  —a—b;     ±{a—b)=a—b  or  b—a. 

If  we  had  written  a^—2ab-}-b-  in  its  equivalent  form 
b^—2ab-^a^,   it   is   seen   that   b—a   is    a   root. 

It  is  usual,  however,  to  give  only  the  square  root  which 
has  its  first  term  positive,  and  we  say  that  the  square  root  of 
a^-\-2ab-\-b'^  is  a-\-b  and  of  a^—2ab-'rb"  is  a—b  or  b—a. 

EXERCrrSE  104  (Oral) 
State  the  square  of  : 

1.  —abc.  2. 

4.  2a— 36.  5. 

7.  a+b+c.  8. 

State  the  square  root  of  : 

10.  IGx^y^.  11, 

13.  a^—2a+l.  14. 

16.  x^+x+i.  17. 

19.  x^+y^+z^+2xy-}'2xz+2yz. 

20.  a^+b^+c^-2ab-2ac+2bc. 

21.  4a2+962+l  +  12a6-4a-66. 

216 


x+l. 

3. 

-x-1. 

a^+1. 

6. 

x^—x. 

a+b—1. 

9. 

2a+b—c. 

la%\ 

12. 

x^+2ax+aK 

4a2-12fl6+962. 

15. 

9x^-30xy+25y' 

a*+2a^+a^. 

18. 

\6x*-i8x^+S6. 

EXTRACTION  OF  ROOTS  217 

149.  Formal  Method  of  Finding  Square  Root.  When  the 
square  root  of  an  expression  of  more  than  tliree  terms  is 
required,  it  is  not  always  possible  to  write  down  the  square 
root  by  inspection. 

Thus,  to  find  the  square  root  of 

9^4- 12a;3+ 10a;2-4a;-|-l. 

Here  we  could  say  that  the  first  term  in  the  square  root  is 
3x^,  and  that  the  last  term  is  either  +1  or  —1,  but  it  is 
evident  that  there  must  be  another  term  as  well. 

Let  us  again  examine  the  square  of  the  binomial  a+^> 
which  is  a^-^2ab-\-b'^. 

The  first  term  of  the  square  root  is  a,  which  is  the  square 
root  of  a^.  The  second  term  of  the  square  root,  b,  may  be 
obtained  in  two  different  ways,  either  from  the  last  term,  b^, 
or  from  the  middle  term,  2ab. 

Let  us  now  see  how  we  could  obtain  the  second  term  in  the 
square  root  from  the  middle  term  2ab.  This  term  is  twice 
the  product  of  a  which  is  already  found,  and  of  the  last  term 
of  the  square  root  which  is  still  to  be  found. 

If  twice  the  product  of  a  and  the  last  term  is  2ab,  then  we 
can  find  the  last  term  of  the  root  by  dividing  2ab  by  2a,  which 
gives  b. 

The  quantity  2a  which  we  use  to  find  the  second  term  in 
the  square  root  is  called  the  trial  divisor. 

Since  a^-\-2ab-{-b^=a^-\-b{2a-\-b),  we  see  that  the  complete 
divisor  is  2a-\-b,  that  is,  the  trial  divisor  with  the  second  term 
in  the  square  root  added  to  it. 

The  steps  in  the  process  are  :  a^-\-2ab-\-b^  \a-\-b 

(1)  The    square    root    of    a^    is   a.     The  a^ 

square  of  a  is  subtracted  from  the  expression  ~~ 

leaving  2a6  +  6^.  _2a  + 6  |  +  2«6  +  6^ 

(2)  The   trial   divisor   for   obtaining   the  2ab-{b^ 
second    term    in    the    square   root    is    2a. 

When  2a  is  divided  into  2ab  the  quotient  is  b,  the  second  term  in  the 
root. 

(3)  The  com.plete  divisor  is  2a-\-b,  and  when  this  is  multiplied  by  b 


218  ALGEBRA 

and  the  product  subtracted  from  2o6  +  6-  there  is  no  remainder.  The 
square  root  is  then  a  +  b. 

It  might  be  thought  that  step  (3)  is  imnecessary,  as  the  root  has 
already  been  found  in  (I)  and  (2).  It  is  unnecessary  if  we  take  for 
granted  that  the  expression  is  a  perfect  square. 

If  you  attempt  to  find  the  square  root  of  a'^-\-2ab-\-4b^  and  do  not 
go  beyond  steps  (1)  and  (2),  you  would  get  the  result  a+b,  as  before. 
This,  however,  is  not  the  correct  result.     Why  ? 

We  can  now  extend  the  method  to  find  the  square  root  of 
a  quantity  of  more  than  three  terms. 


9x«- 12.^3+ 10a;2-4a;+ 1 1  3a 

9x* 

;2-2a;+l 

3x^-2x \  -  12x3  +  10x2- 4a;  + 1 
-12a;'+   ix- 

6a;2-4a:+l  1  6a;2-4.-r+  1 
Qx^-ix+l 

After  finding  the  first  two  terms  in  the  root,  as  in  the  previous 
example,  the  3a;2  — 2a;  is  treated  as  a  single  quantity  and  the  second 
trial  divisor  is  twice  Sx^—2x  or  Qx^  —  'ix 

The  square  root  is  Sx^—2x-\-\. 

150.  Verifying  Square  Root.  We  might  verifj^  the  result 
in  the  preceding  example  by  Avriting  down  the  square  of 
3x2— 2.'r+l.     Verify  in  this  way. 

A  simple  method  of  checking  is  to  substitute  a  particular 
number  for  x. 

When  a;=l,  9.i;^- 12.'(;3+ 10a;2-4a;+ 1  =  9- 12+ 10-4+ 1  =  4, 
and  3a;2-2a;+ 1  =  3-2+1  =2. 

Since  the  square  root  of  4  is  2,  we  presume  the  work  is  correct. 


EXERCISE    105 

Find  the  square  root,  by  the  formal  method,  and  verify  the  results : 
1.     a;2+12a;+36.  2.     Oa^-Ga  +  l. 

S.     9.^2+24x2/+ 16?/2.  4.     25x^-10xy+y\ 

5.     l-18ab+8la%^.  G.     49a*-28a^~b^+4:b*. 

7.     a4+2a3-3a2-4a+4.  8.     'ix*+^x^+5x^+2x+l. 


EXTRACTION  OF  ROOTS  219 

9.     x«-6x3  +  17a;2-24.r+16.  10.     9o4-12a''+34a2_20a-}-25. 

11.     a^-4a36+6a262_4a63+64.        12.     a*-4a3-f  8a+4. 

13.  9a4+12a36+34a262^20a63-f-256^ 

14.  x«-4x5+6a;3+8a;2+4a;+l. 

15.  a:*— 2x3+2x2— X-+1 
a*      4a3      2a-      4a 

17.     a--4a6H-6ac-f462-i26c+9c2. 
18.*  Simplify  a(a+l)(a+2)(a+3)-f-l,  and  find  its  square  root. 

19.  By  extracting  the  square  root  of  x*+4x'+6x2+3x+7,  find  a 
value  of  X  which  will  make  it  a  perfect  square.   (Verify  by  substitution.) 

20.  If  the  square  root  of  .r*— 8x3+30x2-56.r+49  be  x^-\-mx-\-l, 
what  is  the  value  of  m  ? 

21.  Using  factors,  find  the  square  root  of 

(x2+3x+2)(.r2+5x+6)(x2+4x+3). 

22.  Find  the  first  three  terms  in  the  square  root  of  1— 2x— 3x-  and 
of  4-12x. 

23.  When  x=  10,  the  number  44,944  may  be  written 

4xH4x3+9x2+4x+4. 

Find  the  square  root  of  the  latter  and  thus  deduce  the  square  root  of 
44,944. 

151.  In  algebra,  an  expression  of  which  the  square  root  is 
required  is  usually  a  perfect  square.  When  such  is  the  case 
the  formal  method  may  be  greatly  abbreviated. 

Ex.  1. — Find  the  square  root  of 

a;4_4a;3_|_  io.x2_  12.r+9. 

The  first  term  is  x*  and  the  last  is  +  3  or  —  3. 

The  trial  divisor  for  obtaining  the  second  term  of  the  root  is  2x*, 
therefore  the  second  term  is  —  4x'-h-2x^  or  —  2x. 
.•.  the  square  root  is  x*— 2x+3  or  x*— 2a;— 3. 


220  ALGEBRA 

If  we  square  x^  —  2x-\-S,  the  term  containing  x  will  be  twice  the 
product  of  —2x  and  3  or  —12a;.  If  we  square  a;^— 2a;— 3,  the  term 
containing  x  will  be  +  1 2a;. 

We  thus  see  that  if  the  expression  is  a  perfect  square,  the  square 
root  is  x^  —  2x-\-3. 

Check  this  by  putting  x=  1. 

Ex.  2. — Find  the  square  root  of 

4a;''+20x3+13a;2_30a;+9. 

What  is  the  first  term  in  the  square  root  ?  What  is  the  trial  divisor  ? 
What 'is  the  second  term  in  the  root  ?  What  may  the  last  term  be  ? 
What  is  the  square  root  ?     (Verify  your  answer.) 

Ex.  3. — Find  the  square  root  of 

-3a3  +  -25^^4_5^^6  7^2, 

Write  the  expression  in  descending  powers  of  a. 

a«-3a3-|-«|o2-5a+V-- 

The  first  term  in  the  root  is  a~.  The  trial  divisor  is  2a^,  therefore 
the  second  term  is  —  3a^-H2a-  or  —  #a. 

.'.  the  root  is  a^— la  +  f  or  a^  — fa  — |. 

Which  is  it  ?     (Verify  by  squaring.) 

Ex.  4. — Find  the  square  root  of 

4       4 

4a2_4a  +  9 h-^' 

a      a'- 

Here  the  terms  are  already  arranged  in  descending  powers  of  a, 
the  term  +9  coming  between  a  and  -. 

The  first  term  in  the  root  is  2a,  the  second  is  —  4a -^  4a  or  —  1,  and  the 

2 

last  is  +  -. 

a 

Complete  and  verify. 

It  will  be  recognized  that  it  is  only  in  the  most  complicated 
cases  that  it  is  necessary  to  use  the  formal  method  in  full. 
It  is  advisable  to  use  the  contracted  method  whenever 
possible. 


EXTRACTION   OF   ROOTS  221 

E3XBRCISB  106 

Find  the  square  root,  using  any  method  you  prefer.  Verify  the 
results. 

1.  a;*+2a;3— a;2— 2x-+l.  2.  x'^—Ax^+Qx-—'^x+\. 

3.  a*— 6a3+5a2+i2a+4.  4.  a;*+8a;3+12a;2— 16a;+4. 

5.  9a*— 6o3+13a2_4a+4.  6.  x'^+&xhj+lx"y-—&xif-iry\ 

7.  4x^+20x3— 3x2— 70a;+49.  §.  l-10x+27x-2-10.i-3+x«. 

9.  67x2+49+9x4— 70a.--30.i-3.  ^q.  ai2_8^9_(_i8^6_8(^3_f_i^ 

11.  x*+2x3-x+i.  12.  a-*-2x3+fx2— Jx+Jg-. 

13.     c*-6a2+ll --^ -f  1.        14.     l^*_^^_^  +  ?^+l. 
a-      a*  ?/*        2/^        y^        y 

15.    --3x3+a^x2-2x+i  16.     ^*  +  —  +  21^' +  ^+1. 

4  ^  -^  ^'^  25  ^   5   ^   45   ^  3  ^ 

-,r-      e   I     4      4      o     1     •>  -fo      ^''      2x  ,   „       2w  ,   w2 

17.     5H —  2x+x2.  18. +  3 --\-^. 

x^     X  y^       y  X       x2 

19.*  (a;+2/)4-4(x+?/)3+6(x+2/)2-4(x+y)  +  l. 

20.  x2(x-5a)(x-a)+a2(3x-a)2_3a2.r2. 

21.  (a-6)2{(a-6)2-2(a2+62)j.+2(a4+6*). 

22.  (a+6)*-2(a2+62)(a+5)2_|_2(a4^54), 

24.  If  x*+6x^+7x2+ax+l  is  a  perfect  square,  what  is  the  value 
of  a? 

25.  If  the  sum  of  the  squares  of  any  two  consecutive  integers  be 
added  to  the  square  of  their  product,  prove  that  the  result  will  be  a 
square. 

26.  If  4x*+12x3y+A:x2y2_j_6_^^3_j_y4  jg  J^  perfect  square,  find  k. 

27 .  Ji  m^x and  n=  y  —  - ,  show  that 

X  ■        y 


mn+ V(m2+4)(n2+4)=2x«/+  — • 

xy 

28.     Find  the  square  root  of  4x*+8x='+8x2+4x+l.     Check  when 
x=10. 


222  ALGEBRA 

152.  Cube  of  a  Monomial.  When  three  equal  factors  are 
multipUed  together,  the  product  is  called  the  cube  of  each  of 
the  factors. 

Thus,  the  cube  of  2a  or  {2a)^  =  2a.2a.  2a=8a'^, 
the  cube  of  a^  or  {a^)^  =a^  .  a-  .  a^  =  a^, 

and  the  cube  of  3a»  or  (3a3)3      =  Sa^  .  Sa^  .  ^a^  =  21a^. 

The  cube  of  a  monomial  is  found  by  writing  down  the  cube  of 
each  factor  of  it. 

Thus,  the  cube  of  5ab^x  is  125a*6^x*. 

153.  Cube    of    a    Binomial.      Fmd    the       a^+2ab+b'' 
cube  of  a-\-b  by  multiiDlying  its  square  by       "  +^ 


a-\-b.     Find  also  the  cube  of  a—b.  o*+2a26+  afe^ 

{a—b)^=a^—2a^'b+Zab^—b^  a^+3a^b+3ab'~+b^ 

Note  that  m  each  case  the  cube  contains  four  terms,  in 
descending  powers  of  a  and  ascending  powers  of  b,  and  the 
numerical  coefficients  are  1,3,3,  1 . 

The  cube  of  a—b  is  the  same  as  the  cube  of  a-\-b,  except 
that  the  signs  are  alternately  plus  and  minus. 

From  the  forms  of  these  two  cubes,  the  cubes  of  other 
expressions  may  be  written  down. 

Ex.  1.  (x  +  2y)'>  =  x^  +  3x^2y)  +  Sx{2yy~  +  (2y)^, 

=x^  +  6x^y+l2xy^  +  8y^. 
Ex.  2.        (3x-2y)!'  =  (3a;)3-3(3x)2(22/)  +  3(3x)(2!/)2-(2y)3, 

=  21x^-54:X^y  +  3Qxy^-8y^. 


Ex.  3.      {a+b  +  c)^  =  {a  +  b  +  cy, 

=  (a+6)'  +  3(a+6)2c  +  3(a+6)c2  +  c', 
=  a^+3a^b  +  Sab^  +  b^  +  3a^c  +  6abc  +  Sb^c  +  3ac^  +  Sbc^  +  c^, 
=  a^+b^  +  c^+3{a^b+ab'  +  b^c+bc^  +  c'^a+ca-)  +  6abc. 


EXTRACTION   OF   ROOTS  223 

EXERCISE   107  (1-12,  Oral) 


Find  the  cu 

[be  of  : 

1.     -1 

2 

-2a. 

3. 

-3o62. 

4. 

—x^yz^. 

5.     x+y. 

6. 

x—y. 

7. 

m-\-n. 

8. 

p-q. 

9.     x+l. 

10. 

x-\. 

11. 

a2+6. 

12. 

\-a\ 

13.     a;+3. 

14. 

2x—y. 

15. 

2a +  36. 

16. 

I -2a. 

17.     a-46. 

18. 

\-a\ 

19. 

a-\-b—c. 

20. 

a—b—c. 

21.*  Simplify  {a+b)'^+{a-bf  and  {a+bf-ia-b)"^. 

22.  Show  that  {x-{-y)^=x^-\-y^-'r3xy{x-\-y)  and  write  a  similar 
form  for  (x — y)^. 

23.  Simplify  ia+b+c)^+{a+b-cf. 

24.  Show  that  {a-b)^+{b-c)^+{c-af=3{a-b){b-c){c-a). 

25.  Show  that  the  difference  of  the  cubes  of  any  two  consecutive 
integers  is  greater  than  three  times  their  product  by  unity 

26.  When  x=y-\-z,  show  that  x^—y^—z^—Sxyz. 

27.  Two  numbers  differ  by  3.  By  how  much  does  the  difference 
of  their  cubes  exceed  nine  times  their  product  ? 

28.  Three  consecutive  integers  are  multiplied  together  and  the 
middle  integer  is  added  to  the  product.  Show  that  the  result  must 
be  the  cube  of  this  middle  integer.     What  is  the  cube  root  of 

241x242x243+242? 

154.  Cube  Root  of  a  Monomial.  The  cube  root  of  any 
quantity  is  one  of  the  three  equal  factors  which  were 
multiphed  to  produce  that  quantity. 

Thus,  the  cube  root  of  8  is  2,  of  a'  is  a,  of  8x'  is  2x,  of  a*  is  a^,  of 
27a36«  is  3ab\ 

The  cube  root  of  any  power  of  a  letter  is  obtained  by  dividing 
the  index  of  the  power  by  3. 

The  symbol  indicating  cube  root  is  ^"~ 

Thus,  -^^125=5,     ^?^=a»,     VSx^»^2x'y. 


224  ALGEBRA 

155.  Cube  Root  of  a  Compound  Expression. 

The  cube  root  of  a^-{-3a^b-{-3ah"-\-b^  is  «+&, 
and  of  ci^—Sa^b-{-3ab^—b^  is  a— b. 

Therefore,  when  an  expression  of  four  terms  is  known  to  be 
a  perfect  cube,  its  cube  root  can  at  once  be  MTitten  down 
by  finding  the  cube  roots  of  its  first  and  last  terms. 

Ex.  1. — The  cube  root  of  x^  —  6x^y -\- 12xy^  —  8y^  is  x—2y,  since  the 
cube  root  of  x^  is  x  and  of   —  8y^  is  —2y. 

Ex.  2. — The  cube  root  of  a^—2a^b-\-^b^—^%b^  is  evidently  o— §6. 

In  the  cube  of  a-\-b,  the  second  term  is  3a^h.  After  finding 
the  first  term  a  of  the  cube  root,  we  might  have  found  the 
second  term  of  the  root  by  dividing  3a -6  by  Sa^,  that  is,  by 
three  times  the  square  of  the  term  aheady  found. 

Thus,  the  second  term  of  the  cube  root  in  Ex.  1  is 
—  Qx'^y-^Sx^  or  —2y, 
and  in  Ex.  2  is  —2a^b-^3a'  or  — 16. 

Here  three  times  the  square  of  the  first  term  of  the  root  is  the 
trial  divisor,  corresponding  to  twice  the  first  term  in  finding 
the  square  root. 

Ex.  3. — Find  the  cube  root  of 

8x^+l2x^—30x^—35x^+45x^-\-21x-21. 

The  first  term  in  the  root  is  2x^  and  the  last  is  —  3. 
The  trial  divisor  for  finding  the  second  term  of  the  root  is  3(2x")* 
or  12a;*. 

.•.    the  second  term  of  the  root  is  12a;5^12a;*  or  x. 

.'.    the  cube  root  is  2x^-\-x—3. 

It  is  thus  seen  that  it  is  easier  to  find  cube  root  by  inspection 
than  to  find  square  root,  as  in  finding  cube  root  there  is  no 
ambiguity  as  to  the  sign  of  the  last  term  in  the  root. 

156.  Higher  Roots.  Since  {x^)^=x^,  we  may  find  the  fourth 
root  by  taking  the  square  root  and  then  the  square  root  of 
the  result. 


EXTRACTION   OF  ROOTS  225 

Also,  since  {x'^)^=x^  and  {x^)^—x^,  we  can  find  the  sixth 
root  by  taking  the  square  root  of  the  cube  root,  or  the  cube 
root  of  the  square  root. 

Thus,  the  square  root  of  x*  +  8x^  +  24x2-)- 32a;  + 16  is  x'  +  4a;4-4, 
therefore  the  fourth  root  is  x-\-2. 

The  cube  root  of  a;«  — 6x*+ 15a;*  — 20a;3+ ISx^  — 6a;+ 1  is  cc*  — 2a;+l, 
therefore  the  sixth  root  is  x—  1. 


EXERCISE  108  (1-15,  Oral) 

State  the  cube  root  of  : 

3.     -125a%3,        4^     _8(a_ft)3 

6.  x^—Zx'^y-\-^xy'^ — y^. 

8.  8.T='-12.c2+6.r-l. 

10.  64a3- 144^2  + l08a-27. 

12.  21x^—21xhj+^xy--y^. 

16.     ^  -  6.r*  +  '\-2xhf  —  8j/«. 
Ill,      lit-  y^ 

17.*  In  finding  the  cube  root  of  x^+^x^+&x^+lx^+Qx'^-\-Zx-\-\. 
what  is  the  first  term  in  the  root  ?  What  is  the  last  term  ?  What  is 
the  trial  divisor  for  finding  the  second  term  ?  What  is  the  cube  root  ? 
Check  by  substituting  a;=l. 

Find  the  cube  root  and  check  : 

18.     l-6a;+21a;2-44a;H63a;*-54a;5+27a;«. 

.„      x3       a:2      ^  18      27   ,   27 

'"•  27-y+2--^  +  -.-^+^- 

20.  27a«-  108a5  +  171a*-  136a3+57a2-  12a+l. 

21.  (l+3x2)2-a;2(3+a;2)2. 

22.  For  what  value  of  x  will  x^-\'Zcx'^-\-2c^x-\-5c^  be  a  perfect  cube  ? 

23.  Find  the  fourth  root  of  a;*— 4a;3+6a;2— 4a;+l. 

Q 


1. 

-64.             2.     27a3. 

5. 

a:3-f-3.r-+3a;+l. 

7. 

a3+6a24-i2a4-8. 

9. 

a:V+3a;2t/2+3a;y+l. 

11. 

125a;3-75a;2+15a;-l. 

x3      3a;2    ,   3a;      , 

13. 

8         4+2-'- 

15. 

3      „      ,   27       27 

226  ALGEBRA 

24.  Find  the  fourth  root  of  a'^-l2a^+5'ia^-\08a+81. 

25.  Find  the  sixth  root  of 

a;«-12x5  +  60a;*-160a;3+240a;2-192a-+64. 


E3XBRCISE  109  (Review  of  Chapter  XVII) 

Find  the  square  root  of  : 

2.  x^  +  4x^-2x^—l0x^+13x^-6x+l. 

3.  x^'  +  6x'^'>  +  5x»-8x^  + 16x^-8x^  +  4:. 

5.  12a^x-2ea-x^  +  25x*+9a*-20ax^. 

6.  4a;2(7  +  a;2  +  3a)  +  (3«+7)'. 

7.  (a;2  +  5a;+6)(a;2  +  7a;+12)(x2+6a;+8). 

8.  (2x^-x-S)(x^-4x-5)(2x--13x-ir  15). 

9.  4x4  _  20a;3  +  33^2  _  32a;  +  34  -  ^^  +  -^ . 

a;       a;^ 

What  is  the  cube  root  of  : 

10.  27- 1 35a; +  225a;2- 125x3. 

11.  8a;8-12x5+18a;4-13a;3  +  9a;2-3a;+l. 

12.  (a-6)3  +  36(a-6)2  +  362(a-6)  +  63. 

13.  Find  to  three  terms,  the  square  roots  of  : 

l-2a;,    l-a,   4  +  x. 

14.  Find  the  value  of  y  for  which  x'^  —  2{a  —  y)x+y^  is  a  complete 
square  and  prove  by  trial  that  your  result  is  correct. 

15.  The  first  two  terms  of  a  perfect  square  are  49x*  — 28x',  and  the 
last  two  are  +6a;+|.     What  must  the  square  root  be  ? 

16.  Prove    that    the    product    of    any    four    consecutive    integers 
increased  by  unity  is  a  perfect  square. 

17.  Find  the  square  root  of  a*  +  4a^  +  6a^-\-4a-\-\  and  deduce  the 
square  root  of  14,641. 

18.  By  finding  the  cube  root,  simplify 

{a  +  b)^  +  ^a  +  b)^a-b)  +  ^a  +  b){a-b)2  +  {a-b)^ 

19.  Tf  a  =  6+l,  show  that  a»- 63-1  =  3a;;. 


EXTRACTION  OF  ROOTS  227 

20.  Show  that  the  product  of  any  four  consecutive  even  integers 
increased  by  16  is  a  perfect  square.  How  might  the  result  be  deduced 
from  No.  16  ? 

21.  By  inspection,  find  the  values  of 

(a-6)2+(6-c)2  +  (c-a)2  +  2(a-6)(6-c)  +  2(6-c)(c-a)  +  2(c-a)(a-6), 
(2x-y)^~3{2x-yn2x+y)  +  3{2x-ij){2x+yy--(2x  +  y)\ 

22.  To  the  square  of  the  double  product  of  any  two  consecutive 
integers,  add  the  square  of  their  sum.  Prove  that  the  result  is  always 
a  perfect  square. 

23.  Express  in  symbols  :  The  difference  of  the  cubes  of  any  two 
numbers  exceeds  the  cube  of  their  difference  by  three  times  their 
product  multiplied  by  their  difference.     Prove  that  this  is  true. 

24.  The  expression 

8x9-36a;8  +  66x'-87x6+105x5-87a;*  +  61x3-42x2+12j;-8 
is  a  perfect  cube.     Find  its  cube  root  by  getting  two  terms  from  the 
first  two  terms  of  the  expression  and  the  other  two  from  the  last  two 
terms.     Check  when  x=l. 

25.  What  number  must  be  added  to  the  product  of  any  four 
consecutive  odd  integers  so  that  the  sum  may  be  a  perfect  square  ? 

26.  Show  that  the  sum  of  the  cubes  of  three  consecutive  integers 
exceeds  three  times  their  product  by  nine  times  the  middle  integer. 

27.  Find  the  cube  root  of 

(4a;-l)3  +  (2a;-3)3  +  6(4a;-l)(2a;-3)(3a;-2). 
[Note  that  it  is  of  the  form  a^-\-b^+'3ab{a  +  b).] 

28.  If  4cc*+ 12x^4- 5x*  —  2x*  are  the  first  four  terms  of  an  exact 
square,  find  the  remaining  three  terms. 


Q  2 


CHAPTER  XVIII 


QUADRATIC  SURDS 

157.  Surd.  When  the  root  of  a  number  cannot  be  exactly 
found,  that  root  is  called  a  surd. 

Thus,  we  cannot  find  exactly  the  number  whose  square  is 
equal  to  2,  and  we  represent  the  number  by  the  symbol  V2 
and  we  call  V'2  a  surd. 

If  no  surd  appears  in  any  quantity,  it  is  called  a  rational 
quantity. 

By  the  arithmetical  process  of  extracting  the  square  root  of  2,  we 
can  obtain  the  value  of  '\/2  to  as  many  decimal  places  as  we  please, 
but  its  exact  value  can  not  be  found. 

To  four  decimal  places  the  value  of  V'2  is  1-4142.     Find  the  square 
of  1-4142  by  multiplication  and  see  how  closely  it  approximates  to  2. 
We  can  find  geometrically  a  line  whose  length  is  V2  units.     In  this 
square,  whose  side  is  1  imit,  draw  the  diagonal  BD. 
Then,  from  geometry,  we  know  that 

:.      BZ)2=12+12  =  2, 

.-.    BD  =V2. 

On  squared  paper  mark  the  corners  of  a  square 
whose  side  is  10  units.  Measure  the  diagonal  and 
thus  estimate  as  closely  as  you  can  the  value  of  V  2. 

Make  a  diagram  like  this  to  show  how  to  represent 
graphically  lines  whose  lengths  are  V2,  Vs,  V  4,  V5, 
etc.  Take  the  unit  line  1  inch  in  length.  What  test 
have  you  of  the  accuracy  of  your  drawing  ? 

158.  Quadratic  Surd.  A  surd  like  V2  in  which  the  square 
root  is  to  be  found  is  called  a  quadratic  surd.  In  this  Chapter 
quadratic  surds  only  are  considered. 

22S 


QUADRATIC  SURDS  229 

159.  Multiplication  of  Simple  Surds. 

Since  V  2  represents  a  quantity  whose  square  is  2, 

:.   V2xV2=2=V4:, 
also  VlxVd^VSQ,  because  2x3=6. 

Similarly,  we  might  expect  that  V2xV3=VQ.     That  this 
is  true  may  be  shown  by  finding  the  square  of  y/2x  Vs. 
('\/2x  v'3)2=a/2x  V3x  V2x  V3,  [Just  as  {ab)^^a  .b  .a.b.] 
=  V2xV2xV3xVS, 
=2x3=6. 
.'.     V2xV3=V6. 
Similarly,  VSxV5=VT5, 

and  Va  x  Vb—  Vab. 

Therefore,  the  product  of  the  square  roots  of  two  numbers  is 
equal  to  the  square  root  of  the  product  of  the  numbers. 

Since    Vab=VaxVb,      .\   \/l2  =  \/4x  V3=2a/3, 
and 

V56=  V25  X  V2=o  V2  ;    VlSa^=  VWa-  x  V2a=3aV2a. 

Thus,  we  see  that  if  there  is  a  square  factor  under  the  radical 
sign,  that  factor  may  be  removed  if  its  square  root  be  taken. 

Conversely,  5^3=  V25  xV3=Vl5, 

aVb=  Va^  X  V^=  Va^, 
axVmy  =  Va^x^  X  Vmy  =  Va^x'^my. 

160.  Mixed  and  Entire  Surds.  When  a  surd  quantity  is 
the  product  of  a  rational  quantity  and  a  surd,  it  is  called  a 
mixed  surd.  If  there  is  no  rational  factor  it  is  called  an 
entire  surd. 

Thus,  oVS,  aVb,  {a  —  b)Vx  —  y  are  mixed  surds,  and  VS,  V50, 
Vax  +  b  are  entire  surds. 

In  the  preceding  article  we  have  shown  that  a  mixed  surd 
can  always  be  changed  into  an  entire  surd,  and  an  entire 
surd  can  sometimes  be  changed  into  a  mixed  surd. 


230  ALGEBRA 

A  surd  is  said  to  be  in  its  simplest  form  when  the  quantity 
under  the  radical  sign  is  integral  and  contains  no  square 
factor. 

Thus,  the  simplest  form  of  VSO  is  5V2. 


EXERCISE   110  (1  29.  Oral) 

Find  the  product  of  : 

1.     V2,  Vs.                   2.     V5,  Vs.                   3. 

V2,  Vs. 

4.     sVl,  2\/7.              5.     VS,  VS,  V2.         6. 

Vs,  VI 

7.     Va,  Vg,  Vf.  8.     (Vabc)K 

Express  as  entire  surds  : 
9.     2V3.  10.     3v/2.  11.     W5. 

12.     aVb.  13.     SaVl.  14.     b  ./-. 

^    b 


/^ 


-6 


16.  SVa-b.  16.     (a+6)  /v/ — rr  • 
Simplify,  by  removing  the  square  factor  : 

17.  Vs.  18.     V12.  19.     V2O.  20.     V75. 
21.     V27.           22.     V56.               23.     Vl62.  24.     V2a^ 

25.     VIOOO?^.    26.     |V32.  27.     V{a-bf.      28.     -Va^b. 


a 

29.  Solve  x^=2  ;    3^2=27  ;    ^x-=9. 

30.  Show  by  squaring  that 

V3xV7  =  V2T  and  VaxVbxVc^Vabc. 

31.  Show  that  V8=2V2,    by  extracting    the   square  roots  of    8 
and  2  to  three  decimal  places. 

32.*  Describe  a  right-angled  triangle  whose  sides  are  2  inches  and 
3  inches.     Express  the  length  of  the  hypotenuse  as  a  surd. 

33.     By  using  a  right-angled  triangle,  how  could  you    find  a  line 
whose  length  is  VlO  inches  ? 


QUADRATIC  SURDS  231 

34.  If  the  area  of  a  circle  is  66  square  inches,  find  the  length  of  the 
radius  (7r  =  31). 

35.  The  sum  of  the  squares  of  two  surds,  one  of  which  is  double 
the  other,  is  40.     Find  the  surds. 

36.  The  length  of  the  diagonal  of  a  square  is  10  inches.  Find 
the  length  of  the  side. 

37.  One  side  of  a  rectangle  is  three  times  the  other  and  the  area  is 
96  square  inches.     Find  the  sides. 

161.  Like  Surds.  In  the  surd  quantity  SVS,  5  is  a 
rational  factor  and  VS  is  called  a  surd  factor. 

When  surds,  in  their  simplest  form,  have  the  same  surd 
factor,  they  are  called  like  surds  or  similar  surds,  otherwise 
they  are  unlike  surds. 

Thus,  3V'2,  5\/2,  aV2  are  like  surds. 

I  „  _  _ 

and  2V3,  3V2,  W5  are  tmlike  surds. 

162.  Addition  and  Subtraction  of  Like  Surds.  Like  surds 
may  be  added  or  subtracted,  the  result  being  expressed  in 
the  form  of  a  surd. 

Thvis,  3^/2  +  5^2  =  8^2,  just  as  3a+5a=8a. 

7^/3-4^3  =  3^3,  just  as  7a-4a  =  3«. 
a/75- 2V3  =  5\/3- 2^3  =  3^3. 

V50  +  \/32-V'l8  =  5\/2  +  4V2-3V2  =  6\/2. 

The  sum  or  difference  of  unlike  surds  can  only  be  indicated. 

Thus,  V2+\/3  can  not  be  combined  into  a  single  surd,  but  the 
approximate  values  of  V'2  and  Vs  may  be  found  and  added. 

Show  that  V2+V3=V5  is  not  true,  by  finding  the  square  roots 
of  2,  3  and  5  each  to  two  decimals. 

Is  it  true  that  V 4  +  Vo  =  V 13  ? 

EXERCISE  111  (1-8,  Oral) 
Express  as  a  single  surd  : 
1.     3V2+5\/2.  2.     5\/7-3a/7.  3.     2Va+3Va. 

4.     2Vx+5Vx—Vx.         5.     \/8-|-V2.  G.     \/l2+\/3. 


232  ALGEBRA 

7.  Vis- Vs.  8. 

9.*  \/75+Vl2+3\/3.  10. 

11.  V45-V20+V80.  12. 

13.  4\/r28+4V50-5\/l62.         14. 
15. 
16. 


2\/18+3-s/8-5a/2. 
2V63-5\/28+a/7. 
10\/44-4a/99. 


V45-\/20+V'80. 

4  a/ 128 + 4  V50  -  5  V 162. 

Vis + V'20  -  VSO  +  V 180. 

Vl2 + ^98  -  V 1 28  -  \/32  -  VSO. 

Simplify  the  following  and  find  their  numerical  values,  correct  to 
two  decimal  places,  using  the  square  root  table  : 

17.     V75.  18.     -\/63.  19.     V60+a/15. 

20.     VU'l-2Vl2.     21.     Vl28-Vl62.      22.     V56+Vf2+V9b. 

Solve,  finding  x  to  three  decimal  places  : 

23.     a:2^37.  24.     3x^-+5=50.  25.  Ja;^— 4=19. 

26.     31x2=132.  27.     i(3x2-ll)=53.    28.  Jx2=^x2-47. 

29.     The  area  of  a  circle  is  176  square  inches.  Find  its  radius. 


Square 

Roots 

OF  Numbers 

FROM  1  TO 

50. 

n 

V'n 

n 

Vn 

n 

Vn 

n 

v^ 

n 

v/^ 

1 

1-0000 

11 

3-3166 

21 

4-5826 

31 

5-5678 

41 

6-4031 

2 

1-4142 

12 

3-4641 

22 

4-6904 

32 

5-6569 

42 

6-4807 

3 

1-7321 

13  3-6056 

23 

4-7958 

33 

5-7446 

43 

6-5574 

4 

2-0000 

14  3-7417 

24 

4-8990 

34 

5-8310 

44 

6-6332 

5 

2-2361 

15  3-8730 

25 

5-0000 

35 

5-9161 

45 

6-7082 

0 

2-4495 

16  i  4-0000 

26 

5-0990 

36 

6-0000 

46 

6-7823 

7 

2-64.58 

17  4-1231 

27 

5-1962 

37 

6-0828 

47 

6-8557 

8 

2-8284 

18  4-2426 

28 

5-2915 

38 

6-1644 

48 

6-9282 

9 

3-0000 

19  4-3589 

29 

5-3852 

39 

6-2450 

49 

7-0000 

10 

31623 

20  4-4721 

30 

5-4772 

40 

6-3246 

50 

7-0711 

163.     Multiplication  of  Surds. 

3\/2  X  4\/3=3  X  V2  X  4  X  V3, 

=3x4x-\/2x\/3, 
=  12\/6. 


QUADRATIC  SURDS  233 

It  is  thus  seen  that  the  product  of  two  surds  is  found  by 
multiplying  the  product  of  the  rational  factors  by  the  product  of 
the  surd  factors. 

5\/3x2V3=10.3=30, 

also  aVc  X  b  Vc = abc. 

It,  therefore,  follows  that  the  product  of  two  like  surds  is 
always  a  rational  quantity. 

Ex.  1.— Multiply  VEb  by  VtS. 

Here  the  siirds  should  be  simplified  before  multiplying. 
Since  a/50  =  5V2  and  \/75=5V'3, 

.-.   V50xVlB=5V2x5Vl=25V6. 

Ex.  2.— Multiply  2  +  2\/3 

2+2 V3  by  3-\/2.  ^~  ^^ 


Here  the  multiplication  is  performed  in  a  6  +  6v3 

manner  similar  to  the  multiplication  ofa  +  6  — 2v2  —  2v6 

by  x+y.  ~  ~  ~ 

6+6\/3-2\/2-2\/6. 

164.     Conjugate  Surds.     If  we  wish  to  multiply 

5\/3  +  2\/2  by  5\/3-2\/2, 

we  may  follow  the  same  method  as  in  the  preceding  example. 
These  expressions,  however,  are  seen  to  be  of  the  same  form  as 
a-\-b  and  a—b, 

:.     (5\/3+2V'2)(o\/3-2\/2)  =  (5\/3)2-(2V2)2=75-8=67. 

Similarly,  (3+ \/2)(3- V2)  =  9-2  =  7, 

and  (2-Vl0)(2  +  Vl0)  =  4-10=-6. 

Such  surd  quantities  as  these  which  differ  only  in  the  sign 
which  connects  their  terms  are  called  conjugate  surds. 

Note  that  the  product  of  two  conjugate  surds  is  always  a 
rational  quantity. 


234  ALGEBRA 

EXERCISE  112  (1-12,  Oral) 

Find  the  product  of  : 

1.  2\/3,  3\/5.  2.     5\/2,  WS.  3.     aVb,  bVa. 

4.  3\/2,  \/3,  Vs.        5.     (2\/3)2,  (\/2)2.       6.     V2+1,  V2. 

7.  VS  +  a/S,  ^2.  8.     Va+Vft— 1,  Vc. 

9.  a/3+V'2,  ^3-^2.  10.     VrO-3,  VIO+3. 

11.  Vx—Vy,  Vx+Vy.  12.     2\/2+\/.3,  2V2--V'3. 

13.*  3\/6,  4V'2.  14.     3\/2,  4\/7,  J\/2. 

15.  (v'3+\/2)2.  ^Q      {2V5-Vl)^- 

17.  (3\/2+2V3)2.  18.     (Va-f\/6)2. 

19.  4+3 \/2,  5-3\/2.  20.     3a/2+2a/3,  5\/2-3\/3. 

21.  3^5-4^2,  2V5+3\/2.        22.     3\/a-2v'6,  2\/a-3'\/6 

23.  V'5+V'3  +  V'2,  ^5+^3-^2. 

24.  \/7  +  2a/2-V3,  V7-2\/2+\/3. 

25.  Va+6— 3,  •\/a+6+2.  26.     Va+Va—1,  Va—Va—l. 
27.  (\/3+V2+l)2.  28.     (V5+2V2-V3)'- 

29.  (V^6+\/a'^)2.  30.     (3\/.^^-2\/^+^)^- 

Simplify : 

31.  (6-2\/3)(6+2\/3)-(5-a/2)(5+\/2). 

32.  (v/3-\/2+1)2+(v'3  +  a/2-1)2. 

33.  (\/50-\/l8+\/72+\/32)xiV3. 

34.  2(4V3+3V'2)(3\/3-2V2)  +  (5\/2-3\/3)(4V24-2V'3). 

35.  (\/3+V2)(2V3-\/2)(\/3-2\/2)(V3-3\/2). 

36.  By  squaring  VlO+VS  and  VS+Vl,  find  which  is  the  greater. 

37.  The  product  of  5^3+3^7  and  SVS—Vl  lies  between  what 
two  consecutive  integers  ? 

38 .  Find  the  area  of  a  rectangle  whose  sides  are  5 + V2  and  10— 2^2 
inches. 


QUADRATIC  SURDS  235 

39.  The  sides  of  a  right-angled  triangle  are   7+4\/2  and  7  —  4^2 
inches.     Find  the  hypotenuse. 

40.  The  base  of  a  triangle  is  2\/3+3\/2  inches  and  the  altitude 
is  3v'3+2\/2  inches.     Find  the  area  to  two  decimal  places. 

165.     Division  of  Surds. 

Since     VaxVb=Vab,   :.   Vab-^Va^J^^Vb. 

y  a 

„.    .,    ,  /-       /r     Va        la 

Similarly,  ■\/a^vo=       =  a /  -  , 

Vb      ^  b 

and  3Vl5^2V'5=|V3. 

Ex.  1. — Find  the  numerical  value  of  V5-^\/2  or  — -_• 

V2 

(1)  We  might  find  the  square  roots  of  5  and  2  and  perform  the 
required  division. 

V'5-:--\/2  =  2-236h- 1-414=  1-581 

(2)  V5^\/2=V'f=V2^=  1-581. 

,     VB       VBxV'2       ViO      3162      ,  _-, 
yo)     —j^=—^ -^  = -— —  =  — - — =1-581. 

V2       \/2xa/2  2  2 

Here  the  third  method  is  at  once  seen  to  be  simpler  than 
either  of  the  others. 

V5  VlO 

In  (3)  we  changed    — -   into   ,  that  is,  we  made  the 

^       V2  2 

denominator  a  rational   quantity.     This  operation  is  called 

rationalizing  the  denominator. 

Ex.  2.— Find  the  value  of    V>  if  V2=  1-4142. 

V2 

Here,    instead    of  dividing    1    by     1-4142,    we   first  rationalize    the 
denominator. 

T>,«r,  1  lxV2         V2       1-4142 

Ihen  -^^  =  —— ~  =  -— -  =  — —  =  -7071. 

V2       V2xV2         2  2 

Ex.  3.— Divide  QVs  by  10\/27. 
6V8  6x2V'2        2a/2      2xV2xV3      2V6      2x2-4495 


IOV27       10x3\/3       5V3      5xV3xV3        15  15 


=  -3266. 


236 


ALGEBRA 


Ex.  4. — Rationalize  the  denominator  of 


2+\/5 


3+V5 

We  have  already  seen  that  the  denominator  will  be  rational  if  we 
multiply  it  by  its  conjugate  3— V5. 

2+V5  _  (2+V5){3-V5)  _  l  +  \/5  ^  I  +  a/5 
3+V'S  ~  (3  +  V'5)(3-V5)  ~    9-5  4 

Ex.  5.— Divide  5+2^3  by  7-4\/3. 

5  +  2^^3 

Write  the  quotient  in  the  fractional  form     j^t  rationalize  the 

^  7-4V3 

denominator  and  simplify. 


Divide  : 
1.     3^/27^  Vs. 

3.     V72^3\/8. 

5.     A/l8  +  \/i2byV3. 

Rationalize  the  denominator  of  : 


EXERCISES  113  (1-12,  Oral) 

2.      Vi2^\/3. 


10. 


13.* 


16. 


2 
V3 

a 

Vb 


i-Ws 

Va 
Va+Vb 


8. 


11. 


14. 


17. 


4.     VaFc- 

^Va. 

6.     V 

a6+\/ac 

by  Va. 

of: 

10 

V5 

9. 

V5 

3\/5 

12. 

1 

V6 

V2-1 

12 

3\/2-2a/3 

15. 

V3+\/2 
V3-V2 

5\/3-3\/5 

18. 

V7-\-V2 

Vs-a/s 

9+2  Vl4 

Find  the  value  to  three  decimal  places,  using  the  table  : 

1  15  2\/3 

— ^  •  '*{)      — =  •  21  ^     . 

a/3  "   '     Vis  ■     3V2 


19 


22. 


25. 


V3+\/2 
V'3^\/2. 


23. 


26. 


17 


3\/7+2\/3 
2^63-^3^35. 


24. 


Vi-Vs 

Vt+Vs' 

l^(7+4\/3). 


QUADRATIC  SURDS  237 

Solve,  giving  the  value  of  x  to  two  decimal  places,  using  the  table  : 
28.     a;V2=3.  29.     xV3=V2.  30.     xV3=V2+l. 

31.     a;\/3-.rV'2=l.  32.     a;\/5-5=2a;- v/5. 

33.  x%V3-l)=2{VS+l). 

34.  The  area  of  a  triangle  is  2   square    feet.      The    altitude    is 
Vs+VS  feet.     Find  the  base  to  three  decimals. 

35.  Simplify  2+VlO 


4\/2+-v/20-V'l8-\/5 

166.  Surd  Equations.  A  surd  equation  is  one  in  which 
the  unknown  quantity  is  found  under  the  root  sign,  in  one  or 
more  of  the  terms. 

Thus,  Vx+7  =  4c,  Vx-\-Vx—5  =  5,  are  surd  equations. 

Ex.  1.— Solve  Vx^=2. 

Square  both   sides,  a;— 3  =  4, 

x=7. 
Verification:  Vx—'3=Vl  —  S=Vi  =  2. 

Ex.  2.— Solve  V5x- 1  - 2 Vx+S = 0. 

Transpose  2\/.t  +  3,  VSa;— l  =  2\/a;  +  3. 

Squaring,  5a;  —  1  =  4a;  + 1 2, 

.-.     a;=13. 
Verification  :      V5^^^-2a/x  +  3=  \/64-2\/l6  =  8-8  =  0. 

Note  that  in  verifying  we  have  taken  the  positive  square  root  only, 
as  defined  in  art.  63. 

BXE3RCISB   114  (1-8,  Oral) 
Solve  and  verify  : 

1.  2Vx=6.  2.     Vx-5=4.  3.     6-^^=1. 

4.  a/x+2=4.  5.     Wx=V2b.  6.     Vx-b=a. 

7.  m+Vx=n.  8.     1  —  Vx^=3. 

9.  Vx^+9=9—x.  10.     Vx^^Ux+3=x+5. 

11.  V9x^-nx^5=3x-2.  12.     2a;-V'4a;2_i0a;-t-4=4. 

13.  2a+Vx+a^^b+a.  14.   V{x—a)^+2ab+b^^x—a-\-b. 


238  ALGEBRA 

EXERCISE  115  (Review  of  Chapter  XVIII) 

Simplify  : 

1.*  Vs+VIS+VgS.  2.  V 500 +V 80-^20. 

3.  5\/3  +  3V'27-\/"i8.  4.  (4\/5+ \/T8)(4\/5- VlS). 

5.  (6\/6-5)(6\/6+5).  6.  (V'6+ V2  +  2)(  V6- V2-2), 

7.  (a/8+ ^2-2)2.  8.  (V'3-2\/2-l)2. 

9.  5\/27^6\/75.  10.  (\/5-2)^(  V5  +  2). 

11.  (\/l25+V'i5)-^V'320.  12.  (5+ \/3)(5- \/3)-f-(A/l3- \/2). 

13.  Multiply  3\/8  +  2a/3-\/2  by  2V8- \/3  +  4"v/2. 

.  14.  By  how  much  does  the  square  of  \/3  H — ■j=  exceed  the  square 

of  V2  +  4-  ? 

V2 

15.  Show   by   multiplication   that   the   value   of    Vs   lies   between 
1-732  and  1-733.      Which  of  these  is  the  closer  approximation  to  V3  ? 

16.  Which  is  the  greater,  Vu+Vs  or  y/ \2 -\- \/ \b  "i 

17.  The  product  of  3\/2  — 2\/3  and  2^3— V2  lies  between  what 
two  consecutive  integers  ? 

18.  Rationalize  the  denominators  of  : 
4        3\/2      3      /5        /s-^        2^6-2 


A.      3V2      3     /5         .3:5        2 
a/2'    2V3'   2V   6      "^       '     Q^ 


V2      2V3'   2V  6  '     3\/3  +  V'2 
Solve  and  verify  : 

19.     ^^+3  =  4.  20.  V'ix^^=2\/x-2. 

21.     Vx^^5+l=x.  22.  V2x+'l  =  ^Vx. 

23.     Vx235a;+ll  =  x+2.  24.  Vx^-2=l-x. 

25.  Using   the  table,  solve:    a;2=75,   a;2=63,    Ja;2  =  49,   x\/3=V5, 

.-cV'2  +  l  =  V'3. 

26.  Find  to  three  decimal  places  the  values  of  : 

2  1  Jl_        2ViO-V5      3\/2-2 

vl'  \7l'  vl+r  vro+V5'  4V2+1' 


QUADRATIC  SURDS  239 

27.  Find  the  value  of 

(2V'2+ \/3)(3\/2- V'3)(3\/3- V2). 

28.  If  the  sides  of  a  right-angled  triangle  are  VS+l  and   VS— 1, 
what  is  the  length  of  the  hjrpotenuse  ? 

OQ      «•       vf     ^5-1       V5-3         .    V3+V2       \/3-V2 

29.  Simplify    —7= y= and     -^= 7= 7= ;;::  . 

V5-2       •\/5  +  3  \/3-V2       V3+V2 

30.  Find  the  value  to  two  decimal  places  of 

^±^  +  ^^,  whenx=2+\/3,  2/  =  2-\/3. 

31.  Multiply  2V30-3V'5  +  5V3  by  \/3  +  2V2-\/5. 

32.  Multiply  V7'+2V6  by  Vt-WB. 

33.  The  area  of  a  rectangle  is  16\/l()  — 25  and  one  side  is  sVo — ^2. 
Find  the  other  side  to  two  decimal  places. 


CHAPTER  XIX 
QUADRATIC  EQUATIONS 

167.  A  quadratic  equation  has  already  been  defined  in 
art.  104.  In  the  same  article  we  considered  the  method  of 
solving  some  of  the  simpler  forms  of  it. 

Quadratic  equations  frequently  occur  in  the  solution  of 
problems  as  shown  in  the  following  examples. 

Ex.  1. — Find  two  consecutive  numbers  whose  product  is 
462. 

Let  the  numbers  be  x  and  .1;+  1- 

.T(a;+1)  =  462, 
.-.    x^  +  x-462  =  0. 

Ex.  2. — The  length  of  a  rectangle  is  10  feet  more  than  the 
width  and  the  area  is  875  square  feet.     Find  the  dimensions. 

Let  x  =  the  number  of  feet  in  the  width, 

.'.    a;  +  10  =  the  number  of  feet  in  the  length, 
a;(a;+10)  =  875, 
.-.     a;2+ 10a; -875  =  0. 

Ex.  3. — Divide  20  into  two  parts  so  that  the  sum  of  their 
squares  may  be  36  more  than  twice  their  product. 

Let  a;=one  part, 

20— a; = the  other  part, 
a;2  +  (20-a;)2  =  2a;(20-a;)  +  36, 
a;2 -f  400  -  40x + a;2  =  40a;  -  2a;2 + 36, 
4a;2- 80a; +364  =  0, 
a;2-20x  +  91  =  0. 

240 


QUADRATIC  EQUATIONS  241 


EXERCISE  116 

Represent  the  number  to  be  found  by  x  and  obtain,  in  its  simplest 
form,  the  quadratic  equation  which  must  be  solved  in  each  of  the 
following  : 

1.*  The  sum  of  a  number  and  its  square  is  132.     Find  the  number. 

2.  Find  the  number  which  is  156  less  than  its  square. 

3.  The  sum  of  the  squares  of  three  consecutive  numbers  is  149. 
Find  the  middle  number. 

4.  The  product  of  a  number  and  the  number  increased  b\-  6  is 
112.     Find  the  number. 

5.  The  length  of  a  rectangle  is  6  feet  less  than  five  times  the  width. 
The  area  is  440  square  feet.     Find  the  width. 

6.  The  average  number  of  words  on  each  page  of  a  book  is  6  more 
than  the  number  of  pages.  The  total  number  of  words  is  9400.  Find 
the  number  of  pages. 

7.  The  area  of  a  rectangle  is  88  square  inches  and  the  perimeter 
is  38  inches.     Find  the  length. 

168.  Standard  Form  of  the  Quadratic  Equation.  Every 
quadratic  equation  may  be  reduced  to  the  form 

in  which  a,  b  and  c  are  any  known  numbers,  except  that  a 
can  not  be  zero. 

The  term  not  containing  x  is  called  the  absolute  term. 

It  is  frequently  necessary  to  simplify  equations  to  bring 
them  to  the  standard  form,  and  thus  determine  if  they  are 
quadratic  equations. 

Ex.  1.  (a;+l)(2.r+3)=4:r2-22, 

2x2  +  5a;  +  3  =  4x2-22, 
.-.    -2a;2  +  5.T  +  25  =  0, 
2x2 -ox -25  =  0. 

Here  the  equation  is  seen  to  be  a  quadratic.     The  coefficient  of  x*  is  2, 
of  X  is  —  5  and  the  absolute  term  is  —  25. 
Or,  o  =  2,  6= -5,  c=  — 25. 

It 


242  ALGEBRA 

Ex.  2.  —H =1, 

4  a; 

.-.    7x2  +  4(a;-7)  =  4a;, 
7x2-28  =  0, 
a;2-4  =  0. 
Here  a=  1,  6  =  0,  c=  —4. 

T-,      <,  2a;     ,   a;+l 

a;— 1      a;+2 
.-.    2x(x  +  2)  +  (x+l){x-l)  =  S{x-l)(x+2), 
2x-  +  4x  +  a;2-l  =  3x2  +  3x-6, 
a;+5  =  0. 

Here  a  =  0,  6=1,  c  =  5,  and  the  equation  is  not  a  quadratic,  since 
the  coefficient  of  x^  is  zero. 


EXERCISE  117 

Reduce  to  the  standard  form  and  state  the  values  of  a,  b  and  c, 
in  which  a  is  alwa\'S  positive  : 

2,     25a:=6.r2+21. 
4,     2=Ux—12x^-. 

'•    "  +  .  =  2- 

8.     {3x-5){2x-5)=x~+2x-3. 

^      3A--8_5a:-2  IQ         *  5         3 


1.* 

6x2=a;+22. 

3. 

19a;=  15-8x2. 

5. 

— -_5.r. 

7. 

a;2 
8  —  a;  =  — . 
4 

x— 2        a;  4-5  .r— 1      x-\-2      x 

x+l        3a;    _.  2a;    _^— 3_, 

a;+2      a;— 1  ^  '  '     a;— 3         x     ~ 


169.     Solution  by  Factoring 

(1)  When    the   absolute    term    is    zero,  the   equation   can 
always  be  solved  by  factoring. 


QUADRATIC  EQUATIONS  243 

Ex.  1.— Solve  2a;2-3x=0. 

a;(2a;-3)  =  0, 

.-.    a;=Oor  2x-3  =0, 

.".    a;=0  or  |. 
Verify  both  roots. 

Ex.  2.— Solve  ax^+bx=0. 

x{ax  +  b)  =  0, 

.".    a;=0  or  aa;+6=0, 

b 
.-.    x=0  or  —  -  • 
a 

(2)  When  the  middle  term  is  zero,  the  equation  can  always 
be  solved  by  factoring,  or  by  extracting  the  square  root. 

Ex.— Solve  3x2-27=0. 

3(a;-3)(a;+3)  =  0, 
.-.    a;-3  =  0  or  a;+3  =  0, 
x=±3. 
Orthiis,  3x2-27  =  0, 

.-.    x^  =  9, 
.-.     a;=±3. 

(3)  The  equation  is  a  complete  quadratic  when  none  of  the 
coefficients  a,  b,  c  is  zero.  If  the  quadratic  expression, 
ax^-\-hx-\-c,  can  be  factored  by  any  of  the  methods  previously 
given,  the  solution  is  then  easity  effected. 

Ex.  1.— Solve  3a;2_lla;=14. 

3x2- llx- 14  =  0, 
.-.    (x+l){3x-14)  =  0, 
x=  —  1  or  V. 
Verify  both  of  these  roots. 

Ex.  2. — Solve       x'^—mx-\-nx—mn={). 

x(x  — TO)  +  n(x  — m)  =  0, 
.".    (x  — m)(x  +  n.)  =  0, 
x  =  m  or  —n. 

R  2 


244  ALGEBRA 

exercise:  118 

1-12.     Solve  the  equations  in  the  preceding  exercise  and  verify. 

13-19.     Solve  the  problems  in  the  first  exercise  in  this  Chapter. 
(Verify  the  results.) 


Solve  by  factoring  and  verify : 

20.     x^—3ax+2a^=0. 

21. 

a;2_52^0. 

22.     a;2— ma;— 6m2=0. 

23. 

x^—ax—bx-\-ab=0. 

24.     .-c2+2x(a+6)+4a6=0. 

25. 

2ax^+ax~2x=l. 

26.     {x—a)(x—b)=ab. 

27. 

x"—a^—{x—a){b-\-c) 

170.  Consider  the  problem  :  Find  two  numbers  whose 
sum  is  100  and  whose  product  is  2491. 

Let  ;c=one  number, 

100— .T= the  other  number, 

.T(100-a:)  =  2491, 
.-.   .t2-100x+2491-=0. 

To  solve  this  equation  by  the  preceding  method,  we  must 
find  two  factors  of  2491  whose  sum  is  100,  but  this  is  exactly 
what  the  problem  requires  us  to  find. 

The  necessity  is  therefore  seen  for  another  method  of 
solving  the  quadratic  equation  when  the  factors  of  the 
quadratic  expression  cannot  be  obtained  readily  by  inspection. 

171.  Solution  by  Completing  the  Square.  We  know  that 
(^x-{-a)^=x^-\-2ax+a^,  the  middle  term  being  twice  the  product 
of  X  and  a. 

If  the  first  two  terms  of  a  square  are  x^-\-2ax,  we  know  that 
it  must  be  the  square  of  x-\-a,  and,  therefore, 
a'^  must  be  added  to  x^-{-2ax  to  make  a 
complete  square. 

What  is  the  area  of  the   shaded  portion  in 
•^'      "         the  diagram  ? 

Similarly,  x^-\-4x  must  be  the  first  two  terms  in  the  square  of  x-\-2. 
To  make  x^-\-Ax  a  complete  square  we  must  add  2^  or  4.  Also,  x^  —  Sx 
are  the  first  two  terms  in  the  square  of  a;  — 4,  and,  therefore,  4*  or  16 
must  be  added. 


QUADRATIC  EQUATIONS  245 

To  complete  the  square,  it  is  seen  that  the  quantity  to  be 
added  is  the  square  of  half  of  the  coefficient  of  x. 

Ex.  1. — Factor  .r'^+Bx-— 40. 

Add  9  to  a;-  +  6x  to  make  a  complete  square. 

Then  x^  +  6x-4:0  =  x^  +  Qx  +  9-d-40, 

=  x-  +  6x+9~49, 
=  (a;+3)2-72, 
=  (x+3  +  7)(a;  +  3-7), 
=  (a;+10)(x-4). 

Ex.  2.— Factor  a;2+5x-806. 

Add  (f;)^  or  -/'  to  x^-\-5x  to  complete  the  square. 
Then  x^  +  5x-SQ6  =  x^  +  5x+\''-^^-806, 

=  x^  +  5x+ -Y- — -¥-» 

=  (x+g)2-(V)'', 

=  (a;+|+V)(a;+i--V-), 
=  (x  +  31)(.T-26). 

Ex.  3.— Solve  a;2_ioOa;+2491=0. 

Add  50^  or  2500  to  complete  the  square. 

.r2-I00a;+ 2500 -2500 +  2491  =  0, 

a;2- 100^  +  2500-9  =  0, 

(x-50)2-32  =  0, 

.-.   (a;-50  +  3){a;-50-3)  =  0, 

(a:-47)(x-53)  =  0, 

a;-47  =  0  or  a;-53  =  0, 

x  =  47or5.3. 

The   solution   might   be   contracted   by   writing   it   in  the 

following  form  : 

a;2- 100^+2491  =  0. 
Transpose  the  absolute  term,     .'.    x^—  100a;=  —2491. 
Add  2500  to  each  side,     .".   x^- 100a;  +  2500= -2491  +  2500=9. 
Take  the  square  root,  ,*.         x— 50=  +  3, 

a;  =  50±3, 
=  53  or  47. 


246  ALGEBRA 

Here  the  solution  depends  upon  the  same  principle,  but 
assumes  a  simpler  form. 

It  is  thus  seen  that  we  effect  the  solution  of  a  quadratic 
equation  hy  finding  and  solving  the  two  simple  equations  of 
which  it  is  composed. 

Thus  by  the  first  method  of  solving  x-—\00x^24Q\=0, 
we  obtained  the  two  simple  equations  a;— 47=0  and  x— 53=0, 
and  by  the  second  .r— 50=3,  and  a;— 50=  — 3. 

Ex.  4.— Solve  3:r'-+.r=10. 

Divide  by  3  to  make  the  first  term  a  square. 

Add  ( ,1 ) ''  to  each  side,      .-.   a;^  +  Jx + ^V = ¥-  +  uV = W- 
Take  the  square  root,      .'.  ^+i=  +  -V> 

x=+-V — i  =  tor— 2. 
Verify  both  of  these  roots. 

The  steps  in  this  method  are  : 

1.  Reduce  the  equation  to  the  standard  form  and  remove  the 
absolute  term  to  the  right. 

2.  Divide  by  the  coefficient  of  x-  if  not  unity. 

3.  Complete  the  square  by  adding  to  each  side  the  square 
of  half  the  coefficient  of  x. 

4.  Take  the  square  root  of  each  side. 

5.  Solve  the  resulting  simple  equations. 

EXERCISE  119  (1-8,  Oral) 

What  must  be  added  to  each  of  the  following  to  make  a  complete 
square  ? 

1.     x^-\-2x.  2.     a;2— 4a;.  3.     a;2+10a;.  4.     x^—Ux. 

5.     x^-\~3x.  G.     x^—bx.  7.     x^-{-^ax.  8.     x'^—\x. 

Factor,  by  making  the  difference  of  squares,  and  verify  : 
9.     a;2+4.T— 77.  10.     a;^— 54a;+713.         11.     a;^— 2a;— 899. 

12.     a;2— a;— 1640.  13.     a;^— i.ix+¥.  14.     3a;2+16a;— 99. 


QUADRATIC  EQUATIONS  247 

Use  the  method  of  completing  the  square  to  solve  the  following  and 
verify  the  roots  : 

15.     a;2+8x=9.  16.     x^-()x=l.  17.     a-^-lOx-f  9=0. 

18.     a;2-9x+18=0.       19.     .t2+7x+10=0.       20.     x-^x=2. 

21.     2.t2— 3x=2.  22.     2a:2+.r=1081.         23.     6x^+ox=6. 

1 

24.     If  x-+x=lh,  find  the  values  of  .r  +  -  • 

'  •'  X 

172.  Equations  with  Irrational  Roots.  In  all  the  quadratic 
equations  we  have  solved,  we  fomid  tliat  when  we  had 
completed  the  square  on  the  left  side,  the  quantity  on  the 
right  was  also  a  square.     This  would  not  alwaj's  be  the  case. 

Ex.  1.— Solve  .T2-6.r-l-0. 

X^  —  ()X=l, 

:.     x''-6a;+9=10, 

a;-3=±\/T0, 
a;=3±V'IO. 

The  two  roots  are  S+VlO  and  3— ViO.  _ 

We  might  go  a  step  further  and  substitute  for  V  10  its  approximate 
value  3-16. 

The   two   roots  would   then   be 

3±3-16  =  6-16  or  -16. 

If  we  substitute  either  of  these  values  for  x  in  x^  —  6x—l,  the  result 
will  not  be  exactly  0,  as  we  might  expect,  because  \/lO  is  not  exactly 
3-16,  but  tlie  difference  between  0  and  the  value  found  for  a;^  — Ox— 1 
will  be  very  small. 

Ex.  2.— Solve  2.r2+a;=2. 

X^  +  ^X=1, 

:.    x^+^,x+^,  =  H, 

x+k=±vn=±ivv7. 

x=-i±kVvi. 

The  two  roots  are  -l  +  iVT?,  — ^-^VT?,  or  -781,  -1-281,  on 
substituting  V 17  =  4-123. 


248  ALGEBRA 

173.  Inadmissible  Solutions  of  Problems.  When  a  problem 
is  solved  by  means  of  a  quadratic  equation,  it  does  not  follow 
that  the  two  roots  of  the  equation  will  furnish  two  admissible 
solutions  of  the  problem. 

Ex. — A  man  walked  25  miles.  If  his  rate  had  been  one 
mile  per  hour  faster  he  would  have  completed  the  journey 
in  1^  hours  less.     What  was  his  rate  ? 

Let  his  rate  be  x  miles  per  hour. 


At  the  supposed  rate  his  tinae        =  hr. 

■■      X        x+1       * 

Simphfying,  x^  +  a;  — 20  =  0, 

Solving,  a;  =  4  or  —5. 

Therefore  his  rate  was   4   miles  per   hour,   the  other  root  giving  a 
solution  which  is  inadmissible. 


EXERCISE   120 

Solve,  finding  the  roots  approximately  to  three  decimal  places, 
using  the  table : 

1.*  a;2— 4.T=1.  2.     x^—10x+ll=^0. 

4.     a;2+8.r=19.  5.     .r(a:+3)=f. 

Solve,  expressing  the  roots  in  the  surd  form  : 

7.     x^-6x=2.  8.     .r2+8a;=ll. 

10.     4x2-8a:=37.  11.     3.r2-5x-ll=0. 

The  following  problems  reduce  to  quadratic  equations.  In  solving 
the  equations  factor  by  inspection  where  possible  and  verify  the 
results. 

13.  The  sum  of  two  numbers  is  II  and  their  product  is  30.  Find 
the  numbers. 

14.  The  sum  of  the  squares  of  two  consecutive  numbers  is  85. 
Find  the  numbers. 


3. 

.r2+2.r-6=0. 

6. 

2.r2+3.i-4=0. 

9. 

4a;2-4a;=7. 

L2. 

ix^+kx=l. 

QUADRATIC  EQUATIONS  249 

15.  The  difference  between  the  sides  of  a  rectangle  is  13  inches 
and  the  area  is  300  square  inches.     Find  the  sides. 

16.  Find  two  consecutive  numbers  such  that  the  square  of  their 
sum  exceeds  the  sum  of  their  squares  by  220. 

17.  A  merchant  bought  silk  for  §54.  The  number  of  cents  in  the 
price  per  yard  exceeded  the  number  of  yards  by  30.  Find  the  number 
of  yards. 

18.  The  area  of  a  rectangular  field  is  9  acres  and  the  length  is 
18  rods  more  than  the  width.     Find  the  length. 

19.  The  three  sides  of  a  right-angled  triangle  are  consecutive 
integers.     Find  the  sides. 

20.  How  can  you  form  730  men  into  two  solid  squares  so  that  the 
front  of  one  will  contain  4  men  more  than  the  front  of  the  other  ? 

21.  The  owner  of  a  rectangular  lot  12  rods  by  5  rods  wishes  to 
double  the  size  of  the  lot  by  increasing  the  length  and  width  by  the 
same  amount.     What  should  the  increase  be  ? 

22.  If  .r+2  men  in  .t+o  days  do  five  times  as  much  work  as  x+1 
men  in  x—  1  days,  find  x. 

23.  A  rectangular  mirror  18  inches  by  12  inches  is  to  be  surrounded 
by  a  frame  of  uniform  width  whose  area  is  equal  to  that  of  the  mirror. 
Find  the  width  of  the  frame. 

24.  What  must  be  the  radius  of  a  circle  in  order  that  a  circle  with 
a  radius  3  inches  less  maj'  be  ^  as  large  ? 

25.  One  side  of  a  right  triangle  is  10  less  than  the  hypotenuse 
and  the  other  is  5  less.     Find  the  sides. 

26.  A  man  spends  $90  for  coal,  and  finds  that  when  the  price  is 
increased  .$1-50  per  ton  he  wiU  get  3  tons  less  for  the  same  money. 
What  was  the  price  per  ton  ? 

27.  A  man  bought  a  number  of  articles  for  8200.  He  kept  5  and 
sold  the  remainder  for  S180,  gaining  S2  on  each.  How  many  did  he 
buy? 

28.  The  sum  of  the  two  digits  of  a  number  is  9.  The  sum  of  the 
squares  of  the  digits  is  5  of  the  number.     Find  the  number. 

29.  A  number  of  cattle  cost  8400,  but  2  having  died  the  rest 
averaged  810  per  head  more.     Find  the  number  bought. 


250 


ALGEBRA 


30.  How  much  must  be  added  to  the  length  of  a  rectangle  8  inches 
by  6  inches  in  order  to  increase  the  diagonal  by  2  inches  ? 

31.  In    the    figure,   the    rectangle    AO  .  05= rect- 
angle CO  .  OD. 

(1)  If  .40=16,  B0=3,  C0=  8,  find  OD. 

(2)  If  .40=10,  50=4,  CD=IS,  find  OD. 

32.  In  the  figure,  when  0.4  is  a  tangent  to  the  circle, 
OA^=OC  .OD. 

(1)  If  00=  4,  CD=  5,  find  0.4. 

(2)  If  0.4=  8,  0Z)=  10,  find  OC. 

(3)  If  0.4  =  15,  CD=m,  find  OD. 

33.  I  sold  an  article  for  $56  and  gained  a  per  cent, 
equal  to  the  cost  in  dollars.     What  was  the  cost  ? 

34.  The    denominator    of    a    fraction  exceeds  the 
numerator  by  3.     If   4   is    added   to   each    term   the 

resulting  fraction  is  |  of  the  original  fraction.     Find  the  fraction. 

35.  An  open  box  containing  432  cubic  inches  is  to  be  made  from  a 
square  piece  of  tin  by  cutting  out  a  3  inch  square  from  each  corner 
and  turning  up  the  sides.     How  large  a  piece  of  tin  must  be  used  ? 

36.  A  and  B  can  together  do  a  piece  of  work  in  14':-  days,  and  A 
alone  can  do  it  in  12  days  less  than  B.  Find  the  time  in  which  A  could 
do  it  alone. 


EXERCISE  121  (Review  of  Chapter  XIX) 
1 .     What  is  a  quadratic  equation  ? 


Is  (x+  l)(.'c  — 2)(a;4-3)  =  (a;— 4)(a;  -  l)(a;+7)  a  quadratic  equation? 


Solve  it. 


3.     The  sum  of  a  positive  number  and  its  square  is  4.     Find  the 
number  to  two  decimal  places. 

5  8         „         1  J_  _  J. 

a;-8~  12' 


4.     Solve  -_ --  +  -^-  =  3 ;  — -_ 
o—x      o  —  x  x—9 


5.  If  x^j/^  — 6a;?/  — 7  =  0,  what  are  the  values  of  xy  ? 

6.  Are  x  —  4  and  a;2=16  equivalent  equations,  that  is,  have  they 
the  same  roots  ? 

7.  Solve  x-  —  xy-{-y~  =  ^9,  when  y  =  '7. 


QUADRATIC   EQUATIONS  251 

8.  Divide    14   into   two   parts   so   that    the   sum  of  thoir  8([uare3 
may  be  greater  than  twice  their  product  by  4. 

9.  If  {x~2){x-3)  =  l{x-S),  does  it  follow  that  a;-2  =  7  ?     Wliat 
is  the  proper  conclusion  ? 

10.  The  distance  (s)  in  feet  that  a  body  falls  from  rest  in 
t  seconds  is  given  by  the  formula  s=16-l<2.  How  long  will  it  take 
a  body  to  fall  6440  feet  ? 

11.  Solve  3a;2- 4a;- 1  =  0. 

12.  Ten  times  a  number  is  24  greater  than  the  square  of  the 
number.     Does  this  condition  determine  the  number  definitely  ? 

13.  Solve  ^-±|  =  U^f^. 

X—  1       3         2it; 

14.  Find  two  consecutive  odd  numbers  whose  product  is  399. 

15.  Solve(2a;+3)2-2(2a;+3)  =  35. 

16.  The  units  digit  of  a  number  is  tlie  square  of  the  tens  digit 
and  the  sum  of  the  digits  is  12.     Find  the  number. 

^„       „  ,       .-c+10       10       11        7  15 

17.  Solve  = =  TT  ;  —r-k s  =  o  * 

x~5        X        6      a;+5      x—Z       3 

18.  If  a  train  travelled  10  miles  per  hour  faster  it  would  require 
2  hours  less  to  travel  315  iniles.     Find  the  rate. 

19.  Solve  (3.r-7)(2a;-9)-(5.c-12)(a;-6)  =  (x-2)(2.r-3). 

20.*  Find,  to  three  decimal  places,  the  positive  number  which  is 
less  than  its  square  by  unity. 

21.  If  4x2-3a;2/  +  2/2=i4,  find  a;  if  ?/  =  a;+3. 

22.  The  perimeter  of  a  rectangle  is  34  feet  and  the  length  of  the 
diagonal  is  13  feet.     Find  the  sides. 

23.  Solve  a;^  +  (a;  — 4)2  =  40.  State  the  problem,  the  condition  in 
which  is  expressed  by  this  equation. 

24.  A  line  20  inches  long  is  divided  into  two  parts,  such  that 
the  rectangle  contained  by  the  parts  has  an  area  of  48  square  inches 
more  than  the  square  on  the  shorter  part.  Find  the  lengths  of  the 
parts. 

25.  Solve  x^  +  2/'  =  9,  when  y  =  Z  —  x. 

26.  The  diagonal  of  a  rectangle  is  39  feet  and  the  shorter  side  is  yV 
of  the  longer.     Find  the  area. 

27.  If  5  is  one  root  of  x^  —  lx^-\-Q>x-\-20  =  Q,  find  the  other  roots 
to  three  decimal  places. 


252  ALGEBRA 

28.  Find  the  price  of  eggs  per  dozen  when  10  less  in  a  dollar's  worth 
raises  the  price  4  cents  per  dozen. 

29.  The  length  of  a  field  exceeds  its  breadth  by  30  yards.  If  the 
field  were  square  but  of  the  same  perimeter,  its  area  would  be  ^^ 
greater.     Find  the  sides. 

30.  If  8a; =  4,  find  x  to  three  decimal  places. 

X 

31.  The  cost  of  an  entertainment  was  $20.  This  was  to  be  divided 
equally  among  the  men  present.  But  four  failed  to  contribute  anything, 
and  thereby  the  cost  to  each  of  the  others  was  increased  25  cents. 
How  many  men  were  there  ? 

32.  If  a  man  waliied  one  mile  per  hour  faster  he  would  walk  36  miles 
in  3  hours  less  time.     What  is  his  rate  of  walking  ? 

33.  A  polygon  with  n  sides  has  ^n(n  — 3)  diagonals.  If  a  polygon 
has  20  diagonals,  how  many  sides  has  it  ? 

34.  Solve  a2(a;_a)2  =  62(a;  +  a)2. 

35.  A  can  do  a  piece  of  work  in  10  days  less  than  B.  If  they  work 
together  they  can  do  it  in  12  days.  In  what  time  could  each  do  it 
alone  ? 

36.  If  x^+    0  =  84,  find  the  value  of  x^  and  of  x. 

x^ 

37.  The  length  of  a  rectangular  field  is  to  the  width  as  3  to  2 
and  the  area  is  5-4  acres.  How  many  rods  longer  must  it  be  to  contain 
6  acres  ? 


CHAPTER  XX 
RATIO  AND  PROPORTION 

174.  Methods  of  Comparing  Magnitudes.  When  we  wish  to 
compare  two  magnitudes,  there  are  two  ways  in  which  the 
comparison  may  be  made. 

(1)  We  may  determine  by  how  much  the  one  exceeds  the 
other.     This  result  is  found  by  subtraction. 

(2)  We  may  determine  how  many  times  the  one  contains 
the  other.     Here  the  result  is  found  by  division. 

Thus,  if  one  line  is  6  inches  in  length  and  another  is  18  inches,  we 
may  say  that  the  second  is  12  inches  longer  than  the  first,  or  that  tho 
second  is  three  times  as  long  as  the  first. 

Neither  method  of  comparison  can  be  used,  unless  the 
magnitudes  compared  are  of  the  same  denomination,  or 
can  be  changed  into  equivalent  magnitudes  of  the  same 
denomination. 

Thiis,  we  can  compare  3  lb.  and  10  lb.  ;  2  yd.  1  ft.  and  2  ft.  9  in.  ; 
but  we  can  not  compare  5  lb.  and  4  ft. 

175.  Ratio.  When  two  magnitudes,  of  the  same  kind, 
are  compared  by  division,  the  quotient  is  called  the  ratio 
of  the  magnitudes. 

Thus,  the  ratio  of  3  to  4  is  the  same  as  the  quotient  of 
3-1-4,  which  is  usually  \^Titten  f . 
The  ratio  of  3  to  4  is  written  3  :  4, 

.-.     3  :  4=3-4=|. 
Similarly,  a:fe=a-i-6  =  ^. 

253 


254  ALGEBRA 

It  will  thus  be  seen  that  all  problems  in  ratio  may  be 
considered  as  problems  in  fractions. 

176.  Comparison  of  Ratios.  To  compare  two  ratios  we 
simply  compare  the  fractions  to  which  these  ratios  are 
equivalent. 

Ex.  1. — -Which  is  the  greater  ratio,  3  :  4  or  7  :  9  ? 

The  problem  is  at  once  changed  into  :  "  Which  is  the  greater  fraction 

f  ©r  i  ?  " 

To  compare  the  fractions  we  reduce  them  to  the  same  denomination 
in  the  forms  |^  and  f  |,  and  it  is  seen  that  the  latter  is  the  greater.  We 
might  also  comiDare  them  by  reducing  the  fractions  to  equivalent 
decimals. 

Ex.  2. — Which  is  greater,  a  :  a+2  or  a  +  1  :  a+3. 

a      _       a(o  +  3)       _       a^  +  3a 

0^+2  ~  (ra  +  2)(a+3)  ~  (o  +  2)(a+3) " 

o+l  _  (a+l)(a+2)  _    a^  +  3a+2 
a+3  ~  (a+2){a+3)  ~  (a+2)(a  +  3)' 

What  is  the  conclusion  ? 

177.  Terms  of  a  Ratio.  In  the  ratio  a  .b,  a  and  b  are  called 
the  terms  of  the  ratio,  a  being  called  the  antecedent  and  b 
the  consequent.  The  antecedent  corresponds  to  the  numerator 
of  the  equivalent  fraction,  and  the  consequent  to  the 
denominator. 

a  _  antecedent  _    numerator    _  dividend 
'  b       consequent       denominator         divisor 

178.  Equal  Ratios.  Since  a  ratio  is  a  fraction,  all  the  laws 
which  we  have  used  with  fractions  may  also  be  used  with 
ratios. 

(X  771(1/ 

Thus,  since  r  =  — r ,  it  follows  that  a  :  b=ma  :  mb. 
b       mb 

Hence  both  terms  of  a  ratio  m,ay  be  multiplied  or  divided 
by  the  same  quantity  {zero  excepted)  without  changing  the  value 
of  the  ratio. 

Thus,  6  :  9  =  2  :  3,  ^  :  ^  =  3  :  2,^  :  ^  =  a^  :  b^ 


RATIO  AND  PROPORTION  255 


EXERCISE   122  (1  15,  Oral) 

Simplify  the  following  ratios  by  expressing  them  as  fractions   in 
their  lowest  terms  : 


1. 

10:  15. 

2. 

2^:5. 

3. 

45  :  63. 

4. 

15  :  10. 

5. 

$2  :  S6. 

6. 

$2-50:  810. 

7. 

2  ft.  :  3  yd 

8. 

2  days :  12  hr 

9. 

2  ft.  3  in.  :  3  ft. 3  in 

0. 

24rt  :  8a. 

11. 

5ah  :  lOa^. 

12, 

a  +  b:u--b-. 

3. 

a—h  :  a^— 

h^. 

14. 

x—y  :  x—y. 

15. 

X                   X 

-6. 

1 

1 

_. 

17. 

a-2+2.w+?/2  :  x-3+w3. 

x—2  '  x'^—5x-\r& 

18.*  If  12  inches= 30-48  centimetres,  find  the  ratio  of  an  inch  to 
a  centimetre  and  of  a  metre  to  a  yard. 

19.  The  edges  of  two  cubes  are  2  inches  and  3  inches.  Find  the 
ratios  of  their  volumes. 

20.  If  25  francs  =S4-80,  find  the  ratio  of  a  franc  to  a  dollar  and 
of  a  quarter  to  a  franc. 

21.  If  a  metre=39-37  inches,  find  the  ratio  of  a  kilometre  to  a  mile. 

22.  Which  is  the  greater  2  :  3  or  4  :  5,  15  :  37  or  11  :  27,  a:  a-\-2 
or  a+3:a  +  5  ? 

23.  Arrange  in  descending  order  of  magnitude : 

2:3,  3:5,   11  :  15,   13:  18. 

24.  What  is  the  effect  of  adding  the  same  number  5  to  both  terms 
of  the  ratio  7  :  15  ?  What  is  the  effect  of  subtracting  5  from  each 
term  ? 

25.  What  is  the  effect  of  adding  5  to  each  term  of  15:7?  Of 
subtracting  5  ?     Compare  your  results  with  the  results  of  Ex.  24. 

26.  Separate  360  into  three  parts  which  are  in  the  ratio  2:3:4. 
(Let  the  parts  be  2x,  Zx,  4a;.) 

27.  Divide  165  into  two  parts  in  the  ratio  of  2  :  3  ;  510  in  the  ratio 
3:7;    36  in  the  ratio  1^  :  2^. 


256  ALGEBRA 

28.  When  a  sum  of  money  is  divided  in  the  ratio  1  :  2,  the  smaller 
part  is  $20  more  than  when  it  is  divided  in  the  ratio  2:7.  Find  the 
sum. 

29.  What  number  must  be  added  to  both  terms  of  f  to  make  it 
equal  to  4  ? 

30.  What  number  subtracted  from  each  term  of  7  :  10  will  produce 
13  :  19  ? 

31.  What  must  be  added  to  each  term  of  a  :  6  to  produce  c :  d  1 
What  is  the  conclusion  when  c=dt 

32.  If  a  is  a  positive  number  which  is  the  greater  ratio, 

l+2a         l+3a„ 

— ■ or   — ? 

l+3a  l+4a 

33.  The  rate  of  one  train  is  30  miles  per  hour  and  of  another  is 
55  feet  per  second.     What  is  the  ratio  of  their  rates  ? 

34.  Divide  a  line  a  inches  long  into  two  parts  whose  lengths  are  in 
the  ratio  b  :  c. 

35.  A's  income  :  JB's  income=3  :  4,  and  ^'s  expenditure  :  B's 
expenditure =5  :  6.  If  ^  spends  all  his  income,  what  per  cent,  of  his 
income  does  B  save  ? 

36.  Divide  $315  among  A,  B  and  C,  so  that  ^'s  share  will  be  to  fi's 
as  3  :  4,  and  5's  to  C"s  as  5  :  7. 

37.  A  line  is  divided  into  two  parts  in  the  ratio  of  5  :  7  and  into 
two  parts  in  the  ratio  3:5.  If  the  distance  between  the  points  of 
division  is  1  inch,  find  the  length  of  the  line. 

38.  Two  numbers  are  in  the  ratio  of  3:5,  but  if  10  be  taken  from 
the  greater  and  added  to  the  smaller,  the  ratio  is  reversed.  Find 
the  numbers. 

39.  Two  bodies  are  moving  at  uniform  rates.  The  first  goes 
VI  feet  in  a  seconds  and  the  second  n  yards  in  b  minutes.  What  is 
the  ratio  of  their  rates  ? 

179.  Proportion.  A  proportion  is  the  statement  of  the 
equality  of  two  ratios. 

Thus,  3  :  4=15  :  20,  since  |=if. 

Therefore,  3  :  4  =  15  :  20  is  a  proportion,  or  3,  4,  15,  20  are 
eaid  to  be  in  proportion,  or  they  are  said  to  be  proportionals. 


RATIO  AND  PROPORTION  257 

If  a,  b,  c,  d  are  in  proportion  or  a  -.b—c  :  d, 

a      c 
then  -  =  -, 

0       d 

/.     ad— be. 

In  the  proportion  a:b=c:d,  a  and  d  are  called  the 
extremes  and  b  and  c  the  means. 

Since  ad=bc,  it  is  seen  that  the  lyroduct  of  the  extremes  is 
equal  to  the  product  of  the  means. 

180.  Fourth  Proportional.  When  a  :  b=c  :  d,  d  is  called 
the  fourth  proportional  to  a,  b,  c. 

Thus,  if  the  fourth  proportional  to  10,  12,  15  is  x, 

then  10:  12=15:a;or  !^=  '^, 

12         X 

.-.      10a;=  12x15, 

x=lS. 

181.  To  find  a  Ratio,  by  Solving  an  Equation.  From  certain 
types  of  equations  in  x  and  y,  the  value  of  the  ratio  oi  x  :  y 
may  be  found. 

Ex.  1. — If  5x—6y,  find  the  ratio  of  x  :  y. 
Since  ox  =  6y, 

^  =  ly. 
x       6 
y~5' 
the  ratio  of  x  :  y  =  ^  or  6  :  5. 

Ex.  2.— If  3x-{-4:y=3y~lx,  find  - 

3a;  +  7a;  =  3y  —  4y, 
x_        l^ 

"    y~     To' 

If  each  term  in  the  equation  is  of  the  first  degree  in  x  or  y, 
the  ratio  oi  x  :  y  can  be  found,  but  it  can  not  be  found  if 
there  is  a  term  not  containing  x  or  y. 

Thus,  from  2x  —  ly=  10,  the  value  oi  x  :  y  can  not  be  found. 

S 


258  ALGEBRA 

Ex.  3.— If  2.r2-7a:?/+6?/2=0,  find  x:y. 

Factoring,  {x-2y){2x-Sy)  =  0, 

.-.     a;-2t/ =  Oor  2a;-3i/  =  0, 

-  =  2  or  ^  . 

y  2 

Here  there  are  two  values  of  -.     If  we  divide  each  term  of  the  given 

y 

equation  by  y^,  we  get 

+  6  =  0. 


r-<i) 


In  this  form  we  see  that  the  equation  is  a  quadratic  in  -,  and  we 
might  naturally  expect  to  find  two  values  for  the  required  ratio. 

Ex.  4.— If  2x-5ij+  z=0, 

3x-\-2y—2z=0, 
find  the  ratios  of  x,  y,  z. 

If  we  eliminate  z  in  the  usual  way,  we  get 
7a;- 8!/  =  0, 
a;  _  8  _      X      y 

y-r  •■■8  =  7' 

If  we  eliminate  y  we  get 

a;  _    8  .       X  _^   z 

z  ~  19'        •  •      8  ~  T9  ■ 

We  can  combine  these  results  in  the  convenient  form  : 
X       y        z 
8  ^  7  ""  19" 

EXERCISE  123  (1-21,  Oral) 
Find  the  value  of  x  in  the  proportions  : 


1. 

24 

S^x' 

2. 

39 

7~x' 

3. 

2      X 
5~3' 

4. 

x_5 
6      4" 

5, 

3         6 
X      -12* 

6. 

5 

7. 

a      c 

b      x' 

8. 

a      X 
b~~c' 

9. 

4_  X 
x~W 

10. 

3:  7  = 

12; 

;  X. 

11. 

2:3=a;:9. 

12. 

X  :  5=7  :  10. 

BAT  10   AND   PROPORTION  259 

Find  the  value  of  x  :  y, 


13. 

2x=ly. 

14.     3y=l2x. 

15.     2x-y=0. 

16. 

^X  =  il/. 

17.     2x=  —  3y. 

18.     3y +11x^0. 

19. 

x'^=iir. 

20.     ix'^f))/-. 

21.  {x-3y)(x-5y)=0. 

22. 

'<l-l- 

show  that  -  =  -,  and  -  = 
c      d          a 

d 
c 

23. 

Find   a 

fourth 

proportional    to  : 

.      2 

3,    18;      5,  —7,  -10; 

h   h 

i  ;    a,  b,  c 

;    a,  2b 

,  3c. 

24.  Find  a  fourth  proportional  to:  a—b,  {a-{-b)",  a''—b^;  and 
to  a^— 3a+2,  a^— 5a+6,  a^— 5o+4. 

25.  What  number  must  be  added  to  each  of  the  numbers  2,  4, 17,  25 
so  that  the  results  will  be  proportionals  ?     (Verify.) 

2«j.  If  a^x,  b+x,  c-\-x,  d-\-x  are  proportionals,  find  x.  What 
does  the  result  mean  when  be— ad  ? 

27.  Find  a  in  order  that  «  +  3:a+15=3:4. 

28.  ^'s  age  is  to  B's  as  4  :  5.  Five  years  ago  the  ratio  was  3  :  4. 
Find  their  ages. 

29.*  In  an  equilateral  triangle  the  ratio  of  the  altitude  to  either 
of  the  equal  sides  is  a/3  :  2.  If  the  altitude  is  10  inches,  find  the  side 
to  two  decimal  places. 

30.  When  a  line  is  drawn  parallel  to  the  base  of  a  triangle  it  divides 
the  sides  in  the  same  ratio.  In  the  figure,  .45=20,  AD=14:  and 
AC=  15.     Find  A E  and  EC. 

31.  In  the  figure,  the  triangles  ADE  and  ABC  are 
similar.  W^hen  triangles  are  similar  their  correspond- 
ing sides  are  in  the  same  ratio,  so  that  D/. \E 

AD  :  AB=DE  :  BC=AE  :  AC.  ^   ^ 

If  AB=8,  BC=10,  AC=d,  AD^Q,  find  the  lengths  of  all  the  other 
lines  in  the  figure. 

32.  In  the  same  figure,  the  areas  of  the  similar  triangles  are  in 
the  same  ratio  as  the  areas  of  the  squares  on  their  corresponding  sides 
If  AD=20  and  AB^35,  find  the  area  of  ADE  if  the  area  of  ABC  is 
735. 

s2 


260 


ALGEBRA 


33.  The  side  of  the  square  ABCD  is  10  inches  and  EF  is  parallel 
to  DC.  If  the  length  of  AE  is  8  inches,  find  the 
length  of  FC  to  three  decimals. 

34,  If  the  bases  of  two  triangles  are  in  the  ratio 
3  :  4  and  their  heights  in  ratio  8  :  9,  find  the  ratio  of 
their  areas. 

From  these  equations  find  x  :  y, 

13x+5?/=9a;+13// ;    ax-\-bi/=cx-\-dy  ; 
mx—ny=nx-\-my ;     px-\-qy=Q. 

36.  Find  two  values  oi  x  :  y  when 

6a;-— 13.i-^+6^-=0  ;    x'=4iXy-\-5y^. 

37.  If  5rt— 36+2c=0  and  a+6+c  =  0,  find  the  ratios 
of  a  :h,  a  :  c,  a  :  b  :  c. 

38.  Find   the   lengths  of   all  the  other  lines  in  this 
figure. 

39.  If   a  pole  10  feet  high  casts  a  shadow  17 1  feet 
long,  what  will   be   the   length    of   the    shadow    cast,    at    the   same 
time,  by  a  monument  84  feet  high  ? 


3  D  9 


:0  in  a  form  showing  - 

y 


40.  Write  the  equation    3x^—10xy-\-3y- 
as  the  unknown,  and  find  x  :  y. 

41.  A  number  of  two  digits  bears  the  ratio  7  :  4  to  the  number 
formed  by  reversing  the  digits.  If  the  sum  of  the  numbers  is  66, 
find  them. 

42.  The  length  of  a  room  is  to  the  width  as  6:5,  and  the  length 
is  to  the  height  as  3  :  2.  If  the  area  of  the  floor  is  187J  square  feet, 
find  the  dimensions. 

43.  If  4  men  and  3  women  earn  as  much  as  16  boys,  and  6  men 
and  5  boys  earn  as  much  as  10  women,  find  the  ratio  of  the  earnings 
of  a  man,  woman  and  boy. 

44.  If  3ab+2b^  :  2a^—ab=9  :  5,   find  a  :  b. 

45.  When  the  angle  A  is  bisected, 
AB  :  AC=BD  :  DC. 

(1)  If  .4jS=10,  AC=8,  BC^12,  find  BD  and  DC. 

(2)  If  AB=c,  AG=b,  BC=a,  find  BD  and  DC. 


BAT  10  AND   PROPOBTION  261 

46.  The  ratio  of  the  area  of  a  rectangle  to  the  area  of  the  square 
described  on  its  diagonal  is  6  :  13.      Find  the  ratio  of  the  sides. 

47.  The  sides  of  a  triangle  are  7,  10  and  12.  The  perimeter  of  a 
similar  triangle  is  72|.     What  are  its  sides  ? 

48.  If ~  == ^  =  — — •i-' —  ,  find  x  :  y:  z. 

5  7  9 

182.  Mean  Proportional.  When  three  numbers  form  a 
proportion,  it  is  understood  that  the  middle  number  is  to  be 
repeated.  The  three  numbers  are  said  to  be  in  continued 
proportion,  and  the  middle  one  is  called  the  mean  proportional 
between  the  other  two. 

Thus,   4,    6   and   9   are   in   continued  proportion,  since   4:6  =  6:9. 
Here  6  is  the  mean  proportional  between  4  and  9. 
If  X  is  a  mean  proportional  between  3  and  27, 

3        X 

-  =  ^,  .-.      a;2=81,  .-.      a;=+9. 

X       21  - 

.'.    the  mean  proportionals  between  3  and  27  are  +9. 

3        9  3         —9 

Since  q  =  :j^  and  -^  =  -^-   it  is  seen   that  these  are  the   correct 

results. 

Similarly,  if  x  is  the  mean  proportional  between  a  and  b,  then 

ax  .     /-L 

-  =  -,  .-.      x=  +  Vab. 

X         0 

Therefore,  the  mean  proportional  between  any  two  quantities 
is  the  square  root  of  their  product. 

183.  Third  Proportional.  Tf  a,  h,  c  are  in  continued 
proportion,  c  is  called  the  third  proportional  to  a  and  h. 

Thus,  if  X  is  the  third  proportional  to  6  and  1.5, 

4  =  -,  .-.      6a;=22o,  .-.      x=37i. 

15       a;  ^ 


262  ALOEBRA 


EXERCISE   124 

1.  Find  a  mean  proportional  between  4  and  16 ;  2a  and  8a  ; 
4a6^  and  da^b  ;    (a— 6)^  and  {a+b)'^. 

2.  Find  a  third  proportional  to  2  and  4  ;  3  and  30  ;  f^t  and  10«6 ; 
x"—y'^  and  x—y. 

3.  The  mean  proportional  between  two  numbers  whose  sum  is 
34  is  15.     Find  the  numbers. 

4.  Three  numbers  are  in  continued  proportion.  The  middle 
one  is  12  and  the  sum  of  the  other  two  is  51.     Find  the  numbers. 

5.  What  number  must  be  added  to  each  of  the  numbers  3,  7,  12 
so  that  the  results  will  be  in  continued  proportion  ? 

6.*  In    the    figure,    the  angle    BAC  being  in  a 
semicircle    is    a    right  angle.     When  AD  is   drawn 
perpendicular  to   the  Iwpotenuse  it  is   proven    in 
"      D  "*      geometry  that 

AD  is  a  mean  proportional  between  BD  and  DC, 

AB  between  BD  and  BC,  and  AC  between  CD  and  BG. 

(1)  If  BD=  4,  DC=  9,  find  AD. 

(2)  If  BD=  5,  AB=  8,  find  DC. 

(3)  If  BC  =  n,  AC=\2,  find  DC,  AB,  AD.' 

(4)  If  AB^  3,  AC=  4,  find  BC,  AD,  BD. 

7.  How  would  you  use  the  preceding  to  find 

(1)  A  line  whose  lengtii  is  V6  inches  ? 

(2)  The  side  of  a  square  whose  area  is  12  square  inches  ? 

8.  Find  two  numbers  such  that  the  mean  proportional  between 
them  is  4  and  the  third  proportional  to  them  is  32. 

9.  Divide  a  line  21  inches  long  into  three  parts  such  that  the 
longest  is  four  times  the  shortest  and  the  middle  one  is  a  mean 
proportional  between  the  other  two. 

184.  The  following  examples  will  illustrate  a  method  which 
has  many  appplieations  to  problems  with  ratios  or  fractions. 


RATIO  A:ND  proportion  263 

Ex.  l.-If  ^  =  j^,  prove  that  -^^-^  =  ^^-^  • 

Since  7-  =  -,,  let  each  fraction  =  k. 
0       d 

Then  r  =  fc,   .".   a=bk,  -,  =  k,    .'.   c=dk. 

b  a 

Substitute  these  values  of  a  and  c  in  each  side  of  the  identity  to  be 
proven. 

3a2  +  262       3h2i2  +  262       b^{^k^  +  2)       Zk^  +  2 


a^-5b^        b^k^-5b^         b^k^-5)         k^-5 

3c^+2d^  _  3d^k^  +  2d^  _  d^(3fc^+2)  _  3A:''  +  2 
c^-od^   ^  d^k^-5d^   ~  d^k^-d)   ~~    k^-5' 

3a2  +  262  _  3c^  +  2d^ 

a^  — 56^         c^— 5f/- 

Ex    2.— If  -  =  ^  =  "^  ,  prove  ^''^+-^'''+^'^  ^ix±y^\^^ 

Let  x  =  ak,  y  =  bk,  z  =  ck.     Substitute  as  before  and  show  that  each 
of  the  fractions  is  equal  to  k^. 

.0^     ,..  «      f    .^      a+&      e-\-d 

185.     If  1-  =  ^ ,  then  — ~  =  :^  • 

0      a  a—o      c—a 

Prove  this  by  letting  a=^bk  and  c  =  dk,  as  in  the  preceding  examples. 

Here  the  fraction  — ,  was  obtained  by  adding  and  siib- 

a—b  -^  ^ 

trading  the  terms  of  the  fraction  -,  and  — ^  was  obtained 

in  a  similar  way  from  -^  ■ 
This  principle  is  sometimes  useful  in  simplifying  equations. 

Ex.  1. — Solve 
Adding  and  subtracting, 


4a;- 3 

3a -46 

Sx 

6a 

6 

~  8b' 

.      646a; 

=  36a, 

X 

9a 

~  166' 

Solve  also,  in  the  usual  way,  by  cross  multiplication. 


264  ALGEBRA 

Ex.  2.— If  —^ — ^  =  —r-> ^ '   prove  j- = - 

a—b—c-\-d      a—b^c—d  be 

Adding  and  subtracting,  ^^-j-^^  =  ^b:-^^. 


Adding  and  subtracting, 


a-\-d 
6  +  c  "" 

a  —  d 
'  b-c 

a-\-d 
a—d 

b  +  c 
^  b-c 

2a 

26 

2d" 

~  2c' 

a 

b 

'•     d 

c 

a 

d 

EXERCISE   125 

1.  If  -  =  ^,  prove  that  - — -r,  =  ^  — --,. 

6      d^  2a+3b      2c+3d 

2.  li  a  :  b=c  :  d,  show  that 

ma-{-nb  :  ma—nb^=mc-\-nd  :  nic—nd. 

3.  If  a  :  6  =  c  :  d,  prove  a2ftf;_|_ft2g_|_5g_(jj2g_|_gf5(^_|_fj()( 

4.*  If  -  =  -,  find  the  value  of  2^±^f  . 
y      4  6x2/+?/2 

(Here  x=fy,  substitute  for  x  and  simplify.) 

.      rr  --c      2/  c   J  .,         1        t  {2x-{-3y){3x-\-2y) 

5,     If  -  =:^  ■^,  find  the  value  of  ^ ^r    Iw— -^.  • 

2      3  {5x-3ij){3x-5y) 

r.      Tt  X      n       1  «      2    ,.   J  ^,         ,        J.  \2ax—by 

i\.     If     ^  3  and  -  =  -,  find  the  value  of  ^ -,^ . 

y  b      5  2ax-\-6by 

7,     If  -  =  ■   =  -,  prove  that  each  fraction  is  equal  to  ^ ,  that 

a      b      c  a+b+c 

.     .       sum  of  numerators 
IS,  to 


sum  of  denominators 


8.     If  -—  =  ^-  =  ^,  prove  :r{bi-c)+y{c^a)+z{a+b)=0. 
b—c      c—a      a-b 


RATIO   AND   PROPORTION  265 

9.     If  a,  h,  c,  d  are  proportionals,  prove  that  a^c^ac^,  bH+b(P, 
u-c-  and  bH-  are  proportionals. 

10.  If = = ,  prove  that  a=b-\'C 

11.  If  a  :  b  —  b  :  c,  show  that  a  :  c  =  a- :  b'~. 

12.  If  a—b:a^b=c—d:c^d,   prove  a  :  b=c  :  d. 

Ts:  a+b+c+d      a—b+c—d     ,        ^u  i.  ^       c 

13.  If  — ^ — —       = -,  show  that -  = -• 

a+b—c—d      a—b—c+d  b      d 

14.  Solve  ■ — -  = ■ — -  • 

3a;- 46      5a -36 

15.  If  the  sum  of  two  numbers  is  to  their  difference  as  7  to  4,  find 
the  ratio  of  the  numbers. 

16.  If '^^  — '- —    =  — ^  ,    show   that   each   fraction 

6— c+2       c-rt+4       a— 6+6 

equals  |.     (Use  Ex.  7.) 

._      o  ,       ax+bA-c      bx-\-c-\-a 

17.  Solve  ■ — ~=  — — ■ — 

ax—b-\-c      bx-\-c—a 

18.  If  a  :  6=3  :  5,  6  :  c=7  :  9,  c  :  rf==15  :  16,  find  the  ratio  of  a  :  d. 

19.  If  ■-  =  "'  =  - ,  show  that  each  fraction  equals  * ^^     ,  and 

a      b      c  ^         5a-36+2c 

,  ,    mx+7iy-~pz 

also  equals ^ — i-    • 

'ma-\-nb — pc 

20.  Find  two  numbers  such  that  their  sum,  difference  and  product 
are  proportional  to  4,  2,  9. 

21.  If  a,  6,  c  are  consecutive  numbers  and  if  c-—b-  :  6'-— rt-=41  :  39, 
find  the  numbers. 

22.  The  length  and  breadth  of  a  room  are  as  3:2,  and  if  2  feet 
be  added  to  each,  the  new  area  of  the  floor  is  to  the  old  as  35  :  27.  Find 
the  dimensions. 

23.  If  a:  b=c  :  d,  prove  a  :  a+6=a+c  :  a+6-(-c+d. 

„.      „  lOa+6      I2a+6     ,        ...  a      c 

24.  If  '—^  =  — -—  .  show  that  -  —  -,■ 

lOc+d      I2c+d  6      d 

25.  If  a  :  b  —  b  :  c,  then  a'^-'rub  :  b^=b^-\-bc  :  c^. 


266  ALGEBRA 

EXERCISE  126  (Review  of  Chapter  XX) 

Write  as  fractions  in  their  simplest  forms  : 
1-     7^  :  8|.  2.     a;2-t/2  :  (x-y)'^.  3.     a^  +  b^  :  {a  +  b)K 

4-      1 s  :  1-1 5.     a- :1 »  6.     a :c 

x^  X  a  a  c  a 

^-     x^-5x+6'-x^  +  x-l2'  ^-     "'+l+^  =  «-l+a- 

9.     Divide  144  into  three  parts  proportional  to  3,  4,  11. 

10.  What  must  be  added  to  each  term  of  4  :  7  to  make  it  equal 
to  6  :  7  ? 

11.  Write  as  a  proportion  in  two  ways  : 

3.  6  =  2.  9  ;   2  .  5  =  3x;  ab^cd; 

{a  +  b){a-b)  =  3  .  4;  a^-5a+6  =  a^  +  5a  +  4:. 

12.  If  the  means  are  7  and  12  and  one  extreme  is  3,  what  is  the 
other  extreme  ? 

13.  Find  a  fourth  proportional  to  :   7,  15,  35  ; 

a,  a^.  a^ ;  x-\-y,  x  —  y,  x'^  —  y^;  tj r  .  a^  —  b^. 

^  "  ^       a—b  o  +  ft 

14.  Find  two  numbers  in  the  ratio  9  :  5,  the  difference  of  whose 
squares  is  504. 

15.  Two  numbers  are  in  the  ratio  of  5  :  8,  and  if  8  be  added  to  the 
less  and  2  be  taken  from  the  greater,  the  ratio  is  14  :  15.  Find  the 
numbers. 

16.  Find  two  numbers  in  the  ratio  6  :  5  so  that  their  sum  is  to  the 
difference  of  their  squares  as   1:3. 

17.*  If  the  ratio  a  —  x  :  b~x  is  equal  to  the  square  of  the  ratio  a  :  b, 
find  X. 

18.  If2a;+3?/ :  3a;-5?/=9  :  11,  find  a;  :  y. 

19.  If  (5x-72/)(2a;-32/)  =  (4a;-52/)(a;-2/),  find  .r  :  y. 

20.  If  4a; -5y  =  2a; +22/,  find  3a;+2?/  :  2x^%y. 

21.  If  6.^2+  15y2=  19x2/,  find  x  :  y. 

22.  Ifa;2  +  a;+l  :  62(a;+ l)  =  a;2-a;+ 1  :  63(a;-l),  find  x. 

23.  If  2x^y  —  2z  =  0  and  7x  +  6j/ —  92  =  0,  find  x  -.y,  x  :  z  and  x  :  y  :  z. 

24.  If  -  =  ?    find  the  value  of  ^f!^  • 

y       3  3x+l\y 


RATIO  AND   PROPORTION  2G7 

25,  Find  a  mean  proportional  to  a;^ ^  and  y^ ^  . 

y  '(' 

26,  If  ax -{-by  :  bx'\-ay  =  9  :  11  and  a  :  6  =  3  :  2,  find  the  ratio  of 
X  to  y. 

27,  If    r  =  -,  =  ^,  show   that   each   of  these  fractions   is   equal  to 

b       d      f 

ma—nc—pe 
mb—nd—pf' 

28,  Find  two  numbers  whose  sum,  difference  and  product  are 
proportional  to  5,  3.  16. 

29,  If  a:b  =  c:  d,  show  that  -  =  — —  ;    „  -2~ro»,2  =  ^  2  i  o  72 ' 

(ab  +  cd)-  =  (a^  +  c^){b'  +  d% 

30,  If  ; =  — ; ,  =  ; ,  prove  that 

o-j-c  — a       c-\-a  —  o       a-\-b  —  c 

x{b~c)  +  y(c-a)  +  z{a-b)^0. 

31,  If  any  number  of  ratios  are  equal,  show  that  each  ratio  is  equal 
to  the  ratio  of  the  sum  of  all  the  antecedents  to  the  sum  of  all  tiie 
consequents. 

32,  If  3x- 2^ +  42  =  2a;— 3^  +  3  =  0,  find  the  ratios  of  x,  y,  z.  If 
also,  x^ -{- y^ -\- z^  =  150,  find  the  values  of  x,  y  and  z. 

33,  The  hypotenuse  of  a  right-angled  triangle  is  to  the  shortest 
side  as  13  :  5.     If  the  perimeter  is  120,  find  the  sides. 

34,  The  length,  width  and  height  of  a  room  are  proportional  to 
4,  3,  2.  If  each  dimension  be  increased  2  feet,  the  area  of  the  four 
walls  will  be  increased  in  the  ratio  of  10  to  7.  Find  tlie  dimensions  of 
the  room. 

ace  a^  +  c^-l-e^       ace 

^'*-  ^H  =  d=  r  b^w+p  =  bdf • 

36,     If  the  sides  of  a  triangle  are  6  and  8  and  the  base  is  4§,  find 

the  segments  of  the  base  when  the  bisector  of  the  vertical  angle  is 

drawn, 

Q-       re  y  —  z-\-x       x-~y-\-z  .1x0        »,     2 

.i<.     If ; —  =  — ,         - ,  show  that  z^  —  x--\-y^. 

z—x-\-y      x+y+z 

38,  The  incomes  of  A  and  B  are  as  2  :  3  and  their  expenses  are  as 
f)  :  7.     If  A  saves  25%  of  his  income,  what  %  does  B  save  ? 

39.  Find  three  values  of  the  ratio  x  :  y  if 

3{x^-4x^y  +  5xy~-2y^)==2{x^~2x-y-2xy^  +  3y^). 


CHAPTER  XXI 
THE  GENERAL  QUADRATIC  EQUATION 

186.  Type  of  the  General  Quadratic.     The  equation 

is    called    the    general    quadratic    equation,    because    evei-y 
quadratic  equation  may  be  reduced  to  this  form. 

If  the  factors  of  ax'^-\-bx-\-c  can  be  obtained,  the  roots 
of  the  equation  can  be  found  by  solving  the  two  equivalent 
equations. 

187.  Solution  of  Literal  Quadratics.  The  method  of 
completing  the  square  may  be  applied  to  the  solution  of 
quadratic  equations  with  literal  coefficients. 

Ex.  1. — Solve  x^-\-2mx=n. 

Complete  the  square  by  adding  ?n-  to  each  side, 

x^-\-2tnx-\-'m^  =  7i-\-m^. 
Take  the  square  root,  x  +  m=±  V'n  +  m^, 

.'.     x=—'m+Vn-\-m''. 
The  two  roots  are  —m.+  Vn-\-m^,   —m~Vn+m^. 

Ex.  2. — Solve  x^^px-\-q=0. 

Transpose  the  absolute  term,  x^-\-px=  —q. 

7)"  ^2  rtj2  ^2 — -  4o 

Add  '.    to  each  side,  x^  -\-  px -\-  -^  =  —  7  +  "^  =        ^       ■ 

Take  the  square  root,  a;  +  ^  =  ± ~^ ' 

~p       Vp"  — 4^ 
••■•     =^  =  -2"-  2 


THE  GENERAL  QUADRATIC  EQUATION  269 

Ex.  3.— Solve  aa:2+6:c+c=0. 

Divide  by  a  to  make  the  first  term  a  square, 
b         c 

:.    x^  +  -x  +  -  =  o. 

a         a 

.be 
Irauspose,  a;-  -| — x  = 

.  ,  ,     62               ,          ,       6          62         62        c       62_4cjc 
Add  -p- „  to  each,     x-  +  -x  +  j-^^   .-- =       .    „     • 


Take  the  square  root,  x  +  ^r-  =  ± 


h  ,    \  7>2  — 4ac 


'2a  'la 


b        Vb^-iac 
la  2a 

_  -b±Vb'^-4ac 
~~  'la 

The  roots  of  the  general  quadratic  equation  are 

2a 

188.     The  roots  of  the  general  quadratic   might   also    be 

found  by  factoring  as  in  art.  171. 

ax^-\-bx-\-c  =  0, 


f      ,    b     ,    \'b--iac\ 


(   .      b         c\ 
a    x-  +  -a;  +  - 
\           a         a/ 

=  0, 

(/           by-       c         62) 
«lV^+2aj    +a-4^/ 

=  0, 

(f      ,     6\2       62-40C) 

=  0, 

'""^ex   1     ^        V62-4acl 
J  r  ^  2a              2a         1 

=  0. 

2a 

Since  the  product  is  zero,  one  of  the^  factors  must  be  zero.     But 
a  is  not  zero,  as  the  equation  would  not  then  be  a  quadratic. 

6        V62-4ac       ^  ,     6        v'62-4ac       ^ 

.'.     ^  +  o-  -\ f. =  0  or  a;  +  ;r-  —  5 =  0, 

'2a  2a  2a  2a 

_  —  6±\/62— 4ac 


270  ALGEBRA 

BXERCISB   127 

Solve  by  either  of  the  preceding  methods  : 


1.*  x"-2ax^3a^. 

2. 

x2+46a;-562==0. 

3.     x^—Qmx+3m^=0. 

4. 

x--{-ipx—p^=0. 

5.     x^—2ax+b=0. 

6. 

x^+2bx—c=0. 

7.     ax^+2ax=b. 

8. 

ax^'+2bx+c=0. 

9.     ax^—bx—c=0. 

10. 

px^—qx-\-?-=0. 

189.  Solving  by  Formula.  The  roots  of  any  particular 
quadratic  equation  may  be  found  by  substituting  the  values 
of  a,  b  and  c  in  the  roots  of  the  general  quadratic. 

Ex.  1.— Solve  6x-2-7.i;+2=0. 

Here  a  =  6,  b=  —  l,  c  =  2. 

a   u  ^-^   ^    ^u  1         •  -6±V62-4ac 

feubstitute  these  values  in  x= ~ ? 

2a 

+  7+\/49-48 
•■•     ^=       -^12 

7+1        8  6       2        1 

=  "12~=  1^  "''  r2  =  3  °''  2° 

Verify  by  substitution. 

Ex.  2.— Solve  5a;2+6x-l=-0. 

Here  a=5,  6  =  6,  c=  — 1. 

-6+V'36-(-20)       -6+A/56 

•■•    ^  =  —^^10 =  — I^ 

-6+2\/l4       -S+Vli 


10  5 

In  this  case  the   roots    are    irrational,    but,    if    necessary,    we    may 
substitute  for  v/l4  its  approximate  value  3-742,  when  the  roots  become 

-3  +  3-742       -742         -6-742        ,,„  ,  „,„ 

— =  _        or   = —  =  -148  or  —1-348. 

Note. — The   pupil    is    warned    to    be    careful    of    the    signs    when 
substituting,  particularly  when  c  is  negative. 


THE  GENERAL  QUADRATIC  EQUATION  271 

Ex.  3.— Solve  2a:2-5^+6=0. 

a=2,   6= -5,  c  =  6. 

+  5+v'25-48       5+\/^^23 
4  4 

190.  Imaginary  Roots.  In  the  preceding  result  the  numeri- 
cal value  of  the  roots  cannot  be  found  even  approximately, 
for  there  is  no  number  whose  square  is  negative. 

Such  a  quantity  as  V— 23  is  called  an  imaginary  quantity, 
and  the  roots  in  this  case  are  said  to  be  imaginary.  This  is 
merely  another  way  of  saying  that  there  is  no  real  number 
which  will  satisfy  the  equation  2x2— 5a;+6=0. 

191.  Methods  of  Solving  Quadratic  Equations.  When  a 
quadratic  equation  has  been  reduced  to  the  standard  form,  it 
may  be  solved  : 

(1)  By  factoring,  by  insjiedion  or  by  completing  the  square. 

(2)  By  substitution  in  the  general  formula. 

The  pupil  is  advised  to  try  to  factor  by  inspection,  and  if 
this  method  is  unsuccessful,  then  substitute  in  the  general 
formula. 

As  the  general  formula  will  be  used  very  frequently,  it  is 
absolutely  essential  that  it  be  committed  to  memory. 

The  roots  of  ax^-\-bx-\-e=^0  are — 

2a 


BXE3RCISB 

128 

Solve,  using  the  formula  : 

1.*  3a:2-5x+2=0. 

2. 

24a;2-46a;+21==0. 

3.     575a;2-2a;=l. 

4. 

2a;2— 6.T— 1=0. 

5.     24:lx^+5x=l2. 

6. 

2x2- 13a;+ 10=0. 

7.     391a:2+4a;=35. 

8. 

1200x-2-10a;=l. 

9.     x^+xC3b-2a)  = 

■  6ab. 

10. 

2a;2-25x+77=:0. 

11.     6a;2-a;-l=0. 

12. 

1800x2_5a;-l=0. 

272  ALGEBRA 

Solve  by  any  method.  Verify  13-18  : 

13.     27a;2-24a;=16.  14.     15a;2-f7a;-2=0.     15.     V2x^-x-^=0. 

16.     4d;2-17a;+4=0.  17.     A%Qx^-^x=l.         18.     5~26x+5a;2=0 

19.     9a;+4  =  5a;2.  20.     3x2+2^9a;.  21.     'ia-^— 2a;=f 

22.     4a;2-4a;=79.  23.     l-y=y\  24.       1^--  =  ?. 

3      9      a; 

25.     x-^-~=Q.         26.    ^  +  1  =  J-.  27.     a;^--^a;=l 

a;+2  2^23^  12 

28.     (x--4)2-3(a;-9)  =  15.  29.     (.r-2)(a;+3)-x(5a;-9)-2. 

30.     2aa;2+x(a-2)  =  l.  31.     ac^-  =  -^cx. 

a      x 

32.     2a;(x-2)=a2_2.  33.     ^-f?==^  +  -. 

^         X         o         ^ 

3*-     -4t  +  -4o  =  ^-  35.     (x-+2)2+(a:+3)2=:(x-+6)2. 

a;+l      a;+2      a;+3 

36.  x^-xy-Zy^=-\2.     If  ?/=2,  find  a:. 

37.  a;2— 4a;y+x3+«/2+5=0.     If  a:=— 3,  find  ?/. 

x         x-\- 1 

38.  If = ,  find  x  to  three  decimal  places. 

x+\        2x  ^ 

39.  Find  the  sum  of  the  roots  of  a;^— 3a;=20. 

40.  The  area  of  a  square  in  square  feet  and  its  perimeter  in  inches 
are  expressed  by  the  same  number.     Find  the  side  of  the  square. 

41.  The  length  of  a  rectangular  field  exceeds  the  width  by  16  rods 
and  the  area  is  32  acres.     Find  the  length. 

42.  Find  three  consecutive  even  numbers  whose  sum  is  ^  of  the 
product  of  the  first  two. 

43.  A  line  10  inches  long  is  divided  uito  two  segments  so  that  the 
square  on  the  longer  segment  is  equal  to  the  rectangle  contained  by 
the  whole  line  and  the  shorter  segment.  Find  the  segments  to  two 
decimal  places. 

44.  Find  two  numbers  whose  difference  is  3  and  the  sum  of  whose 
squares  is  317. 

45.  The  area  of  a  square  is  doubled  by  adding  5  inches  to  one  side 
and  12  inches  to  the  other.     Find  the  side  of  the  square. 


THE  GENERAL  QUADRATIC  EQUATION  273 

46.  Three  times  the  square  of  a  number  exceeds  eight  times  the 
number  by  unity.     Find  the  number  to  three  decimals. 

47.  Mr.  Gladstone  was  born  in  the  year  a.d.  1809.  In  the  year 
A.D.  x'^  he  was  x— 3  years  old.     Find  x. 

48.  The  area  of  a  rectangular  field  is  half  an  acre.  The  perimeter 
is  201  yards.     Find  the  sides. 

49.  One  root  of  x^—bx-\-d=0  is  8.  Find  the  value  of  d  and  the 
other  root. 

50.  If  a  train  travels  10  miles  per  hour  faster  than  its  usual  rate, 
it  will  cover  480  miles  in  4  hours  less  time.     Find  its  usual  rate. 

51.  Divide  3  into  two  parts  so  that  the  sum  of  their  squares  may 

52.  I  buy  a  number  of  articles  for  84"80  and  sell  for  S5"95  all  of 
them  but  2  at  6  cents  a  dozen  more  than  they  cost.  How  many  did 
I  buy? 

53.  A  straight  line  AB,  12  inches  in  length,  is  divided  at  C  so  as 
to  satisfy  one  of  the  following  conditions.  Find,  in  each  case,  the 
length  of  AC  to  two  decimals  : 

(1)  AC^^2BC\  (2)  AC^=2AB  .  BC. 

(3)  3.4C'2=4.4£  .  BC.  (4)  AC^+^BC-^=2ABK 

(5)  .4C'2-5C2=10sq.  in.  (&)  AC{AB+BC)  =  2  sq.  it. 

54.  I  buy  a  number  of  books  for  $6,  the  price  being  uniform.  If 
they  had  been  subject  to  a  discount  of  5  cents  each,  I  could  have 
bought  6  more  for  the  same  money.     What  did  each  cost  ? 

55.  Solve  the  equation  ax^-^bx+c^O  by  multiplying  by  4a  and 
completing  the  square  of  2ax-\-h. 

o6.     Solve  —J — -\ „ — —  =  -  • 

2—x       2+x        a;2— 4       3 

Verify  the  roots  obtained. 

192.  Equations  Solved  like  Quadratics.  There  are  certain 
types  of  equations  of  a  higher  degree  than  the  second,  which 
may  be  solved  by  reducing  them  to  the  form  of  quadratics. 

T 


274  ALGEBRA 

Ex.  1.— Solve  x^-lOx^+d^-0. 

This  is  an  equation  of  the  fourth  degree,  but  we  might  write  it  in 
the  form  of  a  quadratic,  tlius  : 

(a,.2)2-10(a;2)  +  9  =  o, 

or  if  we  write  y  for  x^  it  takes  the  form 

.-.     (y-9)(y-l)  =  0, 

y=d  or  1. 
Buty  =  x',  .'.  a;^=9  or  1, 

.r=  +  3  or  ±1. 

We  see  that  this  equation  has  four  roots.     This  is  what  we  might 
expect,  as  it  is  an  equation  of  the  fourth  degree  in  x. 
Verify  each  of  the  four  roots. 

Ex.  2.— Solve  (:^2_5^^)2^4(^2_5^)_12^0. 

Here  we  consider  x~  —  5x  as  the  unknown,  whose  value  should  first 
be  found. 

Let  cc^  — 5a;  =  t/, 

2/2  +  4?/- 12  =  0, 

.-.      (y+6)(y-2)  =  0. 


y 

=  ~ 

6 

or 

2. 

x^—5x=—Q, 

or 

a;2-5a;  =  2, 

a;2-5a;+6  =  0. 

2. 

a;2_5a;_2  =  0. 

(x-^)(x-2)  =  0, 

a;  =  3  or 

5+\/25  +  8 
^-           2 

5±V33 

This  equation   has  foiu-  roots,  two  of   which  ai-e  rational   and   the 
other  two  irrational. 

Verify  the  rational  roots. 

Ex.  3.— Solve  {2x^-\-3x-l){2x^+3x-2)=56. 

Let  2x"  +  3x  =  y. 

rp,            u  •            3       o    -3±V'^^9 
J  he  result  is  x  =  jr,  —3, • 


THE  GENERAL  QUADRATIC  EQUATION  275 

Ex.  4.-Solve        -X-+.^+2.=  5- 

-    ,  a;2  +  2a;  3  1 

^^'  "3-=^'  •■■     a^^  =  2/' 

1        26 

y      5 

Complete  Ex. 's  3  and  4  and  verify  the  rational  roots. 

Ex.  5.— Solve  .6-3-1=0. 

Factoring,  {x—l){x~  +  x-\-\)=(), 

x  —  1  =  0  or  a;^  +  x  +  1  =  0, 

x=i  or  X  =  ^ • 

We  thus  see  that  if  one  root  of  an  equation  of  the  third 
degree,  or  a  cubic  equation,  can  he  found  by  factoring,  the 
equation  can  he  completeUj  solved. 

This  equation  might  be  written  ,^^=1,  and  each  of  the  three 
roots  when  cubed  must  give  unity,  which  shows  that  unity 
has  three  cube  roots.  This  is  what  we  might  have  expected, 
as  we  have  aheady  seen  that  unity  has  two  square  roots 
+  1  and  —1. 

EJXBRCISE  129 

Solve  and  verify  the  rational  roots  : 

1.     a;*-5x2+4=0.  2.     a;4-13a;2+36=0. 

3.     V  + 12=317/2.  4.     8.c6-65a;=»+8=0. 

5.     (a;H5a;+6)(a;2-9a;+14):-0.  «.     ^+16_^      ^^     ^  2. 

V     T      T   A  ^     )  25     ^a;2+16 

42 
7.     (a;2-4a;+5)(x2-4x+2)  =  -2.        8.     a;^  +  x  +  1 


9.     (.r2+x+l)2-4(a;*+a;+l)+3=0. 

10.  x-3-4a;2-4x+16=0. 

11.  6('.r  +  -y-35('x+^')+50=0. 

12.  (l+a;+a;2)(x+x2)  =  156. 

T  2 


276  ALGEBRA 

13.*  {x+l)(x+2)(x+S){x-\-4:)^120.     (Multiply    the    first    and    last 
factors  and  the  second  and  third.) 

14.  x{x-\){x-2){x-3)=360. 

15.  Find  the  three  cube  roots  of  8  by  solving  the  equation  x^— 8=0. 

16.  Find   the  four  fourth   roots   of    16   by   solving   the  equation 

17.  Solve  .c^— 19a; +30=0  being  given  that  3  is  one  of  the  roots. 

18.  Solve  12x3—29x2+23^—6=0  (use  the  factor  theorem). 

19.  It  is  evident  that  4  is  a  root  of  the  equation 

.r(a;-l)(x-2)  =  4.  3.2. 
Find  the  other  two  roots. 

20.  Find  the  six  roots  of  8x^—211x^+2:  =0. 

21.  Solve  (x-~xY^-8{x^~x)  +  l2=0. 

22.  Solve  x^  +  — -  +  a;  +  -  =  4.      (Add   to  x^  +    -    the    quantity 

x^  X  \  x^ 

required  to  make  it  the  square  of  x  H j 


EXERCISE  130  (Review  of  Chapter  XXI) 

1.  Explain  the  different  methods  of  solving  quadratic  equations. 
Illustrate  them,  by  solving  in  full  the  equation  3x^  — 4x— 15  =  0,  by 
each  method. 

2.  Solve  323x2 +  2x=l. 

3.  The  difference  of  two  numbers  is  8  and  the  sum  of  their  squares 
is  104.     Find  the  numbers. 

4.  If  X  =  2f  1+^, ),  find  x  to  two  decimal  places. 

5.  What  is  the  price  of  meat  per  lb.  if  a  reduction  of  20% 
in  the  price  would  mean  that  5  lb.  more  than  before  can  be  bought 
for  $3  ? 

6.  Solve  10x2- 19x- 9  =  0. 

7.  The  sides  of  a  right-angled  triangle  are  a,  a— 10  and  a+10. 
What  are  the  sides  ? 


THE   GENERAL  QUADRATIC   EQUATION  277 

8.  Solve  ^  +  -^  =  4. 

a;  —  4       x—b 

9.  The  sum  of  two  numbers  is  45  and  tlie  sum  of  their  reciprocals 
is  -09.     Find  the  numbers. 

10.  Solve  6375a;2-  10a;=  1. 

11.  The  length  of  a  rectangular  field  is  o  rods  more  tiian  the  width. 
The  area  is  3^  o.cres.     Find  the  sides. 

\Qa-\-2n'^ 

12.  What  must   be  the  values  of  n  in  order  that    ,^ — — ;r^    may 

10«  +  21« 

equal  |  when  a  =  nV  ' 

13.  The  perimeter  of  a  rectangle  is  5G  and  the  area  is  192.  Find  the 
diameter  of  the  circle  which  passes  through  its  angular  points. 

14.  Solve  •0075x2  + •75a;=  150. 

15.  By  solving  (x— 2)(a;— 3)  =  (a  — 2)(a  — 3),  find  a  quantity  which 
can  be  substituted  for  a  in  (a  —  2)(o.—  3)  without  changing  its  value. 

16.  Solve  a;3- 2x2 -89^+90  =  0. 

17.  Two  trains  each  run  330  miles.  One  of  them,  whose  average 
speed  is  5  miles  per  hour  greater  than  the  other,  takes  \  an  hour  less 
to  travel  the  distance.     Find  their  average  speeds. 

18.  Solve  f±j^  +  J^^  =  2J. 

19.  Solve^^  +  -^  =  — . 

20.  I  sell  a  horse  for  $96  and  gain  as  much  %  as  the  horse 
cost  in  dollars.     What  was  the  cost  ? 

21.  Solve  (.T2-3a;-5)2+8(x2-3x-5)  +  7  =  0. 

22.  Divide  25  into  two  parts  so  that  the  sum  of  the  fractions  formed 
by  dividing  each  part  by  the  other  may  be  4-25. 

23.  The  sides  of  a  rectangular  field  are  a;+17  and  x— 17.  The 
diagonal  is  50.     Find  the  area. 

24.  Solve  a;2  +  .r+    ,'~  =18. 

x^->rx 

25.*  Solve  (w2_.^2)a;2^2x(?n2  +  n2)  +  m2-n2  =  0. 

26.  Solve  (x-2)(.-c-l)(.T+2)(x+3)  =  60. 

27.  Find  all  the  roots  of  the  equation  x^=  125. 

28.  Since  x^  — 8a;+12  =  (x— 2)(a;— 6),  for  what  values  of  x  will  the 
expression  x*  — 8x+12  be  equal  to  zero,  and  for  what  values  will  it 
be  negative  ? 


278  ALGEBRA 

^^       c,  1  «  i>  a  —  b 

29.     Solve r  =  r- 

x  —  a       x  —  b       a-\-b 

.'50.     8olve  adx  —  acx"  =  bcx~bd. 

31.  The  area  of  a  square  is  trebled  by  adding  10  inches  to  one 
side  and  12  inches  to  the  other.     Find  the  side  of  the  square. 

32.  Solve  a;2(a2-c2)-a;(a6  +  36c)-262  =  o. 

33.  Solve  (a;2-|-6x+8)2  +  3.T(a;2  +  6.T  +  8)  =  0. 

34.  A  man  bought  a  number  of  acres  for  S300.  If  he  had  paid 
$5  more  per  acre,  the  number  of  acres  would  have  been  2  less.  Find 
the  number  bought. 

o.  o     ,  1  111 

35.  Solve  — --,  =-+-+--. 

x+a+o       x       a       h 

36.  Solve  — -. \- = ,-  • 

b  a  x  —  a       x—b 

37.  OX  and  OY  are  two  roads  at  right  angles.  A  starts  at  noon 
along  OX  at  3  miles  per  hour.  B  starts  at  2  o'clock  along  O  F  at  4  miles 
per  hour.     Find  to  the  nearest  minute  when  they  will  be  20  miles  apart. 

38.  Solve  a2a;2-2a3a;  +  a*-l  =  0. 

6c" 

39.  Solve  oa;2— ,  =cx  —  bx^. 

a-\-b 

40.  A  gravel  path  2  yards  wide  is  made  round  a  square  field  and 
it  is  found  that  it  takes  up  xV  of  the  area  of  the  field.  Find  the  area 
of  the  field  in  square  yards. 

41.  Solve  s  =  i?<+16<2  for  <. 

42.  What  positive  integer  is  that,  the  sum  of  whose  square  and  cube 
is  nine  times  the  next  higher  integer  ? 

43.  vSolve  (a:2  +  a;-2)2-4(a;2  +  .r-2)  +  3  =  0. 

44.  The  side  of  a  square  is  34  inches.  Find  at  what  points  in  the 
sides  the  vertices  of  an  inscribed  square  must  be  placed  so  that  it 
may  have  an  area  of  676  square  inches. 

45 .  Write  the  equation  ax^  +  bxy  +  c^/^  =  0  as  a  quadratic  in  -  .    What 

X  V 

are  the  values  of  -  and  of  -  ? 

y  X 

46.  What  positive  integral  value  of  x  will  make  a;*+10x  most 
nearly  equal  to  1000  ? 


CHAPTER  XXII 
SIMULTANEOUS  QUADRATICS 

193.  Consider  the  problem  :    The  sum  of  two  numbers  is  12 
and  the  sum  of  their  squares  is  74.     Find  the  numbers. 

Let  a;  =  one  of  the  numbers, 

12  — a;  =  the  other, 
.-.     a;2  4-(12-a;)2  =  74. 
Solve  this  equation  and  find  a;  =7  or  5. 
If  a;  =7  or  5,  then  12  — a;  =  5  or  7 
/.    the  numbers  are  5  and  7. 

Here  we  have  used  only  one  unknown.     We  might  have 
solved  by  using  two  unknowns. 

Let  X  and  y  be  the  numbers, 

.-.      x  +  y=\2, 
and  a;2  +  ?/-  =  74. 

How  can  we  obtain  from  these  two  equations  the  original  equation 
in  the  preceding  sohition  ? 

194.  Type  I. 

Ex.  1.— Solve  a;+3.y=10,  (1) 

x^-\-xy^=4:.  (2) 

From(l),  a;=10-3t/  (3) 

Substitute  in  (2),  (10-32/)2  +  y(10-3/y)  =  4, 

.-.      100-()0;y  +  9(y2+10i/-32/2  =  4, 

6^/2 -501/ +  96  =  0, 

3.V*- 252/ +  48  =  0, 

(.V-3)(3?y-16)  =  0, 

2/  =  3orJ/. 
279 


280  ALGEBRA 

Substitute  y=  3  in  (3)  and  x==l. 

!»  y  =  ^  "     ;>     !»     a;=— 6. 

There  are  therefore  two  solutions, 

x=  1,  2/  =  3  or  a;=  —6,  y  =  5^. 

x=  1  or  —6, 

i/  =  3  or  5i. 

Verify  by  showing  that  x=l,  y  =  3  satisfies  both  equations  and  also 
x=-6,  2/  =  5i. 

The  pupil  must  note  that  x=l  was  obtained  from  ?/=3, 
not  from  y=5^. 

Therefore,  x=l,  y=5J  is  not  a  sohition,  nor  is  x=  —  6, 
y=3.     Verify  this  by  substitution. 

Equation  (1)  is  a  Unear  equation,  or  an  equation  of  the 
first  degree  in  x  and  y.  Equation  (2)  is  a  quadratic  equation, 
or  an  equation  of  the  second  degree  in  x  and  y. 

A  system  of  equations  of  this  type,  that  is,  where  one  is 
of  the  first  degree  and  the  other  of  the  second  degree,  may 
always  be  solved  by  the  method  of  substitution,  which  does 
not  differ  from  the  similar  method  employed  in  art.  107, 
when  both  equations  were  of  the  first  degree. 

Ex.  2.— Solve  3x~y^5,  (1) 

x^+3xy=l5.  (2) 

From  (I),  2/  =  3x-5,      .-.    a;2  +  3n;(3x-5)=  15, 
.-.   10,r2-15a;-15  =  0, 
2x2- 3a;- 3  =  0, 
3±V3^      3+5-745 


4  4 


2-186  or  -686. 


.-.     y  =  3x-  5=  -Ai^'^^  =1-558  or  - 7-058. 

Here  the  roots  are  irrational  and  it  is  customary  to  leave  them  in 
that  form,  unless  the  decimal  form  is  asked  for. 


SIMULTANEOUS  QUADRATICS  281 


EXERCISE 

181 

Solve  and  verify 

1-6 

1.     x+y^l. 

2.     .T— ?/  =  4, 

3.     x-2y=0. 

xy=l2. 

xy=m. 

x^-y'^=21. 

4.     x-y^3. 

5.     x—y=6. 

fi.     2.r+//=9, 

x^+y^=m. 

x^-y^  =  60. 

a;2-i/2=15. 

7*  x+Sy=U, 

S.     2x+3y=l2, 

9.     3a:— 4?/=2, 

x''+y=21. 

a;2  +  ?/2=13. 

3a;2+2?/2=140. 

10.     x2+3a;!/+«/2+2.r=37,  11.     3x2— 2a;?/+5a;— y=17, 

x—y=Z.  2x—Zy=\. 

12.  If  a;— 3?/=2  and  x'^—xy^2y"=&,  find  the  values  of  x  and  ?/ 
to  three  decimal  places. 

13.  The  hypotenuse  of  a  right-angled  triangle  is  25  and  the 
perimeter  is  56.     Find  the  sides. 

14.  A  is  10  years  older  than  B.  Eight  years  ago  the  sum  of  the 
squares  of  the  numbers  representing  their  ages  was  148.  Find  their 
ages. 

15.  The  diagonal  of  a  rectangle  is  50.  The  difference  of  the  sides 
is  10.     Find  the  area. 

16.  The  area  of  a  right-angled  triangle  is  96  and  the  difference 
of  the  two  sides  about  the  right  angle  is  4.     Find  the  hypotenuse. 

17.  Solve  3x4- 52/^:2,  Zx^-my^-xy^2%=(i. 

18.  If  each  digit  of  a  number  be  increased  by  2,  the  product  of  these 
increased  digits  wUl  be  the  original  number.  When  the  digits  are 
interchanged  the  resulting  number  is  thirteen  times  the  tens  digit  of 
the  original  number.     Find  the  number. 

19.  The  sum  of  the  areas  of  two  squares  is  40  square  inches.  The 
side  of  the  smaller  is  10  inches  less  than  three  times  the  side  of  the 
larger.     Find  their  sides  to   three  decimals. 

20.  Solve  -  +  5-=  14,  i/-l=a;. 

y-      y 

195.  When  both  equations  are  of  the  second  degree  in  x 
and  y,  they  can  not  always  be  solved  by  elementary  methods. 


282 


ALGEBRA 


There  are  special  cases  in  which  they  can  be  solved  without 
difficulty. 


196.     Type  II. 

Solve 

Factoring  (1), 


{x-4ij){x-y)  =  0, 
x=4y  or  x=y. 

We  are  now  required  to  solve  : 


x'^-{-y'^-\-^x- 


=  291 


and 


Substituting  the  value  of  x, 

16,/2  +  y2+122/  =  29, 
17i/2+122/-29  =  0, 
(2/-l)(172/  +  29)  =  0, 
y—\  or  —  ?§, 
x  =  4  or  —  TV• 


-2/2+3x=29^ 
x  =  y    f 


y'+y'-  +  3y  =  29, 
2y''  +  3y-29  =  0, 
■3+\/24T 


y 


-3+\/241 


(1) 
(2) 


Here  there  are  four  solutions  ; 
4  or  — 


X 


1  or 


-3+V241 
4 

-3±V24l 
4 


In  this  type  the  first  equation  contains  only  terms  of  the 
second  degree.  When  that  is  the  case  the  left-hand  member 
may  be  factored  and  each  of  the  resulting  linear  equations 
may  be  combined  with  the  second  equation,  thus  giving  two 
cases  of  Type  I. 


EXERCISE   132 

Solve  and  verify  1-5  : 

1.             x-—y"=0. 

2.     x~—4xy+3y^=0. 

x-+xy+y-=36. 

x^+y^=lO. 

3.     3x^-2xy-y^=0, 

4.     x^+y^+2x=l2. 

x+y+y^=32. 

3x^+2xy=yK 

SIMULTANEOUS  QUADRATICS  283 

5.     4x2+20.r;/+%2=0,  fi.*    •^'"  + '^  =  14, 

ir      y 

2.1//+ 1=0.  .T- 1-7/2. 

7.     6.r2-17xy/+12^-=0,  8.     ,'1:2+2^^=5, 

x'^—xy—y^\.  Qx'^^Ay-^Wxy. 

9.     Find  four  solutions  of  the  equations 

(x--^)(.r-2)=0,  U-+y/-6)(//+3)=0. 

197.     Type  III.     Homogeneous  Equations. 

Solve  x^—xij=Q,  (1) 

y/2+3x-y=10.  (2) 

Multiply  (1)  by  5  and  (2)  by  3  and  subtract,  to  eliminate  the  absolute 
terms,  and  we  get 

,5x2- 14x7/- 3y  2  =  0.  (3) 

This   equation    (3)    is   of   tlie   same    form   as   the   first   equation    in 
Type  II.     Grouping  (3)  with  (1)  we  proceed  as  before. 


Factoring  (3),                     {x—Zy)(Z 

a;  +  2/)  =  0, 

x  =  Zy  or  —ly. 

Substitute  x—Zy  in  (1), 

Substitute  x 

--i7/in(l). 

.-.      9y2_37/2  =  G, 

izV 

Hl2/2  =  6, 

2/-^=l, 

2/2  =  25, 

2/=±l, 

2/=±5, 

.^=±3. 

_•_ 

a;=  + 1- 

Hence  the  four  solutions  are  : 

x  =  3,     ^^     a;=-3,      ^^ 
or                         or 
y=\.             2/=-l. 

x=-\, 

7/ =  .5. 

or 

a;=l. 

Verify  each  of  these  four  pairs  of  roots. 

If  we  had  grouped  (3)  with  (2),  the  results  would  have  been  the  same. 
Show  that  this  is  true. 

In  this  type,  terms  of  the  first  degree  were  absent  from 
both  equations.  The  expression  on  the  left  in  each  is  homo- 
geneous, that  is,  every  term  is  of  tlie  same  degree.  For  this 
reason,  this  is  called  a  homogeneous  system. 


284  ALGEBRA 

The  pupil  should  be  on  the  look  out  for  special  methods  of 
obtaining  from  the  given  equations  an  equation  of  the  first 
degree.  Here  we  might  have  done  so  by  simply  adding  the 
equations  and  taking  the  square  root.  Solve  it  by  this 
method. 

EXERCISE   133 

Solve  and  verify  1-9  : 

1.     3x^-5!j"=28,        2.  2.i-2-3?/2=23,  3.     x^-xy+y^=2\, 

Sxy—4:y^=8.  2xy-3y^=3.  2xy—y^=15 

4.     2x"—3xy^l4:,        5.  x-+xy^66,  6.     x~~xy=54:, 

3y'—x^+l=0.  x'^-y"=U.  xy—y^=18. 

7.     x^+2xy=32,        8.  3x^-5xy+2y"=14:,    9.     x^~-iy^=20, 

2y^+xy=l6.  2x--5xy+3y^=6.  xy=12. 

10.*       x^-3y^^4,     11.  x^+xy+y^=7,  12.     32y^^-2xy+U, 

x^+xy+y^-=2S.  3x^-l=xy.  x^+'kij^=10. 

13.       2x^-9xy+9y^=5,  14.  x^+xy+tj^=7, 

4:X^—10xy+Uy^=35.  2x^+3xy+4y^=24:. 

15.  3x^~3xy-^2y^=2x,  2x^+3y^~-xtj=4:X. 

16.  Find,  to  two  decimals,  the  real  values  of  x  and  y  which  satisfy 
x^—xy=20  and  3xy—y^=50. 

17.  When  a  number  is  multiplied  by  the  digit  on  the  left  the  product 
is  105  ;  when  the  sum  of  the  digits  is  multiplied  by  the  digit  on  the  right 
the  product  is  40.     Find  the  number. 

198.     Special  Methods. 

Since  {X'\-y)^—{x—yy'^-\-4:Xi/,  it  follows  that  if  the  values 
of  any  two  of  the  quantities  .t -(-?/,  ^—y  and  xy  are  given,  the 
remaining  one  can  be  found. 

Ex.  1.— Solve  x-\-y=U,  (1) 

xy=l8.  (2) 

Squaring  (1),  x''  +  2xy  +  y^=l2l. 

From  (2),  4xy  =  72. 

Subtracting,  x'^  —  2xy-\-y'  =  4:9, 

:.     x—y—±7. 


SIMULTANEOUS  QUADRATICS  285 


If                x-[-y=U,                                  If 

x  +  y=n. 

and                 x—y  =  l.                               aiid 

x-y=-l. 

a;  =  9,  y  =  2. 

x  =  2,y  =  9. 

Hence  there  are  two  solutions  : 

a;  =  9  or  2, 

2/  =  2  or  9. 

Ex.  2.— Solve  .T— ?/=ll,  .r//=60. 

Find  {x-\-y)^  by  adding  4a;^  to  (x  —  y)'  and  complete  the  solution. 

Ex.  3.— Solve  .t3+^3^35,  (1) 

x-^ij=5.  (2) 

Dividing  (1)  by  (2),  x^-xy  +  y^  =  7,  (3) 

Squaring  (2),  a;2  + 2x2/ +  2/2  =  25. 

Subtracting,  3xy=lS, 

xy  =  6.  (4) 

Subtracting  (4)  from  (3),        x^  —  2xy  +  y^=\, 

x—y=  +  l. 
Complete  the  solution  as  before. 
Also  solve  by  substituting  x  =  5~-y  from  (2)  in  (1). 

Ex.  4.— Solve  a:4+a;V+2/''=91>  (1) 

.»--+^2/+y'=13.  (2) 

x*-\-x^y^-^y*  =  {x^-{-xy-{-y^){x^  —  xy+y^). 

Dividing  (1)  by  (2),  x^-xy  +  y^==l.  (3) 

Subtracting  (;5)  from  (2),  2xy  =  6, 

xy  =  3.  (4) 

Adding  (2)  and  (4),  x-  +  2xy  +  y''=H\ 

:.      x  +  y=±4.  (5) 

Similarly  from  (3)  and  (4),  x  —  y=  +  2.  (0) 

(5)  and  (6)  can  be  grouped  in  four  ways,  thus  : 

a;+w  =  4,  x  +  v=  —4,  a;  +  w  =  4,  x  +  y=  —4, 

\  a        'or  •^  or  ^  or  ^ 

x  —  y  =  2.  x  — 2/=  — 2.  X— ?/=— 2.  a;— 2/  =  2. 

From  these  four  solutions  are  obtained  : 

x=3,   -3,  1,   -1,  x=+3or+l, 

y=\,  -1,  3,   -3.  y=±lor+3. 


286 


ALGEBRA 


Ex.  5.— Solve        {x+y)^-5{x+ij)-Q--^0, 

xy=8. 


Factoring  (1), 


Now  solve 


{x  +  y-6)(x  +  y+\)  =  0, 
a;+2/  =  6  or  —  1. 

xy  =  8.  xy  =  8. 


(1) 

(2) 


EXERCISE   134 

Solve,  by  finding  x+y  and 

x-y. 

and  VI 

erif y  : 

1. 

xJf-y=8,               2.     :i 

:-y= 

4, 

3.     x2+!/2=25, 

xy=lb. 

xy= 

12. 

x—y=\. 

4. 

a;2+2/'=61,          5.     (: 

v-yY 

'=1, 

6.     x-—xy+ij^=51. 

a;+?/=ll. 

xy 

=30. 

x-y=8. 

7. 

x^+xy+y~=19, 
x+y=5. 

8. 

x^-xy+y^=19, 
a:+2/=13. 

9. 

5x'^  +  xy+5y~=23, 
x+y=l. 

10. 

a;?/ =40. 

11. 

x^—7xy+y'^=  —  101, 
xy=SO. 

12. 

2a;2+3a;i/+2//2=8, 

x-?/=  —6. 

13. 

X3_7/^=19, 

x-y=l. 

14. 

x^+y^=l064:, 
x+y=U. 

15. 

x-—xy^y^=39, 
a;3+?/3=351. 

16.=" 

'  x'+x^y^+y*=2l, 
x''+xy+y^=l. 

17. 

x*-\-x^y^+y*=l33. 
x^—xy+y"=7. 

18. 

x^-xhj'-^y^  =  n, 
xy=2. 

19. 

(x+yr--3{x-hy)-2S-- 

=0,  .X- 

-y=i 

20. 

{x-yy~-l{x-y)  +  l2: 

=0,  xy=\2. 

21. 

x^y^-27xy +180^0,  ; 

«+Z/= 

=8. 

22.     The  perimeter  of  a  rectangle  is  34  inches  and  the  diagonal  is 
13  inches.     Find  the  sides. 


SIMULTANEOUS  QUADRATICS  287 

23.  The  diagonal  of  a  rectangle  is  25  and  the  area  is  300.  Find  the 
sides. 

24.  The  sum  of  two  numbers  is  12  and  the  sum  of  their  squares 
is  72-5.     Find  the  numbers. 

25.  The  product  of  two  numbers  is  270.  If  each  number  is  decreased 
by  3  the  product  will  be  180.     Find  the  numbers. 

2G.  The  sum  of  two  numbers  is  10  and  the  sum  of  their  reciprocals 
is  f*5.     Find  the  numbers. 

27.  Solve  (x-l)(y+2)=9,  2a-//=  15. 

28.  A  and  B  are  two  squares.  The  area  of  A  is  63  square  inches 
more  than  B,  and  the  perimeter  of  4  is  12  inches  more  than  B. 
Find  the  side  of  each. 

29.  Find  two  numbers  whose  product  is  1  and  the  sum  of  whose 
reciprocals  is  2^\7. 

30.  Solve  x^—%if=b%,  x-2ij=2. 

31.  The  sum  of  the  two  digits  of  a  number  is  1  of  the  number. 
The  sum  of  the  squares  of  the  digits  is  4  less  than  the  number.  Find 
the  number. 

32.  The  area  of  a  rectangle  is  1161  square  yards,  and  its  perimeter 
is  140  yards.     Find  the  dimensions. 

33.  Solve  1  +  1  =  -3,  -  -  ^  =  -03. 

X      y  X-       7/^ 

34.  The  sum  of  a  number  of  two  digits  and  the  number  formed  by 
reversing  the  digits  is  121.  The  product  of  the  digits  is  28.  Find  the 
number. 

35.  Find  the  sides  of  a  right-angled  triangle  whose  perimeter  is 
24  inches  and  whose  area  is  24  square  inches. 

36.  Prove,  algebraically,  that  if  two  rectangles  have  equal  areas 
and  equal  perimeters,  they  are  equal  in  all  respects. 

37.  Soh'e  x'^-\-xy-[-y'^=l-15,  x-—xy-\-y^=5-25. 

38.  What  must  be  the  dimensions  of  a  rectangular  field  containing 
7J  acres,  if  the  greatest  distance  from  any  point  in  its  boundary  to 
any  other  point  is  50  rods  ? 


288 


ALGEBRA 


39 .  The  sum  of  the  radii  of  two  circles  is  8  inches  and  the  sum  of 
their  areas  is  |  of  the  area  of  a  circle  whose  radius  is  9  inches.  What 
are  their  radii  '! 

40.  What  must  be  the  length  of  a  rectangular  field  that  contains 
a  square  rods  and  which  can  be  enclosed  by  a  fence  b  rods  long. 


199.     Graphical    Methods.      What   is    the    distance    of    the 
point  P(4,  3)  from  the  origin  0  ? 


Since 


(9P2. 
OP^-- 

OP  -- 


-OM^+MP^, 

:42+32=25, 
^5. 


If  any  point  {xij)  is  the  same 
distance  from  the  origin  that  P  is, 
then  the  point  {x,y)  must  lie  on 
a  circle  whose  radius  is  5  and 
whose  centre  is  0.  But  the 
square  of  the  distance  of  the  point 
{x,y)  from  the  origin  is  x^-\-y^, 


It  is  thus  seen  that  the  equation  x'^'\-y^=25  represents  a 
circle  ivhose  radius  is  5  and  whose  centre  is  the  origin. 

Similarly,  x^-{-y^=lQ,  x^'j-y^=lOO,  x^-iry^=\8,  represent  circles 
with  the  origin  as  centres  and  whose  radii  respectively  are  4,  10,  VlS. 

It  is  seen  that  it  is  a  simple  matter  to  draw  the  graph  of 
the  equation  of  the  circle  in  the  form  x^-\-y^=r^.  All  we 
require  to  do  is  to  describe  with  the  compasses  a  circle  whose 
centre  is  the  origin  and  whose  radius  is  r. 

When  the  radius  is  a  surd  as  in  x^-[-y^=18,  it  is  simpler 
to  find  a  pair  of  values  of  x  and  y  which  satisfy  the  equation. 
Here  .t=3,  y=3  satisfies  the  equation,  and  the  circle  is  then 
described  through  the  point  (3,  3). 


SIMULTANEOUS  QUADRATICS 


289 


200.  Graphical  Solution  of  Simultaneous  Equations 
"Solve  x^-\-y"=2o,  (1) 

x-y=\.  (2) 

(1)  represents  a  circle  whose  radius  is  5. 

(2)  represents     a     straight     hne,     two 
points  on  which  are  (1,  0)  and  (0,  —  1). 

The  graphs  of  (1)  and  (2)  are  shown  in 
the  diagram. 

The   hne   cuts    the  circle  at  the  points 
(4,  3)  and  (-3,  -4). 

.".    the  roots  of  the  given  equations  are 

a;  =  4  or  —3, 
2/ =  3  or  —4. 

201.  Equal  and  Imaginary  Roots. 

Solve,  (1)  a:'^+//'^=18,  x-y=0. 

(2)  a;2+i/2=18,  x+?/=6. 

(3)  a;2+y2^18,  a;+?/=8. 


y 

- 

^ 

"" 

■~~' 

K 

/ 

/ 

/ 

\ 

/ 

\ 

K 

^ 

/ 

X 

1 

/ 

/ 

^ 

/ 

/ 

'^y 

k 

^ 

_, 

^ 

- 

^ 

■A 

y 

~ 

N 

N 

yu 

n 

\ 

J 

/ 

N 

s 

''N 

<y 

/ 

X 

^\«> 

_^ 

^^"^ 

kI 

jJn 

< 

/ 

y 

Ny 

\ 

/ 

' 

N 

\ 

X'l 

/ 

N 

s 

X 

/ 

? 

^ 

\ 

^ 

/ 

J 

\ 

V 

/c 

'>\ 

/ 

\ 

^ 

% 

i.  1 

y 

/ 

1  r 

^ 

The   diagram   shows   that   the 
roots  of  (1)  are 


of  (2)  are 


x*=3  or  —3, 
?/=3  or  —3 
a;=3  or  3, 
y=Z  or  3. 


The  roots  of  (2)  are  equal,   as 
the  line  a;+?/=6  touches  the  circle 
at  the  point  (3,3).    We  might  say 
that  in  this  case  the  line   meets 
the  circle  at  two  points  which  happen  to  be  coincident. 

The  diagram  shows  that  the  line  a;+y=8  does  not  meet 
the  circle  at  all,  and  there  are  no  real  values  of  x  and  y  which 
will  satisfy  (3).     The  roots  in  this  case  are  imaginary. 

Solve  these  equations  by  the  usual  methods  and  see  if  the 
results  agree  with  the  diagram. 

U 


290  ALGEBRA 

EXERCISE  135 

1 .  On  the  same  sheet  draw  the  graphs  of  the  circles  whose  equations 
are  x^+y^=4:,  x^+y^=9,  x^+7j^=l3,  x'^+y^=34:. 

2.  Solve  graphically  x^-'ry^=13,  x—y=\. 

3.  Find  graphically  the  positive  integral  roots  of  x^+y'^~25 
and  2x+3y=18  ;  x^+y^=10  and  2x~y=5.  Approximate  to  the 
other  roots. 

4.  The  sum  of  two  numbers  is  8  and  the  sum  of  their  squares 
is  25.  Show,  graphically,  that  this  is  impossible.  Is  it  impossible 
if  the  sum  of  the  numbers  is  7  instead  of  8  ? 

EXERCISE  136  (Review  of  Chapter  XXII) 

1.  Solve  x+?/  =  28,  a;2-y2  =  336. 

2.  Solve  5a;-2y=12,  25a;2-4y2  =  96. 

3.  The  sum  of  two  numbers  is  10  and  the  sum  of  their  squares 
is  58.     Find  the  numbers. 

4.  Solve  2a;-32/  =  4,  :r2+(y-  =  29. 

5 .  Solve  Sx  -  41/  =  4,  2x^  +  3xy=^56. 

6.  The  sum  of  two  numbers  is  5  and  the  sum  of  their  reciprocals 
is  |.     Find  the  numbers. 

7.  Solve  x^  +  xy  +  2y^-2x-7y  +  5=^0,  x+y^3. 

8.  Solve  a;2  +  a^- 62/2  =  0,  x^  +  3xy-y^  =  36. 

9.  A  field  whose  length  is  to  its  breadth  as  3  to  2  contains  664 
square  rods  more  than  one  whose  length  is  to  its  breadth  as  2  to  1. 
The  difference  of  their  perimeters  is  60  rods.  Find  the  dimensions  of 
each  field. 

10.  Solve  x'2+2a;2/=55,  x?/  +  2y2  =  33. 

11.  Solve  2.t;2  +  3xy  =  8,  y--2xy  =  20. 

12.  The  area  of  a  rectangle  is  300  square  feet.  If  the  length  is 
decreased  by  2  feet  and  the  width  by  3  feet,  the  area  would  be 
216  square  feet.     Find  the  dimensions. 

13.  Solve  x(a;  +  2/)  =  150,  2/(a;+2/)  =  75. 

14.  Solve  x{x  —  y)=l5,  2/(0;  +  ?/)=  14. 

15.  Sodding  a  lawn  at  9  cents  a  square  rod  costs  $108.  If  it  had 
been  10  yards  longer  and  6  yards  wider  the  cost  would  have  been 
half  as  much  again.     Find  the  dimensions. 


SIMULTANEOUS  QUADRATICS  291 

16.  Solve  a;3-2/3=  126,  a;2  +  a:y+2/2  =  21. 

17.  Solve  x^  +  Zxy-by^  +  2x'-y=\2,  x+y  =  7. 

18,*  If  {x  +  yy-l{x  +  y)+l2  =  0  and  xhj^-6xy  +  8  =  0,  find  the 
values  oi  x-\-y  and  cc^/,  and  thus  solve  these  equations  for  x  and  ?/. 

19.  The  product  of  two  numbers  is  28  and  their  difference  is  5. 
Find  the  sum  of  their  squares,  without  finding  the  numbers. 

20.  Solve  8x^+1/^  =  280,  2x+y=lO. 

21.  Solve  y  =  x+V2,x^+y^=l. 

22.  Find  two  positive  integers  whose  sum  multiplied  by  the  greater 
is  192  and  whose  difference  multiplied  by  the  less  is  32. 

23.  Solve  ^  +  ^,=  10,  ^  =  3. 

x^       y^  xy 

24.  If  12^2  — 41a;!/  +  35i/2  =  0,  find  the  values  of  -• 

25.  The  product  of  two  niunbers  is  (5  and  the  difference  of  their 
squares  is  5.     Find  the  numbers. 

26.  Solve  ^  +  -  =  6,  a;-2/  =  4. 

y-     y 

27.  Solve  (x  +  i/)(a;  +  2.v)  =  300,  -  +  =^  =  3. 

y      ^ 

28.  A  regiment  consisting  of  1625  men  is  formed  into  two  solid 
squares,  one  of  which  has  15  more  men  on  a  side  than  the  other.  What 
is  the  number  on  a  side  of  each  ? 

29. 


1       2 

Solve    -  -j —  : 
x      y 

1         4 

=  8,  - +  — =  40. 
X-       y- 

«  ,        1       1 

Solve : 

X      y 

1        4,6         5 

~  12'    a;2"'"2/2~  12 

30. 

31.  The  difference  of  two  numbers  is  15  and  half  of  their  product 
equals  the  cube  of  the  less.     Find  the  numbers. 

32.  Solve  a;2-t-32/2  =  37,  a;y=  10. 

33.  Solve  a;  +  -  =  4,  w--=3. 

34.  Two  men  start  to  meet  each  other  from  towns  which  are  25 
miles  apart.  One  takes  18  minutes  longer  than  the  other  to  walk  a 
mile  and  they  meet  in  5  hours.     How  fast  does  each  walk  ? 

u  2 


292  ALGEBRA 

.    „.      «  ,       1,1      1      1,9       1 

35.  bolve  -  +  -  =  -,   — 2  +  ^  =  Q* 

36.  Solve  (a; +2/) 2- x- 2/ =  20,  a;?/ =  6. 

37.  The  difference  of  the  cubes  of  two  consecutive  odd  numbers 
is  218.     Find  the  numbers. 

38.  Solve  a;*-cc22/2+162/'»  =  28,  x'^  +  -ixy  +  '^y^=\4:. 

39.  Solve  x2  +  y=2/^  +  a;=3. 

40.  Tlie  diagonal  of  a  rectangle  is  d,  and  the  difference  of  the  sides 
is  5.  What  are  the  lengths  of  the  sides  ?  Apply  the  formula  thus 
obtained  to  find  the  sides  of  a  rectangle  whose  diagonal  is  13  inches, 
and  one  side  is  7  inches  longer  than  the  other. 

41.  Solve  9a;2  +  2/^-21(3.r  +  t/)+ 128  =  0,  xy  =  ^.  (Make  the  first 
equation  a  quadratic  in  'ix-^y,  by  adding  to  Qx'-^y-  what  is  necessary 
to  make  a  complete  square.) 

42.  Solve  x2  +  42/--18x-362/+ 112  =  0,  xy  =  %. 

43.  Solve  a;3  +  2/^=12(i,  a;-*/  +  x2/-  =  30. 


CHAPTER  XXIII 

INDICES 

EXERCISE  137  (Oral) 

1.  What  are  the  values  of  3-,  2\  \*,  P",  O^  ? 

2.  Shnplify  3  X  22 ;    SxlO^;    5x0^;    0-^^4. 

3.  When  .t=10,  what  are  the  values  of  : 

.1-3,  6.r2,  200-^.T,  500-f.T2,  6x^^x-  1 

4.  Give  the  values  of  (-1)2,  (-1)3,  (-l)*,(-l)-'\  (-!)"«. 

5.  What  are  the  values  of  (-2)^,  (-2)%  (-2)«  ? 
G.     Find  the  difference  between  2^  and  3-,  2^  and  5'^. 

7.  What  does  x*  mean  ?     How  many  factors  are  there  in  x"'  <x^  ? 

8.  Express  in  the  simplest  form  a^xa^xri*. 

9.  How  many  factors  will  remain  when  x''  is  divided  by  x^  ?    What 
is  the  quotient  ? 

H>.     What  are  the  values  of:    x*^x^,  .t^^-^.t^ 

x^     a^°     Tir-'     a*b^  r, 
x^     o^"      ttt      a%- 

11.  What  does  (a^) 3  mean  ?     Read  its  value  without  the  brackets. 

12.  State  the  value  in  the  simplest  form  of  : 

(^2)2^  (?y3)2,  (y3)3,  {a^)\  (a2)'". 

/  ^'\^ 

13.  What    does    (oft)^    mean  ?     What    does  (  ,  )      mean  ?     Read 

their  values  without  brackets. 

14.  Express  as  powers  of  10  :    100,  1000,  10,000,  lOx  100,  10^  x  10^, 
10=^^103. 

298 


294  ALGEBRA 

15.  Simplify  (-l)2x  (-1)»X  (-1)*  ;    {-ay-x(-a)*x{-a). 

16.  What  is  the  value  of  x  if 

10^=1000,  2^=16,  5^=125,  3^=81  ? 

17.  Express  32,794  in  descending  powers  of  10. 

202.  Definitions  of  «*".  As  a^  is  the  product  of  three  factors 
each  equal  to  a,  so  a'"  is  the  product  of  m  factors  each  equal 
to  a. 

a'^=^a.a.a  .    .    .   to  m  factors. 

Here  it  is  understood  that  m  is  a  positive  integer. 

203.  The  Index  Laws.     We  have  already  seen  that : 

(1)  a3xa^=-a3+^=--a'. 

(2)  a5-ha2=a5-2^a3 

(3)  (a2)3^^2X3^a6, 

(4)  (a6)4=.a454 


Let  us  now  express  these  statements  in  general  form,  using 
letters  to  denote  the  indices. 

(1)  f«'«X  «"=«'""*■". 

(2)  w"~n"=^a' 

(3)  {(i>»)»=a' 

(4)  {aby"=a"'b**K 
a'" 


ttn-n 


iinth 


These  are  called  the  index  laws.  The  letters  m  and  n 
represent  any  positive  integers,  and  in  (2)  m>n  (m  is  greater 
than  n),  to  make  the  division  possible.  The  laws,  as  stated 
in  the  general  form,  may  he  proved  as  in  particular  cases. 


INDICES  295 

204.  Law  I.     Law  for  Multiplication,     a'"  X«"=a'"  +  '^ 

By  definition, 

a'"  =  a  .  a  .  a  .  .  .  to  m  factors. 
a"  =  a-.  a  .  a  .  .  .  to  n  factors, 
o™  X  a"  =  {a  .  a  .  a  ...  to  ni  factors)(a  .  a  .  a  .  .  .  to  n  factors). 
=  a  .  a  .  a  .  .  .  to  (m  +  n)  factors, 
=  «"'+",  by  definition. 

Also,     a'»xa''xaP=a>"+"xaP, 

205.  Law  II.     Law  for  Division.     «."'^a'»=re'«-» 

a'"       a  .  a  .  a  .  .  .  to  ni  factors 
a"       a  .  a  .  a  .  .  .  to  n  factors 

=a  .  a  .  a  ...  to  (m— m)  factors,  if  ni>n, 

=  a™~". 

Here  the  n  factors  in  the  denominator  cancel  with  an  equal  number 
in  the  numerator,  leaving  m  —  n  factors  in  the  numerator. 

If,  however,  n>m,  the  n  factors  in  the  numerator  cancel  with  an 
eqiial  number  in  the  denominator,  leaving  n—m  factors  in  the  denomi- 
nator. 

when  ?w>n,  a'" -i- a"  =  «"*"", 

and  when  n>m,,  a'"  -^  a"  =  — - — 

a"   "' 

206.  Law  III.     Law  of  Powers.     {a'»y^=a''^'\ 

(a'")"=a'"  .  o'"  .  a'"  ...  to  n  factors, 

=  (a  .  a  ...  to   m   factors)  (a  .  a  ...  to   m   factors)  .  .  .    the 

brackets  being  repeated  n  times, 
=a  .  a  .  a  .  .  .  to  inn  factors, 
=  a""'. 
Also,  {(o'")»;''  =  (a""')''=a'""'\ 

207.  Law  IV.    Power  of  a  Product.     {ab)'^=a»b'K 

(ab)"  =  ab  .  ah  .  ah  ...  to  n  pairs  of  factors, 

=  {a  .  a  .  a  .  .  .  to  n  factors)  {h  .  b  .  h  .  .  .  to  n  factors), 
=  o"6". 
Also,  (abc)"  =  (ab)"  .  c"=a"b"c'K 


296  ALGEBRA 


208.     Law  V.     Power  of  a  Quotient.     ,  ,  ,  —  , 


(f 


7-  •  T  .  r  •  •  .  to  n  factors, 
boo 

a  .  a  .  a  .  .  .  to  n  factors 
6  .  6  .  6  .  .  .  to  «  factors' 
a" 

209.  We  have  given  five  index  laws.  They  are  not  all 
independent.  The  second  and  third  laws  may  easily  be 
deduced  from  the  first. 

(1)  When  m>n,  a'"  =  a'"~"xa"  by  Law  I. 

.".     a"^-Ha"  =  a'"~«,  which  is  Law  II. 

(2)  a"*  X  a'"  =  a"'+'"  =  a~"',  by  Law  I. 
Similarly,                        o'"  X  a™  X  a"'  =  «'"+"«+"'  =  o^"', 

and         a"'  .  a'"  .  a'"  ...  to  to  factors  =  a'"+"'+"'  •  •  •  to  n  terms  — c^mn^ 

(a'")"  =  a""',  which  is  Law  III. 

For  this  reason  the  first  law  is  frequently  called  the 
fundamental  index  law. 

EXERCISE   138  (1-18,  Oral) 


Simplify  : 

1.     ary.a^xn^. 

2. 

X^X'X^  —  X''. 

:?. 

(,,•2)4^.1-3. 

4.    (avy->y\ 

5. 

(32)2. 

«. 

(33)2_^(3>); 

7.     {ab)^^a-b. 

8. 

5«^r)^. 

9. 

((-2r-)3. 

6^ 

10.       6-5- 

11. 

(-1)' 

12. 

(ab)^ 

13.     a;"  X  a;''  X  a;".  14.     a--^ .  a'-' .  w" '  >>.      15.  x'"  -  "  X  .r^"'  +  ". 

16.     a;"  +  ''-f-.T«-''.  17.     {a^b^c'^y'.  18.     .x"  + '' .  .r'' + " .  a;"^  + «. 

19.*  fn    X  (- j    X  (- j  .  20.     a;2«  +  ''xx2''  +  <^xa;2« +  ''-<'. 


INDICES  297 

-•  ©"•  ©■"•  ©'"•    -•  ^^i^:x.-- 

23,     •  24.     a;"  +  ''.  a:''  +  ''..r''  +  "  — (;c"  .  .r''.x<=)2. 

25      Express  4"  as  a  power  of  2  and  9®  as  a  power  of  3. 

26.  Divide  27^  by  9^  by  expressing  each  as  a  power  of  3. 

2"x2"-ix22         9"x3»  +  s 

27.  Simplify  ^^ and    ^y^-n    ' 

28.  Solve 

^2x~  i^^jc  +  s  .  43-_2'-*";  93j-  +  3__27'*5  .  2-'' .  4-f .  8-^=16^^"^. 

210,  Fractional,  Zero  and  Negative  Indices.  We  have  defined 
a"*  to  mean  the  product  of  m  factors  each  equal  to  a.  This 
definition  requires  m  to  be  a  positive  whole  number. 

Thus,  the  definition  will  tell  us  what  a"  means,  but  will  not 
tell  us  what  a-  means,  nor  what  a~^  means,  nor  what  a° 
means. 

If  we  wish  to  use  in  algebra  such  quantities  as  a^,  a~^,  a", 
it  is  necessary  that  we  define  their  meanings. 

Now  it  would  be  very  inconvenient  if  we  gave  to  these  new 
forms  of  indices  such  meanings  that  the  index  laws,  already 
established  for  positive  integral  indices,  would  not  apply  to 
them.  We  will,  therefore,  give  to  fractional,  zero  and 
negative  indices  such  meanings  as  will  make  the  index  laws 
vaUd  for  them  as  well  as  for  positive  integral  indices. 

211,  Meaning  of  a  Fractional  Index. 

Since  x'"  X;K"=a;'"+",  then  if  we  suppose  that  the  same 
law  applies  to  fractional  indices,  it  follows  that 

•T^  X  X-  =  X^  +  ^  =  X^  =  X. 

Thus,  x^  when  multiplied  by  x^  gives  the  product  x,  or  the 
square  of  .r^  is  x. 

But  we  have  already  represented  the  quantity  whose  square 
is  X  by  Vx, 


298  ALGEBRA 

That  this  is  a  reasonable  value  to  attach  to  x'^  might  appear  as 
follows : 

We  know  that  x*=  Vx^,  x^=  Vx*,  x'^=  \' x^,  the  index  of  the  quantity 
under  the  root  sign  in  each  case  being  half  of  the  index  in  the  preceding 
case.     If  now  we  take  half  of  the  index  on  each  side  again,  it  would 

seem  but  natural  that  x'^  should  be  equal  to  yfx. 
Similarly,  a;^  X  a;^  X  x'^  =  a;*     *     ^  =  a;. 

a;^=  Va;  (the  cube  root  of  x). 
Also,  x^  =  Vx  (the  fourth  root  of  x), 

and        x"'=  \^x  (the  ntli  root  of  x),  where  «,  is  a  positive  integer. 
Thus,  42  =  Vi  =  2,  125^  =  f'l25  =  5,  32^=  v'32  =  2. 

By  Law  III,  (x^^  =  x^, 

•        x^=^X^. 


Similarly, 


(a;0    =xP, 


.'.     X  —  v'xi',  where  p  and  q  are  positive  integers. 

We  thus  see  that  if  the  same  laws  apply  to  fractional  indices 
as  to  positive  integral    indices   we  are  led  to  the  conclusion 

that  .T''=\/a:^,  when  'p  and  q  are  positive  integers,  that  is, 
ichen  the  index  is  a  fraction,  the  denominator  of  the  fraction 
indicates  the  root  to  he  taken  and  the  numerator  the  power. 

p     /  \_\p         I 
By  Law  III,  a;'  =  Uv   =  (x'')'' , 

p 
.-.     a;9=(v'x)P=^P. 
p 
So  that  x^  means  that  the  p^^  power  of  the  q^^  root  is  to 
be  taken,  or  the  7*''  root  of  the  ^*''  power. 

Thus,  8^=^82=^64=4, 

or  8§  =  (\'/8)2  =  22  =  4. 


INDICES  299 

It  will  be  seen  that  it  is  simpler  to  take  the  root  first  and  then  the 
power. 

Thus,  32'°  =  ('v/32)3  =  23=8. 

212.  Meaning  of  a  Zero  Index. 

By  Law  I,  a"  x  a'"  =  a''+"'  =  a"*, 

a''  =  a"'-Ha"'=  1. 

Therefore,  if  the  same  law  appUes  to  zero  indices  as  to 
positive  integral  indices,  we  are  led  to  the  conclusion  that 
any  quantity  {zero  excepted)  to  the  index  zero  is  equal  to  unity. 

Thus,  3''=1,  (5a;)''=l,  (-2)o=l,  (-ia6)o=l. 

213.  Meaning  of  a  Negative  Index. 

By  Law  I,  a-^xa^  =  a-^+^  =  a'^=l, 

1 

Similarly,  a-PxaP  =  a-i'+P  =  a*^=  I, 

1 

/.     a-p  =       • 
aP 

We  thus  see  that,  any  quantity  to  a  negative  index  is  equal 
to  unity  divided  by  the  same  quantity  to  the  corresponding  positive 
index. 

Thus,  4-  =  l=j^. 

27S       (\y27)2       9 
(a;t)-"'==x-«  =  i- 

Since  x^—x*-^x,  x^=x^^x,  x^  =  x^^x,  what  would  you  naturally 
expect  x°  to  be  equal  to  ?     What  would  you  expect  x" '  to  be  equal  to  ? 

214.  Since  a~''  =  —  and  ai^  =         ,    it    follows    that    any 

ai'  a~P  -^ 

factor  may  he  removed  from  the  numerator  to  the  denominator 
of  a  fraction,  or  vice  versa,  by  changing  the  sign  of  its  index. 


300 

Thus,  g,2 


ALGEBRA 

2  .  62 

^'      b-"    '    y"    ' 

4ct» 

4.r-%3  =  -^ 

a;2 

Ex.— Simplify  ^/S^x^^le^;    (y|^)  '• 


■^82  X  \/l63  =  83  X  16+  =  (2»)«  X  {24)i  =  22  x  23=32. 
/  9a*   \"'       9"*.a-«  16^  64 


Vl66--/  16"?  .  69       9%s69       27a«69 


EXERCISE  139  (1-82,  Oral) 

What  is  the  meaning  of: 

i  1 

1.     a-2.  2.     x^. 

4 

5,     x^.  G.     .T°. 

9.     x~*.  10.     ?/-". 

What  is  the  vahie  of : 


3. 

1 
y-. 

4. 

al 

7. 

a-\ 

8. 

a;-i 

LI. 

.«-* 

12. 

a;""^, 

13.     92. 

14. 

16^. 

15. 

125^. 

16. 

10,0003^ 

17.     42. 

18. 

27^ 

19. 

(i)^- 

20. 

5-2. 

21.     10-3. 

22. 

4-i 

23. 

^  1 
8  -. 

24. 

(-6)«. 

25.     8"i 

20. 

(«")--• 

27. 

(•25)i 

28. 

(•16)t 

29.     (J)-2. 

30. 

(-2)-*. 

31. 

.'^n .  4". 

32. 

2^.  2" ®. 

Write  with  positive  ind 

ices: 

33.     a2ft~3. 

34. 

2«-3. 

35. 

a-2 

36. 

a6-3 

37.  -^^  "^ 

38. 

1 

x~^y 

.39. 

2a;- 1 
By-^ 

40. 

4-3. a;3 
3-2.2/-3 

Write  without  a 

denominator : 

41      2^^. 

42. 

4a3 

43. 

3x 

44. 

5 

*  •      ^2 

a2(c+rf)-2 

INDICES  301 

Simplify : 

_  3                                  1 
45.*   16  i.  4B.     -.  47.     (-04)^2.       48.     (-027)'^. 

32"^ 
49.     2oi-5.  50.     (— 8)~-i       51.     f/g^*.  52.     I6I". 

2-2_2-3  \9gy  \l6 

56.     ^I^Z:^.  57.     fi^y.  58.     (l^y^ 

3-1-2-1  V243/  V  816V 

59.     Solve  j;^=4,  .ry=32,  .r*=27,  .r^2=3,  .i;"^^=8. 

215.     Operations  with  Fractional  and   Negative   Indices.     The 

following  examples  will  illustrate  how  the  index  laws  may  be 
apphed  to  the  multiphcation,  division,  etc.,  of  quantities 
involving  fractional  and  negative  indices. 

The  work  will  usually  be  simj)lified  if  all  expressions  are 
arranged  m  descendmg  or  ascending  powers  of  some  common 
letter. 

Thus,  Q-\-x^^x~''^—x~'^-\-x'^  would  be  Avritten  in  descending 
powers  of  x,  thus  : 

x^-\-x^-\-b—x~'^-\-x~'~. 

Ex.  1.— Multiply  2,r2  +  3-x^  by  'ix^-2-5x~^. 

Ex.  2. — Di\dde  a—b  by  a^—b^. 

(1)  (2) 

3x^-2     -   5x~^  a~ah^ 


,ih^i 


6x  +9a;--    3  a^lr' 

_4x^_    6+    2a;"  ^  0^-0^6^ 

-10-15a;~2  +  5x-i  7~ 


■b 


6a;  +5a;-^-19-13x  •-'  +  5x-i  aM-6 


302  ALGEBRA 

Ex.  3. — Find  the  square  root  of  9a;— 12a;i+10— 4a;~2+a;~^. 


9x 


6a;2-2 


12a;^+   4 


1  _i  -i 

6a;-  — 4+a;  -     6  — 4a;  -  +  x~^ 

Q—4x  -+a;~^ 


Verify  by  squaring   3x'-  — 2+a;  ^   by  the  method   of   art.  93.     Also 
check  by  putting  x=l. 


EXERCISE  140 

Multiply : 

11  11 

1*  x-  +  '3,  X-—2.  2.     a;+a;2  +  l,  a;2  — 1. 

3.     a;2— a;+a;2  — 1,  x^-\-l.  4.     3a;— 2a;2_j-5,  x—2x^. 

5.     a*_i  +  2«~ia2_^l— 2a-2.  6.     (a-a2  +  i)2. 

7.     a;+5a;S+6a;2,  a;^— 1— a;~i.  8.     (a;24-2)*. 

11  11  1        „ 

9.     x-{-x"y--\-y,  x—x-y~-\-y.  10.     {a-  —  \Y. 

Divide  and  verify : 

11.  a + 5a 26 2 -j- 66  by  a 2  _j_ 26 2. 

12.  a;3— a:2+a;— 2— 2x-2— 2a;-3  by  a;2+2+2a;-2. 

4  2    J2  4  2  1     1  _:i 

13.  x'^-\-x-^y^-{-y"'  by  x'''+x'^y'-^'-\-y'^. 

14.  1— 5a;'— a;  by  1— a;^+3x'^,  as  far  as  four  terms. 
Find  the  square  root  and  verify : 

15.  a+6a2-i-9  and  25x-2— 10+a;-2 

16.  a2+4a2+6a+4a2  +  i. 

17.  4x^-20a;^+37a;— 30a;*+9a;3. 


INDICES  303 


18.     49-30a;3-24x""-^+25x3+16a:"i 


1  1 


19.  Show  that  ^ ^     ,^      -  (a^+P)-i. 

5.-0  1  _i 

20.  Divide  x-—x  -  by  x~—x  -. 

21.  Divide     lOa^'"— 32a'"-27a2m^l4  by  2a'"— 7. 

22.  Simplify  (x+a;^+l)-+(^-a;-  +  l)2. 

23.  SimpUfy  (V'a+l)(v^-l)-(\/3a+ V2)(V3^-\/2). 

24.  Find  the  square  root  of  x^— 4x\/a:+10a;— 12V'.r+9. 

216.     Contracted    Methods.      The    following    examples    will 
illustrate  how  contracted  methods  may  be  employed. 

Ex.  1.— Multiply  a;+a;i— 4  by  x^x^^4:. 

If  x  +  x-  be  considered  as  a  single  term,  the  product 

=  (x+x^)2-42,        [(a  +  6)(a-6)  =  a2_{,2.j 
=  a;2  +  2a;*  +  a;-16. 

Ex.  2.— Divide  a+6  by  a^+fe^. 

Since   a  is  the  cube  of  a^  and   6  of  b'-^,  this  is  similar  to  dividing 
x^-\-y^  by  x  +  y. 

Since  {x^  +  y^)^{x-\-y)  =  x''-  —  xy+y^, 

so  (a  +  6)^(o*  +  6^)  =  a^_aM  +  6S. 

Ex.  3. — What  is  the  cube  root  of 

This  is  evidently  the  cube  of  a  binomial  whose  first  term  is  2x*  and 
last  term  is  ~Zy^. 

.'.    the  cube  root  is  2x^  —  ^y^ ,  if  the   given  expression  is  a  perfect 
cube.     Check  when  x  =  t/=  1. 

Using  the  method    of    art.    155,    the  cube  root    of    more 
complicated  expressions  may  be  found. 


304  ALGEBRA 

217.     Factors  with  Fractional  or  Negative  Indices.     If  we  are 

permitted  to  use  fractional  or  negative  indices,  many  ex- 
pressions may  be  factored  which  Avere  previously  considered 
algebraically  prime. 

Ex.  1.     a  —  b  may  be  written  as  the  difference  of  two  squares,  tliu-s 

1  1         i  1 

a  — 6==(a-+6'-^)(a"^  — 6-). 
Ex.  2.     ^x—\  —  2x~^    may    be    factored    by    cross    multiplication. 
The  factors  are  {Zx'^'Ix  -)(a;-  — x  -). 

Ex.  3.     x~ -\- xy -\- y'^  is  an  incomplete  square. 

It  may  be  written  (x-\-y)^  —  {x^y'-)^. 

11  II 

/.   the  factors  are  {x-\-x-y'-' -\-y){x  —  x''y'' -\-y). 

EIXEBCISB  141 

Use  contracted  methods  in  the  following  : 

1.*  Multiply  x^—2  by  x^+2  ;    a-—b'^  by  a2-f62- 

2.  Multiply  a2 _  1  -j-a" 2  by  a^-{- 1  +a" 5. 

1 

3.  Find  the  square  of  x—x-  —  \  and  of  2a—2—a~K 

i  1 

4.  Write  down  l;he  cube  of  a^-j-i  and  of  1— :f-. 

1  i  JL  i 

6.  Multiply  x-{-x~y~-\-y,  x—X"y~-\-y,  x^—xy-\~y'^. 

2  1.    L  2 

G.     Divide  x-\-y  by  x''^—x'^y'^-\-y'K 

7.  Divide  a+2ah^+b-c  by  a^+b^—c^. 

8.  Find  three  factors  of  x"—y^. 

1   1 

9.  Find  a  common  factor  of  a->ra^b"—2b,  a—h. 

i 
a;+a;-— 6        a—b        a"-\-ab-{-b^ 
10.     Simplify   —       ^       »     -3 j'    zr 


INDICES  305 

11.  What  is  the  cube  root  of  .f-— 6.f+12;f'-— 8, 
and  of  a:3-3a;'^+6a:2-7a;2+6a:-3.T^  +  l  ? 

12.  What    is    the    square    root    of    4,f~*+12a;""^-l-9.e~^ 
and    of  a;--+4a;+2-4a;-iH-.T-2? 


EXERCISE  142  (Review  of  Chapter  XXIII) 

1.  State  the  index  laws. 

2.  Explain    how    meanings   are    assigned    to    such    quantities    as 
a^,  a»,  a- 3. 

3.*  When  x=  16,  ?/  =  9,  find  the  values  of  : 

(x-\-y)'',x  -+y  '',  (x'^+y-)^. 

4.  Find  the  numerical  values  of : 

8^,  9~^    v'  125^^  32*,  16-1-5,  .25--i,  (-64)"^. 

5.  Show  that  ^Px  ^^8^=  108. 

6.  Simphfy  i2~^^{J^)^  and  8i2^^(jVrn^- 

7.  Find,  to  two  decimals,  the  value  of  10'  when  x  =  ^,  |,  ^,  §. 

8.  Simplify  2^+ 10«-4*-(|r*  +  0*  +  ( Vt^)-!. 

9.  Find,  to  three  decimals,  the  value  of  (3^)"^  X  V^27. 

10.  Simplify  16^  +  16^- 16"^- 16"^  and  32* -32* +  32"'^ +  32"*. 

11.  Simphfy  5^  x  5*  x  5*  x  16^'^  x  16^''  x  16'''. 

12.  Solve  x'^  =  S,  2^.  4^  =  64. 

13.  Simplify  4^  x  6"*  x  ^^3  and  (8^  +  4^)  x  16"^. 

9«+i        Q-ii  3  2"  — 4  2"~i 

14.  Simplify  ^^  X  Y^^  and  ^1^;--— —^ . 

15.  Reduce  to  lowest  terms  : 

a+SVa+l5      3x^  +  5x^  +  2      a^+ab 


a  +  lVa  +  \2  ^i^j       '    ab-b^ 


306  ALGEBRA 

3      ^  13  11 

16.  Multiply  x-y'  —  2xy-{-4cX-y-  by  x^-j-zy^. 

1    _l  _1    i  JL    _i  _i    i 

17.  Multiply  x'^y  '■^-\-l-\-x  -^y-  by  x'^y  -'— 1  +  x  ^y^. 

18.  Divide  x^  —  y-^  by  x^-\-x^y~^-\-y~^ 

2  _  2  1  _i 

and  a^+l  +  a  ^  by  a^+l+a  ^. 

19.  Divide  a*">-b*"'  by  a"'-6'". 

5  •■'  i 

20.  The   dividend  is   y- +22/2_3y— 2,    the   quotient   is  2/'~2/       1> 

1 
the  remainder  is  3y'^—l.     Find  the  divisor. 

21.  Find  the  square  root  of  (x^x~^)^  —  4(x  —  x^^). 

22.  What  is  the  cube  root  of  la^^  — ^a2^6^  +  §a^62^-/y6»»  ? 

23.  If  x  =  a~+l  and  y  =  a-^+l,  show  that  ^y^^~y  =  a^. 

xy—x+y 

24.  Simphfy  -008*,    1-728^    2-251-5,    -0625"^. 

1  _x 

25.  If  .-r  +  2/  =  o^  and  x—y  =  a  -,  fuid  the  values  of  xy  and  x^+y^ 

in  terras  of  a. 

26.  If  2a=2^  +  2-'-  and  26  =  2^-2--',  find  a^-b^.      ' 

27.  Find  the  square  root  of  (e*— e"^)2  +  4  and  of 

r,    i  .".     -.1  15 

a;'— 4x^2/^  +  lOx^i/—  14x-2/-'  +  13a;?/"  — 6a; -2/'-  + 2/^. 

3  1         -4-  --1-         -1  -\  -8 

28.  Simplify  — j"  ^  — ZT'   ■ 

c^  b  ^  c 

1 

29.  Factor    x-  —  y,    a;  — 5x-  — 6,   a;— 1,    4a— 6^    and 

x^-4:X+  10-  12a;-i  +  9a;-2. 

30.  If  10-3oio3^2,    find  the  value  of  10-«o2»'>  and  lOi'^o^is, 

31.  If  -^'oio^^SO  and  72-0593  =  55^  g^d  the  value  of  7*"»6" 

9ii+l  (9h-1)'(+1 

32.  Simplify   ^^"^  X  ^^;^+3 


INDICES  307 

33.  Solve  3-'+ 1  +  2*  =  35,  3^+2.«+2  =  4L 

34.  Divide  x-2(x^'  —  x'h  +  2(x'^-x~h—x-'>-  by  x^  —  x~K 

35.  Show  that  x'^  ■'  =  (xV x)'  is  satisfied  by  a;  =  2|. 

36.  Find  the  square  root  of 

i{2Vx)^-2x-'  +  x+'lx'+Vx-^-  ^ . 


x2 


CHAPTER  XXIV 
SURDS  AND  SURD  EQUATIONS 

218.  In  Chapter  XVIII.  we  have  ah-eady  dealt  with 
elementary  quadratic  surds.     It  was  there  shown  by  squaring 

that  y/axVh=Vah. 

We  might  now  deduce  it  from  the  index  laws. 

From  Law  IV  (a6)"  =  a"6". 

Letw=^,  .".     (a6)'-=a-6-', 

Va6=\/axV6. 

1       11 
Similarly,  (a6)"=a"6", 

219.  Orders  of  Surds.  We  have  already  defined  a  quadratic 
surd  as  one  in  which  the  square  root  is  to  be  taken.  A  cubic 
surd  is  one  in  which  the  cube  root  is  to  be  taken.  When 
higher  roots  are  to  be  taken  as  the  fourth,  fifth  .  .  .  n^'^,  they 
are  called  surds  of  the  fourth,  fifth  .  .  .  w"'  orders. 

220.  Changing  the  Order  of  a  Surd.  A  surd  of  any  order 
may  be  expressed  as  an  equivalent  surd  of  any  order  which  is 
a  multiple  of  the  given  order. 

n 

Thus,  ^/x=x^  =  x'^  =  x^=x^''\ 

Similarly,  x^  =  x^'=x^=x^  , 

308 


ST'RDS  AND  SURD  EQUATIONS  309 

221.  To  Compare  Surds  of  Different  Orders.  Any  two  surds 
may  be  rednced  to  surds  of  the  same  order  and  their  vahies 
compared. 

Thus,  to  compare  the  vahies  of  \^2  and  f'^3, 

\  2  =  2^  =  2*^=  V'2'=V^8. 

f  3  =  3^  =  3^=  V^=\/9. 
It  is  thus  seen  that  \  3  is  greater  than  A  2. 

222.  Changes  in  the  Form  of  Surds.  Any  mixed  surd  can 
be  expressed  as  an  entire  surd. 

Thus,  2  \V5=  ^23  X  ^5=  i^40. 

av'6=  v^a"  X  Vb=  \^a^. 

V  m  +  n       A     ^  '      m  -\-  n 

Conversely,       \'''250  =  ^^l25  x  ^2  =  5-^2. 
^/^ib^=^/W'x  y/a  =  b^-\/a. 

-^^8T=f/^^27x^3=  -3^3. 

EXERCISE   143 
Express  as  mixed  surds  : 
1.     Vfl,  VlOOa,  VsP,  VSo^fe,  \/32^,  VSGSa^P. 

3.  v'32,  'v/243;  \/^^,  \/6i,  v'8.T2+16a:2/+8i/2 

Express  as  entire  surds  : 

4.  2\/3,  10\/2.  3\^a,  0^5,  ahx^b,  {a-b)Va^. 


310  ALGEBRA 

7.*     Reduce  ^^2,  VS  to  surds  of  the  same  order.    Also  reduce 
^2  and  \/3  ;  V2,  ^3  and  \/5. 

8.  Which  is  the  greater: 

3\/2or2\/3;   sVGorlVS;    \/5  or  \KlO  ;  1-26  or  ^2;    ^3  or  V^5  ? 

Reduce  to  like  surds  and  simplify  : 

9.  VS  +  VlS+VdS.                    10.  \/500+\/80-V'20. 
11.     3\/32+5\/50-i\/l28.          12.  -^T6-\^/i28+\^''250. 
13.     -^96-2^^=^+^324.          14.  v'32+\/l62+\/i260. 

15.  \/75-3\/l2+5\/300+2V'48-7\/l47+3VI. 

16.  xVx-\-y-\-Vx^-{-x'^i/—V{xA-y)^—V{x^—y^){x—y). 

17.  Express  as  equivalent  surds  of  a  lower  order  : 

i/9,  \/l25,  ^/x^,  \/i6xh/%   v^32. 

18.  If  ^2=1-26  approximately,  find  the  values  of  : 

\'/r6,  -^54,  •^2060,  ^KJ,  ^^002,  1^6^. 

19.  Show  that  2  x  V2  x  V2  x  v^4=4. 

223.     Rationalizing  a  Surd  Denominator.    When  the  numerical 

value  of  a  fraction  with  a  surd  denominator  is  required, 
the  value  is  more  easily  obtained  when  the  denominator  is 
rational  (art.  165). 

When  the  denominator  contains  only  two  terms,  it  may  be 
rationalized  by  multiplying  by  its  conjugate  (art.  164). 

EXERCISE   144 
Multiply  : 

1.  2VS,  3V5.  2.     V2ax,  V3ax.  3.     Vx,  Vxy. 

4.  ^4,  -^5.  5.     6VI4,  JV2I.  6.     ^^,  .n''^. 

7.  </a^b,  ^a+b,  i''a^l)^. 

8.  Vx+2,  Vx—S,  Vx—2,  Vx+3.    ' 


SURDS  AND  SURD  EQUATIONS  311 

9.     \/2+V3-a/5,  \/2+V3+\/5. 

10.  ^'a— 1,  ^a— 2,  ^a+3. 

11.  \/6-\/lT,  \'6+\/II. 

12.  (\/r8+Vl2+V8)2. 

What  is  the  simplest  quantity  by  which  the  following  must  be 
multiplied,  to  produce  rational  products  ?  What  is  the  product  in 
each  case  ? 

13.  3V2.  14.     2V'5.  15.      \/32.  16.     V64. 
17.     V5T2.        18.     V2.            19.     V^^^.  20.     \/^. 

21.     3-\/2.      22.     Va+b.      23.     3\/2-2\/3.      24.     aVb-bVa. 
Rationalize  the  denominator  of : 

26.     Vh  27.     Vrs. 


29.     (v/S+VS)-!.  30. 


25.* 

.,  I0V2 

V5 

28. 

4+2\/2 

2+2\/2 

31. 

a+6— c 

V'a2_^62_ft 


_.  32.  . 

Va+b+Vc  Vx+y+Vx—y 

33.  Find,  to  three  decimal  places,  the  value  of 
22^(3\/2-a/7)(2\/2+\/7)  and  of  (5J-\/7)^(3  +  \/7). 

34.  If  X  = and  w  =  -J^ ,  find  the  value  of  x~+y^. 

2+V3  2-V3  ^ 

35.  Simplify  (2\/3-\/2)3-(\/3-\/2)3. 

36.  Simplify  ^^^  +  i±^  and  ^±^  +  ^t^^? . 

3+\/5      4-\/5  I  +  VS       2+A/5 

37.  SimpUfv^^2V5_lo+6V5. 

4-V'5         2+\/5 

38.  Show  that  3  — V?  is  a  root  of  the  equation  .r^— Sx^— 4a;+2=0. 


312  ALGEBRA 

39.  The  three  dimensions  of  a  room  are  equal.  If  the  longest 
diagonal  from  the  ceiling  to  the  floor  is  18  feet,  find  the  length  of  the 
room  to  the  nearest  inch. 

224.  Surd  Equations.  When  an  equation  contains  a  single 
quadratic  surd,  and  the  equation  is  written  with  the  surd 
alone  on  one  side,  the  surd  may  be  removed  by  squaring 
both  sides  of  the  equation  (art.  166). 

If  the  equation  contains  three  terms,  two  or  three  of  which 
are  surds,  the  operation  of  squaring  must  be  performed  twice. 

Ex.  1.— Solve  l-f  Vx=Va;+25. 

Squaring,  l-\-2Vx-\-x=x-\-25, 

:.     2\/^  =  24, 
Vx=U, 
x=  144. 
Verification  1  + Vx=  l  +  -\/l44=13. 

V'x+25=VT69=13. 

Solve  by  squaring  in  the  form  Va;=  Vx  +  25— 1  and  in  the  form 
l  =  Vx-\-25—Vx,  and  compare  the  three  solutions. 

Ex.  2.— Solve  l  —  Vx=Vx-\-25. 

Squaring,  1  — 2'v/x  +  .T=.r+25, 

.-.      -2\/x  =  24, 

a;=144. 

Compare,  line  by  line,  this  solution  with  Ex.  1.  The  answer  is  the 
same  to  both,  although  the  equations  are  different.  We  have  verified 
Ex.  1,  and  we  know  that  x=  144  is  the  correct  result. 

Let  us  now  verify  Ex.  2. 

Verification:  1— -v/i=l— •\/l44=  — 11. 

\/x+25-:VT69=13. 

It  is  seen  that  our  attempt  at  verification  shows  that  .t=  144  is  not 
the  correct  root  of  the  equation  in  Ex.  2. 

If  in  verifying  we  could  say  that  Vl44  is  —12,  the  equation  would 
be  satisfied.  But  this  is  not  allowable,  as  the  symbol  V  always 
represents  the  positive  square  root  (art.  fi3). 


SURDS  AND  SURD  EQUATIONS  313 

This  may  be  explained  as  follows  : 

(1)  The  equation  1  — Vx=v.x+25  is  impossible  of  solution, 
as  Va;-f-25  is  a  positive  quantity,  and  therefore  l  —  Vx  must 
be  positive,  that  is,  x  must  be  less  than  1.  But  it  is  evident 
that  no  value  of  x  which  is  less  than  1  can  satisfy  the  equation. 

(2)  If  we  square  both  sides  of  an  equation,  the  resulting 
equation  is  not  necessarily  equivalent  to  the  given  equation. 

A  simple  example  will  show  that  this  is  the  case. 

Let  .r  =  —  6. 

Squaring,  .".      x'^—36. 

Now  the  equation  x"  =  36  has  two  roots  +6  and  —6,  and  is, 
therefore,  not  equivalent  to  the  given  equation. 

This  is  similar  to  the  case  in  which  both  sides  of  an  equation 
are  multiplied  by  a  factor  containing  the  unknown. 


3), 


The  equation  a;^  — 5x+6  =  0,  which  has  the  roots  2  and  3,  is  not 
eqioivalent  to  the  given  equation. 

225.  Extraneous  Roots.  Roots  which  are  introduced  into 
an  equation  by  squaring  or  multipljdng  are  called  extraneous 
roots. 

Thus,  x=6  in  the  first  ecjuation  and  x=3  in  the  second  are 
extraneous. 

Refer  to  Ex.  4,  art.  145,  where  reference  is  made  to  the  effect 
of  dividing  both  sides  of  an  equation  by  a  factor  containing  the 
unknown. 

We  have  already  seen  the  necessity  of  verifying  the  results 
in  the  solution  of  equations.  In  the  case  of  surd  equations 
there  is  an  added  reason  for  verifying,  for  although  there  may 
be  no  error  in  the  work,  the  root  which  is  found  may  not  be  a 
root  of  the  given  equation. 


Let 

x  =  2. 

Multiply  by  x—3,    .' 

x{x-S)  =  2( 

.     x^-5x+6  =  0. 

314  ALGEBRA 

EXERCISE   145 
Solve  and  verify  1-15.     Reject  extraneous  roots  : 

1.     v2.r-5-3=0.  2.     \/3a:-2=2V'a:-2. 

3.     3xi=x^+4:.  4.     ^5a:-7=2. 

5.     2x^=3.  6.     2f'3x-25-f-3  =  7. 

7.     2(a;-7)*'=(a:-14)5.  8.     Vx+Vx+5=5. 

9.     Vx+45+V^=9.  10.     1  +  ^^+2=^^. 


11.     V:r-t4+Va;+15=ll.  12.     v/4x-2+3x-16  =  2a:+2. 

X- 1_  7\/x+10 

Vx^  '      a/4x-2 


^        x-1  ^^      lVx+\0      ^ 

13.     Va;  +  5=;7-==-  14.         ,-     ^  =4. 


15.     V^-3^Vg+l.  16.*  Vx+4-V^4=^4. 
Vx'+3      Vi-2 

17.     (12+a;)^+a;2=6.  18.  (x+8)^-(x--f3)5=2.ri 

,^      Vx+16      a/x+29  „„  6\/x-ll_2\/x+l 

19.       -— =    -— •  -^u.  — • 

^/x+4:        Vx+n  3Vx  Vx+6 

, ,-        , 36 

21.     \/a;2-a2+62^x-a+6.  22.  \/x+Va;-9= -/===• 

23.     Va;-15+Vx=-7==-.  24.  V'a:-f3+\/x+8=2\/x. 
Vx— 15 

25.     V5+4^-Vx-2Vfe+i.  26.  ^a;3-6x2+llx-5=a;-2 

1  1  OS  Vx+6+V^_o 

27.     5(70x+29)^'=9(14a:-15)^-.  -8-  ^^^^ZV^ZIe  ~ 

29.     _5x-l_^j^V5^  30  V^4V^^^ 


SURDS  AND  SURD  EQUATIONS 
226.     Surd  Equations  Reducing  to  Quadratics. 

Ex.  1.— Solve  :i-+\/:r+5=7. 


315 


Transposing, 
Squaring, 


Verification 


Vx+5  =  7  — cc. 
x  +  5  =49-14.r+a;2 
a;2-15x  +  44  =  0, 
(a;-4)(a;-ll)  =  0, 

x  =  4:  or  11. 
When  x  =  i,x+  Vx^5  =  4  +  V9  =  7, 
When  x=  11,  a;+V'x+5=ll+ VT6=15, 
the  correct  root  is  re  =4. 
a;=ll  is  evidently  a  root  of  x—  Vx  +  5  =  '7' 

Ex.  2.— Solve      V8x+ 1  —  Va;+ 1  =  VSx. 
Transposing,  .-.      V8x+l  =  V3x+Vx+l. 

Squaring,  .-.  8x+l  =  3x+2Vdx^  +  Sx  +  x+l, 

4:X  =  2V3x^  +  3x, 
2x=  V3x^  +  3x, 
4:x^  =  3x^  +  3x, 
x2-3a;=0, 

x=0  or  3. 

Here  we  find  on  verifying  that  both  roots  satisfy  the  given  equation. 

Ex.  3.— Solve       2V2x-\-l  =  3-SVx^. 

Solve  as  in  the  preceding  and  the  roots  are  x=4  or  364,  neither  of 
which  satisfies  the  equation. 

Of  what  equation  is  x  =  4  a  root  ?  Of  what  equation  is  a;  =364  a 
root  ? 

Ex.  4.— Solve    x^-3x-QVx^-SxS  =  -2. 

If  the  surd  is  removed  to  one  side,  we  get 

a;2-3a;+2  =  6V'^^-3x-3. 

If  we  now  square  both  sides  to  remove  the  surd,  we  will  obtain  an 
equation  of  the  fourth  degree  which  we  cannot  easily  solve. 


316  ALGEBRA 

We  may  obtain  the  solution  by  changing  tho  unknown  from  x  to 
V.T^— 3x— 3,  similar  to  the  method  employed  in  art.  192. 
Let 


\/.r2-3x-3  =  ?/, 

x'~Sx—S  =  y-, 

x-  —  3x  =  y-  +  '6. 

in  the  original  equation, 

?y2  +  3-62/=-2, 

,/2^6^-f5  =  0. 

y  =  5  or  1. 

-3a;- 3  =  25, 

or         .T-- 3a;- 3=1, 

-3x-28  =  0, 

x^-3x-4:  =  0. 

7  or  -4. 

a;  =  4  or  —1. 

We,  therefore,  have  foiu*  roots  :    7,-4,  4,  —1. 

Verify  each  of  these  and  show  that  they  all  satisfy  the  given  equation. 
Here  both  values  of  y  were  positive  ;   if  either  of  them  had  been 
negative  it  could  at  once  be  discarded  as  impossible. 

Ex.  5.— Solve  x-\-y—Vx^y^20,  (1) 

xy—2Vxy=l20.  (2) 

From  (1),  Vx+y  =  5  or  —4. 

From  (2),  Vxy=  12  or  -  10. 

Here  the  negative  values  of  the  surds  are  discarded, 
.-.      Vx+y  =  5,    Vxy=l2. 
x-\-y  =  25,      xy=l44. 
Solving  these,  x  =  Q  or  16,  2/=  16  or  9. 

EXBRCISE3  146 

Solve  and  verify  1-17.     Reject  extraneous  roots  : 
1.     a:-f-V'a;=20.  2.     a;— \/a;=20. 

3.     V3x—5+Vx—2=3.  4.     VSx-5-Vx-2=3. 

5.     3a; +\/5x2+l  1+5=0.  6.     3.r+5=\/5a;2+ll. 

7.     —;^  =  5  —  2Vx.  8.     3a;-2V'7.T+4=15. 


SURDS  AND  SURD   EQUATIONS  317 

\/x+16        \/4— x-       T)  ., 

\/4-.i-       \/a;+16      2  ^  ^ 

11.     \/x+a+\/x+6=\/a-6.     12.     V'2a;+5- V'a;-1=2. 

13.  4(x--+a;+3p=3(2.x---+5.c-2)i 

14.  3(.r+\/2-a;2)=z4(a;-\/2-a;2). 

^'      \/3a;2  +  4  +  \/2x2  +1^7"  '      V     2         V    2 

17.     2\/a;+3\/^=12,  18.*  u;(/-\/x7/=30, 

3V'x+2\/^==13.  x-+y=13. 

19.     a:+\/^+2/=28,  20.     ;c+y+A/^+^=30, 

x—  V  .i-;/+?/=12.  X-— (z+a/x— 2/=12. 

21.     a;2+x?/+2/2==91,  22.     x-2+.r2/+2/2=5i, 

a;+V'x^+i/=13.  x— \/a;«/+2/=l2- 

23.  a:2-3x-+6-\/a:2-3x-+6=2. 

24.  x^— .r— Vx^— a;— 6=36. 


y       A/o;      2 

227.     Square  Root  of  a  Binomial  Surd. 

(V'3  +  V2)-=3+2+2v6  =5+2a/6. 
(V5-V3)2=5  +  3-2Vl5  =  8-2\/l5. 
(3-\/2)2=9  +  2-6V2  =11-6^2 

{Va-Vb)^=a-\-b-2Vab. 

The  square  of  \/a-\-Vb  is  made  up  of  a  rational  quantity 
a-\-b,  which  is  the  sum  of  the  quantities  under  the  root  signs, 
and  a  surd  qiiantity  2Vab,  the  ab  being  the  product  of  the 
quantities  under  the  root  signs. 


318  ALGEBRA 

The  form  of  the  square  of  va-l-vb  will  show  us  how  we 
can  sometimes  find,  by  inspection,  the  square  root  of  a 
binomial  surd. 

>/a^b—2Vab  =  Va—V0. 


Ex.  1. — Find  the  square  root  of  7+2'v/l2. 

Here  we  want  two  factors  of  12,  whose  sum  is  7.     They  are  evidently 
4  and  3. 

7  +  2\/r2  =  4  +  3  +  2V473. 


V'7  +  2V'l2=V'4+v'3  =  2+\/3. 


Similarly,  V7-2V'l2  =  2- V3. 

Verify  by  squaring  2+  Vs  and  2—  VS. 

Ex.  2. — Find  the  square  root  of  14— GVs 

To  put  this  into  the  form  a +  6  — 2^06,  first  change  6a/5  into  2^45, 
14-6V5=14-2V'45  =  9  +  5-2V'45, 
.-.       \/l4-6\/5=\/9- V5  =  3- V5. 

EXERCISE  147  (1-7.  Oral) 
Find  the  square  root  and  verify  : 

1.     5+2\/6.               2.     8-2\/l2.  3.  4-!-2\/3. 

4.     6-2  Vs.               5.     10+2  v/24.  6.  15+2\/56. 

7      8+21/7.               8.     7-4\/3.  9.  9+4\/5.      • 

10.     x+y^2Vxy.     11.     x+y—2Vxi/.  12.  2x+2Vx^—y^ 

13.     15-4\/i4.         14.     IS-SVS.  15.  20+V'300. 

1(>.     IO+a/64.           17.     47  + a/360.  18.  57-18\/2. 

19.     a— 2\/a— 1.       20.     4:X -{- 2  V  4^x^—1.       21.  rt— 6— 2\/a— 6— I 


SURDS  AND  SURD  EQUATIONS  319 

Ex.  1. — Find  the  square  root  of  56— 24\/5. 

56-24^/5  =  56-2^720. 

Here  we  require  two  factors  of  720  whose  sum  is  56.  Wlien  tlie 
numbers  are  large,  as  here,  it  may  be  difficult  to  obtain  the  factors 
by  inspection. 

When  this  is  the  case  we  may  represent  the  factors  by  a  and  b  and 
find  the  values  of  a  and  b  from  the  equations 

a6=720, 
a  +  b  =  56. 
Solve  these  equations  by  the  method  of  art.  19-i  or  of  art.  198  and 
obtain 

o=36  or  20, 

6  =  20  or  36. 
The  required  factors  of  720  then  are  36  and  20. 

.-.      56-24\/5  =  56-2V'720  =  36+20-2\/36.  20 

.-.      Vse-  24V5=  V36-  \/20  =  6-2V'5. 
Verify  by  squaring. 

Ex.  2. — Find  the  square  root  of  f  +  Vs. 

9  /-       9  +  4^5       9+2\/20 

4+^^  =  — T— = 4 

the  square  root  is  ^ or  — — (-  1. 

Ex.  3.— Find  the  square  root  of  2\/lO+6V2. 
First  take  out  the  surd  factor  V2,  and  we  get 
2V\0+QV2=  ^2(6  +  2^5), 
the  square  root  =V2(l+\/5). 

EXERCISE   148 
Find  the  square  root  and  verify  : 

1.     94--42V5.  2.     38  +  12Vr6.  S.     47-12\/T5. 

4.     107-24\/l5.  5.     94+6\/245.  6.     10I-28\/T3. 

7.     67+7^72.  8.     28-5^12.  9.     xy+2yVxy^i^-. 


320  ALGEBRA 


10,*  Find  the  value  of  l-^Vl6— 6V7  to  3  decimal  places. 

4 2\/3 

11 .  Find  the  value,  to  three  decimals,  of  the  sq  uare  root  of ^ . 

7+4V3 

12.  By  first  removing  a  simple  surd  factor,  find  the  square  roots  of  : 

7\/2+4\/6,  lO+eVs,  7\/3-12,  59\/2+60. 

13.  Show  that  V  17  +  12^2+^/17-12^2=- 6,  (1)    by    taking    the 
square  roots,  (2)  by  squaring. 


14.  Simplify  V  3+Vl2+V49+8V3. 

,^-  4-2\/3  ,_ 

15.  By  changing  2—  V  3  into x ,  find  the  square  root  of  2— V  3, 

also  of  #  + V2  and  of  V  +^V7. 

16.  From  the  result  of  Ex.  1,  show  that  94— 42\/5  is  a  positive 
quantity  less  than  unity. 

17.  If  .r2(14-6V'5)-=21-8\/5,  find  x  to  three  decimals. 

18.  The   sides    of  a  right-angled    triangle    are    Vs  and    3+2^2. 
Find  the  hypotenuse 


228.  Imaginary  Surds.  When  we  solve  the  equation  x^=Q, 
we  obtain  .t=  ±  3,  and  we  know  that  this  is  the  correct  result, 
for  the  square  of  either  +3  or  —3  is  9. 

If  we  solve  x^=5,  we  say  that  the  value  of  x  is±  V5,  and 
we  can  approximate  to  the  values  of  the  roots  as  closely  as 
we  wish  by  finding  the  square  root  of  5  by  the  formal  method. 

If  we  are  asked  to  solve  x'^——9,  we  might  say  that 
the  solution  is  impossible,  as  there  is  no  number  whose 
square  is  —9.  This  statement  is  correct,  but  we  find  it 
convenient  to  say 

if  .t2=-9, 

then  rr  =  ±  V^^. 


SURDS  AND  SURD  EQUATIONS  321 

Such  a  quantity  as  V— 9  is  called  an  imaginary  quantity, 
and  must  be  distinguished  from  such  quantities  as  5,  —  |, 
^V?,  etc.,  which  are  called  real  quantities. 

We  may  define  an  imaginary  quantity  as  one  whose  square 
is  negative,  or  as  the  square  root  of  a  negative  quantity. 

We  have  already  seen  how  imaginary  quantities  sometimes 
appear  in  the  solution  of  quadratic  equations  (art.   190). 

We  will  assume  that  the  fundamental  laws  of  algebra, 
which  we  have  applied  in  using  real  numbers,  apply  also  to 
imaginary  numbers. 

Thus,  V^^=V'9x  V^  =  3V^^. 

\/^25-f- \/^=4  =  5  V^  +  2  V^  =  7  V^. 

These  examples  show  that  an  imaginary  quantity  can 
always  be  expressed  as  the  product  of  a  real  quantit}''  and  the 
imaginary  quantity  V— 1.  The  quantity  V— 1  is  sometimes 
called  the  imaginary  unit. 

229.  Powers  of  the  Imaginary  Unit.  Any  even  power  of 
V— 1  is  real,  and  any  odd  power  is  imaginary. 

Thiis,  [V^l)^=  -  1,  by  definition. 

.-.      (V^^)*  =  (-l)2=  +  l, 

.-.      (V^^)5  =  (\/^^)*x\/^=  +  V^,  etc. 

230.  Multiplication  and  Division  of  Imaginaries. 
Ex.1.    V^x  V^=V2  .  V^^x  V3  .  V^n^, 

=  \/2.  v'3x(\/^)2=-V6. 
Note  that  the  product  here  is     -  V  0,  not  VG. 


322  ALGEBRA 

Ex.  2.     3\/^4x5V^^  =  6\/^^xl5V'^I, 
=  90(\/^)-=-90. 

EX.3.     ^-y-H>l^.^«.v/9=3. 

V-2  \/2xV-l  V2 

Ex.  4.     (a;  +  2/V^n)''  =  x2  +  2/'(v/^T)2  +  2a;2/V^^, 
=  x2  — 2/2+2a;yV-l. 

Ex.  5.     {a  +  bV'^)(a-bV^^)  =  a^-b^(\/^^l)~=a^+b'- 

3 

Ex.  6.     Rationalize  the  denominator  of jz==^  • 

1  — V  — 2 

Miiltiply  both  terms  by  1  +  V  -  2  and  we  get 

3(1+ V-2)  _  _  3(1+ v;^)  _  1^ v^. 

(l_V-2){l  +  V-2)  l-(-2) 


EXERCISE!   149  (1-9,  Oral) 

1.  Express  as  a  multiple  of  V—~l  :  V— 4,  V— 16,  V— 81,  V— a^ 
\/=^625,  V=9^*,  \/-(^"6)2. 

2.  What  is  the  value  of  (V^l)\  {V^^)\  (V^^)\  (V-f)^  ? 
Find  the  sum  of  : 

3.  V^,  V^,  V^.  4.     V'^25,  V-100,  V^^iO; 
5.     44- V^,  2-\/  — 16.  6.     a+V-ft-,  a-6\/^. 

What  is  the  product  of  : 

7,     V^,  V^.      8.     \/^25,  V-IOO.       9.     \/^^,  V^^^. 
Simplify : 
10.*  3\/-3  +  2\/^75-4\/-12+5a/^^. 

11.  (3  +  5V^)(3-5\/^)  +  (5-3V'^)(5+3\/^). 

12.  (4-3\/^)H(2+6V^)2. 

13.  2  +  (1-a/^).  14.     (-1  +  V^3)  +  (-1-a/-3). 
15.     {a+bV~iy-+(a-bV^^y-. 

IG.     Show  that  -j(-l  +  V^)2=i(-l- V^). 


SURDS  AND  SURD  EQUATIONS  323 

17.  By  finding  the  cube  of  hi  —  l  +  V—S),  show  that  this  quantity 
is  a  cube  root  of  unity,     (art.  192,  Ex.  5.) 

18.  Are  2±  \/^,  the  roots  of  x^—4:X+7=0  ? 

19.  If  a=2-\-SV^-i  and  b=2—3V,^,  show  that  a+6,  ab  and 
a"^+6^  are  real  quantities. 

231.  Impossible  Problems.  We  obtained  the  imaginary 
number  V—9  in  answer  to  the  question,  "  What  is  the 
number  whose  square  is  —  9  ?  "  As  we  have  said,  this  is 
arithmetically  an  impossible  problem. 

When  we  obtain  an  imaginary  result  in  solving  a  problem  we 
may  conclude  that  the  problem  is  impossible. 

Ex.  1. — The  sum  of  a  number  and  its  reciprocal  is  1|-. 
Find  the  number. 

Let  a;=the  number  .".  -  =  its  reciprocal, 

X 

2x2 -3x -1-2  =  0, 
3±\/9^n[6       3±V^1 

.'.       X  =    ;, =   z • 

4  4 

Here  the  roots  are  imaginary,  and  we  conclude  that  there  is  no 
number  which  answers  tlie  condition  of  the  problem. 

In  fact,  it  may  be  shown  that  the  sum  of  a  positive  number  and  its 
reciprocal  is  never  less  than  2. 

Change  H  into  2h  and  solve  the  problem. 

Ex.  2.— For  S30  I  can  buy  x  yards  of  cloth  at  $(10— x') 
per  yard.     Find  x. 

The  total  cost  in  dollars     =x(lO  —  x)  =  '.iO. 
.-.     a;2-10a;+30=0, 

10+ V^^^ 


What  conclusion  do  you  draw  ?     Would  it  be  impossible  if  for  $30 
we  substitute  $25  ?    $20  ? 

y2 


324  ALGEBRA 


SXERCISE   150 

Solve  and  determine  if  these  problems  are  possible  : 

1.  A  line  which  is  10  inches  long  is  divided  into  two  parts  so  that 
the  area  of  the  rectangle  contained  by  the  parts  is  40  square  inches. 
Find  the  lengths  of  the  parts. 

2.  The  length  of  a  rectangle  is  twice  its  width.  If  the  length  be 
increased  10  feet  and  the  width  decreased  1  foot,  the  area  is  doubled. 
Find  the  dimensions.     Solve  also  when  the  width  is  increased  1  foot. 

3.  A  man  has  20  miles  to  walk.  If  he  walks  at  x  miles  per  hour 
it  will  take  him  8— a;  hours.     At  what  rate  does  he  walk  ? 

4.  If  it  is  possible  that  x{\2—x)  —  ZQ^a,  and  a  is  not  negative, 
what  must  the  value  of  a  be  ? 


BXBRCISB  151  (Review  of  Chapter  XXIV) 

1.*  Multiply  l  +  \/3-V2by  \/6- v/2. 

2.  Multiply   V3+  \'2  by  4-  +  ^  • 

V3       V2 

3.  Find  the  square  roots  of  :    14+ \/T80,  25  — 4V2l,  22+ V'420, 
11-V72,  12-6V'3. 

4.  Find  to  two  decimal  places  the  values  of  : 

1  / —        7+^3        3A/8+V20 

Vf^Z      V5  +  2V6,    2v'3  +  V8'      V5+V2   " 

Solve  and  verify  : 

5.  x+\/x^=\\.  6.     V'4a;  +  7+\/4x  +  3  =  6. 

7.     Vx  +  Vx^l  =      ,^-^--        8.     V6x  +  7-V'.r+2=V2\/x+T. 
Va;— 4 

35 


9.    Vx+s  +  Vx-ie  = 


Vx+S 


, , 24 

10.      V19  +  a;—  V15  — a;  =  —    .T^T- 
Vi9+a; 


SURDS  AND  SUED  EQUATIONS  325 

11.  Multiply    V'2+V'3+V5  by    v/5+a/3-V'2   and 

x  +  y+2Vx  +  y   by   x  +  y-2Vx  +  y. 

12.  When  x=2+VTi,   y=2-  Vs,   find  the  value  of 

2x+y      x-2y  _ 
x-y        x+y 

13.  Expand   and   simplify    {V2a  +  Vh+V2a-Vb)^.      Check  the 
result  by  substituting  a=  13,  6=  100. 

14.  Solve  p-\-x—V2px-{-x^=q. 

15.  Find  the  product  of  2  \/3  +  3\/2+ VSO  and  V2+V'3-a/5. 

16.  Find  the  continued  product  of  ^ 

a;- 1+^2,  x-l-V-I,  x+l+Vs  and  x+l-VS. 

17.  Simphfy     7;^--— ;^ 7=-—  + 


Ve+Vs     Vs+i     ^3+^2 

18.  When  x  =  a-\-  \/a-—\,  find  the  values  of  : 

x  +  -  '    a;2  +  — ,    x-  H — :.• 
X  x^  ic-* 

19.  Express  in  the  simplest  form  : 

V'27-  \/8+  ^17  +  I2V2-  ^28-6-^/3 
and  \/ll  +  6v'2+\/l9-4Vl2fV'5-V'2i. 

on       Q-       r*      V12  +  6V3         ,       fm+n  /^T^n 

20.  Simplify r= and  ^/ h  \/  — ; 

VS+l  ^    m— /!.       V   m+n 

21.  If  x=-l  +  2V'^,  find  the  value  of  a;" -12a;. 

22.  Find  the  square  roots  of  7-}-  VlS  and  2a-\-  \/4a-  — 4. 

23.  Solve  2x2  +  6x  -  6  -  Vx'^  +  3a;  -  3  =  45. 

24.  Simplify  (\/.^+  \/3+  v/2)2  +  (\/5+  \/3- \/2)2 

+  (\/5  -  v/3+v/2)2-f-(\/5- V'3-\/2)2. 

25.  Solve    3x2  — 9x+ ll  =  4\/x2  — 3a;  +  5,    giving    the    roots    to    two 
decimal  places. 

Va-\-b  +  Va  —  b       Va  +  6  —  y/a  —  b 


26.     Simphfy 


Va+b—Va—b       Va+b+Va— 6 


326  ALGEBRA 

1 


Show  that  — 7^===^_-  + 


-\/l6+2V63       Vie- 2^63 


28.     Show  that   va+  Vb  cannot  be  expressed  in  the  form  Vx+Vy 
unless  a'^  —  b  is  a  perfect  square. 


29.  S,n.p..fy  f?'-^')'  +  [1=^] 

30.  Simplify  {3-2V2)^ +  {S  +  2V2f. 

.31.     Find  the  value  of  x^-\-x^+x+l  when  .^•=V3^-l. 


CHAPTER  XXV 

THEORY    OF    QUADRATICS 

232.     Sum  and  Product  of  the  Roots. 
Solve  these  equations  : 

(1)  a;2— lLr+10=0.      The  roots  are  10,   1. 

(2)  2x2-  3^_  5^0.        „       „         „      I,  -1. 

(3)  15.T2+26.r+  8=0.        ,,       „         „  -|,  —i. 

In  (1)  the  sum  of  the  roots  =  11,  product  =  10. 
In  (2)  „  „  „  „  „  =  I,  „  =-|. 
In  (3)     „       „      „     „        „      =-f|,         „        =  T%. 

Examine  the  sum  and  the  product  in  each  case  and  state 
how  they  compare  with  the  coefficients  in  the  given  equations. 
Every  quadratic  equation  may  be  rediiced  to  the  form 

a.r2-(-6.T+c=0. 

— 6+\/62— 4ac       ,  —h—Vb^—^ac 


2a                               2a 
For  brevity  represent  these  roots  by  m  and  n, 

-b4-Vb^-^ac-b-Vb^-4:ac      -26 
2a                               2a 

b 
a 

(-6+V62-4ac)(-6-V62-4ac) 
id          mn-y                        ^2 

(_6)2_(V62i:4ac)2      b^-b^-\-4ac      4ac 

c 

4a^  4a^  4a^ 

327 


328  ALGEBRA 

Comparing  these  results  with  the  coefficients  a,  b,  c  in  the 
equation,  we  see  that : 

The  sum  of  the  roots  of  any  quadratic  equation,  in  the  standard 
form,  is  equal  to  the  coefficient  of  x  with  its  sign  changed,  divided 
by  the  coefficient  of  x^,  and  the  product  of  the  roots  is  equal  to  the 
absolute  term,  divided  by  the  coefficient  of  x^. 

coefficient  of  x 


Sum  of  the  roots 
Product  of  the  roots 


coefficient  of  x^ 

absolute  term 

coefficient  of  x^ 

See  if  these  two  statements  apply  to  the  roots  and  co- 
efficients of  the  three  equations  preceding. 

The  formulae  for  the  sum  and  product  of  the  roots  furnish  a 
convenient  means  of  verifying  the  roots. 

Thus,  I  find  the  roots  of  3x^  +  x  —  2  =  0  to  be  |,  —1,  but  the  sum  of 
I  and  —  1  is  —  J  and  the  product  is  —  |,  which  agree  with  the  sum  and 
product  given  by  the  formulae.  Therefore,  these  are  the  correct 
roots. 

Are  the  roots  of  Ux--  iar-60  =  0,  V",  -*  ? 

233.  Reciprocal  Roots.     If  the  roots  of  ax'^-\-bx-\-c=0  are 

reciprocals  (hke  §  and  :f ),  their  product  is  unity,  and  therefore 

c      , 

-  =1  or  c=a. 

a 

So  that  any  quadratic  equation,  in  which  the  coefficient  of 
x^  is  equal  to  the  absolute  term,  ivill  have  reciprocal  roots. 

Thus,  the  roots  of  6a;^—  13a;+6  =  0  are  reciprocals,  since  their  product 
is  I  or  I.     Verify  this  by  finding  the  roots. 

234.  Roots  equal  in  Magnitude  but  opposite  in  Sign.     If  the 

roots  of  ax'^-\-bx-\-c=0  are  equal  in  magnitude  but  opposite 
in  sign   (like  3  and   —3),  their  sum  will  be  zero,  therefore 

=  0  or  6=0. 

a 

So  that  any  quadratic  equation  in  which  the  second  term  is 

missing  will  have  roots  equal  in  magnitude  but  opposite  in  sign. 

Thus,  20;"  — 9  =  0  and  ax^  —  c  =  0  have  such  roots. 
Verify  by  finding  the  roots. 


THEORY  OF  QUADRATICS  329 

BXBRCISE;  152  (Oral) 

State  the  sum  and  product  of  the  roots  of  : 

1.     a:2— 7x+12  =  0.  2.     x^-ox— 11=0. 

3.     x24-6x-fl=0.  4,     2a;^-10x+6=0. 

5.     3x^—12x-l=0.  6.     4x2-17x+4=0. 

7.     ax^—bx—c=0.  8.     o.r'^— (6+c)a;4-cf=0. 

9.    pa;2— 9=0.  10.     ax'^+a=0. 

11.     3x2— 4a;=6.  12.     (a+b)x^-'X+a^--b-^=0. 

13.  Which  of  the  preceding  equations  have  reciprocal  roots  ? 
Which  have  roots  equal  in  magnitude  but  opposite  in  sign  ? 

14.  Are  4  and  5  the  roots  of  x^— 9x+20=0  ? 

15.  Are3+V'2,  3— \/2  the  rootsof  x2— 6.r-L7=0? 

In  which  of  the  following  are  the  correct  roots  given  : 

16.  a;2— 7a-+10=0;  5,  2.  17.     .i-+3.r— 28=0  ;  7,  -4. 
18.     x2-13a;+36=0;  4,  9.  19.     x2-12a;+27=0  ;  4,  8. 
20.     a;2— 4a;— 5=0  ;  5,  1.               21.     2x2— 5a;+2=0  ;  2,  J. 

22.  In  solving  x2—2x— 1.599=0,  one  root  is  foimd  to  be  41.  What 
must  the  other  be  ? 

23.  How  would  you  show  that  1-12.5  and  2- 168  are  the  correct 
roots  of  x2— 3-293x+2-439=0  ? 

24.  If  the  roots  of  Gx^— 10x-)-a=0  are  reciprocals,  what  is  the  value 
of  a? 

25.  If  the  roots  of  mx^—{7n-—9)x-{-m^=0  are  equal  in  magnitude 
but  opposite  in  sign,  what  is  the  value  of  to  ?  What  would  then  be 
the  product  of  the  roots  ? 

235.  To  form  a  Quadratic  with  given  Roots.  First  Method. 
In  the  equation  x^-\-px-\-q=0,  the  sum  of  the  roots  is  —p, 
and  the  product  is  q.  Since  every  quadratic  equation  may  be 
reduced  to  the  form  x^-\-px-\-q=0,  by  dividing  by  the  co- 
efficient of  x^,  any  quadratic  equation  may  be  written  thus  : 

x^—x  (sum  of  roots) + (product  of  roots)  =0. 


330  ALGEBRA 

If  the  roots  are  given,  the  equation  can  at  once  be  written 
down. 

Thus,  tne  equation  whose  roots  are  3  and  5  is  a;*  — .t(3-|-5)  +  3  .  5  =  0, 
or  x2  — 8a;+15  =  0. 

The  equation  whose  roots  are  2+  \/3  and  2—  V  3  is 

a;2-a;(2+\/3  +  2-V'3)  +  (2+V'3)(2-\/3)  =  0,  or  a;2-4a;+l  =  0. 

The  equation  whose  roots  are  a-\-b  and  a  —  b  is  x^  —  2ax  +  a^  —  b^  =  0. 

Second  Method.     The  equation  whose  roots  are  p  and  q 

is  {x—p){x—q)=0. 

The  equation  whose  roots  are  3  and  5  is  {x—S){x  —  5)  =  0,  or 
a;2-8x+15  =  0. 

The  equation  whose  roots  are  j  and  —  f  is  (x— |)(aj+i)  =  0>  or 
(3a;-2)(4a;+3)  =  0. 

The  equation  whose  roots  are  2+^3,  2— V3  is 

{x-2-V3){x-2+  \/3)  =  0,  or  (a;-2)2-3  =  0  or  x^-4:X+l  =  0. 

Either  method  is  simple  enough  to  apply,  but  the  first  is 
probably  easier  when  the  given  roots  are  not  simple  numbers. 

The  second  method  may  be  applied  to  form  an  equation 
with  any  number  of  given  roots. 

Thus,  the  equation  whose  roots  are  2,3,  —  5  is 

{x-2){x-S)(x  +  5)  =  0,  or  .T»-- 19a; +30  =  0. 


EXERCISE  153  (1  16,  Oral) 

State,  without  simplifying,  the  equations  whose  roots  are : 
1.     3,  7.  2.     3,  —7.  3.     —3,  7.  4.     —3,  —7. 

K         1     1  <!         1      1  7         1     1  8         2     3 

9.     a,  a.  10.     — rt,  —6.       11.     3,  0.  12.     0,  m. 

13.     3,4,5.         14.     2,3,-1.      15.     a,  b,  c.  16.     rr,  6,  0. 

Reduce  to  the  simplest  form  the  equations  whose  roots  are  : 
17*  m+n,m—n.        18.     2a—b,2a+b.        19.     S  +  VS,  3— \/5 
20.     If,  -21  21.     -2,  -4,  6.  22.     |,  ^,  J. 

23.     Show  that  1-25  and  4-64  are  the  correct  roots  of 
100a;2-589.T+580=0. 


THEORY  OF  QUADRATICS  331 

24.  Construct  an  equation  in  which  the  sum  of  the  roots  is  7  and  the 
difference  of  their  squares  is  14. 

25.  Form  the  equation  whose  roots  are  a  and  h  where  a^-\-h'^=25, 

2G.     Form  the  equations  whose  roots  are 

a-\-h      a—b  ,_ 

-S.,     --,;     i(4±\/7). 
a—b      a-\-b       -^  ' 

27.  Find  the  sum  and  the  product  of  the  roots  of  : 
(1)  (a;-2)2=5.r-3.  (2)  {x-a){x-b)=ab. 

(3)  x*x-p)^p{x-q).  (4)  {x+ay'+{x+by'^{x+c)^ 

28.  Solve  .T*— 21.t-— 20.i-=0,  being  given  that  one  root  is  5. 

29.  If  one  root  of  x'^—12x-\-a=0  is  double  the  other,  find  the  roots 
and  the  value  of  a. 

30.  If  one  root  of  x^-\-px+4:8=0  is  three  times  the  other,  what  are 
the  values  oi  pt 

236.  Functions  of  the  Roots.  When  m  and  n  are  the  roots 
of  ax^-\-hx-\rC=^, 

h  c 

m-\-n  = ,  mn  =  - . 

a  a 

Here  it  will  be  seen,  that  the  sum  and  the  product  of  the 
roots  do  not  contain  surd  expressions,  while  the  separate 
roots  do. 

If  we  wish  to  find  the  sum  of  the  squares  of  the  roots,  we 
can  do  so  in  the  following  way  : 

m^-^-n'^  =  {m-{-7i)'^—2mn, 

a  a 

-  ^J  _  2^  =:  ^^-^^ 
a^      a  a- 

It  can  also  be  found  by  taking  the  square  of  each  root 
and  adding  the  results  Find  it  that  way  and  see  if  you  get 
the  same  result. 


332  ALGEBRA 

Ex.  1. — When  m  and  n  are  the  roots   of  ax^-\-bx-\-c=^0, 

1  1       7/}'         Ti 

find  the  vahies  of 1 — ,    — | ,  m'^-\-n^,  m—n. 

m      n     n       m 

11       7n-\-n  ft    .   c  b 

m       n         mn  a   '   a  c 

62_  2c 

m       n  _  7n2-)-n*       (7n-\-n)-  —  2mn_a^       a       b"—2ac 
n      m  mn  mn  c       ~      ac 


,,,,,,„„,,,  b^       36c      Sabc—b^ 

m^-\-n^  =  {m-\-nY  —  6mn{m-\-  n)  = ;  +  — „-  =  —  , —  > 

or  m^  +  n^  =  ('m-\-7i)(m,^  —  mn~\-n^)  =  (m-{-7i)\(m-\-n)^  —  Smn}  =  etc. 

/           \2       ,      ,     \9     A             b^       4c       b^~4ac 
(m—n)^  =  {m,-\-ny  —  4'mn  =  — „ =  5 —  » 

Vb^'-iac 

m  —  n  =  + . 

a 

The  same  two  values  of  the  last  expression  might  have  been  found 
by  simple  subtraction,  the  sign  depending  on  the  order  in  which  the 
roots  were  taken. 

Ex.  2. — If  m  and  n  are  the  roots  of  x^-\-px-\-q=0,  find  the 
equation  whose  roots  are  m^  and  n^. 

Here  m-\-n=  —p  and  mn  =  q. 

The  sviin  of  the  roots  of  the  required  equation  is 
m2  +  n2  =  (m  +  n)-—  '2mn=p~  —  2q. 

The  product  of  the  roots  =  w-n^  =  7-. 
.".    the  required  equation  is  x~  —  x(p--~2q)-\-q-  =  Q. 

Ex.  3. — Find  the  equation  whose  roots  are  each  greater  by 
2  than  the  roots  of  6a:2— 13a;— 8=0. 

Solve  the  given  equation  and  the  roots  are  f ,  —  \. 
.'.    the  roots  of  the  required  equation  are  V,  %. 
.'.    the  required  equation  is 

x^-{\t-\-%)x  +  h*  .  1=0,  or  6.-r2-37.r  +  42  =  0. 
We  might  have  solved  the  problem  without  finding  the  actual  roots 
of  the  given  equation. 


THEORY  OF  QUADRATICS  333 

Let  p  and  q  be  the  roots  of  6a;*—  13a;  — 8  =  0. 
Then  P  +  <l  =  ^i  and  M=— I- 

.*.    the  sum  of  the  roots  of  the  required  equation  is 

p  +  2  +  fy  +  2=p  +  5  +  4  =  -V-  +  4  =  -«/, 
and  the  product 

=  (p  +  2)(g+2)=M+2(p  +  g)  +  4=-A  +  V  +  4  =  7, 

.'.    the  required  equation  is 

.t2-=Vx  +  '7  =  0,  or  6a;2- 37a; +42  =  0. 

When  would  the  second  method  be  simpler  than  the  first  ? 

Ex.  4. — Find  the  equation  whose  roots  are  the  reciprocals 

of  the  roots  of  mx'^-\-nx-\-k=^(). 

Let  p  and  q  be  tlie  roots  of  the  given  equation, 

then  P  +  <1= and  pq=  —' 

m  m 

The  roots  of  the  reqviired  equation  are  -  and  —  • 

Find  the  sum  and  product  of       and  -  in  terms  of  m,  n  and  k  and 

p        q 

complete  the  solution. 

Compare  the  new  equation  with  the  given  one  and  see  if  you  could 
not  write  down,  mentally,  the  equation  whose  roots  are  the  reciprocals 
of  the  roots  of  any  given  equation. 

237.  The  following  method  will  be  found  useful  in  solving 
such  problems  as  the  three  preceding. 

Ex.  1. — Find  the  equation  whose  roots  are  each  greater 
by  5  than  the  roots  of  4^2— 5a;+7=0. 

Let  y  be  the  unknown  in  the  required  equation. 

Then  y  —  x-^5  or  x  =  y  —  o. 

Substitute  x  =  y  —  5  in  the  given  equation,  and  the  required 
equation  is  4(y  —  o)^  —  5{y  —  5)-\-7  =  0, 

or  l-y*-40i/+ 100-52/4-25  +  7  =  0, 

or  42/2-4%+ 132  =  0. 


334  ALGEBRA 

Ex.  2. — Find  the  equation  whose  roots  are  the  squares  of 
the  roots  of  ax^~\-bx-\-c—0. 

Let  y  be  the  unknown  in  the  required  equation. 

Then  y  =  x^  or  x=  +  Vy, 

.".     the  required  equation  is  a{±Vyy-{-b{±Vy)+c  =  0, 

or  ay->rC=   +bVy, 

or  a^y^-'i-2acy-{-c^  =  b^y, 

or  a^^/2  +  ?/(2ac  — 62)  +  c-  =  0. 

Solve  Ex.'s  2,  3,  4  preceding,  by  this  method. 


BXERCISE    154 

1.*  If  tn  and  n  are  the  roots  of  X'—5x-\-'S—0,  find  the  values  of 

I    .   I   m      11       ,  ,         ,     o 

— |-  -,  — I — ,  nr-\-'mn->rn-. 
m      n   n      m 

2.  Find  the  sum  of  the  squares  of  the  roots  of 

a;2-7.r+l=0  and  of  3a;2-4a;+5=0. 

3.  If  p  and  q  are  the  roots  of  Sx'^-\-2x—6=0,  find  the  values  of 

-  +  -,   2  +  -T'  p^-pq+q^- 

q      p  p^      q^ 

4.  Find  the  sum  of  the  cubes  of  the  roots  of 

2x-2— 3a;+4=0  and  of  x^— x+a=0. 

5.  Find    the    equation    whose    roots    are    double    the    roots    of 

a;^— 9a:+20=0,  (1)  by  solving,  (2)  without  solving. 

6.  Find  the  equations  whose  roots  are  each  less  by  3  than  the 
roots  of  (1)  a;2-lla;+28=0,  (2)  x^-x-l=0. 

7.  Find  the  equations  whose  roots  are  the  reciprocals  of  the  roots 
of  (1)  2x2+a;-6-=0,  (2)  x^-px+q=0. 

8.  If  m  and  n  are  the  roots  of  3,*'-— 2.r+5=0,  find  the  equations 
whose  roots  are : 

(1)  —  and  -,    (2)  -  and  — ,    (3)  m-  and  n^. 
in  n  n  m 


THEORY  OF  QUADRATICS  335 

9.     Find  the  sum  of  the  squares  and  the  sum  of  the  cubes  of  the 
roots  of  x'^^ax-\-h—0. 

10.  Find  the  equation  whose  roots  are  the  squares  of  the  roots 
of  x--\-px—q=0. 

11.  Find  the  equation  whose  roots  are  each  greater  by  h  than  tlie 
roots  of  ax'^-f-6a;+c=0. 

12.  Find  the  equation  whose  roots  are  the  reciprocals  of  the  roots 
of  x'^-\-x=\. 

13.  If  m  and  n  are  the  roots  of  x^—px-\-q—0,  show   that  m-[-n 
and  mn  are  the  roots  of  x^—x{p^q)^pq=0. 

14.  Form  the  equation  whose  roots  are  m  and  n,  where 

7n--|-w-=20,  TO+W.=  — 6. 

15.  If  m  and  n  are  the  roots  of  x'^-\-px-\-q=0,  show  that  wi+2?i 
and  2m +71  are  the  roots  of  x^-\-Zpx-\-2p^-\-q^:^Q. 

16.  If  p  and  q  are  the  roots  of  ax'^-\-hx-\-c=0,  Hnd  the  vahie  of 
p*-\-p'^q'^-\-q^  in  terms  of  a,  b  and  c. 

238.     Character  of  the  Roots  of  a  Quadratic  Equation. 
Solve  the  equations  : 

(1)  x^—6x-[-  9=0,  the  roots  are  3,  3. 

(2)  6a;2+  a;- 15=0,     „       „        „     |,  -|. 


(3)  5x^+lx-  2=0, 

(4)  2x^—Sx-\-  2=0, 


)J  5> 


-7±V89 

10 
3±V^ 

4 


In  (1),  the  roots  are  equal.  VVe  might  say  that  there  is 
only  one  root,  but  we  prefer  to  say  that  there  are  two  roots, 
which  in  this  case  happen  to  be  eqiial. 

In  (3),  the  roots  are  irrational,  but  we  can  approximate 
to  their  values  by  taking  the  square  root  of  89. 

In  (4),  the  roots  are  also  irrational,  but  we  can  not  even 
approximate  to  their  values.  Here  the  roots  are  imaginary, 
while  in  each  of  the  others  the  roots  are  real. 


336  ALGEBRA 

These  statements  might  be  written  thus : 

In  (1),  the  roots  are  equal,  real  and  rational. 
In  (2),  the  roots  are  unequal,  real  and  rational. 
In  (3),  the  roots  are  irrational  and  real. 
In  (4),  the  roots  are  irrational  and  imaginary. 

If  we  examine  the  roots  of  the  general  quadratic  equation 
we  will  see  the  reason  why,  under  particular  conditions,  there 
is  this  difference  in  the  character  of  the  roots. 

The  roots  of  ax'^-^bx-\-c=0  are 

— 6+\/62— 4ac       ,  — 6— \/62— 4ac 
and 


2a  2a 

From  these  roots  we  may  conclude  : 

( 1 )  If  the  particular  values  of  a,  b,  c  are  such  that  6^— 4ac=0, 

then  the  roots  are  equal,  for  each  is  evidently  equal  to  —  ^  • 

In  equation  (1),  «=  1,  b=  —6,  c  =  9. 

.-.      b2-4oc  =  36-36  =  0. 

(2)  If  b'^—4:ac  is  a  perfect  square,  then  its  square  root  can 
be  found  exactly  and  the  roots  are  rational. 

In  equation  (2),  a=6,  6=1,  c=  — 15. 

.-.      62- 4ac=  1  +  360  =  361  =  192. 

(3)  If  b^—4ac  is  not  a  perfect  square,  but  is  positive,  the 
roots  are  real  but  irrational. 

Find  the  value  of  6-  — 4cfc  in  equation  (3). 

(4)  If  b'^—4:ac  is  negative,  the  roots  are  imaginary. 

Find  the  value  of  b^-iac  in  equation  (4). 

Hence,  the  roots  of  ax2+bx-|-c=0  are  real  and  equal  if 
b^ — 4ac— 0,  real  and  unequal  if  b^  — 4ac  is  positive,  imaginary 
if  b^— 4ac  is  negative,  real  and  rational  if  b^— 4ac  is  a  perfect 
square. 


THEORY   OF  QUADRATICS  337 

239.  The  Discriminant.  We  see  then,  that  we  can  deter- 
mine the  character  of  the  roots  of  a  quadratic  equation 
without  actually  finding  the  roots.  All  we  require  to  do  is  to 
find  the  value  of  &-— 4ac. 

This  important  quantity  is  called  the  discriminant  of  the 
equation  ax'^-\-hx-\-c=0. 

Ex.  1. — Determine  the  character  of  the  roots  of  : 
(1)  3a:2+5a:-ll=0.        (2)  12a:2-25.r+12=0. 

(3)     x^-  X-+  3=0.        (4)     2,1-2- 16^+32=0. 


The  value  of  the  discr 
in(l)is     157, 
in  (2)  is       49, 
in  (3)  is  -11, 
in  (4)  is         0, 


minant  (6-  — 4ac) 

•.    the  roots  are  real  and  irrational, 

■.   the  roots  are  real  and  rational, 

".   the  roots  are  imaginary, 

•.   the  roots  are  real  and  equal. 


Ex.  2. — For   what   values   of    k   will   4.t2— A;a;-{-4=0  have 
equal  roots  ? 

The  roots  will  be  equal  if  6^  — 4ac  =  0,  : 

that  is,  if  A,-2-64  =  0  or  if  A-=  ±8. 

Substitute  these  values  for  k  and  see  if  the  roots  are  equal. 

Ex.  3. — Show  that  the  roots  are  rational  of 

3mx2— a;(2m+3w)+2»i=0. 

Here  6^  — 4ctc  =  (2m+3«)^  — 24mn, 

=  4m^  —  1 2m>t  +  9/1^  =  ( 2^?i  —  3ji)-. 
Since  b^—^ac  is  a  square,  the  roots  are  rational. 
Verify  by  finding  the  roots. 

EXERCISES  165  (1-5.  Oral) 

1.  What   is    the   discriminant   of   x-"--f4x+4=0  ?     What   is    tho 
character  of  the  roots  ? 

2.  What  is  the  nature  of  the  roots  of  x^-\-^x-\-2—Q  ? 

3.  What  is  peculiar  about  the  roots  if  b^—4:ac=0  ? 

4.  What  kind  of  roots  have  .r-— 5a; +7=0,  :c-  —  6.r 4-9=0, 

a;2-.x--6=0,  x^—4.x—Q=0  ? 

5.  If  the  discriminant  is  -25,  what  is  the  character  of  the  roots  ? 

Z 


338  ALGEBRA 

Determine  the  character  of  the  roots  of  : 
6.*  2:r-+5a;+3=0.  7,     3x'—lx—5=0. 

8.     4x2+7.T+15=0.  9.     9a:2— 12.r+4=0. 

10.     abx'-+x(a^+b'^)+ab=0.  11.     x^—mx—\^0. 

12.  Show  that  x^-^-ax+b^O  has  real  roots  for  all  negative  values 
of  i. 

13.  If  9x^-\-12x-\-k—0  has  equal  roots,  find  k. 

14.  If  «a;-—10a;+ff=0  has  equal  roots,  find  a. 

15.  Show  that  the  roots  of  x'^—x{l-\-k)-\-k=0  are  rational  for  all 
values  of  k. 

16.  If  .T-+2a:(l+«)+«"=0  has  equal  roots,  find  a. 

17.  By  solving  the  equation  x" — 4:X-\-5=k,  show  that  if  x  is  real, 
k  cannot  be  less  than  1. 

18.  Show    that    the    roots  of   — | \ =  0   are   real   if 

X       x-\-a       x^b 

a^—ab-\-b^  is  positive. 

19.  Eliminate  y  from  the  equations  y=mx+c  and?/^=4ax,  and  find 
the  value  of  c  if  the  resulting  equation  in  x  has  equal  roots. 

20.  If  2mx^ -\-{5m-\-2)x-'r(im-'rl)=0  has  equal  roots,  find  thevalnos 
ot  m  and  verify. 

240.     Factors  of  a  Quadratic  Expression. 

When  m  and  7i  are  tb.e  roots  of  ax^-\-bx-{'C=0, 

h  c 

a  a 

,\     ax^-\-hx-\-c=a(x"-\ — x -\ — ) 
\  a         a/ 

=a{x^—{m-{'n)x-\-mn] 

=a{x—'m){x — n). 

So  that,  if  m  and  ri  are  the  roots  of  ax^-{'bx-{-c=0,  the 
factors  of  the  quadratic  expression 

ax^-\-  bx-\-c  are  a(x—m){x—7i). 


THEORY  OF  QUADRATICS  W.V.) 

We  can,  therefore,  find  the  factors  of  a  trinomial  like 
ax'^-\-hx-\-c  by  solving  the  corresponding  equation. 

Ex.  1.— Factor  6.i--+.r-40. 
Solving  by  formula,  we  find  the  roots  of 

6x2 +  a;- 40  =  0  are  |,  —  f. 
.-.      Ga;2+a;-40  =  6(x-fi)(a;  +  f) 
=  (2x-5)(3a;+8). 

Ex.  2.— Factor  12.r2_47a;+40. 
The  roots  of  the  corresponding  equation  are  ^,  f, 
.-.      12a;2-47a;  +  40=12(x-|)(,r-.:;) 

=  (4a;-5)(3a;-S). 

241.  Character  of  the  Factors  of  a  Trinomial.  Since 
a.r-+6.r+c=0  has  equal  roots  when  & "^—400=0,  it  follows  that 
ax^+bx+c  has  equal  factors,  or  is  a  square,  tvhen  b^— 4ac=0. 

Thus,  in  Sx^-SOx+To,     62 _4cjc  =  900 -900  =  0. 

.'.  3x^—30x+75  is  a  perfect  square  when  the  numerical  factor  3 
is  removed. 

//  b^—4ac  is  a  jJerfed  square,  the  expression  ax~'\-bx-\-c  has 
two  rational  factors,  for  under  this  condition  the  corresponding 
equation  has  rational  roots. 

Thus,  in  20x2-a;-12,     62-4ac  =  961=312. 

20x'^  —  a;— 12  has  rational  factors.     Find  the  factors. 

242.  Surd  Factors  of  a  Trinomial.  When  we  say  that  a 
trinomial  can  be  factored,  we  usually  mean  that  it  can  be 
expressed  as  the  product  of  rational  factors. 

As  we  have  seen,  this  can  always  be  done  when  h'^—4:ac 
is  a  perfect  s([uare. 

When  there  are  no  rational  factors  we  may  use  the  pre- 
ceding method  to  find  surd  factors. 

Ex. — Find  two  surd  factors  of  a:-— 6.r4-4. 

If  a;2-6a;+4  =  0,   x  =  -^.^  "^  =  3±  V5. 


•      x^-6x+i  =  (x-S-Vo){x-3-i-\/5). 
Verify  by  multiplication. 


Z2 


340  ALGEBRA 

EXERCISE   156 

Factor,  by  trial  if  you  can,  otherwise  by  solving  the  corresponding 
equations  and  verify  : 

1.  3a;2-17,r+10.  2.     20a;2  +  3x--108. 

3.  x2-2x--1763.  4.     1800a2-5a-l. 

5.  299.t;-  +  10x-l.  6.     221a;2-458aa;+221a2. 

7.  Show  that  12.c-— 15x+4  has  no  rational  factors. 

8.*  x^-\-4:X—'3  has  no  rational  factors.  Find  two  surd  factors 
of  it. 

9.     If  x^—8x-\-k  is  a  perfect  square,  find  k. 

10.  If  ax'"—kx-'r9a  is  a  perfect  square,  when  the  factor  a  is  removed, 
find  L 

11.  Express  .r-— 6.f— 11  as  the  product  of  two  surd  factors. 

12.  Factor  144.?;*— 337.c-.^''+144//*.  When  this  expression  is 
equal  to  zero,   find   four  values  of  the  ratio  of  x  to  y. 

13.  If  .r— 2  is  a  factor  of  120.^3— 167.1--— o,;--|- 56,  find  the  value  of 
a  and  find  the  other  two  factors. 

14.  By  finding  the  square  root  of  ax'^^bx-\-c,  find  the  relation  which 
must  connect  a,  b  and  c  when  this  expression  is  a  perfect  square. 

243.     A    Quadratic    Equation    cannot    have    more    than    two 

Roots.      We  have  .seen  that  the  e(|uation  ax'^-\-bx-\-c—'d  has 
two  roots,  and  since  this  equation  represents  every  quadratic, 
it  follows  that  every  quadratic  equation  has  two  roots. 
It  cannot  have  more  than  two  roots. 

Let  m  and  n  be  the  roots  of  ax'^+hx-\-c=0. 

Then  ax^^hx^c=a{x—m){x—n)   (art.  240) 

«(.}•— rn)(.x—w)=0. 

Since  this  product  is  zero,  one  factor  must  be  zero.  But 
a  is  not  zero,  for  the  equation  would  }iot  then  be  a  quadratic. 
Therefore,  eitJier 

•X'— m=0  or  .T— 71=0. 


THEORY   OF   QUADRATICS  341 

But  no  values  of  x  other  than  in  and  //  will  nuikc  cither  of 
these  quantities  equal  to  zero. 

m  and  ti  are  the  only  roots. 

Since  the  quadratic  equation  aa:'-+6.T+c=0  has  only  two 
roots,  then  the  quadratic  expression  ax'^+bx  +  c  can  he  resolved 
into  linear  factors  in  only  one  way. 

EJXBRCISB  157  (Review  of  Chapter  XXV) 

1.  What  is  the  sum  and  the  pi-oduct  of  the  roots  of  a.r'  +  6a;  +  c  =  0  ? 

2.  Under  what  condition  are  the  roots  of  ax"-\-hx^c  =  {)  reciprocals  ? 
When  are  they  equal  in  magnitude  but  opposite  in  sign  ? 

3.  When  are  the  roots  of  oa;2-i-6x  +  c  =  0,  (1)  equal,  (2)  real, 
(3)  imaginary,  (4)  rational  ? 

4.*     Ifj)-r(/  =  4  and  pq  =  o,  find  the  values  of 

5.  Find  the  sum  and  the  product  of  the  roots  of 

(3.T-2)(.T-3)  =  (.T-l)(.-r-o). 

6.  Find  the  equation  whose  roots  are  each  one-lialf  of  the  roots 
of  4.r2-20.T^21  =  0. 

7.  Find  the  sum  of  the  squares  of  the  roots  of  S.r^— lla;+ 1  =  0. 

8.  Find  the  equation  whose  roots  are  twice  as  great  as  tlie  roots 
of  24x2-38x+15  =  (). 

9.  For  what  value  of  k  w  ill  .r-—  ]Ox  =  k  have  equal  roots  ? 

10.  Find  the  equation  whose  roots  are  m  and  n  wlian   OT*  +  n^=74 
and  7Hn  =  35. 

11.  Factor   52o6.T2  +  ,r     1   and   221a;2-8a;- 165. 

12.  Form  the  equation  whose  roots  are  w  and  n  where  m^-f-n'  =  28 

and  7M  +  n  =  4. 

13.  Find  the  siun  of  the  roots  of  {x  —  a)^  +  (x  —  b)-  =  (x ~ c)-. 

14.  Construct  the  equation  whose  roots  are  the  reciprocals  of  the 
roots  of  17a;-  +  53x  — 97  =  0. 

15.  Express  x^-{-6z+7  as  the  product  of  two  linear  factors. 


342  ALGEBRA 

16.  Construct  the  equation  whose  roots  are  each  greater  by  7  than 
the  roots  of  2a;2+lla;-21  =0. 

17.  Find  the  equation  whose  roots  are  each  three  times  the  roots 
oi  ax^  +  bx-\-c  =  0. 

18.  If  ?/i  and  ■»,  are  the  roots  of  ax'^-\-bx-\-c  =  0,  find  the  equation 

whose  roots  are  —  and  —  • 
n  m 

19.  Show  that  {a-{-b  +  c)x'^—2x{a  +  b)-\-a-{-b  —  c  =  0  has  rational 
roots.     What  are  they  ? 

20.  If  one  root  of  x"^  —px-^q  =  0  is  double  of  the  other,  show  that 

21.  If  m  and  n  are  the  roots  of  x^-{-px-\-q  =  Q,  show  that  p  and  q 
are  the  roots  of  x--\-x{m-\-n  —  m)i)  —  inn{7n'\-n)  =  0. 

22.  Show  that  the  equation  ym.x-\ — -j  ='kax  has  equal  roots  for 
ail  values  of  m. 

23.  Find  the  values  of  k  for  which  the  equation 

x^  +  x{2^k)  +  k-ir^l  =  0 
has  equal  roots. 

24.  Since  a;^  — Sx— 20  =  (a;— 10)(a;  +  2),  for  what  values  of  x  is  the 
expression  a;^  — 8a;— 20  equal  to  zero  ?  For  what  values  is  it  negative  ? 
For  what  values  is  it  positive  ? 

25.  Show  that  it  is  impossible  to  divide  a  line  6  inches  in  length 
into  two  parts  such  that  the  area  of  the  rectangle  contained  by  them 
may  be  10  square  inches. 

26.  For  what  values  of  k  is  4x-2  — a;(/i;+8)4-A;  +  5  a  perfect  square? 
Verify  your  result. 

27.  Find  the  sum  of  the  cubes  of  the  roots  of  x~-{-m,x-Yn  =  0. 

28.  Find  the  sum  of  the  squares  of  the  roots  of 

a;2^a;(l+a)  +  |(l+a  +  a-)  =  0. 

29.  Find  the  sum  and  the  product  of  the  roots  of 

O'        .        b       _      c 
x  —  a      X  —  b       X  ~  c 

30.  If  the  sum  of  the  roots  of  ax^  —  Qx-\-  12a  =  0  equals  their  product, 
find  a  and  verify. 

31.  It  is  evident  that  x  =  a  is  one  root  of  {x--c){x  —  b)  =  {a  —  c){a  —  b). 
Find  the  other  root. 


THEORY  OF  QUADRATICS  343 

32.  If  X---  ox— 3a  and  x^—  lla;  +  3a  have  a  common  factor,  it  must 
be  a  factor  of  their  difference.  Make  use  of  this  to  find  the  value  of  a 
for  which  x~  —  5x—Za  =  0  and  a;^— llx+3a  =  0  will  have  a  common 
root.     Verify  by  finding  the  roots. 

33.  The  absolute  term  in  an  equation  of  tlie  form  a;^+pa;  +  g  =  0  is 
misprinted  18  instead  of  8.  A  student  in  consequence  finds  the  roots 
to  be  3  and  6.     What  were  the  roots  meant  to  be  ? 

34.  If  m  and  n  are  the  roots  of  aa;2  +  ^-'c  +  c  =  0,  show  that  m-\-n 

and 1 —  are  the  roots  of  acx^-\-bx{a-\-c)'\-b"  =  (). 

m      n 

35.  Two  boys  attempt  to  solve  a  quadratic  equation.  After 
reducing  it  to  the  form  x^-{-px-\-q  =  0,  one  of  them  has  a  mistake  only 
in  the  absolute  term  and  finds  the  roots  to  be  1  and  7.  The  other  has 
a  mistake  only  in  the  coetficient  of  x,  and  finds  the  roots  to  be  —  1 
and  —12.     What  were  the  correct  roots  ? 

36.  Express  a;-  +  26.T  +  c  as  the  product  of  two  linear  factors  in  x. 


CHAPTER  XXVI 
SUPPLEMENTARY  CHAPTER 

Additional  Examples  in  Factoring 

244.     Product  of  two  Trinomials.     If  we  multiply 
a— 2b— 3c  by  2a—b-^c 
the  product  may  be  written 

The  first  three  terms  of  the  product,  which  do  not  contain 
the  letter  c,  are  evidently  the  product  of  a— 2b  and  2a— 6. 

The  last  term,  —  Sc^,  is  the  product  of  —3c  and  c. 

If  we  wish  to  factor  the  product  of  two  trinomials,  we  may 
do  so  by  the  method  of  cross  multiplication,  which  we  used  to 
factor  a  trinomial. 

Ex.  1.— Factor  2a^--5ab-^2b^-5ac^bc~3c^. 

First  factor  2a^  —  5ab  +  2b^,  and  then  choose  such  factors  of  —3c'' 
as  will  give  the  remaining  terms  in  the  product  when  the  complete 
multiplication  is  performed : 

2o2-5a?)  +  262_5ac  +  6c-3c2 

a  -2b  -3c 

2a  -   b  +   c. 

If  the  terms  of  the  factors  are  written  under  the  terms  from  which 

they  are  obtained,  it  is  not  difficiilt  to  obtain  by  trial   the  factors  of 

an  expression  of  this  type. 

Ex.  2.— Factor  4a2+362_i2c2-8a&-8ac. 

Arrange  the  expression  thus  : 

4a  2  -  8a&  +  36  2  -  8ac  -  1 2^^ 

2a  -   b  +    2c 

2a  -36  -    6c 

Show  by  multiplication  that  these  factors  are  correct. 

344 


SUPPLEMENTARY  CHAPTER  345 


EXERCISE   158 

Write,  mentally,  the  products  of  : 


1. 

a— 26+c 
a—  b—c 

2. 
5. 

8. 

3x-^+l 

Zx^y-2 

a-6+4 
2a- 6 

3ra-2«,  +  l 
2m— 3?i+4 

3. 

6. 
9. 

3a-46+  c 
2a—  6— 2c 

4. 

3.r+  y-4t 
2x-3«/+3 

2a -36 -5c 
2a +36 

2x—oy-\-  z 
3x—2y—3z 

7. 

a- -2a +3 
a2+3a-2 

Factor  and  verify  : 

10.  a--4a6+462-a+26-12. 

11.  2x'^+xy-Qy^+2xz+Uyz—izK 

12.  2a2+662-3c2+76c-5ca-7a6. 

13.  2x^-lxy-22y^-5x+35y-3. 

14.  6a2+a6-1262-2a+316- 20. 

15.  x^—xz—6z^—2xy-\-Gyz. 

16.*  Divide  the  product  of  Ga^- 5a6+62+lla— 46+3  and 
a+6-2  by  3a2  +  2a6— 6^— 5a+36-2. 

17.  Reduce  to  lowest  terms 

4:p^+2lq'^—lSr^+33qr+&rp-31pq 
4:p^—7pq+3q^--2pr-\-3qr-6r^ 

18.  If  3x-\-2y—5z  is  a  factor  of 

3x'^-\-axy—Qy-  |-6,r2+c</2—  lOs-, 
what  are  the  values  of  a,  6  and  c  ? 

19.  Write  the  expression  x'^-\-xy—2y^—x-\-\0y—\2  in  the  form 
a;2+a;(?/— 1)— (2?/2— 10(/+12).  Solve  the  corresponding  equation  for 
x  and  thus  find  the  factors  of  the  given  expression. 

20.  Solve  .r2-5ax  +  6a-+7.T^17a  + 12=0,  (1)  by  factoring,  (2)  by 
the  general  formula. 


346  ALGEBRA 

'21.     Express,  iii  the  factor  form,  the  L.C.M.  of 

6«,- — 5ab-\-b'^-{-ac—c'^ 
and  6f/--|-a6  — 26^— ac+46c  — 2c2. 

245.     Sum  and  Difference  of  Cubes.     We  have  seen  that 

a^  +  b^  =  (a  +  h){a"-ab  +  b^),        a^-b^  =  (a-b)(a^  +  ab  +  b"), 
{a  +  b)^  =  a^  +  Sa-b  +  3ab"  +  b^,     (a-b)3  =  a^--3a'^b  +  3ab^-b^. 
Similarly,     {a  +  b)^  +  c^  =  {a  +  b  +  c){(a  +  b)^-c{a  +  b)  +  c^\, 
and  (a-6)3-c3  =  (o-6-c)l(a-6)-  +  c(a-6)+c2}. 

Ex.  1.— Factor  a^^b^^c^—Sabc. 

Add  to  a^-{-b^  svifficient  to  make  the  sum  the  cube  of  a-\-b,  that  is 
add  3a26  +  3a62. 

Then  a^  +  b^  +  c^-3abc, 

=  a^  +  b^  +  Sa~b  +  Zab^  +  c^-3aV)-Sab''"-3abc, 

=  {a  +  b)^  +  c^-3ab{a  +  b  +  c), 

=  {a  +  b  +  c)\{a  +  b)^--c{j-\-b)  +  c--3ab\, 

=  {a-\-b-\-c){a"-\-b^-\-c^  —  ab'-bc~ca). 

The  factors  of  this  expression  are  important,  and  the  pupil 
should  endeavour  to  retain  them  in  memory. 

The  expression  is  the  sum  of  the  cubes  of  three  quantities 
diminished  by  three  tim,es  their  product. 

One  factor  is  the  sum  of  the  three  quantities,  and  the  other  is 
the  sum  of  their  squares  diminished  by  the  sum  of  their  products 
taken  two  at  a  time. 

We  should  recognize  expressions  which  are  of  the  same 
form  as  this  type  expression. 

Thus,  a^  +  fe^  — c*  +  3a6c  may  be  written  in  the  form 
a3  +  63_|_(_c)3-3o6(-c), 
and  it  is  now  seen  to  be  the  sum  of  the  cubes  of  a,  b  and  —  c,  diminished 
by  three  times  their  product. 

The  factors  of  n^-\-b^  —  c^-\-Sabc  may  at  once  be  written  down  from 
the  factors  of  the  type  form  by  merely  substituting  —  c  for  c. 

.-.      a3  +  63-c3  +  3a6c  =  (a  +  6-c)(a2  +  62  +  c2-a6  +  6c  +  ca). 
a3_53_c3_3„;,c  =  a»  +  (-6)'  +  (-c)S-3a(-6)(-c), 
=  {a-b-c)(a^+b'^+c^  +  ab-bc+ca). 
8x^  +  27y^-z^+lSxyz  =  {2x)^  +  {3y)^  +  {-zy-3(2x)(3y)i-z), 

=  (2x  +  3y-z){4-x'^  +  dy^  +  z^-6xy  +  3yz  +  2zx). 


SUPPLEMENTARY   CHAPTER  347 

Ex.  2.— Factor  a-^+h^-\-\  —  oah. 

a3-i-63-|_i_3a6==rt3  4,ft3_i_i3_3„(f,  .  1. 

=  {a  +  b+\){a-^b--\-\-ab  —  a~h). 

Ex.  3. — Find  one  factor  of 

(•^•+2/)'+(^+2)'+(^+•^')'-3{.^;+y)(^+:;)(^+.T). 

This  is  of  the  same  form  as  a^  +  6*  —  c'  —  3a6c,  where  a  =  x-{-y,b  =  y-\-z, 
c=z-\-x. 

One  factor  is  x-\-y-ry-r^  +  z-rX  or  2{x-~y-\-z). 

The  other  factor  is  lengthy,  but  is  easily  written  down. 

Ex.  4.— If  a+6+c=0,   show  that  a^+b^+c^=3abc. 

This  is  equivalent  to  showing  that  a^-\-b^-\-c^  —  3abc  =  0. 

Now  this  quantity  will  be  equal  to  zero,  if  one  of  its  factors  is  zero. 
But  a  +  6  +  c  is  already  seen  to  be  a  factor,  and  since  it  is  given  equal 
to  zero, 

•       „3_L?,3a-c3-3a6c  =  0.  ov  a^  +  b^^c^  =  3abc. 

We  have  thus  shown  that  if  the  sum  of  three  quantities  is 
zero,  the  sum  of  their  cubes  is  equal  to  three  times  their  product. 

Prove  this  also  by  substituting  ~b  —  c  for  a. 

Ex.  5.— Show  that 

(a-6)3+(6-c)3+(c-a)3=3(a-6)(&-c)(c-a). 

Here    the    sum    of    a  —  h,    b  —  c,    c~a    is    zero,  and    therefore    the 
result  follows  at  once  from  the  preceding  theorem. 
Similarly,  (a-L2fo-3c)»  +  (6  +  2c-3a)3  +  (c  +  2a-36)3 

=  3(a  +  26-3c)(6  +  2c-3a)(c  +  2a-36), 

since  the  sum  of  a  +  26  — 3c,  6  + 2c  — 3a,  c-f  2a— 36  is  zero. 

SXBRCISE   159 
Factor : 

1.  (a  +  26)^-c3.  2.  a^~{b-cf. 

3.  (a+6)3+8c3.  4.  {a+bf+{c+df. 

5.  (.r-2/)3-(a-6)3.  6.  {2x-y)^+(x-2tj)^. 

7.  (3a-6)3-(a-36)3.  8.  8(3a-6)3-27(2a-3fty' 

9.  a3— 6'+c3+3a6c.  10,  8x^+y^+z^-6xyz. 


348  ALGEBRA 

11.     0.3+6^^  l  +  3r/6.  12.     l+c-3-r/3  +  3crf. 

13.*  8.t-'-/y-'-125z3-30a:y3.  14.     {a+bf+c^+l-3c{a+b). 

What  is  the  product  of  : 

15.  a—b—c  and  a^~\'b--{-c''-\-ab-\-ac—hc. 

16.  2x—y+Sz  and  4:X^+y^+9z^+2xy—6.xz+3yz. 

17.  l—a—bandl+n-+b^-\-a+b—ab. 

18.  2fl-36-4  and  4r/--  +  6a64-962_l26+8a+16. 

What  is  the  quotient  of  : 

19.  l-a^+b'^+Sab  by  l-a+6. 

20.  21m^—n^—l—Qmn  by  3m— %— 1. 

21.  a3+12563-l  +  l5ff6  by  a2+2562+l— 5a6+a+56. 

What  is  one  factor  of  : 

22.  (4«  +  36)3-(rt+26)3. 

23.  (.t2-3x+7)3+8. 

24.  (a2_3ff+2)3-(a2-5a+7)3. 

25.  (a+6)3+(c+rf)3-H-3(«+i)(c+(^). 

26.  Prove  that   the  difference  of   the    cubes    of    ia^-\-a+l    and 
2rt2— 2a4-3  is  divisible  by  the  product  of  2a— 1  and  a  +  2. 

27.  Show  that  a^-\-b^+c^—3abc  is  equal  to 

i(rt+6+c){(a-6)2+(6-c)2+(c-a)2|. 

28.  Write  down  a  quantity  of  the  type  o^  +  b^-\-c^—3abc,  of  which 
3x—2y-\-z  is  a  factor.     What  is  the  other  factor  ? 

29.  If  a+b—c=0,  show  that  a^+b^+3abc=c^. 

30.  If  x=y+z,  show  that  x^=y^+z^+3xyz 

31.  If  a  +  b+c=0,   show   that 

(a  +  2b)^  +  (b  ^  2cP  +  (^  +  2a)3=3(«  +  26)(6+2r)(c+f^,i). 

32.  Show  that 

(x-yr  +  {y-zf-^(z--xf-3{x-y){y-z){z-x)=0. 

33.  Show  that  {a  +  3b-4:cf  +  (b+3c-4a)^+(c+3a-ib)^ 

=  3(a+36-4c)(6+3c-4fl.)(c-t-3a-46). 


SUFPLEMEXTABY   CHAPTER  349 

34.  If  x=a  —  b,  y=a+b,  z=2a,  show  that  x^+y^+3xyz=z^. 

35.  Find  the  Vcalue  of  a^-b'-^+c^+dabc  when  a=-32,  6  = -46,  c=-14. 

36.  Reduce  to  lowest  terms  : 


2a2_5a6^362+ac-3c2  {x-2yf+{2y-z)^+{z-x)^ 

37.  Find  two  factors  of  the  first  degree  of 

{ax+by+azf+(bx+ay+bzf. 

38.  When  x=b+c,  y=c+a,  z=a+b,  prove  that 

^.sj_y3j^z^_^^yz=2{a^+b^+c^-3abc). 

39.  Prove  that  a^+b-+c-—ab—ac—bc  is  unaltered  if  a,  b,  c  be  each 
increased,  or  each  decreased  by  the  same  quantity. 

40.  Solve  (,r-o)3  +  (6-.r)3-i-(f,_6)3=o. 

246.  Grouping  Terms.  We  have  already  seen  (art.  91) 
that  we  can  frequently,  obtain  a  factor  of  an  expression  by  a 
suitable  arrangement  of  the  terras. 

The  following  examples  will  give  further  illustrations  of 
this  method. 

Ex.  1.— Factor  a^b-c)'\-b'\c—a)^c-{a—b). 

Arrange  in  descending  powers  of  a,  and  the  expression 
=  a-{b-c)-a(b^-c-)  +  bc(b-c), 
=  {b  —  c)(a^  —  ab  —  ac-irbc), 
=  (6  —  c)  \a{a  —  6)  —  c{a  —  b)\ , 
=  (b  —  c){a  —  b)(a  —  c). 

When  the  factors  are  written  in  cyclic  order  (art.  140), 

aHb-c)  +  b'-(c-a)+cHa-b)=-{a-b)(b-c)(c-a). 

This  expression  may  also  be  factored  by  writing  it  in  the  equivalent 
form  (a2-62)(6-c)-(a-6)(62-c2). 

In  this  form  a  —  b  and  b  —  c  are  seen  to  be  factors.  Complete  the 
factoring  by  this  method. 

The  expression  a{b'^  —  c");-b(c-  a-) -'r c(a- ~  b-)  differs  only  in  sign 
from  a^b  —  c)+b"[c     a)4-c-(a  — 6), 

.-.      a(b^—c-)  +  b{c~-a^)  \  c{a^-b^)  =  {a-b){b-c)(c-a). 

Also,  ab{a  —  6)  +  bc(b  —  c)  +  ca{c  —  a)  =  —  (a  —  6  )(6  —  c)(c  —  o). 


350  ALGEBRA 

Ex.  2.— Factor  a^b—c)  +  b^{c—a)^c^{a—h). 

The  expression     =a^{b  —  c)  —  a{b^  —  c^)  +  bc(b-  —  C'), 

=  (b  —  c){a^  —  ab-  —  abc  —  ac^  +  b''c-{-bc-). 

Now  arrange  the  second  factor  in  jDowers  of  b,  and  proceed  as  before 
and  obtain  ~{a  —  b){b  —  c)(c  —  a)(a  +  b  +  c). 

Factor  also  by  using  the  second  method  of  Ex.  1,  writing  the 
expression  in  tlie  form  (a^  —  b^)(b  —  c)  —  {a  —  b){b^  —  c^). 

What  are  the  factors  of  a{b^-c^)  +  b{c^-a^)  +  c{a^-b^),  and  of 
ab(a"'-b^)  +  bc(b^-c^)  +  ca(c--a')  ? 

Ex.  3.— Factor  a^b  +  c)^b~{c-^a)+c-{a^b)+2abc. 

Arrange  in  descending  powers  of  a,  and  the  expression 
=  a^{b  +  c)+a{b'''  +  2bc  +  c^)  +  bc(b  +  c), 
=  {b+c){a^+ab  +  ac  +  bc), 
=  {b+c){a+b)(a  +  c)  =  {a  +  b)(b  +  c){c+a). 

Ex.  4.— Factor  (a--b^)x^-+{a"+b-):v+ab. 

Expressions  of  this  kind,  when  written  in  descending  powers  of  x, 
are  easily  factored  by  cross  multiplication  in  the  usual  way. 
{a^-b")x^  +  {a^  +  b^)x  +  ab 
(a  +b  )x  -\-a 

(a   —b  )x  -f-6 

The  factors  are  {a  +  b)x  +  a  and  {a  —  b)x-{-b. 

EXERCISE  160 
Factor  and  verify  1-8  : 

1,  acx^-]-x{ad-\-bc)+bd. 

2.  mpx^-\-xy{qm—2^n)  —  nqy'^. 
i.     x-{a^-b-)+iabx-iu--b-). 

4.  {p~-q-)!r  +  2!j{p-  +  q-)+2J'-r- 

5.  x%a'{-b)+x{a+2b+c)  +  b+c. 

G«     x-{a^—a)+xi2a~—3a+2)+a-—2a. 

7.  a^b-\-c)+a{b~+3bc+c^)+bc{b-^c). 

8.  ab{a-\-b)+bc{b-}-c)-{-ca{c-\-a)-\-3abc. 


SUPPLEMENTARY  CHAPTER  351 

9.*  x^{ij-z)  +  if{z-x)^z^x-y). 

10.  xy{x—y)+yz{y—z)+zx{z-x). 

11.  x{y^—z-')+y{z^~-x'-)+z{x'~-y% 

12.  a{h^-c^)  +  h{c'^-a^)+c{a^-h^). 

13.  a2(64— c4)+62(c«-a*)+c2(a*-fc4). 

14.  Divide 

a»(6-c)+63(c-fl,)+c='(rt-6)  by  a2(^_c)4_ft2(c_«)^c-(a-6). 

Solve  and  verify  : 

15.  ahx'^—x{ad-\-hc)-{-cd=Q. 

16.  {a-—b-)x^—4:abx—a-—b'^. 

17.  .r2(a-&)  +  a2(6_.r)+62(,i;_„)=0. 

18.  «6x-2— a;(a'-+62)_j_a2_^,2^0. 

19.  {a--ah)x^-ir{a-+h-)x=ah+h\ 

20.  Find  a  common  factor  of 

ahx--\-x{a-—2ah—h-)—u--{-h^  and  a"x-—a^x—ab—b^. 

247.  The  Factor  Theorem.  We  have  already  seen  that 
any  expression  is  divisible  by  x—a,  if  the  expression  vanishes 
when  we  substitute  a  for  x  (art.  101). 

Any  expression  whose  value  depends  on  the  value  of  x  is 
called  a  function  of  x  (art.  114). 

Any  function  of  x  may  be  conveniently  represented  by  the 
symbol /(a;),  which  is  read  "  function  a:." 

The  factor  theorem  might  be  stated  thus  : 

/(■r)  is  divisible  by  x—a  if  /'{a)=0. 

Thus,  if  f{x)  =  x^-7x-+llx-2, 

/(2)  =  8-28+22-2  =  0 
.".     x*  — 7x-+  llx  — 2  is  divisible  by  x—2. 

If  f{x)  =  x^-4x^a  +  5xa^--\-\0a^, 

then  /(-a)=-a='-4a'-,5a3+10a3  =  0, 

x^  —  4x'^a-\-5xa^  +  l0a^  is  divisible  by  x-{-a. 


352  ALGEBRA 

248.  Factors  of  x"±a**.    We  have  already  seen  that 

x^  —  a^  =  {x  —  a){x-\-a), 

x^  —  a^  =  (x  —  a){x^-\-xa  +  a-), 

x*-a*={x-^-a^){x^  +  a-)  =  (x-'a)(x+a)(x^+a^). 

Here  we  see  that  x—a  is  a  factor  of  each. 
Is  x  —  a  a,  factor  of  x^  —  a^  ? 
When  we  substitute  a  for  x, 

x^  —  a^  =  a^  —  a-'  —  Q, 
x  —  a  is  a  factor  of  x^  —  a^. 

( 1 )  Is  a;  —  a  a  factor  of  x"  —  a"  ? 
When  we  substitute  a  for  x, 

X"  —  a"  =  a« — a" = 0- 
x"  —  a"  is  divisible  by  x  —  a. 

(2)  Is  .r  +  a  a  factor  of  x"  —  a"  ? 
When  we  substitute  —a  for  x, 

a;"— a"  =  (  — a)"— a". 
Now  {  —  a)"  — a"  will  be  equal  to  zero  only  when  {  —  a)"— a",  and  this 
is  true  only  when   n  is  even, 

x"  —  a"  is  divisible  by  a;4-«  when  n  is  even. 

Thus,  rc^  — a^,  a;*  — a*,  a;^  — a®,  etc.,  are  divisible  by  x-{-a,  but  x^  —  a^, 
x^  —  a^,  etc.,  are  not  divisible  by  x  +  a. 

(3)  Is  x  +  a  a  factor  of  x"  +  a"  ? 

Examine  this,  as  in  the  preceding,  and  show  it  is  a  factor  only  when 
n  is  odd. 

(4)  Is  a;  — a  a  factor  of  x"-\-a"  ? 

We  thus  conclude  that,  when  n  is  a  positive  integer, 

(1)  x"— a"  is  always  divisible  by  x—a. 

(2)  x°— a"  is  divisible  6?/  x+a  when  n  is  even. 

(3)  x"-Ka°  is  divisible  by  x+a  when  n  is  odd. 

(4)  x°+a"  is  never  divisible  by  x—a. 

249.  Quotient  on  dividing  sc"±a"  by  x±a. 

(1)         =  x+a.  =x^  +  x^a-\-xa"  +  a''. 

x—a  x—a 

=x--]-xa-{-a^.         =x*  +  a;*a+a--«^+a;a-'4-a*. 


SUPPLEMENTARY  CHAPTER  353 

Verify  these  results  by  division  or  multiplication. 
Notice  that  the  signs  are  all  positive,  and  that  the  powers  of  x  are 
descending  and  those  of  a  are  ascending. 

Similarly, =x^-j-x^a-\-x^a'-^xhi^-i-x^a*-\-xa^-{~u''', 

and  =x'^~'^4-x"~^a-{-x'^"^a^-\-  .  .  .  -}-xa"~'^-\-a"~'^. 

x  —  a 

(2) =  x  —  a.  =x^—x^a+xa^  —  a^. 

x-\-a  x-\-a 

Verify  and  note  the  peculiarity  in  the  signs. 

x-{-a    ""      "'       x-\-a 

(3)        5!±^'==a;2_a;a_,_as.         ^^^'^-=x^-x^a+xht''-xa^  +  a^. 
x  +  a  x-\-a 

x'  +  a'  x^"+^  +  a^""^'- 

Write  down  the  value  of        ,  —  and  of    ; • 

x-\-a  x-\-a 


EXERCISE   161 

1.  If /(a;)=a;3-8.r2+19.r-12,  find  the  values  of 

/(I),  /(2),  /(3),  /(4),  /(5). 
What  are  the  factors  of  .r^— 8.r-+19.T— 12  ? 

2.  If  f{x)=x*--2x^~x"-\-2x,  find  the  values  of 

/(2),  /(I),  /(O),  /(-I), /(-2). 
What  are  the  factors  oif{x)  in  this  case  ? 

3.  Prove  that  x^*— ^^^  is  divisible  by  x—y  and  x-{-7/. 

4.  Prove  that  x^^—  1  is  divisible  bj^  .r— 1,  x+1,  x^-\-l,  x^-\-l. 

5.  Prove  that  x'^+t/^^  is  divisible  by  .r+.?/  and  that  .r^-|-32  is 
divisible  by  x+2. 

Write  down  the  quotients  in  the  following  divisions  : 

.r3+y3  ^      «lr^''  8      "''~^^^  0       ^'-1 

x-j-y  '      a—b  '      a-}-b  '      x—l 

^y^xM:32  ^^      x'-8l  ^2      ^'°-«'         13     («+^)lzl. 

x+2  '       x+S   '  '      x"—a  '  '     («+&)+!  ' 

A  A 


354  ALGEBRA 

14.     State  one  factor  of  : 

x^-b"",  (i'-^h\  a;3-64,  m3+— ,,  {x+yf-l. 

What  is  the  product  of  : 
16,     «"+«-+«+ 1  and  a— 1. 

16.  m*— m^+m^— m+1  and  m+1. 

17.  a^+a'^b^+a^b^+h^  and  a^-b^. 

18.  Prove  that  a;64-3.r*+4a;2+224  is  divisible  by  X'+l. 

19.  Show  that  x-^y,  x'^-\-y',  x^+y^,  x•*+?/^  x^+y'^  and  x-^^+i/ia 
are  factors  of  x^^—y-^. 

20.  If  a;— a  is  a  factor  of  x^-]-px-\-q,  find  the  relation  between  a,  p 
and  q.  • 

21.  If  f{x)=m,x^-\-nx-{-r,  find  /(a)  and  show  that  f{x)—f{a)  is 
divisible  by  .r— a. 

22.  If  .r— 1  is  a  factor  of  x''^—k\c"-'rlOkx—10,  find  the  values  of 
k  and  verify. 

23.  Write  down  the  quotient  when 

1        1 

(1)  x—a  is  divided  by  x^^—a^. 

1       1 

(2)  x-\-a  is  divided  by  x^-{-a^. 

1       1, 

(3)  x—a  is  divided  by  x"—a^. 

1  X 

(4)  x'+rt  is  divided  by  a;"' +«■"•. 

250.  Symmetrical  Expressions.  An  expression  is  said  to  be 
symmetrical  with  respect  to  any  two  letters  if  it  is  unaltered 
when  those  two  letters  are  interchanged. 

Thus,  x-{-y  and  x^-{-y^  are  symmetrical  with  respect  to  x  and  y, 
but  x^-\-xy  is  not  symmetrical. 

Similarly,  a  +  6  +  c  and  ab-{-bc-\-ca  are  symmetrical  with  respect 
to  a  and  b,  b  and  c,  c  and  a,  for  if  any  two  be  interchanged  the 
expressions  remain  unaltered. 

251.  Cyclic  Symmetry.  An  expression  is  said  to  be 
symmetrical  with  respect  to  the  letters  a,  h  and  c,  if  it  is 
unaltered  when  a  is  changed  to  h,  b  to  e  and  c  to  a,  that  is, 
when  the  letters  are  taken  in  cyclic  order. 


SUPPLEMENTARY   CHAPTER  355 

Thus,  a^  +  6-  +  c'  — ra6  — 6c  — ca  is  symmetrical  with  respect  to 
a,  b  and  c,  for  when  the  letters  are  changed  in  cyclic  order  tiie  result  is 

b^-\-c^-{-a^  —  bc  —  ca  —  ab, 

wliich  is  equal  to  the. given  expression. 

The  expression  a'  +  6*  +  c^  — 3a6cd  is  symmetrical  with  respect  to 
a,  b  and  c,  but  not  with  respect  to  a,  b,  c  and  d. 

The  only  expression  of  the  first  degree  which  is  symmetrical 
with  respect  to  a,  b  and  c  is  a-\-h-\-c  or  some  multiple  of  it  as 
A'(a+6+c). 

There  are  two  expressions  of  the  second  degree,  a^-\-b'^-\-c^ 
and  ab-\-bc-{-ca,  and  the  sum  of  any  multiples  of  these,  such  as 

A;(a2+62_|-c2)4-Z(a6+6c+ca), 

which  are  symmetrical  with  respect  to  a,  6,  c. 

252.  Symmetry  applied  to  Factoring.  The  factor  theorem 
may  be  applied  to  the  factoring  of  many  symmetrical 
expressions. 

Ex.  1.— Factor  a{b--c^)+b{c^—a-)  +  cia^-b^). 
If  we  put  a  =  h,  the  expression  equals  zero, 
a  —6  is  a  factor. 

Since  the  expression  is  symmetrical  and  a  —  b  is  shown  to  be  a  factor, 
it  follows  that  b  —  c  and  c  —  a  must  be  factors. 

We  have  thus  found  three  factors  each  of  the  first  degree.  But  the 
given  expression  is  of  the  third  degree,  and,  therefore,  there  cannot 
be  another  literal  factor.     There  may  be  a  numerical  factor. 

Suppose  A;  is  a  numerical  factor, 

.-.      a(62-c2)  +  6(c2-o2)  +  c{a2_62)  =  A;(a-6)(6-c)(c-a). 

Since  this  relation  is  true  for  all  values  of  a,  b,  c, 

let  a=l,  6  =  2,  c  =  0, 

then  1(4-0) +  2(0- l)  +  0  =  A;(l-2)(2-0)(()-l), 

.-.      2  =  2Jfc,  orfc=l, 

.-.      a(62-c2)J-6(c2-a2)  +  c(o2-62)  =  (a-6)(6-c)(c-a). 

In  finding  the  value  of  /.%  any  values  of  a,  6,  c  may  be  used  provided 
they  do  not  make  both  sides  of  the  identity  vanish  on  substitution. 

A  A  2 


356  ALGEBRA 

Ex.  2. — Factor 

(a+&  +  c)3+(a-6-c)3+(&-c-a)H(c-a-/>)3. 

If  we  put  0  =  0,  the  expression  vanishes, 

.■.    a  must  be  a  factor,  and,  therefore,  h  and  c. 

Complete  the  solution  as  before,  and  show  that  the  expression  equals 
24a6c. 

Ex.  3.— Factor   a\h-c)^h\c-a)-]-c\a-h). 

As  in  Ex.  1,  show  that  a  —  b,  b  —  c,  c  —  a  are  factors. 
Since  the  expression  is  of  the  fourth  degree  it  must  have  another 
factor  of  the  first  degree. 

The  remaining  factor  must  be  of  the  form  A;(a  +  6  +  c). 

.-.     a^{b-c)  +  b^{c-a)  +  c^(a-b)  =  k{a'-h){b-c){c-a){a  +  b-]-c). 

Substitute  numerical  \-akies  for  a,  b  and  c  and  show  that  the  factors  aro 

-{a-b){b-c){c-a){a  +  b  +  c). 

Ex.  4. — Simplify 

(a-6-2c)2+(6-c-2a)'-+(c-a-26)24-(a+fe+c)2. 

This  expression  is  symmetrical  with  respect  to  a,  b  and  c  and  is 
of  the  second  degree. 

In  the  simplified  result  there  can  be  only  two  kinds  of  terms,  squares 
like  a-  and  products  like  ab. 

The  coefficient  of  a-  in  the  resvilt  is  1  +  4+ 1  + 1  or  7, 

one  part  of  tlie  result  is  7(a-  +  ?;2  +  c^). 
The  coefficient  of  ab  is  —2  —  4  +  4  +  2  =  0, 
.".     the   complete   result   is    l{a^-\-b^-\-c^). 
Check  by  letting  a  =  b  =  c=\. 

Ex.  5. — Simplify 

(a+&)(a+6-2c)  +  (6+c)(6+c-2a)  +  (c+a)(c  +  a-26). 
The  coefficient  of  a^  in  the  result  is  1  + 1  or  2, 

one  part  of  the  result  is  2{a'^-{'b^^c^). 
The  coefficient  of  ah  is  2-2-2  or  -2, 

the  other  part  of  the  result  is  —  2(a6  l-6c  +  ca), 
/.     the  complete  result  is  2(a*  +  6-  +  c^  —  a6  -6c  — ca). 


SUPPLEMENTARY  CHAPTER  357 

EXERCISE  162  (1-12.  Oral) 
With  respect  to  what  letters  are  these  symmetrical : 
1.     a-\-b.  2.     a+c— 6.  3,     x--\-ij^-\-xy. 

4.     ah^hc^ca.  5.     a^-j-ft^+c^— 3a6c. 

G.     x^-'-^y'+x-y.  7.     Z{p'^+q"+r-)-2{pq-\-qr+rp). 

8.  What  is  the  simplest  expression  of  the  first  degree  whicli  is 
symmetrical  with  respect  to  x  and  ?/  ?     a,  b  and  c  ?     a,  b,  c  and  rf  'i 

9.  W^hat  expression  similar  to  «--(-fc-+3a6  is  symmetrical  with 
respect  to  a,  b  and  c  ? 

10.  SimpUfy 

(a+6)2+(6+c)2+(c+r02     and     {a-b)-+{b-c)-+{c-a)~. 

11.  If  a -|- 6  is  a  factor  of  any  expression,  synuuetrical  with  respect 
to  a,  b  and  c,  what  other  factors  must  it  have  ? 

12.  When  {a-irb)^-\-(b-{-c)^-\-{c-\-a)^  is  simplified,  the  coefficient 
of  «■'  is  2,  of  a-h  is  3  and  of  abc  is  0.  What  must  the  simpUfied 
form   be  ? 

Simplify  : 

13.*  {a-b+c)-+{b-c+af+{c-(i  +  b)\ 

14.  (a+b){a+b—c)  +  {b+c){h+c-a)  +  {c+a)ic+a—b). 

15.  (a;— ?/)(i)a;-|-jDi/— z)  +  (y— 2)(;)?/+pz— z)+(z— x)(ps+j9.-r— t/). 

16.  (ff-fe)3+(6-c)3-f(c-a)='. 
Factor  : 

17.  .t2(?/— z)+?/2(z— a;)-|-22(a;_2/j. 

18.  xy{x—y)+yz{y-z)+zx{z—x). 

19.  a2(5^c)_^j2(c^„)^c2(a+6)  +  2a6c. 

20.  («,+6+c)3-(fr-f6-c)3-(6+c-a)3-(c+a-6)'- 

21.  (a;-2/)3  +  (y-z)3  +  (2_;,.)3. 

22.  a(6+c)2-{-6(c+a)2+c(a+fe)--4a6c. 

23.  ab{a^--b^-)  +  bc(b^-c^)+caic^-a^). 

24.  rt2(ft4-c*)  +  62(c4_(j4)^c2(a4_64). 


358  ALGEBRA 


Simplify  : 


25. 


^iy+z)        ,       y{z+x)       _^     z{x+y) 
{x-y){z-x)^  {y-z)(x-y)  '   {z-x){y-z) 


26.  ^^_----  + y- + -^— . 

{x-y){x-z)       {y-z)(y-x)      {z-x){z-y) 
ah  he  ca 


{c-a){c-h)   '   (a~h){a-c)^  (b-c){b-a) 
28.     ^ ""- +  -^ + 


29. 


bc{a  —  b){c—a)      ca(h—c){a^b)      ab(c—a){b—c) 
b^—ac  c^—ba  a^—cb 


(a—b){b—c)      (6— c)(c— a)      (c~a){a—bj 


bc{b-\-c)  ca{c-\-a)       ^      ab{a-\-b) 


31. 


32. 


a— 6)(a— c)      {b—c)(b—a)      (c—a)(c—b) 

x^  y^  z^ 

{x^){z-x)+(y-z)ix-y)  +  {z-x){y'^z) ' 

(a-6)3+(6-c)3+(c-a)3 
a(62_c2)+6(c2_a2)+c(a.2-fo2) ' 


33.  Simplify  {a+b+c)^-{b+c)^-{c+af-(a  +  hf+a^+b^+c^ 
being  given  that  a  is  a  factor  of  it. 

34.  Show  that  a—b  is  a  factor  of 

a"(b~c)+b"{c—a)+c"{a—b). 
What  may  be  inferred  regarding  otlier  factoi-s  ? 

35.  An  expression  is  symmetrical  in  x,  y  and  z  and  each  term  is  of 
two  dimensions.  When  x=7j=^z=l,  the  expression  equals  ]  5,  and  when 
x—1,  y=2,  z=3,  it  equals  64.     Find  the  expression. 

3G.  Point  out  wherein  it  is  obviously  impossible  for  the  following 
statements  to  be  true  : 

(1)  (a-+b^+c^){a+b+c)=a'^+b''i+a%b+c)+b^c+a). 

(2)  n^+b'^+c-^—:iabc=^{a+b+c){a^+b-+c^-—3ab). 

(3)  {a—b){b—c){c—n)  =  ah--{-b'^c-\-ca^—a^-  —  bc~—ba". 


SUPPLEMENTARY  CHAPTER  359 

253.  Identities.  We  have  already  had  many  examples  of 
algebraic  expressions  which  are  identically  equal,  that  is, 
which  are  equal  for  all  values  of  the  letters  involved. 

Thus,  {x  +  y){x  —  y)  —  x"  —  y~, 

{x-\-y  -\-z)'^=x'^  -\-y^  -{-z-  -{-'2xy  -\-2xz-\-2yz, 

a^  +  6*4-c^  — 3a6c  =  (a  +  6-|-c)(a-  +  6-  +  0-  — a5  — 6c  — ca). 

Any  of  these  may  be  shown  to  be  identities  by  performing 
the  operations  necessary  to  remove  the  brackets  on  one  side, 
when  the  result  is  the  same  as  the  other  side. 

Ex.— Show  that  (a+6  +  c)=^ 

=a3+&3^c=^— 3a6c+3(a+6+c)(a6-|-&c+ca). 

Here  the  cube  of  a-\-h-\-c  may  be  fomid  by  multiiaHcation  or  by  any 
other  method. 

The  brackets  are  then  removed  from  the  right  and  the  terms 
collected. 

The  two  sides  are  now  the  same,  which  shows  that  the  given  statement 
is  an  identity. 

We  might  also  hav^e  changed  the  second  side  into  the  first  by 
factoring,  thus  : 

(a3  +  63  +  c3-3a6c)  +  3(a  +  6  +  c)(a6  +  6c+ca), 
=  (a+6  +  c){a2-|-62-fc2-a6-6c-ca)  +  3(a  +  6  +  c)(a6  +  6c  +  ca), 
=  {o  +  6  +  c)(a2  +  62  +  c2  +  2a6  +  2ac  +  26c), 
=  (a  +  6  +  c)^,  which  jaroves  the  proposition, 

254.  When  two  expressions  are  to  be  shown  equal,  the 
result  may  frequently  be  obtained  by  showing  that  their 
difference  is  zero. 

The  difference  may  be  zero, 

(1)  because  all  of  the  terms  cancel,  or 

(2)  because  it  has  a  factor  which  is  equal  to  zero,  identically, 
or  which  is  given  equal  to  zero. 


360  ALGEBRA 

Ex.  1. — Prove 

(a-6)3+(6-c)3+(c-a)3=3(a-6)(6-c)(c-a). 

Here  we  may  prove  that 

(a"6)3  +  (6-c)=»+(c-a)3-3(a-6)(6-c)(c-a)  =  0, 

(1)  by  removing  the  brackets  when  all  the  terms  cancel, 

(2)  by  observing  that  (a  — fe)  +  (6  — c)  +  (c  — a)  is  a  factor  of  the 
expression  and  this  factor  is  identically  equal  to  zero  (art.  245). 

Ex.  2. — If  a-\-b^c,  show  that  a^-\-bc=b^-\-ca. 

Here,  as  in  the  preceding,  we  may  show  that  a^-\-bc  —  b^~ca  =  0, 
by  showing  that  a-\-b  —  c  is  a  factor  of  it  and  this  factor  is  given  equal 
to  zero,  or  by  substituting  c  =  a-\-b  in  each  side  or  in  the  difference. 

Solve  this  problem  both  waJ^s. 

Ex.  3.— If  a+6+c=0,  show  that 

(a+6)(&+c)(c+a)+a&c=0. 

For  a-\-b  substitute  —  c,  for  6  +  c  substitute  —a,  and  for  c-\-a 
substitute  ~b  and 

(a  +  ft)(6  +  c)(c  +  o)  +  a6c  =  (-c)(-a)(-6'+a6c  =  0. 
Ex.  4. — If  2s=^a-\-h-^c,  prove  that 

s2+(s-a,)2+(5-6)2+(s-C)2  =  a2  +  62^c2. 

When  the  first  side  is  simplified  it 

=  4s2_2s(a  +  6  +  c)  +  a2  +  62  +  c2, 

=  4s2-2s(2s)  +  a2  +  62  +  c2, 

=  a2  +  'j"  +  c^,  which  was  required. 

Of  course,  this  could  have  been  proven  by  substituting  the  value  of 
s  at  once.     It  is  visually  easier,  however,  to  substitute  in  the  last  step. 

EXERCISE    163 
Prove  the  following  identities  : 

1.  a(?>+c)2+6(c+a)2+c(a+6)2— 4a6c=(a+6)(6+c)(c+a). 

2.  (.T+^)HxHy*=2(.r2+.Ty+y2)2. 

3.  (a  +  ft)3+(a-6)='  +  6a(«+i)(f/-?;)=8a='. 


SUPPLEMENTARY  CHAPTER  361 

4.  2{a^+b^+c^-3abc)={a+b+c)\{a-b)-+{h-cy-  +  (c-n)^l. 

5.  a{b-c)^+b{c-a)^+c{a-b)^={a-b){b-c){c-a)(a+b+c). 

If  a+6+c=0,  show  that : 

6.  (3a-26+4c)2-(2a-36+3c)2=0. 

7.  a^-\-b^—c^-]-2ab=0  and  c^—ab—b'^—ac. 

8.  (a+6)(6+c)+(6+c)(c+a)+(c+a)(a+6)=a6+6c+ca. 

9.  aH6*+c*=2a262^262c2+2c2cf2. 

10.  (3rt-6)='  +  (36-c)3+(3c-«)3  =  3(3a~6)(36-r)(3c-a). 

11.  a{b^-+bc+c^-)+b{c^+ca+a-)+c{a-+ab+b-)=0. 

12.  If  a+6=l,  prove  that  {a-—b-)'=a^^b^—ab. 

13.  If  a;+y=22,  prove  that  -^—  +  ^-  =  2. 

a; — 2      y — z 

14.  If  a  =  yiZl,  b  =  ^-^^,  c  =  ^^:Z^,  show  that  a+b+c+ahc=0. 

X  y  z 

._      „  1   ,      1  2  .,112 

lo.     If  -  H — ,,  prove  that  --!--  =  -. 

a      a — c      a—b  a      b      c 

16.  If  .T  +  A  =  y,  show  that  x^  +  -,  =  y-~2  ;  o;^  4-  —  =  ?/3  —  3)/  ; 

X      '  a;^  x^ 

^*  +  -,  =  y'-  4/y2  +  2. 

X* 

If  2s=a-i-6-|-c,  show  that : 

17.  s(.s— rf)+(-s— '')(-5— r)=&'^- 

18.  a(.s-a)+6(s-6)+c(.s-c)+2.s2=2(«6+6c+ccf.). 

19.  {.s-ay-+{s-b)^+{s-cy^+2{s-a){s-b)+2{s-b){s-c) 

+  2(s— c)(s— a)=s- 

20.  (2rt.s+ic)(2ft.s+m)(2c.s+a6)  =  (a+6)2(6+c)2(c-|-«)2. 

5— a      5—6      s—c      s      s(s— «)(s— 6)(.s— c) 

22.  16s{s—a){s-b){s-c)  =  2b^c^2c^a^+2rJa--a*~b*—c*. 

23.  If  6  +  -  =  1.  c  +  -  =  1,  prove  a  +  -  =  1  3^nd  afic  =  —1. 

c  a  0 


362  ALGEBRA 

24.*  If  a  +  -  =  3,  find  the  value  of  a^  +  1 . 
a  o* 

25.  If  a=x{b-{-c),  b=y{c-\-a),  c=z{a-\-b),  show  that 

xy-{-yz-\-zx-\-2xyz—  1. 

26.  If  x-\-y=a  and  xy=b-,  find  the  values  of  .-r'4-^"  and  x^-\-y^ 
in  terms  of  a  and  b. 

27.  Eliminate    x    and    y    from    the    equations    x-{-y=a,    xy=b^, 
x^-'ry"=c'^. 

28.  Eliminate  x  and  //  from  ;c+2/=ct,  .t^+2/^=^^  a;^4-?/^— c^. 

29.  If  a:=o+&— c,  y^b^c—a,  z=c-\-a — b,  show  that 

x^-\-y^-{-z^—3xyz=4:{a^-{-b^-\-c^—3abc). 


EXERCISE  164  (Review  of  Chapter  XXVI) 

1.  Show  that  x^-\-y^-\-z^  —  3xyz  is  divisible  hy  x+y  +  z,  and  hence 
show  that  (6-c)»  +  (c-a)»  +  (a-fe)3  =  3(a-6)(6-c)(c-a). 

2.  Prove  that 

/„.,„         „\/l         1         1\       fb   .   c\fc    .   a\fa   ,    h 
(  o, 


V-  +  ^^  +  ^;)(  ^  +  6^ +  ^j-(e  +  6JU  +  cJU +  «;  =  '• 

3.  If  a  +  6  +  c  +  d  =  0,  prove  that 

(a  +  6)(a  +  c)(a  +  f/)  =  (6  +  c)(6  +  c/)(6  +  o). 

4.  Prove  that  (a  — 6)"  +  (6  — c)"4-{c  — o)"  is  divisible  by 

(a-6)(;)-c)(c-o). 
when  n  is  an  odd  integer. 

5.  If  n  is  a  positive  integer  prove  that  12"— 1  is  divisible  by  11, 

232"  n+1  by  24,  72"- I  by  48. 

6.*  Write  down  a  quantity  of  the  same  type  as  x^-\-y^-\-z^  —  Zxyz 
of  which  }fX-\-\y  —  \z  is  a  factor. 

7 .  Show  that  a,  a  —  x  and  a  —  2x  are  factors  of 

{a-b){a-b-x){a-{-2b-2x)-{-b(b-x){Za-2b-2x). 

8.  Show    that   (cc  +  y)"  — x"  — y"   is   always   divisible   by   xy{x-{-y), 
when  n  is  an  odd  integer. 

9.  If  (y  — a)(l— a)  =  (2/  — fc)(l  —  ?j)=.t,  find  x  in  terms  of  a  and  ft  only. 


SUPPLEMENTARY   CHAPTER  363 

10.  If  .'c  +  2/  +  2  =  0,  prove  that 

(1)  x'^^xy  +  y-  =  y^  +  yz-^z'^  =  z'^-\-zx-{-x-. 

(2)  {x-\-y-zY^{y  +  z-xY^{z+x-ij)^-^24.xyz^0. 

11.  Simplify     .-,„—t^7-—-.+  ~Ji^—7^7u-.^-„^  + 


bc(a  —  b){a  —  c)       ca{b  —  c){b  —  a)      ab(c  —  a)(c  —  b) 
12.     Solve  (.r-o)3  +  (.r-6)3  +  (a;-c)3  =  3(.'c-a)(.-c-fe)(.r-c). 
1.3.     Show  that  {a  +  b)^  —  a^  —  b^  =  5ab{a  +  b){a-  +  ab  +  h^). 

14.  If  2s  =  a+b-\-c,  show  that 

(1)  s(s  —  b)-\-{s  —  a)(s  —  c)=ac. 

(2)  s'^  +  {s—a){s—b)-{-(s—b){s-c)  +  {s  —  c)(s~a)  =  ab  +  bc  +  ca 

(3)  (s-a)3  +  (s-c)3  +  36(s-a)(.s-c)  =  63. 

15.  Prove    that   a"(b^-c-)  +  b"{c~-a^)  +  c"{a'^-b^)    is    divisible    by 
(a  — 6)(6  — c)(c  — a)  and  find  the  quotient  when  n  =  3. 

16.  Simplify     —■ j-- :  +  rn —r r  + — rr  • 

^     -^     a{a  —  b)(a  —  c)       b(b  —  c)(b  —  a)       c{c  —  a){c  —  6) 

17.  If  x  =  a~  —  bc,   y  =  b^  —  ca,   z  =  c^  —  ab,  prove  that 

ax  +  by-{-cz  =  {a  +  b  +  c)(x-\-y  +  z). 

„       ^.       ,..       a3(b  -  c)  +  b^c  -  g)  +  c»(a.  -  b) 

18.  Snnphfy     — -y ,„   ,    , ,..   ,    ,     — rrj-  • 

19.  If  ab-\-bc-{-ca  =  0,  show  that 

(1)  (a  +  6  +  c)2  =  o2  +  62  +  c2. 

(2)  {a  +  b  +  c)^  =  a^  +  b^  +  c^-3abc. 

(3)  (a  +  6  +  c)*  =  a4  +  64-l-c<-4a6c(a  +  6  +  c). 

20.  Show  that  cc"+i  — a;"  — a;+ 1  is  divisible  by  (a;— 1)-,  when  n  is  a 
positive  integer. 

21.  Write  down  the  quotient  on  dividing 

a;*  — a*  by  .r  — o,  .-r^-f  1  by  x~-\- 1,  a^  —  32  by  a  — 2. 

22 .  Factor  a;^  - 1  —  3(x^  -  1 )  +  4(a;2  -  1 ). 

23.  Simplify r^, r  +  two  similar  fractions. 

^     -^     (a  —  b)(a  —  c) 

24.  Show  that  x{y^  —  z^)-{-y{z^  —  x^)-'fz{x^  —  y^)  is  not  altered  when 
X  is  changed  to  x  +  a,  y  to  y  +  a,  z  to  z  +  a. 

25.  If  .'c^  =  a;+l,  show   that  x-'  =  5x-\-3. 


364  ALGEBRA 

26.  Find  two  linear  factors  of 

{ax+by  +  {bx  +  cy  +  {cx  +  ay~3(ax  +  b}{bx  +  c)(cx  +  a). 

27.  If  x^  +  y^  =  z^,  show  that  {x^  +  y^-z^)^  +  27x^y^z^  =  0. 

28.  If  a+b+c=0,  prove  that 

a3  +  63  +  c3  +  3(o  +  fe)(6  +  c)(c  +  a)  =  0. 

29.  A  homogeneous  expression  of  two  dimensions  is  symmetrical 
in  X,  y,  z.  Its  vahie  is  42  when  x  =  y  =  z=2  and  is  16  when  x=l,  y=2, 
z  =  0.     Find  it. 

30.  Eliminate  x  and  y  from  x-\-y  —  a,  xy  —  b,  x^-\-y-  =  c. 

31.  li  x-\-y  =  ^  &nd  x^ -\-y'^  =  b,  find  the  values  of  a;^ +  2/^  and  x*  +  ?/*. 

32.  If  a  +  6  +  c=10anda6  +  6c+ca  =  31,find  the  values  of  a^  +  b^+c* 
and  a^  +  fc^+c^  — 3a6c. 


.«.  *-•'•    •  *-*•" 


ANSWERS 

TO 

HIGH    SCHOOL   ALGEBRA 


ANSWERS 


No  answers  are  given  to  elementary  examples,  oral  examples  or 
examples  which  ma_y  be  verified  or  checked  without  difficulty.  In  each 
exercise  the  number  of  the  first  example  to  which  the  answer  is  given 
is  marked  with  a  star. 


Page  8 

15.  108,  38,  10,  32,  60.     16.  3,  14,  39,  0.       17.  9,  29,  18.       19.  2. 
21.  25.  22.  44,7.  23.  154,616. 

Page  10 

9.  47.  10.  70.         12.  10a;+10.  13.  xft.E.  14.  15a;. 

16.  2a3+2a2-j-3a.        17.  11   ox. 

Page  12 
24.  37.        25.  17.        26.  34         27.   1  28.   1.         29.  h. 


Page  14 


30.    ('-  +  —^  hours.       31.  (5x+20?/-7z)  cents.        32.    ^±?^  cents. 
33.  5a;+102/+502.         34.  1234,  4019.         35.  -3,  -02,  2,  15,  05,  -03. 

36.    1  +  - .  37.  20.  20.  38.  24.  39.  2k 

X      y 

367 


368 


7.  7a+6b—15c. 
10.  4a+46+4c. 
13.  8a-4&. 
16.  2x+2y—z. 


ALGEBRA 

Page  38 

8.   10.r2- l4,*+9. 
11.  4.a-8b+3c-5d. 
14.  6a2+862-bc2. 


9.   lOa-76. 

12.   8a,-— (j^+52. 

15.  3a+36+3c+3d. 


18.  0.         19.  15X+5//. 

23.  2a-^6-ic, 


Page  40 
20.  Abe.        21.   lOa^+ab. 


22.  4z/2. 


13.  -3a;^.        14.  0. 
18.  0.        19.  X. 


Page  41 
15.  4p-.        16.  10m— 3n. 


17.  6.V-4Z. 


Page  44 
15.  3a-26.  16.  2«+5c.  17.   -3x^.  19.  4a2_4a_l5. 

20.  2b.  21.   13r— 2;.  22.  36— 5c— 2a.  23.  x^+Qx~5. 

24.  2a2+a-12.  25.  a+b+c.  26.  2a;-3. 

27.  10a;3+2a;2+8a;+2. 


Page  46 

11.  3x+2^.  12.   — 2rt  — 36.  13.  a+b.  14.  6— a. 

15.  3a+6-3c.  16.  3x~-3.  17.  7.  18.  11 

22.  G,  4,  4,  6,  10.         23.  4rt+46-15c,  4a-46+4c-4rf,  y,  0. 


ANSWERS  369 

Page  47 

1.  a.               2.  4.r-f6.                3.  5n.               4.  5?.  5.  46,8a. 

6.  2a+26+6c.          7.   -.r+y-52.          8.  3a-26-2c.  9.  14,  G. 

10.   -7.                11.  y-x.                12.  31.                13.  «-2c,  2c-a. 

14.  9m-2«.              15.  4.r-9.              16.  a;- 12.  17.  26-4c. 

18.  56— 5a,  a+36— 4c,  7a— 6— 6c.  19.  ■2x.  20.  3x-6. 
21.  a-lb+hc.  22.  52-3a.-.  25.  20.  26.  l+2x.  27.  7. 
28.  7n+4a;— 2m.        29.  3-a-6-c.        30.  5c-36. 


Page  52 
19.  1,  4,  5,  -3,  -1,  -8,  -9,  7.  20,  3,  16,  35. 

21.  a^  -a\  -8,  -1,  1,  81,  32.  22.  29,  81.  23.  24. 

25.  90.        26.  6.       27.  30.       28.  23.        29.   -20.        30.  -50. 
31.   -100. 

Page  53 
13.  8a+76+9c.  14.  x-iij.  15.  3m.  16.  9a-b. 

17.  4a+|6.  18.  6x^+8x.  19.  a^  20.  a:3-9a;24-i0a:. 

21.   -4a6.  27.  6x2-15.r.  28.  la^-5a. 

29.  2x1/,  3a;2+x//+3//2,  x-+5xij+ij'K 

Page  56 

19.  2a:2+4a:-4.        20.  5a2_8a-22.        23.  214.  24.  ia--9b^. 

25.  2x2+2//2,  4x7/.                   26.  a2+a6+462.  27.  14a;+30. 

28.  x^-6x-l.         29.  2.r-10.          31.   12.r24-12.  32.  3x2+ 10. 

33.   Ion.         34.  3.t2-l-12«-+14. 

B  B 


370 

ALGEBRA 
Page  60 

18.  1. 

19.  x2-3a:+2. 

20.  2^. 

23.  a^ 

24.  a;+13. 

21.  5.  22.  a-h. 


Page  60 

2.  9,  16,  -12,  25,  -7,  27,  -64,  91.  3.   1,  -1,  1,  16,  -27. 

4.  in-.  5.  8«2_9a.  6.  ^a^--^}f-.  8.  30a+406. 

9.   12,T'-+12.  10.  4.r2+12.r!/-9;/2.  13.   13m2+13n2_24»m. 

16.  4.r2.  18.  5a2_3a6_462.  20.  3a2-12a.+14. 

21.  6.r2-2xy-6y2.  22.  4-a.  23.8,19.  24.  .t*-16. 

25.  8a2_9a-l,  6-lOa,  3a-4.  26.  a'^-b^-,  2,  a^-b^. 
27.  2062- 56c.        28.  0. 

Page  81 

22.  7,  -2.         23.   -8.         24.  5,  -2.         25.  5,  6.         26.  i,  2. 

27.  6,  1. 

Page  83 

21.  4,5.        22.   -3,-3.        23.  4,9.        24.  5,3.        25.   12,12. 

26.  19,  3.         27.   15,  -56. 

Page  92 

28.  2(1.  29.  1.  30.  .r  +  1.  31.  3.i-— 8.  32.  x+5,  a+b. 
33.  2(x-2)(a:-3).  34.  3(a+4)(a-3).  35.  x{x-7){x-l). 
36.   ±5,  ±1. 

Page  93 

17.  7,  -11.         18.   10,  -4.         19.  5.         20.   10  in.         21.  7  in. 

22.  14  in.         23.   3i  in. 


ANSWERS 


371 


17.  2x2+2. 

21.  5m'-  —  \0mn. 

24.  .r--4.        25.   18fl-lo 

28.  36x.         29.  9a2-8ai+96-'. 


Page  95 

18.  2a-+2b-.  19.  5a;2 

22.  5x-+24:xi/-5y-. 


26.   3x--4a-^+G^ 
31.  8. 


-5. 

20.  iab 

23. 

■Sx-+I2x+U 

2. 

27.   16X-34 

Page  98 

25.  5a2-5.  26.   3«--86-.  27.  0.  28.   Idq^—ijiq. 

29.  x*-a*.         30.   jr..        31.   3(.i:+^)(.r-7/).        32.  5{x+2)ix-2). 
33.  a(a+l)(a— 1).  34.   m{x-a){x+a).  35.  5(1 +  3p)(  1-3^3). 

36.  (a:+2/)(a;-2/)(a;2+2/2).  37.  ^^ji^r){B-r). 

38.   (a;+l)(a;-l)(a+6).  40.  a'^-2ab-3b'-.  41.  2.t,  7. 

42.  4,  ±8. 

Page  102 

22.  .T(a:+l)(a;-l),  3(.-c-2)(a-+2),  a(a-l)(a.-2).       23.  8,-2;  2,-1. 
24.  2.  26.   2(.r-2)(a;+2)(a;2+4),    (ffl+2)(ffl-2)(a+3)(a-3), 

2w(m  +  3)(m-3),    {x+u){x-y)(a+b){a-b).  27.  2b^+2c-+4. 

28.  3x2-52/2.  30.   12a6-3862.  31,  43,23,17,13. 

32.  3x+6?/. 


Page  104 

13.  a  +  2b.         14.  r/-6.         15.  m-n.        16.  x+j/. 
18.  a-2.         19.  .r-3.         20.  y-\.         21.  a+b. 
23.  2(3a+26).        24.  a(rt-l).        25.  2,  3. 


17.  TO+2. 
22.  x-5. 


22. 


27. 


x+1 


x—y 
x+2y' 


23. 


y 
2/-1' 


28.  a+1. 


Page  106 
24 


29 


x-2 
x^3' 
a2+l 


25. 


TO  +  3 


30.  x2+]. 


26. 


a+46 


BB  2 


372 

ALGEBRA 

Page  107 

,0.1 

11.  4 

12.  ^'^.        13.  "^^^ 
ay                 x-\-2 

14.  "-^ 

a— 5 

15.  1. 


Page  108 

10.  «>+!).          11.  3a;(a;+2).          12.  ab{b+c).  13.  2(x-2-l). 

14.  .r(.r+/y)-.                 15.  {x+l){x-l){x-2).  16.  ab{a-b). 

17.  («+6)(a-?>)-.            18.  a;(x-l)(a;+l).            19.  4x(x-l)(a;+l). 

20.  (^_l)(^+l)(,y_2). 


Page  109 

22.  ^^±^.       23.  ^^^.       24.  ^±^.  25.  ^"il^.        26.  0. 

6                           8                           12  a;2-//2 

27.^.        28.--^^.       29.2^-1^-.  30.             ^^+^^ 


G(x+2)  ab{a-b)  (a+l)(a+2)((7-f  3) 

2 


31. 

(a-l)(a+2) 


Page  111 
2.  .r-2,60(a;-2).      3.  x+y,xy{x-^!/).      4.  x-2,(a;-2){x+4)(.T-5). 

5    "±^  _^   ^"  ^        6    4       7     1        8       —  ■^—  2(a^)  a+6 
a    '  x—y  4b'  c'         '     '         '  ^'         '  x—S'  x+3'  3(x-2)'  a—b  ' 

9.-^,-  10.  ^f^,^.  11.^,1.  12.0. 

13.^.        14.   l,?^i:^.        15.0.         16.  A.         17.      '^^ 


46c  15      xz  ■-"■  a2-62 

18.  -'^.        19.  1.         20.  («+6)-.         21.  3.         22.   +4. 


AN8WEBS  .373 

Page  116 


18. 

Ix+G.           19. 

a+b.            20.  'S—x—ij. 

21.  3x+3//+3z 

22. 

22x-+24:X-U. 

23.   10a;2-oa;+15. 

24.  x2-i-8a;-12 

29. 

a.           30.  2. 

31.   4r,^. 

Page  119 

17.  3a;2-12x+ll.               18.  0.               19.  2ad-\-2bc.              20.  0. 

21.  12a;2+12.             22.   l-x^.             23.  a;4-10a;3+35a;2_50a;+24. 

24.  a;*-10.r2+9.                25.  «6_i.                 26.   13.                 27.  0. 

28.  0.                29.   l  +  'ix+Gx^+lOx"^.                30.  2a:3+9x2+3a;-l. 

31.  28x^+x^y-33x^y^+3lxh,3+20xij*-l2y^.     33.  2-a;+4a;2-2a;3. 

34.  195.         35.  51.          36.  abx*+x^b^-ac)+adx^+x{bd-c^)+dc. 

37.  p^x^-\-x(pr — q^)-\-q>',  x\a^ — a)-\-x'^{a^-ra — 1)  —  1. 

38.  2ahj^-2bhj+2bc  39.  px^'+x{p^+3p+3). 


Page  123 

30.  x2+2a:+l.  31.   —x^+9x^-].  32.  a'^+a+l,  a^—a-\-l. 

33.   -2x^/.         34.  2a.         35.  0.  36.  a-2.         37.  a24.3a-2. 

38.  a;2+a;?/+2/^.  39.  2ax.  40.  x+c.  41.  x-{-p  —  l. 

42.  ax—b~c  43.  oy+a+l. 


Page  125 
1.4.  2.    -.5.  3.  22.  4.  ?/-!-5.  5.  2,7/3.  g.   -2.v3. 

7.1  +  ^-       8.1  +  -^,.       9.2--^.       10.  .5a; -3+-?-. 

x+l  a-6  0+6  x+2 

11.   l+a;+x2^.a.3_  12.   l-a;+a;2-x3.  13.   l+2a;+2x2+2x3. 

14.   l+2a+3a2+a3.         15.  n-3.         16.  6.         17.  a;-4,  7. 


374  ALGEBBA 

Page  126 

15.  a-^-^b^^c^-'iahr.        16.  a^-'2m.        17.  .i-'-^-4x+8.        19.  21. 

20.  llx2-7,r-8.  22.  a;2(c-a)+.r(fZ-6)  +  (/-c).  23.  7. 
26.  5//*.  27.  a;+6j/-22.  28.  2(a6+6c+ca). 
29.  bx'^—bcx^^x{ac—a+h)-hc.  30.  35.  31.  .r*— 4.'c2+12x. 
32.  3.f-8.  33.  3+.  34.  9.  35.  6.  36.  p-+p-2. 
37.  -52.  38.  4.  40.  x''+2a;3+3a;2+2.r+l.  41.  9. 
45.  x'^+x%b+p)^x-{q+bp+c)+x{bq+pc)+cq.  46.  .3a  +  26— c. 
47.  x^+x^+X"{a-a-)-^x{\  —  -2a)~\.  48.  a^^2a%c+A.ab''-c'^+%b\^. 
49.  a;3— a;2(a+6+c)+a;(a6  +  6c+ca)— a6c.                                  50.  6— c. 

51.  3.t5— 10.r*  +  3a;3— 14.r2  — 7.r.  52.  .r24-Z/'+-'*^.'/— 2.^;— 42/+4. 

Page  129 

21.  3(.r-2)(a;2+l).  a(.c-l)(y-l).  22.  a+b,  x-l. 
23.  (2.i--.y)(5,r-33),  «6(a+c)(a-3i).  24.  a:-3. 
25.   (6;r-7y)(8a  +  56).           26.  (.r+//)(.x+;y+4),  («-6)(2a-26-l). 

Page  131 
34.   14.r2+19vy2.  35.  3«2+362  +  3c2.  36.  2.T*+6.r2+2. 

37.  3a2+3?j2_|-3c2_2a6-2ac-26f.  38.  8(x2— z2--a;?/+?/2). 

45.  3(.T+1)2,  fl(a+26)2.  46.  (a+6+2c)2,  (a+6_c-rf)2.  48.  3. 
49.   14.  50.  {x^-^y-){a^+b^+C').  51.  {ax-\-hyY^{ay-hxf. 

52.  0. 

Page  134 
11.  a--b^"-c^~+2bc.  12.  4.1-2+ 12.ri!/+9i/2- 25. 

13.  /)2_4^2_9^2.[_i2^r.  14.   l+.T2+.r*. 

15.  a^-b'^-\-c^--d^-2ac-2bd.  16.  a2^4ft2_c2_4^2_4(ji_^4c^, 

44.  2(.r+2)(:r-2),    a(a.+  l)(a-l).    {a-x){a+x)(a'^+x'^). 

45.  .5(a-64-2c)(a-6-2c),    (.T-36)(.r-6)(.r-56). 


ANSWERS  375 

Page  134  (continued) 

46.  {b+c){b—c){a-{-d){a-d),    {ni-b+c){a+b—c){a—b+c){a-b—c). 

47.  {ax-{-c-{-by){ax^c—bi/),    (to— ?i+3»i?i)(m— ?i— Sm?!). 

48.  (a;+l)(a;-l)(3a;-2),   x{x-l){x-'3){x+3). 

49.  2a-—2ab+2bc—2c^.  50.  {x-\-y){x—i/){x+y+a){x+ij—a). 

51.  2a2— 6a+l,    12xz~24:i/z,    24a+9a2-G«3,    20.i-2,(/2-40.rV 

52.  a*4-6*+c4-2a262_262c2_2c2a52.  53.  (x-y){y-z)(z-x). 
54.  (a-6)(c-a). 

Page  137 

13.  2(.r2+2a;+2)(a;2-2.r+2),   x{x^+x+l){x^—x+l). 

14.  {a-b){a+b){3a-b)i3a+b). 

15.  (a;2-a;+l)(a;2+a;+l)(a;*-a;-+l). 

16.  (a+6+c)(a+6-c)(a-6+c)(a-6-c).  17.  (aHBKSa^+l ). 

Page  139 

31.  3(a;+8)(a;-9).  32.  2(a+l)(a  +  3).  33.  a;(3a;-l)(2a;-l). 

34.  {x-\-l){x-l){x+2){x-2).  35.  a(a-l)(a+l)(a-3)(o+3). 

36.  (a  +  l)(a-l)(3a  +  l)(3a-l).  37.  {x+l)(x+3){x-l){x+5). 

38.  ix-2){x-l){x+l)(x-l0).      40.  x--5.r+6.      41.  4x-2-16a;+15. 

42.  ±1,   ±11,   ±19,   ±41.  43.  33a2-38a6-862. 

44.  ix+y){x+4:y+l).         45.  (3a+26)(a-6+2).         46.  x^-\-l. 

Page  141 

22.  2(a-2)(a2+2a+4).  23.   'My+3)(y^--3y+9). 

24.  a(a+l)(a2-a+l).  25.  ?>((/  +  6)(a2_aft+62)_ 

26.  {a^+b^)(a*-a^b^+b*).     27.  (a;-L?/+a)(.r24-2x2/+?/2_aa;-ai/+a2). 

28.  a;(x2-6a;+12).         29.  (2a-6)(a2-a6+62).  30.  2a(a2+362). 

31.  x+y.         33.  a2(a4-6a26c+1262c2),  2/2z(3a:-2/2)(9a;2+3a;2/2+2/2z2). 

34.  (a-6)(a+6)(a2+62)(a2_a6+62)(a2_^c^ft^62)((j4_a262^54), 

35.  (a;+l)(a;-2).  36.  2. 


376  ALGEBRA 

Page  143 
7.  {x-l)i2x^-9x-4:).  8.  (.r-l)(.r+l)(x-2). 

9.  ix-l)(x-2){x+3).  10.  (x-2)(.r-3)(.r+5). 

11.  (a-l)(a-2)(a+4).  12.  {a+bfia-2b).  16.   -12. 

17.  (a-6)(a+26)(a+36).         20.8,-4.         21.2,3. 

Page  147 
4.  2a;2+2a2-262.  5.  4:X^.  6.  4a2+462+4c2. 

7.   14a2+1462+14c2+14a6— lOac— 226c.  8.   19997. 

9.   14,860,000.         10.-5.         11.  2a+197.  30.  (T-2)(4a;-9). 

31.  3a(2a-6)(4rt2-f2a6+62).  32.  8{a+c)(c-a-b). 

33.  (3a--4)(4.r+5).  34.  4(3o-5)(9a24-15a+25). 

35.  {x+y){x-y+l).  36.  {x-3)(x+3)ix^-  +  2). 

37.  (a:-^)(a:2_a;2/+2/').  38.  {x+Uy)(x-12>j). 

39.  (a-6+c)(a-6-c).  40.  (.r+2/)3.  41.   {x-3b){ax-2). 

42.  (a+26)(rt-26-3).  43.  {2x-y){2x+y+a). 

44.  (a+6)(a+fe+c).  45.  (a-6)(a-6-l). 

46.  {x—y){T^+xy+y^+x+yi-l).  47.  a6(a+6)(a— 6)2. 

48.  (2a+56)(2a-56+l).  49.  96(4a2+2o6+62). 

50.  {x^+^xy-y^){x^--4:xy-y^).        S^.  (a^-~b^+a-3){a'^-b^-a+3). 
52.  {x-l)(x^-lOx-3).  53.  (a-l)(3a2-2a-10). 

54.  (a;+l )(.-»;- I)(c+l)(c2-c+l). 

55.  (a+l)(a~l){a+2)(a^+l){a-—2a+4:).         60.  (o— 6)(6— c)(c— a). 

61.  (.r— 2)(2a;+3)(3a;-2). 

62.  (x--l)(a;-2)(a;-3)(a;-4),    {x+l)(x^3){x-2)(x-G). 

63.  1,  5,  —6  ;   0,  1,  6,  -7.        65.  x^—c^-.        66.  «2-62+c2+2oc. 
67.   -4,  5.  68.  {x-a){2x+a+b). 

69.  {ab+cdf—{ac-'rb(jy-,   {ab—cd)--{ac—bdf. 


ANSWERS  377 

Page  153 

16.  30.  17.   13.  18.   1,3,10.  19.  29.  20.   1,3,5. 

21.  IG. 

Page  155 
6.  45.        7.  4.        8.   1. 

Page  169 

6.  About  2  h.  35  m.  after  A  started  ;  31  m.  from  Toronto. 

7.  (a)  At  10.55,  2  m.  from  C  towards  D.     (h)  22  m.,  17  m.     (r)  11.10. 

Page  172 

5.  1.3.  6.  A  square,  16.  7.  Right-angled,  4.  8.   M. 
9.  54.                 10.    IB;  .5,  6^,  8.                  11.  6. 

Page  177 

3.  .5,  10,  13.  4.  6|.  5.   13.  6.  7|,  4.  7.  .30,  30. 
8-.  (1,  -7),  (-3,  -17),  (5,  3).          9.   112i.          10.  24.  17.  24. 

18.(4,4).  25.  (3,  2),  90°.  26.  $1200,  12th.      ^ 

Page  181 

I.  4xt/H,  24ax^y*z^.       2.  a;— »/,  a-7y(.r-— //').       3.  a-\-h,h(a~-h){a'{-h)^. 

4.  x-3,  (x-3)(.T-4)(.T+3)(a:4-5).     5.  o  +  5,  (a  +  5)(a+3)(rt-7)(rt-2). 

6.  S{x-2),  3{x+\){x+2){x-2r.  7.  x-y,  y{x-y){x+z). 

8.  m— 2,  im^n^{m-\-2){m—2)-{m^-\-2m-\-'i). 

9.  2(a2+a6+62),  6a(a3_63).  iq.  cr-Lfe-c,  a(a4-6-c)(a+6  +  c). 

II.  a-\-h-\-c,  {a-\-h-\-c){a—b—c){b—c—a){c—a—b). 


378  ALGEBRA 

Page  181  {continued) 

12.  x^-xii^>f,  {x^y){x^-yxhf-^^y% 

13.  3,i;-2,  (3.f-2)(a;+3)(d;-3)(2a;-3). 

14.  5x-l,  (5.c-l)2(5.r+l)(2a+3c).     15.  a:-3,  .r(.r-3)(a;-2)(.T2+5). 

16.  u—v,  {u—v){u^v){u^-{-v"){u'^-\-uv-\-v'^). 

17.  a;2_8^  (.c---8)(a;+2)(a;+3).  19.    -a. 
20.  a;2_3_^^_|_2y2^  a;2+a;//— B.y^. 

Page  182 

1.  a;-l,(:c-l)(;c-2)(^2-5.f+3).     2.  a-\,(a-\){a-5){a^-\Sa-\). 

3.  x-2,  (a;-2)(a;2+4)(2a;2-3x-6). 

4.  a-1,  (a-l)(a2+i)(3a2+(7,+6). 

5.  a;-l,a;(x-l)(.r+4)(a:2+a;-6).       6.   {x-2){x-+5x+\){x--2x-\). 

7.  ^^^^ ,  ^— .        8.  .r3-6a:2+lla;-6,  x3-9.c2+26a;-24. 

a2+2a6-1562     2x+4 

Page  186 

1.  x-S.  2.  (a-3)(re-4).  3.   2(3a;2+2:c+2).  4.  2x-9. 

5.  262-6-5.  6.  3a;-7//.  7.  a-2.  8.  x-S. 

9.  3a2(a-l).  10.  x-1  11.  {x-'i){x  +  \){x\-2){x^-x+\). 

12.  (x+l)(.j;+2)(x+3)(a;+4).  13.  (2.c+3)(3.r-4)(.i;2+3a;-l). 

14.  {x-\){x-2){x-'i){x-^).  15.  (5a;2-l)2(4a;2+l)(5.r2+a;+l). 

16.  3.         17.  35.         18.  .r2+5x-14.         19.   11. 

Page  187 

1.  a;-ll,  (.r-9)(a;-I0)(;c-ll)(.r-13). 

2.  a;-3,  (x--3)(:f-12)(a:2-2;(x2+3x+9 

3.  a-b,  (a-6)2(a+6)(n2 4.^6+62). 

4.  a:+3,  .T(x  +  3)(a;+2)(.T-4)(a;-5). 

5.  (2a+l)(a-3),  (2CT  +  l)(a-3)(rr  +  3)(2a-l). 


ANSWERS  379 

Page  187  {continued) 

6.  x-b,  {x-a){x-b){^-c).     7.  x-\,{x-\){x-2){x-Z){x+2){x+Z). 

8.  {.i--l)(x+3),  (a;-l)(a:+3)(.r2+a;+4)(a;2-6x-4). 

9.  (a^3)(2a+l),  {a-2){2a+l){a+2f{a+Zf. 

10.  (x--//)-,  (a--2/)2(a--22/)2(a;+2?/)2. 

11.  x-—x}j+y-,{x-—xy+y^f{x-+xif+y%  13.  3. 
14.  x*-a;2a2+a*.               17.   1,  3.               18.  x2-3a;+2,  x^-Qx+5. 

2x+3 

Page  191 

7.  «^+l  8.^.  9.        ^-^      •  10.      ^+^ 


a2+a+l  ^--2  4a2+3a_6  2a;3(a;+l) 

11     ^1        12  --^-'-^  13  "'-3  .     14     ^-3 

■  x+1  ■   12x2-7x--4  ■  a«— 2a3+2a— 5  2a;-l 


Page  192 
1.  ^^!±^\  2.      ^^  3.     ^^y  4. 


a2-fe2  .r2_y2                      x2-2/2                      a2— 7a+10 

_       2ah  g       2a^            ^       2xj/            g^             3.r                   g    ^ 

a^—b'  '   I— a*            '  x-—y"            '  {x+y)(2x—y) 

10.  3                         ^^     3x^-5xy-2y\          ^^           5 


(x+4)(x+5)(a;-f7)  x^-y^  x^-5x+6 

13.^.        14.-^.        15.  ?^2^.        16.0.         17.     ,1     . 

X— //  a^—b^  x—y  x^—\ 

18.        ^  19.   - .  20.  0.  21.  2. 

2a-36  (a:-l)(a;-2)(a;-3) 

a 
2(a+l)*  a;2_^2'  ^43^ 

_S       —         29      ^^' 


22.  0.  23.  1.  24.  :.-^.  25.      ^^         *^ 


26.-^^.        27.      ^.       28.  -„^„.       29.^.       30.2. 


380  ALGEBRA 


Page  195 


1 

x—a 

(ix{a-\-x) 

5 

26  + 3a 

ah{a—h) 

g 

a;2 

a;2— 2/2 

1/| 

a2+c2 

ac(a— 6) 

IT 

a;+?/ 

2.   ;r  -  „,  3.   ^^.  4. 


2a:  .       -32/ 


2a— 36                        a;— 2  a;^— Qy^ 

«     x'^-\-ax  _     a;2— a;4-2  _        1 

6.    ~ •  7.   — „ — —  8. 

a(x—a)                         x^—\  x—2 

10.  0.         11.       ^     •        12.    — ^-.  13.    ^. 

6— 3a                  3a+2x  x"^—! 


15. "^ — ,   .  16 


(c— a)(c— 6)  (a;— a)(a;— 6) 

/        ^2  ■'3-  r-4-2-         "^9.  0.  20.  0.  21.  0. 


22.1.  23_  ^  n-ff  ^^  -^^-^,^^.,  _  24.-1.  25.0. 

{^-y){y-z){z-x) 

26.  <i.  27.   ^„— ,„^ .  28  ^^^ 


(a+6)(a2+62)  (a;2-9)(a;2-25) 

16a: 


29. .         30. 

(a+l)(a+2)(a+3)(a+4)  (x2_l)(a:2-9) 


Page  198 

1.1.      2.^^.     3.^-=^.     4.  ^^±^'.      5.  -i?*   .     6.  -i^. 

a*— 62  a;+4  a  a^—b~  a;*+j/2 


1  4.  a- ft 

-.  8.  a«+-.  9.  1.  10.  1.  11.  ^. 

a^  a*  a;-3 


12.  ^^"-^^        13.  "+*-'.        (4.  ?  +  ^+l.        15.  ^'-1  +  ^' 
ax  o— 6+c  ij      '^  j/2  a;2 


ANSWERS  381 


Page  198  (continued) 

20.  ^~  .         21 .  '^^  .        22.   1.         23.     1-  .         24.  ^(^+^6) 
a2-a;2  a^  a-8  a-66 


Page  200 

1.  _^.       2.1?^.       3.-^.       4.—.       5.1.        6.  -- 

106c  56  a^—b^  xy  x  b 

7.^^.  8.^.  9.1.  10.  ^^+i-.         11.^'- 

x^+y^  a^  xy{xy—\) 

■»2.  ^!-     13.  ?.     14.  a+6.     15.  a+6.     16.  -?g^^.    17. 


t3 


Page  201 

1.  J^.      Z.""^^.      3.       ^^'     •  4.0.        5.-?-.       6.^. 
a«-6*                 6c                  a;2-9i/2  \-3»  x 

7    1.      8.  l±^^       9    ^H^.      10.  ^+^  11.        "+^ 


2                 2a;                  6+c                 x2+4a;+3  a^—ab-^h^ 

12.^^!+^'.        13.      2ia+^.         14.  i±^.  15.0.         16.1. 
a-\-b                   a^-{-ax+x^                     2 

17.         ^?ZlL^     .         18.  J_.        20.  x.  21.   1.         22.  1. 

(2a;+3)(.3x-2)  x-1 

25.  a;+x2.       26.  'J.     27.   -1.      32.   -+1^  33.  ,       -J''      ,-• 

6                                    a+6  (a+6)(a2+62) 

34.  0.        35.       'ix^-lOx —                 3g  ^         3^      xy^ 

(x-l)(a;-2)(.T-3)(a;-4)  x+y 


382  ALGEBRA 


Page  210 

-_        cd—ab                     .      —a  ^_          ,               ._         ab 

13.     — , ,•              4. •            15.  a—b.             1,6. • 

a+b—c—d                      ra+o  a+b~c 

17    ^.       18    ""+"^+^^  19    ??.       20     — ^^          21     "^ 

■  a+6*            ■        a+b       '  '    2  '            '           2    '             '    b' 

22.  ^''.              23.  a-b.  2^.'^^^.               ;»5.  ^,    "^ . 

a^  2                             ■)-\-c  b-\-c 

__     an-\-b  am—b         __       oA  __     Safe --3a-         __      mn{a-\-b) 

m-\-n    m-\-n                 a—b  o  +  3                    ?mi — m — n 

on         ab—cd  „.       25      2s— In    2s— an 

a-^-b—c—d  a+l       n             n 

__               ,    ,    sr—s4-a    s—a  „_     2s— qt^    2s—2at 

32.  s-sr+rl, ^- ,  -.  33.      ^-f-,  —        . 

s—l  2t             t^ 


Page  213 

13.  a,  -b.  14.  ^i:^,  ^1^.  15.  2a.  -b.  16.  a,b. 

a^—b^    a^—b^ 

17.    -!-^ ^-S  --!-^ ^^-  18.  b+a,b—a.  19.  ^a,  lo. 

fe.^Ci— 6jC2    a^c.i-a.TPi 

22.  c,  0,  a.         23.  4a-36. 


Page  214 

25.   ''"'-'-^«'.  26.  afec.  27.  5,5.  28.  21^i  ,  27,"',. 

a— 36 

29.  2, -i.  30.   J,  2.  31.    -3.  32.  $543,  $457. 

33.   ^.  36.  $16400,  $13600.  37.   1540,  880,  616. 

38.  ^^"""1+^,  2«Z:^"^.  39.  2aH26^.  40.  35. 

2n  n^ — n 

41.   $2100,  .$560.         42.    182040. 


ANSWEBS  383 

Page  219 

18.  a2+3a  +  l.  19.  6.         20.   -4.  21.  (.f4-l)(.T+2)(.r+3). 

22.   l-x-2a;2,  2-3a;-|a;2. 


Paste  221 
19.  {x+!/)~-2{x+ij)  +  l.  20.  ;i-2-3ox+«2.  21.   a^+fts, 

22.  sHft'  23.  .r2  +  2  +  i.         24.    ~6.         26.   13. 


Page  223 

21.  2a3+6rt62,  6o26+2fe3.  23.   2a3+2634-6a26+6a62+6ac-  +  66c2 

27.  27.         28.  242. 


Page  225 

17.  a:2+a;^i.  18.   l-2x-+3.r2.  19.   •:^_i+?. 

3  .r 

20.  3a2-4a+l.  21.   l-x^.  22.  4c  23.  x-l. 

24.  a-3.  25.  .r-2. 


Page  226 

1,  3x^—4:xy+2y\  2.  x^+2x^-:ix  +  l.  3.  a;6+3a;4-2:r2+2. 

4.  ix2— |x+l.  5.  5x^—2ax—3a-.  6.  2a;2+3a  +  7. 

7.  (a;+2)(a;+3)(x+4).      8.  {x+l){x-5)i2x-3).      9.  2x2-5x-+2-- ■ 

X 

10.  3— 5a;.                                  11.  2x2-a;+l.  12.   a. 

13.  l-a:-|x2,  l_|a-ia2,  2+|.r-J  x^.                     15.  7a,-2-2x-|. 

18.  8a3.           21.   0.   -82/3.           24.  2.c3-3a;2+a;-2.  25.   16. 

27.  6x-4.  28.   7x2-2x+l. 


384  ALGEBRA 

Page  230 
32.  vT3.  34.  V2I.  35.  2\/2    4V2.  36.  5V2 

37.  4a/2,  12  V  2. 

Page  232 

9.   10\/;3.    ,  10.  7\/2.         11.  5\/5.         12.   -3\/7.          13.  7\/2. 

14.  8v/ll.  15.  7a/5.         16.   -41/2.         17.  8-66.        18.  794. 

19.  11-62.  20.  5-20.  21.  -141.  22.  25  46. 
23.  ±6-083.  24.  ±3-873.  25.  ±6-782.  26.  ±6-481. 
27.   ±9-592.  28.   ±13-711.        29.  7-483. 

Page  234 
13.  24\/3.  14.   UVI.  15.  5+2v'6.  16.  21-W35. 

17.  30  +  12\/6.  18.  a+b+2Vab.  19.  2+3^2. 

20.  12  +  \/6.  21.  6+1/10.  22.  6a+(3b-l3Vab. 
23.  6  +  2v/l5.  24.  4v'6-4.  25.  a+b-6-V~a+b.  26.  1. 
27.  6  +  2\/3+2\/2+2V6.  28.  16+41/10-21/15-41/6 
29.  2a  +  2i/a2^2.  30.  13j,--5y-12i/x2-"^.  31.  1. 
32.  12-4\/2.  33.  6\/6.  34.  70.  35.  30-5\/6. 
36.  V8+1/7.  37.  42,  43.  38.  46.  39.  9\/2. 
40.  30-92. 

Page  236 
13.  14+'8l/3.        14.  6v/2+4i/3.        15.  5+2i/6.        16."^-^-- 
17.  1/15.       18.   ^'^~^.       19.  -577.        20.  3-536.        21.  -817. 


AN;SWERS  385 

Page  236  {continued) 
22.  -318.         23.  1-491.         24.    084.        25.  1-225.         26.  -894. 
27.  -072.  28.  2-12.  29.  -82.  30.   1-39.  31.  3-15. 

32.   11-71.  33.   ±2-73.  34.  1-008.  35.   \  2. 


Page  238 

1.  12a/2.  2.  12V5.  3.   lOVS.  4.  62.  5.  191. 

6.   -4V2.         7.  22-12v/2.         8.   12-4V6-2\/3+4V2.         9.  ^. 
10.  9-4V5.  11.   1.  12.  2\/l3+2\/2.  13.  74+11V6. 

14.  |.  15.   1-732.  16.  \/l2+\/T0.  17.  1,  2. 

18.  2\/2,  iVe,  iV30,  iVli,  1{W2-2VS). 

25.  ±8-661,   ±7-937,   ±9-899,  1-291,  -518. 

26.  -817,  -447,    414,  -757,  -337.  27.  25^3.  28.  2^2. 

29.  ^^~      ,  4V6.        30.  2-02.        31.  30.        32.  5.        33.  4-83. 


Page  241 
1.  a;2+«-132=0,  2.  x2-a;-156=0.  3.  a;2-49=0. 

4.  a;2+6a;-112=0.       5.  5a;2-6x- 440=0.        6.  a;2+6a;-9400=0. 
7.  x-2^19a;+88=0. 

Page  242 

1.  6,  -1,  -22.      2.  6,  -25,  21.      3.  8,  19,  -15.  4.  12,  -11,  2. 

5.  1,  -10,  9.          6.  2,  -.5,  2.          7.   1,  4,  -32.  8.  5,  -27,  28. 
9.  2,  -19,  44.        10.  2,  -5,  -3.         11.  0,  2,  7.  12.  0,  1,  -1. 

C  C 


386  ALGEBRA 

Page  248 

1.  4-236,  --236.  2.  7-828,  2172.  3.   1-646,  -3-846. 

4.   1-916,  -9-916.  5.  -232,  -3-232.  6.  -851,  -2-351. 

7.  S±Vn.  8.   -4±3\/3.  9.  |±V2.  10.  l±|V4i. 

5±V157  ^2.   -i±^V22. 

6 


Page  251 

20.  1-618.  21.  2i,  -1.  22.  5,  12.  23.  6,  -2. 

24.   14,  6  or  16,  4.  25.  .r=l  or  2,  2/=2  or  1.  26.  540. 

27.  3-236,  -1-236.       28.  20c.        29.  60,  90.        30.  1-449,  --949. 

31.   20.  32.  3  m.  per  hr.  33.  8.  34.  «'+«^    "'""^ 


-6'    a+b 


35.  20,  30.        36.  x=2  or  ^.        37.  4. 


Page  255 

18.  2-54,  1-0936.                       19.   8  :  27.                       20.  -192,  1-302. 

21.   3937  :  6.336.                                         22.   4:5,    11  :  27,  a+3  :  a+5. 

23.   11  :  15,   13  :  18,  2  :  3,  3  :  5.              31.  "^^^  ■  32.  i±^ . 

c—d  l+4a 

33.  4  :  5.         34.     "'^       "^  ■        35.   10.         39.  206m  :  an 

b+c    b+c  — 


Page  259 
29.  11-55.         30.   10^,  4i.         31.  AE=6^,  DE=lh         32.  240. 

33.9-899.  34.2:3.  35.  2,  '^-^  ^?^,  -  ?  ■ 

a — c    m—n        p 


ANSWERS  -387 

Page  259  (continued) 

36.  ;■;  or  H,  o  or  -1.  37.  -  =  ^  =  -     . 

5      3       -8 

38.  AC=20,  AE^o,  DE=i.  39.  147  ft.  40.  3  or  J. 

43.  5:4:2.  44.   :i  or  -i  45.  G^    5A  ;     ^"^  .  ^  . 

46.  2  :  3.         47.   17^,  25,  30.         48.   110  :  15  :  17. 


Page  262 
6.  6  ;  74  ;  IIJ^,,  5,  4^^^  :  5,  2|,  H.         8.  2,  8.         9.   3,  6,  12. 

Page  264 
4.  f|.  5.   -17*.  6.  §i.  14.  ?^  15.  V. 

17.    ~^  .         18.   7  :  16. 

Page  266 
17.     "^    .       18."^.        19.2,.^.        20.  |.        21.  3,  §.       22.5. 

23.  f  =  ^  =  ^.  24.  iV  25.  :n/-l.  26.  i. 

3       4       5  Zi/ 

32.    ±10,   ±,5,    +5.         38.  41;-;.         39.   1,3,4. 


Page  270 
1.  3a,  —a.  2.  b,  —56.  3.   3m±mV'6.  4.   — 2p±p\/5. 

5.  cr±\/^2ii^.  6.   -fttVftMlc.  7.    -1±     A  +  1. 

CC  2 


388  ALGEBRA 

Page  271 

1. 1,  §.         2. 1, 1.         3.  jjj,  -oV  4.  i±wn. 

5.  tV,  -VV-           6.  :L3±iV89.           7.   ^,  -^.  8.  ^^l.  _j^. 

9.  2m,  -36.            10.  7,  V.            11-  i  -5-  12.  l^,  -^. 

19.  ,''^±  JoVm.        20.  ■i±l\/51.        21.  ;],  -^.  22.  |±2\/5. 

32.   l±^V2.        33.   tVe.        35.   l±2v'6.        38.  2414,  --414. 

39.  3.                       43.  6-18,  3-82.                       46.  2-786  or  --120. 
53.  703,  8-78,  8,  2-29,  642,  impossible. 


Page  276 
13.  1,  -6,  -|±A\/^39.  14.  6,  -3,  flW^TT. 

15.  2,   -1±V^.  16.    ±2,   ±2V^.  17.  3,  2,  -5. 

18.   1,  I,  |.  19.   -i±|A/=23. 

20.  3,  h,  -i±iV^,  -i±|V^.  21.  2,  3,  -1,  -2. 

22.   1,  1,  -i±iV5. 

Page  277 

25.  ^^-— ^.  ^~^.      26.  -4,3,  -J.±i\/^^T5.      27.  5,  -#±5^/^. 
TO+TO    m—n 

29.  a+6±A/a2q^&+P.       30.  ^,  -- •      31.   15.       32.    — ,  —  . 

c        fl  a— c  a-'rC 

33.    -1,   -2,  -4,  -8.  34.   12.  35.   -a,  -b. 

36.  a+h,  0,  "^+^".      37.  5.10  p.m.      38.  a±^.     39.  ^,  I^'^. 
rt->rb  a  a-\-b  a+b 


ANSWERS  389 


Page  277  (continued) 


40.  6076  nearly.  41.   -^-^"'+%.  42.  li. 

32 

43.   -J;±iv2T,  -i±iVl3         44.   10  in.  from  a  corner.        46.27. 


Page  281 

7.  (5,  2),  (-^,  V)-        8.  (3,  2),  (^.,  |g).        9.   (6,  4),  (-?f,  -Jf). 
10.  (4,  1),  (-1,  --/).  11.  (2.  1),  (-5,  -V). 

12.   (2-525,  -175),  (-2-275,  -1-425).  19.  4-196,  4-732. 

20.  (-2, -1),  (-^,  1). 

Page  283 

6.  (2,  1),  (19±|1V_5    -7^5)_ 

7.  (4,  .3),  (-1,-3),  (3,2),  (-1,  -I). 

8.  (1,  2),  (-5,  -10),  (-3±^89    -9±3v^\ 

\         4  16        / 

9.  (2,  4),  (3,  3),  (2,  -3),  (-3,  -3). 


Page  284 

/     23         n   \ 
'^-  (-^-*H*V!1'*^>  13.(±4,±l).(±l.3^/,4,±5^/^). 

14.  (±1,  ±2),  (±^^,  +^).  15.  (0.  0),  (1,  1).  i^„  ^). 

16.  (±6-32,  ±3-16).         17.  35. 


390  ALGEBRA 

Page  286 
16.  (±2,  ±1),  (±1,  ±2).  17.  (±3,  ±2),  (±2,  ±3). 

18.  (±2,  ±1),  (±1,  ±2),  (±V^1,   +2\/~),  (±2V^,  tV-I). 

19.  (5,  2),  i-h,  -I).       20.  (6,  2),  (-2,  -6),  (|±W57)(-|±i\/57). 
21 .  (5,  3),  (3, 5),  (6,  2),  (2,  6).       39.  7-32, -68.      40.  ^b+^Vb^-lQa. 

Page  291 

18.   (2,  2),  (2,  1),  (1,  2),   (2±\/2,  2  +  \/2),  {^^±W^,  ?,+W-l). 


19.  81.  21.  ^-^-.  ^J.         23.  [a,  3J,  ^3,  6j.         24.  §.  |. 

27.  (±10,   ±5),  (±5\/2,   ±5^2). 

36.  (3,  2),  (2,  3),  (-2±\/^,  -2  +  \/^). 

39.  (-1,  2),  (2,  -1),  (-i±^\/l3,  -i±i\/T3). 

^Q_  ^+V2^'-g'^  _s+V2^2-s2_         ^^_  ^^^  ^^^  ^2,  2),  (1,  12),  (f,  6). 

42.  (4,  2),  (2,  4),  (8,  1).         43.  (5,  1),  (1,  5). 


Page  296 

he 

19.  — .        20.  3^4''  +  4''.        21.   1.         22.   1.         23.  0".         24.  1. 
25.  22",  312.        26.  3.        27.  2,  9         28.  2,  7,  3,  2. 


Page  301 

45.   I  46.  8.  47.  625.         48.   ll^.         49.   125.         50.  J. 

51.   ,V  52.   32.  53.  4.  54.   5.  55.  ^,         56.   }^. 

57.  #-.         58.   ^a^y".         59.   16,  8,  81,  1,  \. 


ANSWERS  391 

Page  302 
1.  x+x^—6.        2.  xi—1  3,  x'—l         4.  3x2— 8a;'-+9.T— 10x5 

5.  a— l+4a"^— 4a-i.  6.  a^~2a^  +  3a—2a^  +  l. 

7.  x++4x— 11x2— 6x3^.  8.  x2+8x'^'+24x+32x2  +  16. 

9.  x2+x?/+r/2.       10.  ai—3a+3a^—l.        20.  x^+x+l+x^^+x-^. 
21.  5a2'«+4a'"-2.  22.  2x2+6x+2.  23.   l-2a. 

24.  X— 2^x4-3. 


Page  304 
1.  x-4,  a3_i3_  2.  a+i+a-\ 

3.  x2— 2x?— x+2x5  +  l,  4a2-8a+4a-i+a-2 

4.  a^+3a+Za^  +  l,  1— 3x2  +  3x— x'^.  5.  x^+xV+y*- 
6.  x^+i/l               7.  a^+fti+ci                8.  (x+2/)(a----2/-)(a:^+A 


i  1  r2_LS        2  1     1 

i-^-b--^.  10.  ^If^t^,  a3+a^&3- 

x3-3 

11.  x^-2,  x-x^4-l.         12.  2x-2+3x-V  x+2-x-i. 


9.  a2_62.  -10.  ^iXf,  a3+a^&3+6^-  a-V^+6. 

x3-3 


Page  305 

3.  5,  v„  49.                 4.  4,  .;,,  25,  4,  ,V  8,   ,V  6.  5,i„  2. 

7.  31G2,  1-778,  1-333,  5-62.                          8.  4.  9.   1-732. 

10.  9f,  -lig.              11.   100.                 12.  4,  2.  13.  ^,  H. 

14.  f,  f.             15.  ^^^,  3x^+2,  -^-^—  .  16.  x2t/*+8xV 
Va+4                  a^6-62 


392  ALGEBRA 

Page  305  (continued) 
17.  a;^?/-i  +  l+.r~^?y.  18.  x^—y~^,  a^— l+a~^. 

19.  a^'"-^a^"'h"'^a"%-"'-\-h^"'.        20.  2/+27/-  +  1.        21.  x—2—x-\ 
2!2.  ha'-—%bK      24.   -0016,  1-44,  3-375,  8.      25.  Va -1\  Va  +  l 

26.   1.  27.  e'-+e-^  x^— 2a;?/2-j-3a;ii2/— 2/2.  28.  06c. 

30.  4,  32.  31.  2750.  32.  J.  33.  2,  3. 

34.  a;3+2a;^+l+2x"^+a;"t         36.  2a;+a;^—a;~l. 


Page  310 

7.  ^4,  ->^27;  v^Ie,  V'27;   ^64,  ^ST,  ?L25. 

8.  3V2,  5\/6,  Vs:   1-26,  ^5.  9.   12\/2.  10.   12\/5. 

11.  33\/2.        12.  3^2.         13.  7\'T2.        14.   10\/2.        15.  9\/3. 

16.  0.  17.  \/3,  a/5,  .t\/^,  2/^4^,  \/2. 

18.  2-52,  3-78,  12-6,  -63,  -126,  1-26. 

Page  311 
25.  3\/rO.  26.   IVS.  27.  fVB.  28.  V2. 

29.   1(2a/2-V3^.  30.  \/rt2+62+6.  ,  31.  \/^6-Vc. 

1  ,, 

32.  -(x-Vx^-y^).  33.  2-517,  1-354.  34.   194. 

35.  27(\/.3-\/2).         36.   ^Z(7-V5),  2\/5.  37.  ^^(18-34^5) 

39.   10  ft.  5  in. 

4 


ANSWERS  393 

Page  314 

16.  No  root.  17.  4.  18.   5^.  19.   100.  20.  9. 

21.    -a.  22.  25.  23.  ()4.  24.  Xo  root.  25.   fc^  • 

2«— 6 

26.  3.        27.   10.        28.   10.        29.   V.         30.   4^ 

C2  +  1 


Page  317 

18.  (4,  9),  (9,  4).  19.  (4,  10),  (16,  4).  20.(17,8). 

21.  (9,  1),  (1,9).         22.  (2,  i),  (|,  2).  23.2,1.  24.  7,  -G. 

25.  (2,  8),  (8,  2). 

Page  320 
10.  2-823.        11.  -196. 

12.  2^(2+\/3),  5T(V.5+1),  3+(2-\/3),  2T(5\/2+3).  14.   l+VS. 

15.  ^r\  ^^t\  ^^t^-        17.  2-309.         18.  24-3V2. 
a/2         V2         VS 


Page  322 

10.  25\/^.  11.  68.  12.    -25.  13.   1+V^. 

14.   ~^~^~^.        15.  2a2_262. 
2 

Page  324 

•1.  2+2\/2-2\/3.            2.  2  +  ,';\/6.             4.  1-98,  3-15,  1-39,  3-55. 

5.  9.          6.  Hg.          7.  7!.          8.  7,   -1.  9.  20.           10.   13. 

11.  6+2\/l5,  a;24-2.r?/+j/2-4x-4?/.  12.    ;\/3. 

13.  4a+2\/4a2-6  14.  A(M2_2pfl_^o2\  15,  12. 

2? 


394  ALGEBRA 

Page  324  (continued) 

16.  x*-lx^+2x+2.  17.  0.  18.  2a,  Aa^-2,  Sa^-%a. 

,~         2m 
19.  4,  7  20.   v/3,      ,__=-•  21.  5. 

22. y^  ,  \/a+\+\/a-\.  23.  4,  -7.  24.  40. 

25.  2-62,  -38.  26.  ^Va^^^^.  29.   1.  30.    10\/2. 

b 

31.  16+9\/3. 


Page  330 

17.  x^—2mx+m^—n^=0.  18.  x^— 4aa;+4a2— 62=o. 

19.  a:2-6x+6=0.  20.   16a;2+8a;-63=0. 

21.  a;3-28x-48=0.  22.  24a;3-26a;2+9a;-l=0. 

24.  4x2 -28.r+ 45=0.  25.  x2-7x+12=0. 

26.  a;2(a2-62)-2.T(a2-j-62)4-a2_52^0,  4x2-16x+9=0. 

27.  9,  7;  a+6,  0;  2;?,  pq;  2c-2a-2b,  a^+b^-c\ 

28.  0,  5,  -1,  -4.        29.  4,  8,        30.    ±16. 

Page  334 

I.  If,  6^,  22.  2.  47,  -II.  3.  -2g,  1^,  6J. 
4.  _5g,  i_3a.  5.  x2-18x+80=0. 
6.  x2-5x+4=0,  x24-5x+5=0.       7.  6x2— x— 2=0,  5.^2—^:^+1=0. 

8.  5x2-2x+3=0.  15x2  +  26x+15=0,  9x2+26x4-25=0. 

9.  a'~-2b,  'iab-a^  10.  x^-x{p^+2q)+q^=0. 

II.  ax'--x{2ah-b)+ah^-bh+c=0.  12.  x2-4x-4=0. 
14.  x2+6a:+8=0.             16.   i(62-a«)(62_3ac). 


AN8WEBS  395 

Page  338 

6.  Rational.  7.  Real  and  irrational.  8.  Imaginary. 

9.  Real  and  equal.  10.  Rational.  11.  Real.  13.  4. 

14.    ±5.  16.   -L  19.  -•  20.  2,  -j. 

m 


Page  340 

8.  {x+2  +  Vl){x+2-Vl).  9.  16.  10.   ±6ff. 

11.  (.c-3+2\/5)(x-3-2\/5). 

12.  (3.r-4i/)(3:c+42/)(4x-3  )(4.t  +  3//)  ;   ±|,   ±|. 

13.  174;  (8a:+7)(15a:-4).  14.  62=4ac. 


Page  341 

4.  6,  ^,  |,  4,  -14.        5.   B,  L        6.   16.x-2-40a;+21=0.       7.   12*. 
8.  6x2- 19a;+ 15=0.  9.   _25.  10.  a;2±  12a; +35=0. 

11.  (72x+l)(73.T-l),  (13.T+ll)(17a;-15).  12.  a;2-4a;+3=0. 

13.  2a+26— 2c.  14.  97a;2— 53a;- 17=0. 

15.  {x+3+V2){x-\-3-V2).  16.  2a;2-17a;=0. 

17.  a.T2-f36a;+9c=0.  18.  acx^—x(b^—2ac)+ac=0. 

19.   1,  ^±^-^.      23.    ±12.       26.    ±4.       27.  Zm,n-m^       28.  a. 
a-\-b-\-c 

29.    ^"L,    _^_   .        30.   i        31.  c+6-a.        32.  a=8or0. 
a+b—c     a+b—c 

33.  8,  1.  35.  6,  2  36.  (x+b+Vb^^}{x+b-\/b^^). 


396  ALGEBRA 

Page  345 

16.   2«-ft+3.  17.  y-7g+3r^  ^g^   -7,1,19. 

p—q+r 

19.  (.T-?/+3)(.T  +  2/y-4).  20.  2a-3,  3a-4. 

21 .  (2ffl-?>+c)(3a-6-c)(3fl+2fe-2c. 


Page  348 

13.  {2x-y-5z){Ax'^+y^+25z''-+2xy+\0xz-5ijz). 

14.  (a+6+c+l)(a2+62_^c2+2a6— ac— 6c— a— fe-c+1). 

15.  a^—b^—c^—Zahc.  16.  8a;3—?/3+ 2723+18x2/2. 

17.  l-f(3_63_3g,j_  18.  8a3_2763-64-72o6. 

19.  l+a2_|_^2_^p(_ft_^(j5_  20.  ^m'+n^-\-\  +  Zmn+Zm—n. 

21.  a+56-1.  22.  3ffl+6.  23.  a;2-3.r+9.  24.  2a-5. 

25.  a+6+c+d-l.  28.  27a:='-8?/H3^+18.ryz.  35.  0. 

36.  «!±^M:^6n:.ac-6c^    x+2^^  ■  37.  (,+,+,)(«+,). 

2a— 36+ 3c  2 

40.  a,  6. 

Page  351 

9.  (.r— ?/)(?/- z)(a:— 2).  10.  (x—y)(y—z){x—z). 

11.  (.r-;v)(?y-2)(2-aO-                           12.  (ff-6)(6-c)(c-a)(o+6+c). 

13.  (f/-6)(6-r)(c-a)(fl  +  6)(6+c)(c+rt).  14.  a+6+c. 

15.  ^,    'I.        16.  ^^^    ^-^-         17.  a,  6.  18.  ^^   ""^ • 

a     b  a—b     b-\-a  a  h 

19.  t    ^.  20.  ax~a-b. 

a    b—a 


ANSWEMS  397 

Page  353 
10.  a,-«-2:i;3+4a;2-8.t-+16.  11.  .i^-3x-+9x-21. 

12.  x^+x^a+x*a'^+X'a^+a'^.  13.  (a^bf-{a+b)-+a+b—h 

14.  x—b,  u+b,  x—4:,m+-,x+y—l.         15.  a^-l.         16.   /w^+1. 
m 

17.  a8-68.  20.  a2^a?)+g=0.  22.   1,9. 

3  1     1  2         2  1     _1_  3         4  :•.     1  2     2  13  4 

4  :i   X        2    2         1    a         4 

x-^  —x-^a-^  -\-x^a^  —x^a''  -\-a" . 


Page  357 

13.  3(a2+62_|_c2)_2(a6+6c+ca).          14.  2(a2+6Hc-).           15.  0. 

16.   —Z{a%—ab'^+b'h—bc-+c^a—ca-).  17.  {x—y){y~z){x—z). 

18.  (x-«/)(2/-2)(x'-z).            19.  (a+6)(6+c)(c+a).  20.  24a6c. 

21.  Z{x-y){tj-z){z-x).  22.   (a  +  6)(6+c)(c+a). 

23.  — (a-6)(6-c)(c-a)(a+6+c). 

24.  («-6)(6-c)(c-a.)(a+6)(6+c)(c+a).  25.  1.              26.  1. 

27.   1.      28.   -4--        29.  0.       30.  a+6+c.       31.   -{x^y+z). 
abc 

32.  3.  33.  6a6c  35.  3(x2+.(y2-fz2)-f-2(a;2/+jy2+za;)- 


Page  362 
24.  18.  26.  a2-262    a^^Sab-.  27.  a2=c24-262. 

28.  aH2c=»=3a62 


398  ALGEBRA 

Page  362 
6.  W+^,y^-,hz^+l^yz.  9.  x^{a-\){\-h).  11.  ^^. 

12.   Ua+b+c).     15.    -(a6+6c+ca,).      16.     "^  ■    18.   -'(a+ft+c). 

aoc 

22.  (a;-l)(x3-2.r2+2.r+2).  23.  -a-b-c.  26.  (a;+l)(a+6+c). 
29.  3(a;2+2/2+22)  +  |(^2/+^2+2x-).  30.  a2=26+c.  31.  9,17. 
32.  38,  70. 


T     H     BEST  PRINTING  CO.    LIMITED.  TOKONI 


/• 


cmrs